Вы находитесь на странице: 1из 196

ATOM & ITS STRUCTURE

E) Its atomic number is 10


1. There is some information given below about the
number of proton and neutron of X and Y atoms;
I. X atom has three protons more than that of Y atom, 7. Which one(s) of the following might be equal for neutral
II. X atom has two neutrons more than that of Y atom, P and S atoms?
III. the proton number of X atom equals to its neutron I. Neutron numbers
number, II. Proton numbers
given that, if the atomic mass number of X atom is 24 what III. Electron numbers
is the atomic mass number of Y atom ?
A) I only B) II only C) III only
A) 9 B) 15 C) 18 D) 19 E) 21 D) I and II E) I, II and III

8. The relations among the number of protons (p), the


2. Which one might be equal for the two different number of neutrons (n) and the number of electrons (e) are
elements? given below,
I. p = e II. p < n III. e > n
A) Atomic numbers Which statement(s) is (are) true for a neutral atom always?
B) Atomic mass numbers
C) Atomic weights A) I only B) II only C) I and II
D) Number of protons D) II and III E) I, II and III
E) Electron numbers at ground state

9. I. An atom having the same number of protons and


3. Which of the following is the same for all atoms of an neutrons
element? II. An atom having the same number of neutrons and
electrons
A) Nuclear charge III. An atom having the same number of electrons with its
B) Number of neutrons atomic number.
C) Atomic mass number Which one(s) of the given data cannot be exactly define a
D) Total number of protons and neutrons neutral atom?
E) Total number of electrons and neutrons
A) I only B) II only C) III only
D) I and II E) I, II and III
4. If the atomic mass and atomic number of an element
are 2x+ 8 and x, respectively, what is the neutron number of
this element? 10. Which one of the following is heavier than an electron
1840 times?
A) x B) 8 C) 2x D) x+4 E) x+8
1 2 4
A) 1 H B) 1 H C) 2 He
3 3
5. Atomic number and atomic mass number of beryllium D) 1 H E) 2 He
are 4 and 9 respectively. According to this, beryllium atom
has
I. 4 electrons, II. 4 protons, III. 4 neutrons, 11. X and Y are isotopes. And their atomic masses are
Which one(s) of the givens above is (are) correct? different. The reasons of this difference are given as follow.
I. Difference in their number of electrons
A) I only B) II only C) III only II. Difference in their number of protons
D) I and II E) I, II and III III. Difference in their number of neutrons
Which statement(s) is (are) true?

6. Atomic mass number and atomic number of fluorine are A) I only B) II only C) III only
19 and 9 respectively. D) I and III E) II and III
According to this, which one is correct for fluorine ions?

A) It has 10 neutrons 12. I. (-) ion is formed when electron is added to the X atom
B) It has 9 electrons II. (+) ion is formed when electron is removed from Y atom
C) It has 10 protons III. An isotope of Z is formed if a Z atom captures a neutron
D) Its atomic mass number is 20

2
Which one(s) is (are) true for X, Y, Z atom?

A) I only B) III only C) I and II 18. How many electrons are there in a compound formed
D) I and III E) I, II and III by 11X and 16Y?

A) 27 B) 34 C) 38 D) 43 E) 51
13. Which one of the following ions has the charge of -3?

p n e 19. For the ion of 11 H -1


A) 7 7 10
B) 13 14 10 I. It has one neutron and one proton
C) 12 12 10 II. It has one proton and two electrons
D) 19 20 18 III. It has one proton and one electron
E) 15 16 10 Which one(s) is (are) true?

A) I only B) II only C) I and III


+3
14. If an X atom gives 3 electrons, it becomes X which of D) I and III E) III only
the following can be deduced by using this information
only?
I. The difference between number of electrons and protons. 20. X+3 ion, whose mass number is 70, has 28 electrons.
II. The total number of protons and neutrons. What is the number of neutrons of X?
III. The difference between the charge of nucleus and ion
A) 22 B) 29 C) 39 D) 41 E) 42
A) I only B) II only C) I and III
D) II and III E) I, II and III
21. X-2, Y-1 and Z+1 ions have the same number of electrons
and their numbers of neutrons are one more than their
+2 -2
15. For the X and X ions of an atom X; which of the numbers of electrons. How can the neutral X, Y, and Z be
following; arranged in order of their atomic mass number?
I. The charge of nucleus
II. The number of protons A) Z>X>Y B) Z>Y>X C) X>Y>Z
III. The number of electrons D) X>Y>Z E) Y>X>Z
is (are) certainly the same?

A) I only B) II only C) I and II 22. For the neutral sulphur atom which has 16 protons and
D) II and III E) I, II and III 16 neutrons;
I. The atomic number of sulphur is 32
II. S-2 has 18 electrons
16. Consider the species of X, X+1 and X-1 for the same III. The charge of its nucleus is +16
atom; Which statement(s) is (are) wrong?
I. Their numbers of neutrons are different.
II. They are arranged X-1 > X > X+1 in order number of A) I only B) II only C) I and II
electrons. D) II and III E) I and III
III. Their charges of the nuclei are the same
Which statement(s) is (are) true?
23. What is the atomic mass number of Fe+3, which has 23
A) I only B) II and III C) I and III electrons and 30 neutrons?
D) I and II E) I, II and III
A) 50 B) 53 C) 56 D) 47 E) 55

17. Which one of the following is true, for the numbers of


14 16 24. X+1 ion has 18 electrons and 20 neutrons. What is the
protons, electrons and neutrons of NO3-1 ion? ( 7N , 8O) mass number of X?

p n e A) 37 B) 38 C) 39 D) 40 E) 41
A) 15 15 16
B) 21 42 21
C) 30 31 30 25. 207Pb has 125 neutrons. How many electrons are there
D) 30 30 30 in one Pb atom?
E) 31 31 32

3
A) 86 B) 82 C) 78 D) 62 E) 42 Which statement(s) is (are) true?

A) I only B) II only C) I and II


26. Ions X+2 and Y-3 have the same number of electrons. D) II and III E) I, II and III
What is the number of protons Y, if the atomic number of X
is 12?
33. If the compound of XaYb formed between 13X and 16Y,
A) 18 B) 15 C) 12 D) 10 E) 7 what is the ratio of a / b?

A) 2/3 B) 3/2 C) 1/3 D) 3/1 E) 1/2


27. X-1 has 74 neutrons and 54 electrons. What is the
difference between the atomic mass number and the
number of protons? 34. I. Atomic mass number
II. Proton number
A) 53 B) 74 C) 54 D) 127 E) 47 III. Neutron number
Which one(s) is (are) the same for all neutral atoms of
elements in the nature?
28. I.X has 11 protons and 12 neutrons.
II. Y has 16 protons and 16 neutrons. A) I only B) II only C) III only
What is the molecular mass of a compound which is formed D) I and III E) II and III
between X and Y?

A) 17 B) 55 C) 38 D) 78 E) 40 37
X and 35
35. 17Y are isotopes of each other. What is the
number of neutrons of X?
29. If X+3 ion takes 3 electrons and Y-2 ion gives 5 electrons,
their number of electrons will be the same. What is the A) 18 B) 17 C) 19 D) 20 E) 24
difference between the atomic numbers of X and Y?

A) 1 B) 2 C) 3 D) 4 E) 5 36. The isotopes of hydrogen are 11 H , 12 D , 31T and the


35 37
isotopes of chlorine are 17 Cl , 17 Cl . What are the heaviest
30. For the atom of 52Cr which has 28 neutrons; and the lightest compound formed between isotopes of H
I. Its atomic mass is 52 amu. and Cl ?
II. Its atomic number is 24
III. The ion of Cr+3 has 21 protons The heaviest The lightest
Which statement(s) is (are) true? 3 37
Cl 1 35
A) 1T and 17 1H and 17 Cl
2 35 2 37
A) I only B) II only C) III only B) 1 D and 17 Cl 1D and 17 Cl
D) I and II E) I and III 3 35 1 37
C) 1T and 17 Cl 1H and 17 Cl
1 37 3 35
D) 1 H and 17 Cl 1T and 17 Cl
31. The one molecule of X3Y2 has 50 electrons. If the ions 3 35 1 35
of X+2 and Y-3 have the same number of electrons, what is E) 1T and 17 Cl 1H and 17 Cl
the atomic number of X?

A) 10 B) 11 C) 12 D) 7 E) 8 37. Consider the table,


Number of Atomic mass
Atom neutrons number
32. X 18 35
Atomic Atomic Mass Number of Y 20 37
Element
Number Number neutrons
I. X and Y are isotopes of each other.
X 17 ? 20
Y 16 32 ?
II. The atomic number of X is 17.
Z ? 39 20 III. Neutral atom X has fewer electrons than Y.
T ? 35 18 Which statement(s) is (are) true?
Fill in the blanks completely in the table. Considering X, Y,
Z, T atoms; A) I only B) II only C) I and II
I. X and T are isotopes of each other. D) II and III E) I and III
II. The heaviest of them is Z.
III. The compound of YT2 is formed from Y and T.
4
38. The neutral atoms of X-3 and Y+5 ions are isotopes. So, A) n>p=e B) p=n=e C) p=n>e
their D) n>p>e E) none
I. Charges of nuclei are the same.
II. Atomic mass numbers are the same.
III. Numbers of electrons are the same. 46. The ion X+2 has 10 electrons and atomic mass number
Which statement(s) is (are) true for the ions of X-3 and Y+5? of 24. What is the number of protons in the atom X?

A) I only B) III only C) I and II A) 10 B) 12 C) 24 D) 34 E) 54


D) II and III E) I, II and III

47. The ions X-3 and Y+2 are isoelectronic. If X has an


39. Which one of the following is wrong for an atom? atomic number of 15, what is the atomic number of Y?

A) Protons and neutrons are found in the nucleus. A) 15 B) 16 C) 18 D) 19 E) 20


B) The number of protons always equals the number of
neutrons.
C) Electrons are found around the nucleus in certain energy 48. Which one of the following statements is wrong for the
levels. isotope atoms?
D) Protons are positively charged and electrons are
negatively charged particles. A) Chemical properties are same
E) Number of electrons is equal to number of protons at B) Physical properties are different
neutral state. C) Atomic numbers are same
D) Mass numbers are same
E) Proton numbers are same
52 6+
40. How many protons, neutrons and electrons do 24 Cr
have?
49. In a neutral atom
A) 24, 28, 18 B) 28, 18, 24 C) 52, 24, 30
A) The number of neutrons is equal to the number of
D) 30, 24, 52 E) 18, 28, 24
electrons
B) The number of protons is equal to the number of
electrons
41. 15P5+ and X1- have the same number of electrons. What C) The number of protons is equal to the number of
is the atomic number of X?
neutrons
D) The atomic number is equal to the mass number
A) 15 B) 19 C) 9 D) 20 E) 10 E) The atomic number is equal to the number of neutrons

42. Cr+3 has 21 electrons. What is the atomic number of 35


Cr? 50. Which one of the following is correct for 17 Cl ?
Protons Electrons Neutrons
A) 48 B) 23 C) 43 D) 21 E) 24 A) 17 18 17
B) 18 17 18
C) 17 17 18
127 a D) 18 18 35
43. 53 X has173 particles (p, n, e)totally. What is the
E) 17 18 35
numerical value of a?

A) –7 B) +1 C) +2 D) +3 E) +7
51. The atom with 26 electrons and 30 neutrons has an
atomic and atomic mass number of ..........
Atomic number Atomic mass number
44. If the atomic and the mass number of an atom are 50 A) 26 30
and 119 respectively. What is the number of neutrons in the B) 30 56
nucleus of the atom? C) 26 56
D) 30 26
A) 69 B) 50 C) 75 D) 119 E) 109 E) 26 46
45. What is the relationship between proton (p), neutron (n)
40
and electron (e) of the atom 20 Ca ? 52. The ions A-1, B-3, C+2, and D+4 are isoelectronic. Which
one has the largest atomic number?

5
60. Atoms of 126C and 146C has a difference of two
A) A B) B C) C D) D E) they are equal
A) electrons
B) protons
53. I. atomic number II. neutron number C) nucleus
III. electron number IV. mass number D) neutrons
Which one(s) of the above is (are) different for the isotopes
of an element?

A) I and II B) II and III C) II and IV


D) IV only E) II only

54.
Number of Number of
Element
Protons electrons
X 12 10
Y 8 10
Z 16 18
T 19 18

Which of the atoms X, Y, Z and T is (are) anions?

A) Y only B) X and Y C) Y and Z


D) Y, Z and T E) T only

55. One of the following is not true about a solid structure.

A) Atoms are arranged in row.


B) Atoms are vibrating forwards and backwards.
C) Atoms are held tightly together.
D) Atoms move about and change places.

56. The electron shell that is closest to the nucleus of the


atom is complete when it contains

A) one electron
B) eight electrons
C) two electrons
D) six electrons

57. What is the smallest particle of a compound?

A) electron B) atom C) molecule

58. The number of protons contained in the nucleus of an


atom is

A) atomic mass
B) atomic number
C) atomic weight

59. What is the smallest particle of an element?

A) electron B) atom C) molecule

6
CHEMICAL CALCULATIONS
1. How many moles of atoms are there in 1 mole of 9. I. Contains 1 mole of NH3 molecule
Fe4[Fe(CN)6]3? II. Contains 1 mole of nitrogen atom
A) 43 B) 23 C) 27 D) 38 E) 41 Ill. Contains 3 moles of hydrogen atom
Which one (s) is (are) true for 17 grams of NH3 molecule?
2. I. NO gas that contains 3.01x1024 nitrogen atoms (N: 14, H: 1)
II. 3.01x1024 molecules of NO gas A) I only B) II only C) land llI
Ill. NO gas that contains 3.01x1024 atoms D) I and II E) I, II and III
Which of the following is the correct order of the number
of moles of NO gas, according to the above statements? 10. The following numbers of protons, electrons and
A) l=ll>lII B) l=II=Ill C) l>II>Ill neutrons are given for one mole of hydrogen cation.
D) l=II<III E) II>I=III (1H+);
I. Has 6.02x1023 protons
3. I. 0.1 gram-formula of CaCl2 II. Has 1.204x1024 electrons
II. 0.2 gram - ion of SO4-2 Ill. Has 6.02x1023 neutrons
III. 0.3 gram - atom of Fe Which one(s) is (are) true? (H: 1)
IV. 0.4 g H2 A) I only B) II only C) Ill only
What is the correct order of the number of moles of the D) l and Ill E) I, II and Ill
atoms, the ions, and the compounds, according to the given
statements above? 11. What is the volume, in cm3, of 1.204x1022 CO2
(H: 1, O: 16, Ca: 40, Fe: 56, CI: 35.5, S: 32) molecules at STP?
A) III>II=IV>I B) I>II>III>IV C) II>I>III=IV A) 56 B) 112 C) 224 D) 448 E) 560
D) IV>III>II>I E) III>II=I>IV
12. What is the volume in liters of 3.2 grams of SO2 at
4. I. 23 grams of Na is 1 gram-atom of Na STP? (S: 32, O: 16)
II. 6.02x1023 CO2 molecules are 1 molecule-gram A) 0.56 B) 1.12 C) 2.24 D) 3.36 E) 11.2
III. 6.02x1023 oxygen molecules are 16 grams
Which of the statements given above is (are) true? 13. What is the mass, in grams, of 6.72 L of SO3 at
(Na: 23, 0:16, C: 12) STP? (S: 32, O: 16)
A) III only B) I and II C) I and III A) 8 B) 16 C) 24 D) 32 E) 40
D) II and III E) I, II and III
14. I. Atomic number II. Mass Ill. Volume
5. The following information is given for NO, of a gas at STP are given. At least which one(s) of the
I. 6.02x1023 NO molecules weigh 30 grams above statements is (are) required to find its mole number?
II. 30 grams of NO gas is 22.4 liters at STP A) I only B) Ill only C) I and Ill
III. There are 6.02x1023 atoms in NO gas, which occupies D) I and II E) II and III
22.4 liters at STP
Which one (s) of the statements given above is 15. What is the mass of C3H4 gas that contains
(are) true for the compound NO? (N: 14, O: 16) 1.204x1022 molecules? (C: 12, H: 1)
A) l only B) l and ll C) l and Ill A) 0.4 B) 0.8 C) 1.2 D) 2.4 E) 8
D) II only E) I, II and III
16. What is the mass of CaCO3 compound that has
6. I. PO4-3 II. SO4-2 Ill. NO3- 0.25x NA atoms? (CaCO3: 100, NA: 6.02x1023)
Compare the molar masses of the compounds that are A) 2.5 B) 5 C) 25 D) 50 E) 75
formed from the element Al with above anions. (Al: 27, P:
31, S: 32, N: 14, O: 16) 17. What is the volume of CH4 molecules that
A) II>III>I B) contain 6.02x1023 atoms at STP?
I=II>III C) III>I>II A) 1.12 B) 2.24 C) 3.36 D) 4.48 E) 22.4
D) III>I>III E) II>I>III
18. What is the mole number of Fe3O4 that contains
7. How many moles of atoms are there in 0.2 moles 28 gram-atom of oxygen atom?
of X2Y3? A) 4 B) 7 C) 14 D) 28E) 56
A) 0.2 B) 0.4 C) 0.8 D) 1 E) 1.4
19. How many gram-atom of oxygen are there in
8. Which one of the following does not occupy 22.4 19.6 g of H2SO4? (H: 1, O: 16, S:32)
liters at STP, if it is 1 mole? A) 0.2 B) 0.4 C) 0.6 D) 0.8 E) 1
A) He B) SO2 C) N2 D) H2O E) O2

7
20. How many moles of Y atoms are there in "a" number A) I only B) II only C) I and II
of X2Y5 molecules? (NA : 6.02x1023) D) I and Ill E) I, II and Ill
A) 5a/NA B)axNA/5 C) 5NA/a
D) 5NA E)a/5NA 31. I. He II. CH4 Ill. SO3
Which one of the following is the order of the given gases
21. What is the mass of CaSO4 that contains 0.04 above that have equal masses, according to their number of
moles of oxygen atoms? (Ca: 40, S: 32, O: 16) atoms?
A) 2.72 B) 1.86 C) 1.36 D) 0.68 E) 0.34 (He: 4, CH4: 16, SO3: 80)
A) lI>l>lll B) II>III>I C) I>II>III
22. What is the mass of 2.408x1024 molecules of D) III>I>II E) III>II>I
P4? (P: 31)
A) 31 B) 124 C) 248 D) 496 E) 620 32. Which one(s) of the following statements is(are)
correct for the CH4, NH3 and H2O molecules that
23. How many atoms are there inCH4 that contain equal number of hydrogen atoms
occupies 3.36 L at STP? I.H2O has the greatest volume at STP
A) 4.5x1023 B) 4.5x1022 C) 6x1022 II. CH4 has the smallest mass.
D) 3x10 23
E) 9x1023 III. NH3 has the greatest mole number.
(C: 12, H: 1, N: 14, O: 16)
24. How many atoms of hydrogen are there in 5.1 A) I only B) II only C) Ill only
grams of NH3? (N: 14, H: 1) D) I and Ill E) II and Ill
A) 1.204x1023 B) 1.806x1023 C) 1.204x1021
D) 5.418x1023 E) 1.806x1020 33. I.2.24 L of CO2 molecules at STP
II. CO2 molecule that contains 0.1 mol of atom
25. How many gram-atom of hydrogen are there in III. 0.1 mol of CO2 molecule
C3H4 that contains 21 moles of atom? Which one is the correct order according to the
A) 1 B) 3 C) 4 D) 8 E) 12 statements given above about masses of CO2?
(C:12, O: 16)
26. Which one of 10 grams of the compounds given A) l=II=Ill B) I>II>Ill C) I<II<Ill
below, has the largest volume at STP? (N: 14, O: D) I=III<II E) I=Ill>II
16)
A) NO B) N2O C) N2O3 D) N2O4 E) N2O5 34. There is 2.24 liters of C3H4 gas in the closed container
(constant volume) at STP.
27. Which one of the following contains 1 mole of I. 0.1 mol N2O
atom?(C: 12, H: 1, O: 16) II. 0.05 mol NO
A) 1 mol of CH4 III. 0.9 mol C3H4
B) 16 grams of O2 Which one(s) of the above(s) must be added to the
C) 22.4 L of CO2 at STP container in order to have a total number of 6.02x1023
D) 6.02x1023 molecules of CO atoms? (H: 1, O: 16, C: 12, N: 14)
E) 30 grams of C2H6 A) I only B) II only C) I and II
D) II and Ill E) I and Ill
28. What is the volume of NH3 gas, at STP, that
has the same number of hydrogen atoms as in 6 35. Which of the statement(s) below is (are)
grams of C2H6? (C: 12, H: 1, N: 14) correct?
A) 2.24 B) 4.48 C) 6.72 D) 8.96 E) 11.2 I. 1 gram-atom of hydrogen is 1 gram
II. 1 gram of hydrogen gas occupies 22.4 liter at STP
29. Which one of the following has the greatest III. 6.02x1023 hydrogen atoms weigh 1 gram
number of molecules in its structure?(C: 12, H: 1) A) I only B) II only C) I and II
A) 0.2 mol of C2H4 D) I and Ill E) I, II and Ill
B) 9 grams of C2H6
C) 6.02x1022 molecules of C2H4 36. I. 4 gram-atom of He gas
D) 11.2 L of CH4 at STP II. 4 grams of calcium atom
E) 19.5 grams of C2H2 III. 4 moles of hydrogen gas
Which one of the following is the right order according to
30. 3.6 grams of H2O. (H: 1, O: 16) the masses given above?
I. Contains 0.6 moles of atoms (He: 4, Ca: 40, H: 1)
II. Occupies 4.48 L of volume at STP A) I>III>II B) I>II>III C) III>lI>I
Ill. Contains 0.4 grams of hydrogen D) III>I>II E) I=ll<lll
Which one(s) is (are) correct?

8
37. I.1 liter of NO2 gas at STP I. Volumes at STP
II. 1 mole of NO2 gas II. Number of molecules
Ill. 1 molecule of NO2 gas Ill. Number of atoms
Which one is the correct order according to the A) I and II B) I and
masses given above for NO2 gas? (NO2: 46) III C) II and III
A) II>III>I B) II>I>III C) I=II=III D) III only E) II only
D) III>II>I E) I>II>III
46. Which one(s) of the statements given below
38. I. It has 11 moles of protons is (are) true for the molecules of NO, NO2 and N2O3
II. It has 10 moles of electrons which contain the same number of oxygen atoms?
Ill. It has 7 moles of neutrons I. NO has the largest volume at STP.
Which one(s) is (are) true for 1 gram-ion of NH4+ ? II. N2O3 has the greatest mass.
14 1
( 7 N , 1H ) Ill. N2O3 has the least number of moles.
(N: l4, O: 16)
A) I only B) II only C) I and ll A) I only B) II only C) I and II
D) I and III E) I, II and Ill D) II and Ill E) I and Ill
39. How many moles of protons are there in 1.6 47. C2H2(g), C3H4(g) and C2H6(g) compounds are formed
32 16
grams of SO2? ( 16 S , 8 O ) by using the same amount of C. According to this which
A) 0.4 B) 0.6 C) 0.8 D) 1 E) 1.2 one(s) can be deduced by using only the given data above?
I. Comparison of the number of moles
40. How many moles of neutrons are there in 15 II. Comparison of the volumes at STP
60 80 Ill. Comparison of the number of atoms
grams of CoBr3 ? ( 27 Co, 35 Br ) A) I only B) I and II C) II and Ill
A) 2.8 B) 4.2 C) 8.4 D) 10.2 E) 15 D) I and Ill E) I, II and Ill

41. I. N2O II. NO2 Ill. N2O4 48. I. X2O3 II. YO Ill. Z2O5
Which one is the right order according to the masses of the For which of one molecule of given compounds, the ratio
given compounds above that have the same moles of of mass of oxygen to the number of oxygen atoms is equal
oxygen atoms? (N:14, O:16) to the mass of 1 oxygen atom?
A) I>II=III B) I=II=III C) I>II>III (O: 16)
D) II>III>I E) I=III<II A) I only B) II only C) Ill only
D) I and Ill E) I, II and Ill
42. I. Contains 1 mol of oxygen in its structure
II. It is 21.6 grams 49. How many liters of volume does 6 grams of
Ill. Contains 1.204x1023 nitrogen atoms C3H4 occupy at the conditions where 6 grams of
Which one(s) is (are) correct for 0.2 moles of N2O5? (N: C2H6 occupy 6 liters volume? (C: 12, H: 1)
14, O: 16) A) 1.5 B) 3 C) 4.5 D) 6 E) 9
A) I only B) Il only C) I and II
D) II and Ill E) l, II and III 50. At the same conditions there are two elastic
balloons which are non permeable. 6 g of C2H6(g) is
43. I. It is 0.3 mol put into the first balloon and 6 g of C3H4(g) is put
II. It has 0.3 gram-atom of S into the second one. In order to equate the volumes
III. It has 14.4 grams of oxygen of the balloons, what should be done? (C:12, H:1)
Which one(s) is (are) correct for 24 grams of SO3 gas? (S: A) Adding 2 g of C2H6 to the first balloon
32, O: 16) B) Adding 2 g of C3H4 to the second balloon
A) I only B) II only C) I and ll C) Removing 2 g of C2H6 from the first balloon
D) I and III E) I, II and III D) Removing 4 g of C3H4 from the second balloon
E) Since their volumes are the same, there is no need to
44. I. H2S II. NH3 III. CH4 change anything.
Which one of the following is the right order of the gases
compared according to their number of atoms that are in 51. In a closed container, there is x grams of CO2 gas.
different containers and have equal number of molecules? Then x grams of C3H8 is added. By using this data:
A) I=II=III B) I>II>III C) III>II>I I. Mass is doubled
D) III>I>II E) I>III>II II. Number of moles is doubled
III. Number of atoms is doubled
45. For PH3 and H2S gases having the same Which one(s) of the statements above is (are) true? (C: 12,
masses, which one(s) is(are) the same? H: 1, O: 16)
(P: 31, H: 1, S: 32) A) I only B) II only C) I and II

9
D) II and III E) I, II and Ill III. Formula mass of Fe3O4
at least which statements above must be known?
52. 73.6 grams of XBr 2 is 0.4 moles. A) I only B) Ill only C) l and II
What is the molar mass of X? (Br: 80) D) II and III E) I and III
A) 24 B) 40 C) 36 D) 56 E) 64
61. Molecule Mass of X
53. 60 grams of X2S3 contains 21.6 grams of X. What is XO2 m1
the molar mass of X? (S: 32) X2O3 m2
A) 11 B) 18 C) 27 D) 33 E) 56 X2O5 m3
Compare the masses (m1, m2, m3) of atom X in
54. The total mass of X is 11.2 grams in both 0.2 each compounds and XO2, X2O3 and X2O5 which
moles of XY2 and 0.3 moles of X2Y3. contain equal amount of oxygen atoms? (O: 16)
What is the molar mass of X? A) m1>m2>m3 B) m1>m3>m2 C) m3>m1>m2
A) 14 B) 16 C) 31 D) 32 E) 56 D) m2>m1>m3 E) m2>m3>m1

55. 0.4 moles of X2CO3 compound are 42.4 grams. 62. In order to find the molar mass of X in X3H4
What is the molar mass of X? (C: 12, O: 16) compound.
A) 19 B) 23 C) 39 D) 46 E) 55 I. Molar mass of hydrogen
II. Formula mass of X3H4
56. 0.02 moles of X2S3 weighs 3 grams and 1 mole III. Mass of X3H4
of X2S3 contains 76 moles of neutrons. At least which statement(s) has (have) to be known?
I. It has 13 protons A) I only B) I and II C) I and III
II. It has 14 neutrons D) II and III E) I, II and III
III. Its molar mass is 54 amu.
Which one(s) is (are) true for one neutral X atom? 63. By using Pt electrodes, 7.2 grams of H2O is
32
( 16 S) electrolyzed. What are the mole numbers of the
produced O2 and H2? (H: 1, O: 16)
A) Il only B) l and Il C) I and Ill H2 O2
D) II and Ill E) I, II and III A) 0.4 0.2
B) 0.2 0.4
57. For 0.5 moles of SO2 gas, which has a molar C) 0.4 0.4
mass of 64 grams, the following information are given; D) 0.3 0.3
I. It weighs 32 grams E) 0.8 0.4
II. It occupies 11.2 liters at STP
III. It contains 3.01x1023 atoms 64. NO and H2O are produced when NH3 is burnt.
IV. It is 0.5 gram-molecule When 3.4 grams of NH3 is burnt, how many liters
Which one(s) is (are) true? of NO is produced at STP? (N: 14. H: 1, O: 16)
A) I, II and IV B) III and IV C) II, III and IV A) 1.12 B) 3.36 C) 4.48 D) 6.72 E) 11.2
D) I, II and III E) II and III
65. According to the following equation
58. The molar mass of X3H8 is equal to the molar C + HNO3  CO2 + NO2 + H2O
mass of XO2. Find the atomic mass of X. 2.4 grams C is reacted with concentrated HNO3
(H: 1, O:16) acid. How many liters of NO2 are produced at STP?
A) 12 B) 8 C) 23 D) 31 E) 14 (C: 12, Balance the equation first)
A) 5.6 B) 11.2 C) 16.8 D) 17.92 E) 22.4
59. 3.01x1022 molecules of X2Y3(g) weighs 5.1
grams. In order to find the mass of element X in the
66. How many grams of Pb are reacted with 8
compound grams of oxygen in order to produce Pb3O4?
I. Volume of X2Y3 at STP (Pb: 208, O: 16)
II. Formula mass of X2Y3 A) 104 B) 208 C) 52 D) 156 E) 78
Ill. Molar mass of Y
Which one(s) should be known? 67. In a closed container the following reaction take
A) I only B) II only C) III only places.
D) I and III E) II and Ill
N2 + 3H2  2NH3
To produce 25 moles of NH3, how many moles of
60. In order to find the total number of atoms in
H2 are needed?
Fe3O4 only whose mass is given
A) 12.5 B) 25 C) 37.5 D) 50 E) 645
I. Molar mass of iron
II. Molar mass of oxygen

10
68. KCl and O2 are produced due to the decomposition of A) I only B) II only C) I and II
1 mole of KClO3, by using heat. D) II and Ill E) I, II and Ill
How many liters of O2 are produced at STP?
A) 11.2 B) 16.8 C) 22.4 D) 33.6 E) 44.8 76. All of the HI in a closed container is
decomposed according to the equation,
69. According to the following equation. 2HI  H2 + I2
MnO2 + 4HCI  MnCI2 + 2H2O + Cl2 I. Total mass in the container (m)
To produce 11.2 liters of Cl2 gas at STP, how many grams II. Mole number of HI (n)
of MnO2 must be used? (Mn: 55, O: 16) III. Total number of moles in the container (nt),
A) 21.75 B) 43.5 C) 87 D) 174 E) 191 the m, n, nt versus time graphs are given below.

70. 1 mole of II.


I.
I. Na II. Zn III. Al
m n
metals are put into 3 different HCI solutions respectively.
Compare the mole number of H2 gases produced from the
2x 2x
reactions?
(HCI solutions are excess) x
A) I>II>III B) III>II>I C) I>IIl>ll
D) II>III>I E) I=II=III

71. If equal number of moles of the following gases time time


are burnt, which one needs the greatest amount of
O2, in moles? III.
A) CH4 B) C2H2 C) C2H4 D) C2H6 E) C3H4 nt

72. If 1 mole of CaCO3 is decomposed into its 2x


components x
I. 1 mol CaO and 1 mol CO2
II. 1 mol CO2 and 0.5 mol O2?
III. 1 mol O2 and 1 mol CaO
time
Which one(s) of the given above can be obtained? Which one(s) is (are) correct?
A) I only B) II only C) I and Ill A) I only B) II only C) III only
D) I and II E) I, II and Ill D) I and II E) I and III

73. Which one of the given choices doesn't 77. By the reaction of 11.2 grams of XOH and 4.4
produce 22.4 liters of H2 gas at STP, when 1 mole of it is grams of YO2, 13.8 grams of X2YO3 and some amount of
reacted with enough HCI? H2O are produced.
A) Mg B) Ca C) Zn D) Na E) Ba Find the molar masses of X and Y? (H: 1, O: 16)
X Y
74. X metal, whose mole number is only known, A) 39 12
reacts with HCI. To calculate the volume of H2 gas B) 23 14
produced at STP, which one(s) should be known at least? C) 12 39
I. Valency (number of valance electrons) of the D) 108 32
produced X metal E) 23 12
II. Mass of the produced salt
III. Temperature of H2 produced. 78. P4 + 5O2  2P2O5
A) I only B) II only C) I and II P2O5 + 3H2O  2H3PO4
D) II and Ill E) I and Ill How many grams of P4 must be used in order to have 0.4
mol of H3PO4 according to the above
75. When NH3 is burnt, NO and H2O are produced. reaction? (P: 31)
According to the data, which of the following(s) is (are) A) 3.1 B) 6.2 C) 12.4 D) 15.5 E) 18.6
true?
I. The mole number of formed NO is equal to
the mole of NH3 burnt. 79. 19.5 grams of Zn is reacted with excess HCI to
II. Equal mole numbers of NO and H2O are give H2 gas and the produced H2 gas reacts with enough
produced. N2. What is the mass of NH3 formed?
Ill. The mole numbers of reactants are equal to (Zn: 65, N: 14, H: 1)
the mole numbers of products for this reaction. A) 1.7 B) 3.4 C) 5.1 D) 6.8 E) 13.6

11
each other. If one of the reactants is used up.
80. 3 moles of SO2 gas react with 4 moles of O2. Which I. 0.6 mol of H2 is unused.
one is the excess and how many moles is it? II. 8.96 L of H2O is produced at STP.
A) 1 mol O2 B) 2.5 mol O2 C) 1.5 mol O2 III. 1.6 grams of O2 gas remains unreacted
D) 1 mol SO2 E) 2 mol SO2 Which one(s) of the statements given above is
(are) correct? (H: 1, O: 16)
81. 0.02 moles of C3H4 reacts with 2.408x1022 O2 A) I only B) II only C) III only
molecules. How many grams of CO2 are produced D) II and Ill E) I and II
after the reaction? (C: 12, O: 16)
A) 2.64 B) 0.22 C) 0.33 D) 0.44 E) 1.32 89. Equal masses of SO2 and O2 gases are
reacted to produce SO3 gas.
82. At the same conditions, 9 liters of N2 and 9 liters At the time when one of the gases is used up,
of H2 gases react in order to produce NH3. which one of the followings remains in the container?
Which gas will be excess and how many liters? A) SO2 only B) O2 only C) A mixture of SO3 and O2
A) 1 liter of N2 B) 3 liters of H2 C) 6 liters of N2 D) SO3 only E) A mixture of SO3 and SO2
D) 6 liters of H2 E) 4 liters of N2
90. For S and O2 in the equation below
83. Ammonia is formed by the combination of 1 mol S(s) + O2(g)  SO2(9)
of nitrogen and 3 moles of hydrogen. In order to get I. If x g of each of them are reacted, x g of SO2 is
4/3 moles of excess nitrogen from the reaction of 5 produced.
moles of hydrogen, how many moles of nitrogen are II. If x mol of each of them are reacted, x mol of SO2 is
needed? produced.
A) 1 B) 2 C) 3 D) 4 E) 5 III. If x L of each of them are reacted, x L of SO2 is
produced.
84. There is a mixture composed of 5 moles of Al Which one(s) is (are) correct given above?
and S atoms in a closed container. At the end of the A) I only B) II only C) I and II
reaction, there is 0.5 moles of molecules in the D) II and Ill E) III only
container. What is the mole number of the
unreacted atoms? 91. 2 mol of S and 2.5 mol of O2 are reacted in the
A) 0.5 B) 1 C) 2 D) 2.5 E) 7.5 closed container, according to the equations below.
S + O2 SO2
85. Equal volumes of N2 and H2 gases are reacted SO2 + 1/2O2  SO3
to give NH3 at the same conditions. After using all samples, which one(s) of the followings will
After one of the reactants is completely used V liter of NH3 be in the container?
gas is produced. Which one of the reactants remains A) 2 mol SO2
unreacted and how many V liters? B) 2 mol SO3
A) V/2 liter N2 B) V liter N2 C) V/4 liter H2 C) A mixture of 0.5 mol SO2 and 1.5 mol SO3
D) V liter H2 E) 2V liter N2 D) A mixture of 1 mol S02 and 1 mol SO3
E) A mixture of 1.5 mol SO2 and 0.5 mol SO3
86. 72 grams of CaS is formed by the reaction of 40
grams of Ca and a little amount of S. Which one(s) 92. In the compound CO2, the ratio of carbon to
of the following statements might be true for the oxygen by mass is 3/8. If 20 g of carbon is reacted
initial amount of S? (S: 32, Ca: 40) to 32 g of oxygen;
I. 1 mole I. 8 g of carbon is excess
II. More than 1 mole II. 44 g of CO2 is produced.
Ill. Less than 1 mole III. 9 g of oxygen is excess
A) I only B) II only C) Ill only Which statement(s) given above is (are) correct?
D) I and II E) I and Ill A) I only B) II only C) I and Ill
D) I and II E) II and III
87. 3 moles of CS2 and 5 moles of O2 gases are
reacted according to the following reaction
CS2 + 3O2  CO2 + 2SO2
In order to allow all the gases to react, how many moles of 93. When 3 L mixture of N2 and H2 gases are
which gas must be added? reacted to give NH3, 0.6 L of N2 remains
A) 3 mol O2 B) 10 mol O2 C) 2 mol CS2 unreacted. According to this,
D) 1 mol CS2 E) 4 mol O2 I. Volume of N2 is 1.2 L in the first mixture
II. Volume of H2 is 1.8 L in the first mixture
88. 2 grams of H2 and 0.2 moles of O2 react with Ill. 1.2 L of NH3 is produced after reaction.

12
Which one(s) is (are) given above correct? II. 224 L of air is consumed.
A) I only B) II only C) I and II III. 44.8 L of H2O is produced
D) II and III E) I, II and Ill Which statement(s) given above is (are) correct?
A) I only B) Ill only C) I and II
94. 6 L of SO2 and 4 L O2 gases are reacted to give D) II and Ill E) I, II and Ill
SO3. After reaction;
I. 6 L of SO3 is produced 98. In an elastic balloon, 4.48 L of SO2 and 3.36 L of O2
II. 1 L of O2 is excess gases are reacted to produce SO3 gas at STP. At the same
III. 4 L of SO2 is used. conditions,
Which one(s) is (are) given above correct? I. 1.12 L of O2 gas is excess
A) I only B) I and II C) I and III II. 4.48 L of SO3 gas is produced
D) II and III E) I, II and Ill Ill. The volume of the balloon is 5.6 L.
Which one(s) is (are) true?
95. S + O2  SO2 A) I only B) II only C) I and II
S + 3/2O2  SO3 D) II and Ill E) I, II and Ill
According to the equations given above,
I. If equal number of moles of S is used, equal 99. 2H2S + 3O2  2H2O + 2SO2
masses of SO2 and SO3 are produced. According to this equation, H2S and O2 with the amount
II. If equal masses of oxygen are used, equal given below
masses of sulfur are consumed. I. 0.1 mol H2S and 0.2 mol O2
III. If equal masses of sulfur are used, equal moles of SO2 II. 0.2 mol H2S and 0.5 mol O2
and SO3 are produced. III. 0.4 mol H2S and 0.7 mol O2
Which statements(s) given above is (are) correct? Which one of the following is the right order according to
A) I only B) Ill only C) I and II the mole number of oxygen, which remains unreacted?
D) II and Ill E) I, II and Ill A) I=II=III B) I>II>III C) III>II>I
D) II>I>III E) II>III>I

100. When 20 g of impure CaCO3 is reacted with sufficient


96. There is 2 mol of N2 gas in a closed container. HCI solution, 1.12 L of CO2 is produced at STP. What is
8 mol of H2 gas is added to the container slowly to the purity percentage of CaCO3?
get the reaction of N2 + 3H2  2NH3 (CaCO3: 100)
According to this reaction the graphs of A) 25 B) 50 C) 60 D) 75 E) 80
I. Mole number of NH3 produced
II. Mole number of N2 left in the container
III. Mole number of H2 added.
101. There is N2O4 gas in a closed container. When
I. II.
NH3 N2
50% of N2O4 is decomposed according to the reaction
I I below
4
N2O4 + heat  2NO2
4 In the container;
2
I. Mass is constant
II. Mole number is increased by 1.5
IIl. Molecule number is halved.
t time Which statement(s) given above is(are) correct?
t time
A) I only B) II only C) I and II
III. D) II and Ill E) Ill only
I H2
4
4
2

102. There are equal moles of Zn and KClO3 in different


Which graph (s) is (are) correct?t time containers of A and B respectively. Concentrated HCI is
A) I only B) I and II C) Il and III added to container A, and the container C is heated until
D) I and III E) I, II and Ill decomposing of all KCIO3. The gases, which produced
from containers, are collected in the container B.
97. 1 mol CH4 gas is burnt at STP
HCl
I. 22.4 L of CO2 is produced.

13
elements X which has 16 protons, 16 neutrons and Y
which has 8 protons, 8 neutrons. According to this. How
many "amu" are there in one molecule of each compound?
XY2 XY3
A) 64 80
B) 32 48
C) 32 64
D) 80 64
E) 48 32
Which gas will remain unreacted and what will be the 108. What is the empirical formula of the nitrogen
percentage by mole of that gas? oxide, if 7.6 g of nitrogen oxide sample contains 4.8 grams
A) 66.6% O2 B) 48% H2 C) 25% O2 of oxygen. (N: 14, O: 16)
D) 24% H2 E) 10% O2 A) N2O B) NO C) N2O3 D) N2O5 E) N2O4
103. When 50 % of 4.6 g of N2O4 is decomposed to 109. If molecular mass of XYO4 is equal to the
NO2, what is the total mole number of gases in the molecular mass of X2Y, what is the molar mass of
container after the reaction? (N: 14, O: 16) X?(O:16)
A) 0.01 B) 0.025 C) 0.05 D) 0.075 E) 0.1 A) 44 B) 40 C) 3 D) 48 E) 64

104. If 25% of 2 mol of COCI2 is decomposed into 110. General formula of a hydrocarbon that contains
CO and Cl2 in the container, what is the total mass of gases 10% hydrogen by mass is CnH2n-2. According to this
in the container? (C: 12, O: 16, Cl: 35.5) information, what is the value of "n" in CnH2n-2?
A) 198 B) 148.5 C) 99 D) 49.5 E) 44.5 A) 1 B) 2 C) 3 D) 4 E) 5
105. 50 % of COCI2 is decomposed in the closed 111. What is the empirical formula of a compound
container according to this equation: which has 50% S and 50% Cu by mass in its structure?
COCl2  CO + Cl2 (Cu: 64, S: 32)
Which graph shows how the total mole numbers of gases A) CuS B) Cu2S C) CuS2 D) Cu2S3 E) Cu4S
change versus time? n
n 2x
A) B) 112. Which one of the following metals reacts with 3
gram - atom of oxygen to form 1 gram-formula
compound?
3x
A) AI B) Ca C) Na D) Mg E) K
2x
113. X and Y elements form two compounds;
n n time 1. compound : 7 g of X with 89 g of Y
time
C) D) 2. compound : 14 g of X with 445 g of Y
4x If the formula of the first compound is XY, which
2x one may be the formula of the second compound?
2x A) XY2 B) X2Y C) X2Y2 D) X2Y5 E) X3Y2
x
114. X: is a base which has 2 hydroxides and its 1
time time mole weighs 74 g.
Y: is an acid which is a monoprotic acid and its
n 1 mole weighs 63 g.
E) What is the mass of 1 mole of the salt formed by the
4x reaction of X and Y? (H: 1, O: 16)
A) 137 B) 119 C) 164 D) 193 E) 182

106. The mass of2xoxygen in 15 g nitrogen oxide is


8 g. What is the empirical formula of the compound? 115. When we consider two compounds of nitrogen
(N: 14, O: 16) time and oxygen; first compound contains 7 g of N and 8 g of
A) NO B) N2O C) NO2 D) N2O3 E) N2O5 O, second compound contains 7 g of N and 12 g of O. If
the first compound is NO, which one is the formula of the
107. XY2 and XY3 compounds are formed from second compound?
A) NO2 B) N2O C) N2O3 D) N2O4 E) N2O5

14
116. 122. Molar masses of some molecules are given
ny
ny below;
9 AC2: 44, B2C: 18 C2: 32
6 8 According to this information, what is the molecular mass
3 6
4 of A3B8 in the reaction below?
2 A3B8 + 5C2  3AC2 + 4B2C
A) 30 B) 40 C) 44 D) 50 E) 62
nx 1 2 3 123. Burning 1 mole of a compound C3Hx(OH)y with
1 2 nx
4 enough oxygen, gives 4 moles of water. If only this
3
Variation of X to Y by mass is given in the figure. information is known, which one of the following can be
According to this information, what are the formulas of 1st calculated?
and 2nd compounds? I. Values of X and Y in the compound
II. Mole number of CO2 produced.
First compound Second compound Ill. Mole number of O2 consumed.
A) XY3 XY2 A) I only B) II only C) I and II
B) X2Y XY3 D) II and III E) I, II and Ill
C) XY2 XY3
D) X2Y3 XY 124. The number of protons is equal to the number
E) XY3 X2Y3 of neutrons of element X whose the atomic number is 20.
The number of neutrons is by 10 more than the number of
117. 0.1 mole of X2O compound weighs 4.4 grams. protons in element Y whose the atomic number is 35. What
0.1 mole of X2On compound weighs 10.8 grams. is the percentage of Y by mass in the compound formed by
According to this, what is the value of n in X2On X and Y?
compound? (O: 16) A) 20 B) 40 C) 60 D) 65 E) 80
A) 1 B) 2 C) 3 D) 4 E) 5
125. Molecular masses of X2Y3 and X2Z3 are 150g and 102
118. 0.25 moles of O2 is used to burn 0.1 mole g respectively. According to this;
compound of C2Hx(OH)2. According to this information, I. Atomic mass of Y is greater than Z.
what is the value of x in the compound of C2Hx (OH)2? II. The ratio of Y to Z by atomic mass is 150/102.
A) 2 B) 3 C) 4 D) 5 E) 6 Ill. The percentage of X by mass in the compound of X2Z3
is more than that of X2Y3.
119. CaCO3  X + CO2 Which one(s) is (are) exactly true?
X + 3C  Y + CO A) I only B) Ill only C) I and Il
Y + 2H2O  Z + Ca(OH)2 D) I and III E) I, Il and Ill
Which one can be Z in the reaction above?
A) C2H4 B) H2CO3 C) C2H5OH D) C2H2 E) C2H6 126. There are 4 g of X in 5.6 g compound formed by
X and Y elements. If this information is only known;
120. By taking X grams from the compound of
I. Empirical formula of the compound
CuSO4 nH2O, the water is evaporated completely. To find II. The percentage of X by mass in the compound
the value of n in the compound, which one(s) should be III. Molecular mass of the compound.
known? Which one(s) of the above can be calculated?
I. Molar mass of H2O A) I only B) II only C) I and II
II. Molar mass of CuSO4 D) II and Ill E) I, II and III
Ill. Mass of CuSO4 after evaporation 127. The following gases C3H4, C2H2 and CH4 each
A) I only B) II only C) III only of which contain 1 mole of hydrogen atoms are mixed.
D) I and II E) I, II and III What is the total mass of the mixture?
(C: 12, H: 1)
121. There are 3.2 g of X in 8 g of compound A) 27 B) 82 C) 41 D) 54 E) 39
formed from X and Y elements. Which of the following
can be calculated with this information only? 128. If a mixture of NO, NO2 and N2O4 gases which
I. Molecular mass of the compound. have equal numbers of oxygen atoms is 0.7 mole.
II. Percentages of the elements by mass in the What is the mass of the mixture? (N: 14, O: 16)
compound. A) 4.6 B) 9.2 C) 7.8 D) 15.6 E) 30.4
Ill. Empirical formula of the compound.
A) I only B) II only C) Ill only 129. 11.2 L mixture of He and O2 gases weighs 6.2
D) I and Il E) II and Ill

15
g at STP. What is the percentage by mol of O2 gas in the contains 15 moles of each of N2 and H2 gases, reacts to
mixture? (He: 4, O: 16) produce NH3. At the end of the reaction, 20 moles of gases
A) 10 B) 15 C) 30 D) 60 E) 72 remained in the container. What is the mole number of
NH3 in this container?
130. 2.5 moles of a mixture composed of X3 H4 and A) 5 B) 10 C) 12 D) 15 E) 20
X2H6 , which have equal number of hydrogen, weigh 90
grams. What is the atomic mass of X? (H: 1) 138. 1.9 moles of oxygen is used to burn 0.5 moles
A) 6 B) 7 C) 12 D) 14 E) 55 of a mixture of C3H8 and C2H4 gases. How many
moles of C2H4 is there in the mixture?
131. X g of CH4 gas is added into a container that A) 0.1 B) 0.15 C) 0.2 D) 0.25 E) 0.3
has X g of SO3 gas. For the gas mixture,
I. The number of mole is increased by 6 times 139. 0.9 moles CH4 and C2H4 gas mixture is burnt with 2.2
II. The mass is increased by 2 times moles of O2. What are the moles of CH4 in the mixture?
III. The number of atoms is increased by 5 times A) 0.2 B) 0,4 C) 0,5 D) 0.6 E) 0.7
Which one(s) of the statements given above is (are)
correct? (CH4: 16, SO3: 80) 140. 0.5 moles of a mixture of C2H2 and C2H4 gases
A) I only B) II only C) I and II is saturated with 0.8 moles of H2 gas to obtain C2H6. What
D) Il and III E) I, II and III is the mole number of C2H2 in the mixture?
A) 0.1 B) 0.2 C) 0.25 D) 0.3 E) 0.4
132. Equal masses of C2H6 and X gases are mixed. If 15.68
liters of the mixture weighs 24 grams at 141. 6 liters of a mixture of CH4 and CS2 gases at
STP, what is the molecular formula of X? STP is burnt with enough oxygen. When initial conditions
(C: 12, H: 1) are obtained after the reaction, 11 liters of product is
A) C2H2 B) C2H4 C) C3H4 formed. How many liters of CH4 are there in the mixture?
D) C3H6 E) C4H8 A) 4 B) 3.5 C) 2.5 D) 2 E) 1

133. A mixture of He-SO2 and CO gases occupies 142. A mixture of equal moles of SO2 – H2 –O2
16 liters. First, the mixture is passed through a base gases is placed in a closed container and reacted with each
solution. The volume decreases by 4 liters. Then, other. For this reaction, which one(s) of the following
remaining mixture is burnt. If 4 liters O2 gas is used during statements is (are) correct?
this process, how many liters of He gas are there in the I. All of the substances are consumed.
mixture? II. H2O and SO3 are produced.
A) 4 B) 6 C) 8 D) 10 E) 12 III. Number of molecules in the container decreases
IV. The mass decreases
134. When 4.48 L of a mixture of CO and SO2 A) I and IV B) I, II and IV C) I,III and IV
gases at STP is passed through NaOH solution, volume D) II and IV E) I, II and III
decreases to 1.12 liters. According to this information, the
mixture is; 143. 6 liters of each of NH3 and CI2 gas mixture react
I. 25% CO by mole according to the equation given below at constant
II. 75% SO2 by volume temperature.
III. 12.72% CO by mass 2NH3(g) + 3CI2(g)  N2(g) + 6HCI(g)
Which one(s) of the statements given above By this equation which one(s) of the following statement(s)
is(are) true? (C: 12, O: 16, S: 32) is (are) correct?
A) I only B) I and II C) I and III I. Mixture turns into products completely
D) Il and Ill E) I, II and Ill II. Total volume of gases after the reaction is 14 liters
III. 2 L of NH3 gas remains unreacted
135. 20% by volume of a mixture of C2H4 and A) II only B) Ill only C) I and III
C3H8 gases are C2H4. How many liters of O2 are D) I and ll E) II and Ill
needed to burn 5 liters of this mixture?
A) 12 B) 15 C) 20 D) 23 E) 25 144. The ion Fe+3 has 23 electrons and 30 neutrons. What is
the atomic mass of Fe atom?
136. 20 L of a mixture of N2 and H2 gases is A) 23 B) 26 C) 30 D) 56 E) 49
reacted completely to give NH3.
What is the initial volume of N2? 145. What is the molecular weight of (NH4)3PO4 ?
A) 3 B) 4 C) 5 D) 10 E) 12 (N: 14, H: 1, P: 31, O: 16)
A) 54 B) 113 C) 149 D) 172 E) 108
137. A mixture in a closed container which
146. What is the volume of 0.25 mol SO3 gas

16
at STP ? A) 0.5 B) 0.75 C) 0.89 D) 1.5 E) 2
A) 56 L B) 22.4 L C) 44.8 L D) 2.24 L E) 5.6 L
161. The mass of the mixture of 0.4 mol of X2O and 0.2
147. How many atoms are there in 0.1 mol of Ca(NO3)2 ? mol of XO2 is 26.8 g. What would be the atomic weight of
A) 9x6.02x1023 B) 6.02x1023 C) 0.9x6.02x1023 X? (O: 16)
D) 90x6.02x1023 E) 9/(6.02x1023) A) 28 B) 24 C) 14 D) 12 E) 7

148. What is the mass of 0.2 mol of Ag? (Ag: 108) 162. If 0.1 mol of a sample of XS2 weigh 7.6 g what are
A) 216 B) 2.16 C) 108 D) 10.8 E) 21.6 the mass of single X atom? (S: 32)
A) 2x10-23 B) 12.6x10-23 C) 12
149. What is the mole number of 3.01x1023 Fe atom? D) 76 E) 12x6.02x1023
A) 5 B) 50 C) 0.05 D) 0.5 E) 2
163. 0.1 mol of N2On is 10.8 g. What is the value of n? (N:
150. What is the mole number of 6.4 g S atom? (S: 32) 14, O: 16)
A) 0.2 B) 2 C) 0.5 D) 5 E) 0.02 A) 1 B) 2 C) 3 D) 4 E) 5

151. 0.02 mol of Y2O5 weighs 2.16 g. What is the molecular 164. 14.3 g a sample of XI4 is 0.02 mol. What must be the
weight of the compound? atomic weight of X? (I: 127)
A) 10.8 B) 108 C) 216 D) 1.08 E) 21.6 A) 32 B) 71.5 C) 103.5 D) 143 E) 207

152. How many molecules are there in 5.6 L CO2 at STP? 165. In 0.4 mol of iron oxide, there are 1.2 mol of Fe and
A) 6.02x1023 B) 1.05x1022 C) 1.5x1023 1.6 mol of oxygen atoms. So the formula of the iron oxide
D) 6.02x1021
E) 5.6x1023 must be………
A) FeO B) Fe2O C) Fe2O3 D) Fe4O3 E) Fe3O4
153. What is the volume of 6.8 g of NH3 at STP?
A) 22.4 L B) 8.96 L C) 2.24 L D) 89.6 L E) 0.89 L 166. 3-mol mixtures of SO2 and CO gases weigh 120 g.
What is the mole number of SO2 in the mixture? (C: 12,
154. What is the mass of one iron atom? (Fe: 56) O: 16, S: 32)
A) 56/(6.02x1023)g A) 1 B) 2 C) 3 D) 0.5 E) 1.5
B) 56x6.02x1022 g
C)6.02x1023/56 g 167. What is the volume of 1.2x1024 H2 molecules at
D) 1/(6.02x1021 )g STP?
E) 1x6.02x1023 g A) 17.92 B) 44.8 C) 22.4 D) 11.2 E) 6.72

155. How many moles of atoms are there in 0.2 moles of 168. 9.5 g a sample of X2O3 gas has a volume of 2.8 L at
X2Y3? STP? What is the atomic weight of X? (O: 16)
A) 0.2 B) 0.4 C) 0.8 D) 1 E) 1.4 A) 7 B) 28 C) 2.8 D) 14 E) 0.7

156. What is the volume in liters of 3.2 grams of SO2 at 169. What is the empirical formula of a compound
STP? (S: 32, O: 16) consisting of 7.8 g of K, 7.1 g of Cl and 9.6 g
A) 0.56 B) 1.12 C) 2.24 D) 3.36 E) 11.2 of O ? (K: 39, Cl: 35.5, O: 16)
A) KClO2 B) KClO C) KClO3 D) KClO4 E) K3ClO
157. What is the mass of 2.408x 1024 molecules of P4? (P:
31) 170. 20 g of XO3 (g) occupies a volume of 5.6 L at
A) 31 B) 124 C) 248 D) 496 E) 620 STP. What is the atomic weight of X? (O: 16)
A) 8 B) 16 C) 32 D) 64 E) 128
158. 73.6 grams of XBr 2 is 0.4 moles. What is the molar
171. A compound with a known molecular weight of 28,
mass of X? (Br: 80)
contains 85.71% carbon and 14.29% hydrogen. What is the
A) 24 B) 40 C) 36 D) 56 E) 64
molecular formula of the compound?
A) CH2 B) C2H4 C) C3H8 D) C2H2 E) CH
159. How many grams of water must be produced by the
reaction of 2 mol of H2 with 3.01x1023 oxygen molecules?
(H: 1, O: 16)
A) 4.5 B) 9 C) 18 D) 27 E) 36 172. 0.01 mol XBr3 weigh 2.67 g. What is the atomic
weight of X? (Br: 80)
A) 27 B) 19 C) 12 D) 40 E) 52
160. What would be the density, at STP, of the mixture
obtained by the mixing of 0.2 mol of O2 and 9.6 g of CH4
gases? (O: 16, C: 12, H: 1)

17
173. When propyne gas, C3H4, burns in air carbon A) 2n B) 3n C) (2n+2)/2 D) (3n+1)/2 E) 3n+1
dioxide and water are produced. How many moles of
C3H4 must be burned to produce 1.2 mol of CO2? 183. To 8 g of CH4 gas some C2H6 was added. If in the
A) 0.1 B) 0.2 C) 0.3 D) 0.4 E) 0.05 mixture there are 4 mol H atom what would be the mass of
C2H6 added? (C: 12, H: 1)
174. A liquid weighing 0.4 g was vaporized and found A) 5 B) 7.5 C) 10 D) 15 E) 20
to have a volume of 112 mL at STP. What is the
molecular weight of that liquid? 184. Equal masses of He and CH4 gases were mixed. What
A) 20 B) 40 C) 60 D) 75 E) 80 was the mol percentage of He in the mixture? (He: 4,
175. What would be the volume in L of 96 g of SO2 CH4: 16)
at STP? A) 20 B) 40 C) 50 D) 60 E) 80
A) 33.6 B) 11.3 C) 3.36 D) 22.4 E) 11.2 185. In a mixture of He-CH4 the number of He atoms is
equal to the number of H atoms. According to this what
176. Which would be the simplest formula of the would be the percentage CH4 by mass in the mixture? (He:
compound that consists of 75% by mass of carbon 4, CH4: 16)
and 25% of hydrogen? A) 12.5 B) 25 C) 33 D) 50 E) 67.5
A) CH B) CH2 C) CH3 D) CH4 E) C2H5
186. What would be the molecular formula of the
177. The following information is given for three compound whose density at STP and general formula are
compounds; 0.8 g/L and CnH2n?
I. 1 mol of MX is 72 grams. A) C2H4 B) C3H6 C) C4H6 D) C5H10 E) CH2
II. 0.1 mol of MY2 is 21.6 grams.
III. 3.01x1023 molecules of M2X3 is 80 grams. 187. I. 1 mol of XYZ4 is 120 grams.
According to these what are the atomic weights of X, II. 20% of this compound by mass is X.
Y and M? III. In this compound, mass of Z is twice of Y.
M X Y According to these what are the atomic masses of X, Y and
A) 56 16 80 Z?
B) 28 16 40 X Y Z
C) 28 32 80 A) 12 16 32
D) 56 32 40 B) 12 32 16
E) 42 24 60 C) 24 32 16
D) 24 16 32
178. In a 14.8 gram sample of M(OH)2 , there are 6.4 gram E) 24 32 48
of oxygen. What is the atomic weight of M? (O: 16, H: 1)
A) 24 B) 32 C) 40 D) 56 E) 74 188. 0.5 mol of XH3 is 8.5 gram. What is the mass of 5.6
L X2 gas at STP? (H: 1)
179. Equal masses of X and oxygen elements react with A) 1.75 B) 3.5 C) 7 D) 10.5 E) 14
each other and form XO3. If in this reaction 1/3 of X were
in excess, what would be the atomic weight of X? (O: 16) 189. I. The mass of 1 mol of XH3 and 1 mol of Y3H4 is 57
A) 6 B) 24 C) 32 D) 48 E) 64 grams.
II. The ratio of the atomic mass of X to that of Y is 7/6
180. At the same conditions, 100cm3 A2 gas reacted with According to these, what would be the atomic masses of X
200 cm3 B2 gas. As a result of reaction, 80-cm3 product and Y? (H: 1)
was formed and 20 cm3 A2 remained unreacted. What
would be the equation of this reaction? X Y
A) A2 + 2B2  A2B2 A) 7 6
B) 2A2 + 4B2  A4B8 B) 10 14
C) 2A2 + 5B2  2A2B5 C) 12 14
D) A2 + 5B2  2AB5 D) 14 12
E) 4A2 + 5B4  2A4B5 E) 24 18

181. What is the ratio of carbon atoms to the hydrogen in 190. Equal volumes of N2 and H2 gases react with each
the mixture of 0.2 mol of CH4 and 0.4 mol of C2H6? other to form NH3 gas. At same conditions if the volume of
A) 1/10 B) 5/16 C) 3/10 D) 10/3 E) 16/5 NH3 formed and excess gas is 16 L, what would be the
volume of gases at initial?
182. When the following reaction is balanced what would A) 24 B) 28 C) 32 D) 36 E) 48
be the coefficient of O2?
CnH2n+2 + O2  CO2 + H2O 191. I. 0.25 mol of an organic compound is 15.5 gram.

18
II. When 0.2 mol of it is burnt 0.4 mol of CO2 and 0.6 mol 202. What is the mass in grams of 1.12 L of CH4 gas
of H2O are produced. at STP?
What would be the molecular formula of the compound? A)
A) C2H6 B) C4H14 C) C4H3O D) C3H10O E) C2H6O2 0.8 B)
8 C) 80
192. The volume and the mass of the mixture of NO and D) 80 E) 400
N2O3 gases are 6.74 L at STP and 13.6 gram, respectively.
What would be the mole number of oxygen atoms in the 203. What is the molar mass of glucose, simple sugar? (C:
mixture? (NO: 30, N2O3: 76) 12, H: 1, O: 16)
A) 0.5 B) 0.6 C) 0.9 D) 1 E) 1.2 A) 18 B) 36 C) 90 D) 180

193. The mass of 5.6 L of a gas at 0oC and under a 1atm 204. How many moles are 4.4 grams of CO2? (C: 12, H: 1,
O: 16)
pressure is 11 g. What would be the mass of one molecule
A) 0.05 B) 0.01 C) 0.1 D) 0.2
of this gas?
A) 11/(6.02x1023) B) 44/(6.02x1023) C) 44 205. How many liters at STP are 6.4 grams of SO2? (S:32,
D) 11x6.02x1023 E) 44x6.02x1023 O: 16)
A) 2.24 B) 4.48 C) 22.4 D)
194. 7/19 of a nitrogen oxide compound is nitrogen 44.8
by mass. What is the formula of the nitrogen oxide?
(N: 14, O: 16) 206. What is the mass in grams of 0.2 mol of NH3? (N: 14,
A) NO B) N2O C) NO2 D) N2O3 E) N2O5 H: 1)
A) 0.17 B) 1.7 C) 3.4 D) 34
196. How many moles are 8.8 g of CO2 gas?
(C: 12, O: 16) 207. What is the number of atoms in 0.1 mol of CH4?
A) 5 B) 6.02x1023 C) 3.01x1023 D)
A) 0.1 B) 0.2 C) 0.3 D) 0.4 E) 2
3.01x1022
197. What is the mole number of 10.26 g of Al2(SO4)3? 208. C3X4, which occupies 11.2 L volume at STP, weighs 20
(Al2(SO4)3:342) g. What is the atomic mass of X?
A) 0.02 B) 0.03 C) 0.04 A) 0.1 B) 1 C) 12 D) 20
D) 0.3 E) 3
“Answer questions 209-211 according to following chemical
198. What is the volume in L of 0.1 mol of SO3 gas at reaction.”
STP? Cr2O3 + 2Al → 2Cr + Al2O3
A) 22.4 B) 11.2 C) 2.24
D) 0.224 E) 1.12 209. What is the mass of chromium produced if 2.7 grams of
aluminum is used? (Cr: 52, Al: 27, O: 16)
A) 5.2 B) 52 C) 10.4 D) 104
199. What is the mass of He gas that contains
6.02x1024 atoms in its structure? (He: 4) 210. How many grams of Cr2O3 are needed to produce 10.2
A) g of Al2O3?
80 B) A) 304 B) 152 C) 15.2 D) 5.1
60 C) 40
D) 4 E) 0.4 211. What is the number of molecules produced by the
reaction of 0.5 mol of Al with excess Cr 2O3?
200. How many moles of atoms are there in 0.3 moles 1 23 1 23
of O3 gas? A) 0.5   6.02  10 B) 1   6.02  10
2 2
A)
0.9 B) 1 23 1 22
C) 0.5   6.02  10 D) 0.5   6.02  10
0.3 C) 0.2 3 2
D) 3.01x1023 E)
1.5x10-23 “Answer questions 212-214 according to following chemical
reaction.”
201. What is the total mole number of atoms in 0.5 2Cu2S + 2O2 → 4Cu + 2SO2
mol of C6H12O6? 212. If 44.8 L of O2 at STP is used with excess Cu2S, how
A) many grams of Cu are produced? (Cu: 63.5, S: 32, O: 16)
12 B) A) 2.54 B) 30.8 C) 61.6 D) 254
6 C) 3
D) 1.2 E) 0.3 213. How many liters of O2 gas at STP are necessary in
order to obtain 2.24 L of SO2?
A) 44.8 B) 22.4 C) 11.2 D) 2.24

19
214. Which of the following gives the mass of Cu2S used to
obtain 3.2 g of SO2?
32 3.2 2 64 3.2 2
A)   B)  
159 64 2 159 32 2
32 3.2 1 32 3.2 2
C)   D)  
159 64 2 127 64 2

215. Which of the following balanced reaction equations


best represents the chemical change given below?
“Calcium metal reacts with water and produces calcium
hydroxide and hydrogen gas.”
A) Ca + H2O → Ca(OH)2 + H2
B) Ka + 2H2O → Ka(OH)2 + 2H2
C) Ca + 2H2O → Ca(OH)2 + H2
D) Ka + 2H2O → Ka(OH)2 + H2

216. What is the atomic weight of the element in Y2O5,


whose 0.02 mol weighs 2.16 g? (O: 16)
A) 108 B) 54 C) 27 D) 14

217. What is the percentage, by mass, of H in C3H4? (C:12,


H: 1)
A) 10 B) 20 C) 40 D) 90

218. What is the empirical formula of the compound which


consists of 66% Zn, 32% O, and 2% H by mass? (Zn: 66, O: 16,
H: 1)
A) ZnOH B) Zn(OH)2 C) Zn2OH D)
ZnHO2

20
CHEMICAL REACTIONS
I. Soluble in alcohol 6. I. Physical properties
II. Number of Protons
II. Evaporates at 78oC
III. Number of molecules
Ill. Flammable with O2
Which one(s) may change after a chemical reaction?
Properties of X are given above.
A) I and II B) I, II and Ill C) II only
Which one(s) is/are chemical process?
D) I and Ill E) III only
A) I only B) II only D) I and II
C) Ill only E) II and Ill
7. N2(g) + 3H2(g)  2NH3(g)
In the reaction above which one(s) is/are conserved?
1. I. H2O(l)  H2O(s) I. Number of moles
II. Ca  Ca+2 + 2e II. Number of atoms
Ill. 2Al + 3S  Al2S3 Ill. Total volume
Which one(s) is/are chemical and physical A) I only B) II only C) I and II
events? D) II and Ill E) I, II and III
Chemical Physical
A) I, II III 8. What will be the coefficients of NaOH, AI(OH)3
B) I, III II and Na2SO4 respectively, if we accept the coefficient
C) I II, III of Al2(SO4)3 as 1 for the below reaction?
D) II I, II
Al2(SO4)3 + NaOH  AI(OH)3 + Na2SO4
E) II, III I
A) 6, 2, 3 B) 3,1, 2 C) 1,3,4
238
D) 3,2,6 E) 6,1,4

234 4
2. I. 92 U 90 Th + 2 He + Energ
II. Na + Energy  Na+1 + e- 9. CaCO3  X + CO2
Ill. H2O(g)  H2O(s) + Energy X + 3C  Y + CO
Classify the changes above as radioactive, chemical Y + 2H2O  Z + Ca(OH)2
and physical change what is the formula of Z?
Radioactive Chemical Physical A) C2H4 B) H2CO3 C) C2H5OH
A) l III II D) C2H2 E) C2H6
B) III Il I
C) I II III 10. 2X + 5/2O2  2NO + 3H2O
D) II I III What is the formula of X?
E) II III I A) NH3 B) N2H4 C) N2 D) H2 E) NO2

3. If an atom reacts chemically, which one(s) will 11. R - C = O + 7/2O2  3CO2 + 3H2O
change |
I. Number of neutrons OH
II. Number of electrons Which one is the R, below?
Ill. Charge of nucleus A) CH3 B) CH4 C) C2H5
A) I only B) II only C) Ill only D) C2H6 E) C3H7
D) I and II E) II and Ill
12. 1C3HX(OH)3 + 7/2O2  CO2 + H2O
4. 3MgO + 2H3PO4  Mg3(PO4)2 + 3H2O What is the value of x in C3HX(OH)3?
Considering the reaction above, which of the given A) 3 B) 4 C) 5 D) 6 E) 7
quantities change?
I. Total mass 13. 6FeSO4+2HNO3+3H2SO43X + 2NO+ 4H2O
II. Number of atoms What is the oxidation number of S in the compound
Ill. Number of moles X?
A) Ill only B) II only C) I and II A) –2 B) +2 C) +3 D) 4 E) +6
D) I and Ill E) II and Ill
14. NO-3 + 2H+ + e-  X + H2O
5. I. Mass is conserved. Which one is the X, below?
II. Mole number is conserved. A) NO B) N2O C) NO2
Ill. Number and kind of atom is conserved. D) N2O3 E) N2O4
Which one(s) is/are exactly true for a chemical
reaction? 15. P2O5 + 3H2O  2X
A) I only B) Ill only C) I and Ill What is the oxidation number of P in the X?
D) I and II E) I, II and III

21
A) –2 B) +2 C) +3 D) +4 E) +5 In a balanced equation, which one of the statements
above expresses the coefficient ratios of reactants
16. Under the same conditions, from the reaction of 2 and products?
volume of X2 and 8 volumes of Y2, 4 volumes of Z is A) I only B) Ill only C) I and II
formed and 2 volumes of Y2 remains unreacted, what D) I and III E) I, II and III
is the formula of Z?
A) XY3 B) X2Y C) X2Y3 D) XY E) X2Y2 22. 5 liters of X2 and 6 liters of Y2 gases at the same
condition are reacted to form XY3. Considering this
17. l. CH4+ O2  CO2 + H2O reaction.
II. Al + O2  Al2O3 I. 3 liters of X2 remains unreacted.
Ill. Al2(SO4)3 + Ca  CaSO4 + Al II. 4 liters of XY3 is formed.
When the equations are balanced, which of them have the Ill. Gases are used up completely
same total number of coefficients in both sides? which of the statements is(are) correct?
A) I only B) I and II C) I and III
A) I only B) II only C) Ill only
D) II and Ill E) III only
D) I and II E) I and Ill
23. One mole of I. Na II. Zn Ill. Al metals are
18. CaO + H2O  X added to the solutions of HCI. Which of the below
X + CO2  Y + H2O gives the correct relation between the number of
for the compounds of X and Y given in the reactions
moles of evolved H2 in each reaction?
I. Compound of X is Ca(OH)2 A) I>II>III B) III>II>I C) I>III>II
II. The color of solution of X turns blue, if turnsole dye D) II>III=I E) I=II=III
is added.
III. Compound Y is CaCO3. 24. Which one of the following is not an example for
Which of the statements is(are) correct? acid-base reactions?
A) I only B) II only C) I and II A) 2KOH + H2SO4  K2SO4 + 2H2O
D) I and Ill E) I, II and III B) Na2O + 2HNO3  2NaNO3 + H2O
C) CaCO3 + 2HCI  CaCI2 + CO2 + H2O
19. 2Al(s) + 6HCl(l)  2AICl3(s) + 3H2(g) D) Mg + 2HCI  MgCl2 + H2
For the reaction, the following statements are given:
E) NH3 + HCI  NH4CI
I. 2 atoms of Al react with 6 molecules of HCI
II. While 2 moles of AICI3 is formed, 67.2 liters 25. I. NH3 + H+  NH4+
of H2 gas is evolved at STP. II. Ca+2 + Ba  Ba+2 + Ca
Ill. By the reaction of 2 moles of Al, 2 moles of III. CH3COOH + SO3-2  CH3COO- +HSO3-1
AICI3 and 3 molecules H2 forms. which of the above is(are) acid-base reaction?
Which of the statements is (are) true? A) I and III B) I and II C) I only
A) I only B) II only C) Ill only D) II and III E) I, II and III
D) I and II E) I and III
26. I. CaCO3  CaO + CO2
20. Consider the reaction: II. KCIO3  KCI + 3/2O2
3MgO(s) + 2H3PO4(l)  Mg3(PO4)2(s) + 3H2O(l) Ill. HgO  Hg + 1/2O2
Which of the following is a common name for the
I. 2 mol of H3PO4 reacts with 3 mol of MgO above reactions?
II. 1 volume of Mg3(PO4)2 and 3 volumes of A) Synthesis B) Analysis C) Combustion
H2O are formed by reacting 3 volumes of D) Displacement E) Exothermic
MgO with 2 volumes of H3PO4
Ill. 1 mole of Mg3(PO4)2 and 3 moles of H2O
are formed by the reaction of 3 moles of
MgO compound with 2 mole of H3PO4 27. Consider the formation of water by the reaction of
which of the statement(s) is(are) correct? hydrogen with oxygen
A) I only B) II only C) I and Ill I. Displacement reaction
D) II and Ill E) I, II and Ill II. Combustion reaction
III. Synthesis reaction
21. I. Ratio between number of atoms Which of the statements is (are) true for the above
II. Ratio between volumes reaction?
Ill. Ratio between numbers of moles. A) I only B) ll only C) III only
D) I and III E) I, II and III

22
B) Zn + NaOH 
28. In an open container KCIO3(s) is dissociated into C) Ca + HCI 
KCI(s) and O2 (g) by heating. D) Ag + HNO3 
I.K II. Cl III. O E) Mg + H2SO4
For which of the above elements there is a change in
the number of their atoms during this process? 37. I. Mg + HCl 
A) II only B) Ill only C) I and II II. Na + H2O 
D) II and Ill E) I, II and Ill III. CaC2 + H2O 
In which of the above reactions, hydrogen gas is
29. H2 + 1/2O2  H2O + heat released?
For the above reaction A) I only B) I and II C) I and III
I. Analysis reaction D) II and III E) I, II and III
II. Synthesis reaction
III. Exothermic reaction 38. Na is an alkali metal
Which of the statements is (are) true?
I. Na + H2O 
A) I only B) II only C) III only
II. Na + O2 
D) II and III E) I, II and III
Ill. Na + HCI 
Which reaction(s) take place?
30. When a metal is added to the test tube in which
A) I only B) II only C) I and III
there is concentrated HNO3, NO2 gas is released.
D) II and III E) I, II and III
According to this information
I. Ag II. Cu Ill. Mg
Which of the above can be the added metal? 39. I. Analysis II. Combustion III. Rusting
A) I only B) II only C) Ill only In which of the above, oxygen is needed throughout
D) I and II E) I, II and III the reaction?
A) II only B) II and III C) I and III
D) I and II E) I, II and III
31. I. Zn II. Ag Ill. Cu IV. Mg
metals are added into concentrated HCI solution.
Which of them do not react? 40. To separate argon from a mixture of SO3, SO2,
A) I and II B) I, II and III C) II and III CO and Ar
D) I, II and IV E) I and IV I. Burning with O2
II. Passing through a base solution
Ill. Passing through an acid solution
32. Into a strong base solution;
In which order should the needed steps be applied?
I.Al II. HNO3 II. ZnO
A) First II then I B) First I then II C) First I then III
are added. In which addition(s) a chemical reaction
D) First II then III E) First III then I
can be observed?
A) I only B) II only C) II and Ill
D) I and III E) I, II and III 41. What is the formula of the compound represented
by X in the following
balanced equation ?
33. I. K2O + CO2 
3X + 2Al  Al2(SO4)3 + 3Cu
II. ZnI2 + Br2 
A) CuO B) CuS C) CuSO4
III. CuSO4 + Zn  D) Cu(OH)2 E) CuSO3
Which of the above reaction(s) take place under
appropriate conditions?
A) I only B) II only C) Ill only
42. In chemical reactions,
I. Atoms are always conserved
D) I and Ill E) I, II and III
II. Volumes are always conserved
35. I. CuO + HCI  III. Mass is always conserved
II. ZnCI2 + Na  IV. Number of molecules is always conserved
III. NaOH + K Which ones of the statements given above are
Which of the above reaction(s) take place under correct?
appropriate conditions? A) I and III B) I, II and III C) II and III
A) I only B) II and III C) II only D) III and IV E) II, III and IV
D) I and II E) I, II and III
43. X2(YO4)3 + Y2O2  YO2 + XO2
36. In which of the following, the evolved gas is What is the coefficient of YO2 in the balanced
different from the others? equation?
A) K + H2O  A) 4 B) 5 C) 7 D) 6 E) 10

23
C) Hydrogen
44. Which one of the following reaction is (synthesis) D) Smoke
reaction?
A) A2 + B2  2AB 54. Which of the following is an example of chemical
B) A + 2BCl  ACl2 + B2 change?
C) C2H6 + 7/2O2  2CO2 + 3H2O A) The vaporization of naphthalene balls
D) 2KCIO3  2KCl + 3O2 B) The burning of wood
E) None of them C) The dissolution of salt in water
D) None of the above
45. SO3 + X  2H+ + SO4-2 What is the
formula of the X ? 55. Wood that has burned and formed carbon dioxide
A) 2H+ B) H2O C) O-2 D) 2H3O+ E) XO and water vapor has gone through a
A) Chemical change
46. Which one of the following is combustion B) Physical change
reaction? C) Change in state
D) Change in color
A) N2 + 3H2  2NH3
B) H2O  H2 + 1/2O2
C) CaCO3 + heat  CaO + CO2
56. When you put magnesium in dilute hydrochloric
acid, hydrogen gas bubbles off quickly. While in water,
D) C3H8 + 5O2  3CO2 + 4H2O bubbles of hydrogen gas appear very, very slowly. This
E) Fe + CuSO4  FeSO4 + Cu shows that
A) Magnesium reacts slowly with water and quickly
47. The gas which play an important role in the with acid.
process of burning is (are): B) Hydrogen reacts quickly with water and slowly with
A) Nitrogen C) Oxygen acid.
B) Hydrogen and Oxygen D) Hydrogen C) Magnesium reacts quickly with water and slowly
with acid.
48. The reaction between an acid and a base which D) Hydrogen reacts slowly with water and quickly with
produces salt is known as acid.
A) Neutralization
B) Vaporization 57. Changes in the chemical structure of substances
C) Fusion are called………
D) Bonding A) Physical change B) Chemical change
C) Nuclear change D) Color change
49. The chemical reaction 2H2 + O2 results in E) Shape change
A) 2H2O2
B) 2H2O 58. …………..are substances which affect the speed
C) H2O of chemical reaction, generally written above the arrow
D) O2H2 sign.
A) Acids B) Bases C) Salts
50. When the equation: MgO + HCl → MgCl2 + H2O D) Catalyst E) Oxides
balanced with the lowest possible whole number
coefficients, the sum of the coefficients is 59. Which of the following properties remains constant
A) 2 B) 4 C) 5 D) 6 during a chemical reaction?
51. The compound expected when Br2 reacts with A) The volumes of atoms
aluminum is B) Total volume
A) AlBr B) Al2Br C) Al2Br3 D) AlBr3 C) Physical states
D) Total mass of substances
52. The process whereby acid and base are combined E) Total number of molecules
is called
A) Distillation 60. If the following reaction were balanced what would
B) Neutralization be the coefficient of H2O?
C) Titration H2SO4 + Al(OH)3 → Al2(SO4)3 + H2O
D) Filtration A) 3 B) 4 C) 5 D) 6 E) 7

53. The main product of burning is 61. What is the formula of substance represented by
A) Oxygen X in the following balanced equation?
B) Carbon dioxide 4X + 5O2 → 4NO + 6H2O

24
A) NH3 B) NH2 C) N4H12
D) NH E) NH4

62. If the following chemical equation were balanced


what would be the sum of the coefficients?
C2H6 + O2 → CO2 + H2O
A) 6 B) 7 C) 8 D) 9 E) 9.5

63. Which of the following changes is not physical?


A) Painting the wall
B) Cutting of papers
C) Melting of ice
D) Dissolving of sugar
E) Cooking of egg

64. If the following reaction is balanced what will be the


coefficients of water?
C6H6 + O2  CO2 + H2O
A) 1 B) 2 C) 3 D) 4 E) 5

65. What must be the formula of compound represented


by X in the following balanced equation?
N2 + 3H2  2X
A) NH2 B) N2H6 C) NH3
D) NH E) N3H

66. Which one(s) of the following properties remain(s)


constant in a chemical reaction?
I. Total mass of substances
II. Physical states
III. Types and the numbers of reacting substances
A) I only B) II only C) I and III
D) II and III E) I, II and III

67. If the following reaction is balanced what will be sum


of the coefficients?
Ba(OH)2 + HCI  BaCI2 + H2O
A)
3 B)
4 C) 5 D) 6 E) 7

25
COMPOUNDS

1. Which of the following mixtures has the lowest electrical conductivity?


A) HCl + NaOH B) NaOH + H2O
C) HCl + NaCl D) C6H12O6 + H2O
E) NaCl + H2O

2. Xn + 2e-  X0
Ym  Y0 + 3e-
According to above equations, which one of the following compounds can be formed between X and Y ?
A) X2Y3 B) X3Y2 C) XY2 D) XY3 E) X3Y

3. Which one of the following compound cannot be produced between the metals 12Mg, 19K and nonmetals 17Cl, and 8O?
A) K2O B) KCl C) MgO D) MgCl2 E) MgK2

4. Which one of the following compounds can be formed between 12Mg and the element X
in K3XO4 ?
A) Mg5X2 B) MgX C) Mg3X2
D) MgX2 E) Mg2X2

5. The atomic mass number of element X is 27, and if it has 14 neutrons, which compounds can be produced when
element X combines with element Y having an atomic number 8 ?
A) XY B) XY2 C) X2Y D) X3Y2 E) X2Y3

6. Which one of the following elements can have different oxidation number in its compounds?
A) Na B) Ca C) Al D) Mn E) Zn

7. What is the oxidation number of C


in H2C2O4 ?
A) +1 B) +2 C) +3 D) +4 E) -2

8. What is the oxidation number of S in SOCl2? (8O, 16S, 17Cl)


A) -2 B) +1 C) +2 D) +4 E) +6

9. What is the oxidation number of Co in K3[Co(NO2)6] ?


A) +1 B) +2 C) +3 D) +4 E) -6

10. I. SO3 II. SCl2 III. K2S


According to given compounds above, which one of the following choices for oxidation numbers of S given correctly?
I II III
A) +3 +2 +1
B) +6 +2 -2
C) +6 -2 +2
D) +4 +2 +4
E) +6 -1 +2

11. In which of the following oxidation numbers of Mn in MnO4- and MnO4-2 respectively are given correctly?
A) +7, +6 B) +3, +4 C) +6, +7
D) +2, +3 E) +5, +7
12. In which oxide the oxidation number of oxygen is different than others?
A) SnO2 B) MnO2 C) PbO2
D) CrO2 E) BaO2

13. In which one of the following compounds there is no oxygen with oxidation number, -2?
A) H2O B) Cl2O C) OF2 D) BaO E) Na2O

14. In which one of the following compounds chlorine has the highest oxidation number?
A) HClO4 B) HClO3 C) HClO2

26
D) HClO E) HCl

15. Some sulfate salts of metals are given as follows.


I. CuSO4
II. Pb(SO4)2
III. Fe2(SO4)3
Which of the following gives the correct comparison of oxidation numbers of metals in these salts?
A) II>III>I B) I>II>III C) III>II>I
D) I>III>II E) II>I>III

16. I. K2Cr2O7
II. K2CrO4
III. CrO3
In which of the following, comparison of oxidation number of Cr in given compounds above, is shown correctly ?
A) I=II=III B) I>II>III C) III>II>I
D) I=III>II E) I=II>III

17. Which one of the following compounds is named wrongly?


A) FeCl3 Iron (III) chloride
B) CuCl2 Copper (II) chloride
C) CCl4 Carbon tetrachloride
D) P2O4 Diphosphorus tetra oxide
E) N2O Nitrogen dioxide

18. Which of the following compounds is named wrongly?


A) K2CO3 Potassium carbonate
B) Ca(CH3COO)2 Calcium acetate
C) CCl4 Carbon tetrachloride
D) PbSO4 Lead sulfate
E) NH4 Cl Ammonium chloride

19. Polyatomic Ions Name


I. AsO4-3 Acetate
II. Cr2O7-2 Choromate
III. ClO- Hypochlorite
Which one(s) of the naming given above is (are) named wrongly?
A) I only B) III only C) I and II
D) II and III E) I, II and III

20. Which one of the following is wrong for acids?


A) They conduct electricity
B) They have sour taste
C) They give H+ ions in water
D) They change the color of litmus paper to blue
E) When they react with Zn, H2 gas is evolved

21. A solution is prepared by dissolving X in the gas phase in water. In this solution the number of H + ions is greater than
that of OH- ions. According to these statements. Which of the following(s) is (are) correct ?
I. Gas X gives H+ in water
II. The solution changes the color of litmus to blue
III. The solution conduct electricity
A) I only B) II only C) I and III
D) II and III E) I, II and III

22. For the concentrated solutions of HNO3 acids;


I. They conduct electricity
II. It dissociates in water by giving OH- and NH4+ ions.
III. When they react with noble metals, H2 is produced.
Which one(s) of the above is(are) always correct ?

27
A) I only B) II only C) I and III
D) II and III E) I, II and III

23. An aqueous solution contains more H+ ions than OH- ions.


For this solution;
I. Strong acid
II. Weak acid
III. Weak base
Which one(s) of the above cannot be true?
A) I only B) III only C) I and II
D) II only E) II and III

24. When metal X reacts with HCl acid, a gas is produced. Which one of the following can not be
X?
A) Na B) Mg C) Zn D) Ag E) Fe

25. For the concentrated solution of strong acid


I. It has sour taste
II. It contains more H+ ions than OH- ions
III. When it reacts with Zn, H2 is produced.
Which of the above is (are) always correct?
A) I only B) II only C) I and II
D) II and III E) I, II and III

26. The substances X, Y and Z are added to different containers, which contain water. When litmus is dipped into the
resulting solutions it becomes red in the first one and becomes blue in the second and third one. In this case which one of
the following may be correct for X, Y and Z?
X Y Z
A) KOH NH3 HCl
B) HCl KOH NH3
C) HCl HCl KOH
D) NH3 HCl KOH
E) KOH HCl NH3

27. I. They do not react with bases


II. They react with acids
III. They react with water
Which one of the following oxides have the properties given above?
A) Acidic oxides B) Basic oxides C) Neutral oxides
D) Peroxides E) Amphoteric oxides

28. When basic oxides react with water, basic solutions are formed. Which one of the following is not a basic oxide?
A) Na2O B) CaO C) K2O D) CO2 E) MgO

29. When acid oxides react with water acidic solutions are produced. If CO2 is dissolved in water, in the resulting solution
which ion exists in a very small extent?
A) CO3-2 B) HCO3- C) H+ D) OH- E) H3O+

30. The reaction of basic oxides with water produces basic solutions. According to this fact; following oxides
I. K2O II. CaO III. CO2
react with water to produce aqueous solutions. Which of these aqueous solution(s) change(s) the color of red litmus paper to
blue?
A) I only B) II only C) I and II
D) II and III E) I, II and III

31. I. AgCl
II. Na2CO3
III. KAl(SO4)2
Which one of the following gives the correct classification of salts above?

28
I II III
A) Neutral Acidic Basic
B) Acidic Basic Neutral
C) Acidic Basic Double
D) Basic Double Acidic
E) Basic Acidic Double
32. A metal X reacts with HCl and produces H2, but it does not react with NaOH. In this case for oxide of X, XO.
I. XO reacts with both acids and bases
II. XO is a basic oxide
III. When XO reacts with water, basic solution is produced.
Which of the following is true?
A) I only B) II only C) I and II
D) II and III E) I, II and III

33. When acidic and basic oxides are dissolved in water acids and bases are produced.
NH3, H2S and SO3 are added to three different containers, which initially contain water. What are the colors of solutions
when some litmus solution is added to these containers?
I II III
A) Blue Red Red
B) Red Blue Blue
C) Blue Blue Red
D) Red Red Blue
E) Red Red Red

34. The following information are given for the oxides of X, Y and Z.
I. The reaction of X with water produces base
II. Y does not react with both acids and bases
III. Z reacts with both acids and bases
According to these information which one of the following can be correct for X, Y and Z?
X Y Z
A) Na2O CO ZnO
B) CO2 NO Al2O3
C) SO3 CO2 CaO
D) CaO CO2 ZnO
E) MgO N2O CO2

35. I. SO2 + H2O 


II. Na2O + HCl
III. NO + H2O 
Which of the reaction(s) above occur?
A) I and II B) III only C) I only
D) I and III E) I, II and III

36. The reactions;


X + HCl  Salt + H2O
X + NaOH  Salt + H2O
are given. Which one of the following can not be X?
A) ZnO B) Al2O3 C) Zn(OH)2
D) Al E) Al(OH)3

37. Al2O3 reacts with NaOH and HCl as in the following reactions;
Al2O3 + 2NaOH  Na3AlO3 + 3/2 H2
Al2O3 + 3HCl  AlCl3 + 3/2H2
In this case;
I. Al2O3 is an amphoteric oxide.
II. Na3AlO3 is a salt.
III. The aqueous solution AlCl3 conducts electricity.
Which of the statements above is(are) correct ?

29
A) I only B) II only C) I and II
D) I, II and III E) II and III

38. I. In general, oxygen containing nonmetals are acid oxides.


II. In general, oxygen containing metals are basic oxide.
III. The reaction of basic oxides with water produces bases.
a) Na + 1/2O2  Na2O
b) C + O2  CO2
c) Na2O + H2O  2NaOH
Which one of the following is correct if you match the statements and the reactions above?
A) Ib IIa IIIc B) Ib IIc IIIa C) Ic IIa IIIb
D) Ia IIb IIIc E) Ic IIb IIIa

39. I. They react with acids


II. They react with bases
III. They do not react acids
IV. They do not with bases
Which of the statements above can be said for the following compound groups?

Acidic Basic Neutral Amphoteric


oxide oxide oxide oxide
A) I and IV II and III III and IV I and II
B) II and III I and IV I and II III and IV
C) II and III I and IV III and IV I and II
D) II and III I and IV I and II I and II
E) I and IV II and III II and IV III and IV

40. For Mg and Al metals;


I. Producing H2 with NaOH
II. Producing H2 with HCl
III. Producing H2 with H2O
Which one(s) of above is(are) common?
A) I only B) II only C) III only
D) I and II E) II and III

41. Which of the following is not peroxide?


A) K2O2 B) H2O2 C) BaO2
D) Na2O E) CaO2

42. XO2 + NaOH  Salt + water


YO2 + NaOH  Salt + water
Z2O3 + NaOH  Salt + water
Z2O3 + HCl  Salt + water
Which one of the following classifications is correct for X, Y and Z if the reactions above take place?
X Y Z
A) Nonmetal Metal Nonmetal
B) Metal Amphoteric m. Nonmetal
C) Nonmetal Nonmetal Amphoteric m.
D) Metal Nonmetal Metal
E) Metal Amphoteric m. Nonmetal

43. 9Fe + 8HNO3  3X + 8NO + 4H2O


Which type of oxide is X in the reaction given above?
A) Acidic oxide B) Basic oxide C) Peroxide
D) Amphoteric oxide E) Mixed oxide

44. Examples for acids, bases and salts are given below.
I. Acids: HCl, H2S, CH4

30
II. Bases: NaOH, Ca(OH)2 , NH3
III. Salts: NaCl, K2SO4 , Al2(SO4)3
In which one(s) there is(are) wrong examples?
A) I only B) I and II C) II and III
D) I and III E) II only

45. The aqueous solutions of H2SO4 and KOH are good conductors but NH3 and HCN are poor conductors. For equal
volume and concentration of
I. H2SO4 and KOH
II. H2SO4 and NH3
III. HCN and KOH
are mixed. Which one of the final solutions above is acidic?
A) II only B) I and II C) I and III
D) II and III E) I, II and III

46. When double salts dissolve in water, they ionize. When complex salts dissolve in water the complex part remains same.
Similar to this, when KAl(SO4)2.12H2O and K3[Fe(CN)6] dissolve in water, which ion can not exist in the solution?
A) K+ B) Al+3 C) SO4-2 D) Fe(CN)6-3 E) Fe+3

47. Fe3[Fe(CN)6]2 is an example of


A) Acid salt B) Basic salt B) Neutral salt
D) Double salt E) Complex salt

48. An acid, HX with molecular mass 81 g/mol and a base, Y(OH)2 , with molecular mass 56 g/mol form a salt. What is the
molecular mass of this salt?
A) 119 B) 137 C) 49 D) 78 E) 182

49. When a mixture of equal masses of He-CO2-NH3 gases is passed through KOH, 12 g remains unreacted. What is the
mass of initial mixture?
A) 6 B) 12 C) 18 D) 24 E) 30
50. When a mixture of 24 grams Na-Zn-Ag is added to water, then the masses of mixture decreases by 4 grams. The
remaining metals are added to NaOH solution and it is observed that 8 grams metal remains unreacted. What is the mass of
Zn in the initial mixture?
A) 4 B) 8 C) 12 D) 16 E) 20

51. When a mixture of 30 L of He- CO- NH3 gases is passed through HCl solution, the volume of mixture decreases by 8
L. The remaining require 6 L of oxygen for burning. What are the volumes of each gas in L in the initial mixture?
He CO NH3
A) 10 12 8
B) 12 8 10
C) 10 10 10
D) 8 6 16
E) 16 6 8

52. When a mixture of 16.9 g Zn-Cu-Ca is put into HCl solution, 6.4 g solid remains at the bottom of container. After that
same mixture is put into NaOH solution and 10.4 g solid remains unreacted. According to these statements, what are the
masses of Zn-Cu-Ca in gram in the mixture?
Zn Cu Ca
A) 6.5 6.4 4.0
B) 6.4 9.4 1.0
C) 3.2 8.7 5.0
D) 4.0 3.2 10.5
E) 8.1 6.4 0.6

53. The formula of ionic compound formed between 12X and 9Y is


A) XY B) X2Y C) XY2 D) XY3 E) X3Y

54. An aluminum, 13Al, can form a compound by giving up............electrons.


A) 1 B) 2 C) 3 D) 4 E) 5

31
55. What is the oxidation number of Y in X2YZ4 ? (11X, 8Z)
A) +6 B) +4 C) +2 D) -2 E) -6

56. Given formulas and their names of some compounds.


Which one(s) of them is WRONG?
I. ZnCl2 : Zinc chloride
II. N2O4 : Dinitrogen tetra oxide
III. AgS : Silver sulfate
IV. Cu2O : Copper(II)oxide
A) I only B) III only C) I and III
D) II and IV E) III and IV

57. Which of the following compounds is lead(II)iodide ?


A) Pb(IO3)2 B) PI2 C) PbI2 D) Pb2I E) PbI4

58. Which one of the following element has variable charges in its compound?
A) Na B) P C) Mg D) Li E) Al

59. Which one of the following is the symbol of silicon?


A) S B) SO4 C) Si D) C E) Cl

60. Which compound has covalent bond between its atoms?


A) SO3 B) NaCl C) MgO D) Mg(NO3)2 E) CaBr2

61. Which one of the following compounds contains ionic bond between atoms?
A) H2O B) CO2 C) NO2 D) Kl E) CO

62. Which of the following is true about water molecules?


A) A water molecule is made up of one atom of hydrogen and two atoms oxygen.
B) A water molecule is made up of two atoms of hydrogen and one atom of oxygen.
C) A water molecule can easily split.
D) A water molecule can be easily squashed.

63. The chemical name of common table salt is


A) Sodium chloride C) Ammonium chloride
B) Ammonia D) Sodium hydroxide

64. Sodium hydroxide is an example of


A) Base
B) Salt
C) Element
D) Acid

65. One of the following is not an acid.


A) CH3COOH
B) HNO3
C) H2O
D) HCl

66. The following are characteristics of acid except A) It neutralizes a base


B) It tastes sour
C) It turns blue litmus paper into red
D) It feels soapy

67. Which of the following is an acid?


A) NaCl
B) H2O
C) HCl

32
D) NaOH

68. The chemical formula CO2 stands for


A) The element carbon dioxide
B) An isotope of carbon dioxide
C) A molecule of carbon dioxide
D) An atom of carbon dioxide

69. What bond is formed when atoms share one or more pairs of electrons?
A) Ionic B) Covalent C) Physical

70. What type of chemical bonding would you expect to take place in the making of sodium chloride?
A) Ionic B) Covalent C) Metallic

71. Water is a combination of hydrogen and oxygen atoms. It is considered as


A) atom B) compound
E) element D) solution

72. Two or more substances mixed together but not chemically joined form a
A) solvent B) compound
C) element D) mixture

73. An element's ability to combine with other element is shown by its


A) atomic number
B) valence number
C) no. of electron

74. Electrons that are involved in chemical bonding are called


A) ions
B) isotopes
C) valence
D) anion

75. What is the formula of compound formed between Zn +2 and CN-1 ions?
A) ZnCN B) Zn(CN)2 C) Zn2CN
D) Zn2(CN)2 E) ZnCN2

76. What is the name of the compound formed between Na + and Cr2O7-2 ions?
A) Sodium dichromate
B) Sodium chromate
C) Sodium monochromate
D) Disodium monochromate
E) Sodium chromide

77. Which of the following formulas represents diphosphorus pentoxide?


A) PO2 B) P2O4 C) P4O5
D) P2O5 E) PO
78. Which of the following compounds is called a base?
A) NaCl B) CO2 C) O2
D) NaOH E) CH4

79. An acid is a compound-producing……… ion to its water solution.


A) OH- B) O-2 C) Cl-
D) H+ E) NH4+

80. Which one of the following is an example of a neutral salt?


A) NaCl B) Ca(CN)2 C) CH3COOH
D) HCl E) Na2CO3

33
81. Which of the following compounds is not an oxide?
A) CO2 B) NO2 C) SO2
D) C2H5OH E) Cl2O

34
MATTER
1. Melting Boiling 8. “ Elements are shown by symbols and molecules
Compound Point(oC) Point (oC) are shown by formulas”
X 27 85 By using this statement;
Y -25 26 I. Table salt
Z 0 24 II. Naphthalene
The melting and boiling points of compounds X, Y III. Iron powder
and Z at constant pressure are given above. Which one(s) can be shown by a formula?
According to this data, what would be the states of X, A) I only B) I and II C) II and III
Y and Z at room temperature (25 oC)? D) I and III E) I, II and III
X Y Z
A) Solid Liquid Gas 9. I. As soon as dropped in acid, a gas evolves
B) Liquid Solid Solid II. When burnt, it forms carbon dioxide and water
C) Gas Solid Liquid III. When electrolyzed, two separate substances
D) Solid Gas Liquid are obtained
E) Solid Liquid Liquid Which one(s) of the above clearly states that
substance X is a compound?
2. 20 grams of both water and water vapor at 100 A) I only B) III only C) I and III
o
C are put into 1-liter bottles separately. D) II and III E) I, II and III
I. Number of H2O molecules
II. Density of H2O molecules 10. X: Substance with same kinds atoms
III. Average distance between H2O molecules Y: Substance with same kinds molecules
Which one(s) of the above is (are) different for the Z: Substance with different kinds of atoms
two bottles? T: Substance with different kinds of molecules
A) I only B) I and II C) II and III Which one is wrong about the substances X, Y, Z and
D) I and III E) I, II and III T?
A) X is element, Y is compound
3. Which one of the following substances does not B) Z is compound, T is mixture
contain only one kind of particles? C) Y is compound, T is mixture
A) Water B) Sulfur C) Acetic acid D) Z is mixture, Y is compound
D) Salty water E) Nitric acid E) T is compound, X is mixture

4. Iron metal reacts with sulfur to form iron (II) 11. I. Contains more than one kind of atoms
sulfur. In this process which properties of iron given II. Combines with definite mass proportions
below do(es) not change? III. Only chemical changes occur through the
I. Structure of the nucleus process
II. Color Which one(s) is the common property of compounds
III. Chemical property and mixtures?
A) I only B) II only C) III only A) I only B) II only C) I and II
D) II and III E) I, II and III D) I and III E) I, II and III

5. Which one of the following is not a mixture? 12. I. They are pure
A) Soil B) Natural gas C) Water vapor II. Cannot be dissociated into more simple
D) Air E) Oil substances by chemical means
III. Temperature remains constant throughout the
boiling process
6. Some examples of elements, compounds and
Which one(s) is the common property of compounds
mixtures are given below.
and elements?
I. Element: Copper, Iron, Hydrogen
A) I only B) III only C) I and II
II. Compound: Water, Air, Carbon
D) I and III E) I, II and III
III. Mixture: Soil, Milk
Which one(s) is correct?
A) I only B) I and II C) II and III
D) I and III E) I, II and III 13. Which one of the following is sufficient to prove
that ethyl alcohol, C2H5OH, is a pure compound?
A) Solubility in water
7. Which one of the following is a homogeneous
B) Boiling at 78oC
mixture?
C) Having a density of 0.78 g/cm3
A) Milk B) Paint C) Fog
D) Giving CO2 and H2O when burned with O2
D) Salty water E) Aryan

35
E) Being transparent and colorless II. Volume at the same temperature
III. Boiling point
14. By mixing which one of the pairs can you obtain a Which one(s) of the above is (are) not same?
heterogeneous mixture? A) I only B) II only C) I and II
A) Hydrogen + Helium D) I and III E) I, II and III
B) Water + Chalk dust
C) Water + Ethyl alcohol 21. Which one of the following is not a distinctive
D) Water + Carbon dioxide property for substances?
E) Water + Table salt A) Melting point B) Density C) Solubility
D) Volume E) Boiling point
15. Gas Density(g/cm3) Color Flammability
X 0.5 Colorless Inflammable 22. For two samples of a pure liquid at same
Y 0.9 Colorless Inflammable temperature and pressure with different masses;
Z 0.9 Bright yellow flammable I. Number of particles in its structure
T 1.2 Yellow flammable II. Boiling points
Q 1.5 Yellow Inflammable III. The quantity of heat given to pass vapor phase at
Properties of X, Y, Z, and T are given above. Since Z boiling point
is a mixture of two gases, what are these gases? Which one(s) is (are) different?
A) I only B) I and II C) II and III
A) X and Y B) Y and T C) Y and Q
D) I and III E) II only
D) X and T E) T and Q

16. “Alcohol is infinitely soluble in water” 23. A metal rod is heated for a while. In this process
Which one of the following would be the best which of the following will change?
description for alcohol -water mixture? I. Length
A) Homogeneous mixture II. Density
B) Emulsion III. Volume
C) Heterogeneous mixture A) I only B) I and II C) II and III
D) Suspension D) I and III E) I, II and III
E) Colloid
24. Sugar is slowly added to the sugar solution and
17. I. Rusting of iron dissolved. In this process which of the following will
II. Boiling an egg change?
III. Blurring of limewater with the CO2 gas I. Density of the mixture
Which one(s) is (are) chemical process? II. Boiling point of the mixture
A) I only B) II only C) I and II III. Mass of the water in the mixture
D) II and III E) I, II and III A) I only B) I and II C) II and III
D) I and III E) I, II and III
18. For a substance in solid state, below processes
are applied 25. When sodium metal is exposed to air, it loses its
I. Ionized in water metallic color and becomes dull. In this process,
which of the following properties will change?
II. Gas evolved when dropped in acid
III. Liquefied by heating I. Melting point
for which one(s), does the chemical property of the II. Density
substance remain unchanged? III. Mass
A) III only B) I and II C) I and III A) I only B) I and II C) II and III
D) II and III E) I, II and III D) I and III E) I, II and III

19. Which one of the following below depends on the 26. “Sugar dissolves in water whereas naphthalene
quantity of a pure matter? does not” in order to separate a sugar-naphthalene
A) Density mixture and to obtain pure sugar;
B) Boiling point I. Evaporation
C) Solubility II. Dissolving in water
D) Mass III. Filtration
E) Thermal expansion coefficient Which processes in which sequence must be
followed?
A) III, II, I B) II, III, I C) I, II, III
20. For equal masses of water and salt-water mixture
D) III, I, II E) I, III, II
under 1 atm pressure.
I. Density at the same temperature

36
27. Table salt and naphthalene is dissolved in water D) III and IV E) I, II and III
in order to be separated. Then it is observed that
naphthalene does not pass through the filter paper
when mixture is filtered. After filtrate is evaporated,
salt precipitates.
In this process, from which property of substances is
used to separate the mixture? 36. Which one of the following graphs shows the
A) Solubility B) Density C) Melting point density-volume relation of a pure substance at
D) Viscosity E) Boiling point constant temperature?
A) d B) d C) d
28. I. Salt from salty water
II. Naphthalene from sand-naphthalene mixture
III. Gasoline from petroleum
For which one(s) fractional distillation must be applied
V V V
in order to separate given mixtures above?
A) I only B) III only C) I and II
D) d E) d
D) II and III E) I, II and III

29. 1 kg of water at 25oC is heated to 400oK. What is


the change in the temperature in oC of water?
A) 375 B) 327 C) 600 D) 102 E) 100 V V

30. How many mL of water overflows when a 89 g of 37. I. Compounds contain different kinds atoms
copper block is put into a beaker containing fully with II. Elements consist of the same kinds atoms
water? (Density of copper is 8.9 g/cm 3) III. There are only protons and neutrons in an atom
A) 0.1 B) 1 C) 10 D) 100 E) 110 Which statement(s) is (are) true?
A) I only B) I and II C) II and III
31. In order to separate a mixture of olive oil and D) I and III E) I, II and III
water, which property of water should be used?
A) Solubility B) Density C) Color 38. Which one of the following isn’t characteristic
D) Mass E) Volume property of the matter?
A) Boiling point B) Density C) Solubility
32. Which of the following properties depends on the D) Volume E) Flexibility
amount of matter?
A) Boiling point B) Solubility C) Expansion 39. Which ones of the following mixtures are
D) Density E) Volume homogeneous?
I. sugar + salt II. sand + water III. oil + water
33. Which one is not characteristic property of IV. sugar + water V. alcohol + water
liquids? A) III and V B) I, II and III C) I and V
A) Solubility B) Elasticity C) Melting point D) IV and V E) II, IV and V
D) Density E) Conductivity
40. Which one of the following is an emulsion?
34. The graph, given aside, is A) oil + water
the mass- volume relations for Mass(g) B) sugar + water
X, Y and Z solids. Which one X C) alcohol + water
of the following is correct for Y
D) sand + sugar
the densities of these solids? E) sand + water
Z
A) dz>dy>dx
B) dx>dz>dy 41. Which of the following statements is true?
C) dx=dy=dz A) Water can only evaporate at 00C
Volume
D) dx>dy>dz (cm3) B) Water can also evaporate at lower temperatures
E) dz>dx>dz but the evaporation is slower
C) Water can evaporate only when it is boiling.
35. I. Water II. Air III. Oxygen D) Water cannot evaporate when temperature is low.
IV. Sugar-water solution
Which one(s) of the following given above is (are) 42. In freezing, liquid changes to solid. What
pure substances ? happens in condensation?
A) II only B) I and III C) I, II and IV A) gas changes to liquid

37
B) solid changes to gas without passing the liquid C) homogenous mixture
state D) heterogeneous mixture
C) liquid changes to gas
D) gas changes to another form 50. One of the following substances is known to
cause ozone depletion in the atmosphere, thus
causing global warming.
A) carbon-tetrachloride
B) boron-triflouride
C) chlorofluorocarbon
43. What happens to the atoms of this substance as D) aluminum hydroxide
it is heated?
A) atoms move faster so the substance eventually 51. Ten grams of solid carbon dioxide (dry ice) were
contracts. placed in a tightly covered jar for one day at room
B) atoms move faster so the substance eventually temperature. At the end of the day, it was observed
expands. that the solid became gas. You should expect that the
C) atoms move slower so the substance eventually CO2 gas would have a mass of
expands. A) less than 10 g
D) atoms move slower so the substance eventually B) 10 g
contracts. C) about 15 g
D) exactly 15 g
44. If heat is continued, the atoms of solid begin to
loose attraction, which hold them together, then 52. Which of the following substances will boil first?
……… occurs. A) acetone
A) boiling C) freezing B) alcohol
B) change of state D) crystallization C) water
D) ether
45. The stronger intermolecular force of attraction
exist in 53. Which of the following is not a physical change?
A) liquid A) melting of ice
B) solid B) dissolving of salt water
C) both liquid and gas C) condensation of air to form water droplets on a
D) gas surface of a cold glass
D) oxidation of a metal
46. What happens when air is heated?
A) It expands. 54. Chemical properties of elements are defined by
B) It contracts. the
C) It condenses. A) electrons
D) It decomposes. B) ionization energy
C) protons
47. The major contributor to the pollutants found in D) neutrons
acid rain are
A) toxic metals 55. Density of a substance is mass per unit volume. It
B) fossil fuels is a measure of how close the particles of matter are
C) industries packed together. The density of a piece of brass is
D) oxides 8.4 g/cm3 If its mass is 84 grams, find its volume.
A) 2 cm3 B) 5 cm3 C) 0.2 cm3 D) 10cm3
48. Which of the following would most easily be
filtered out of water? 56. A graduated cylinder is filled with water to a level
A) ink of 66.4 mL. When a piece of copper is lowered into
B) sand the cylinder, the water level rises to 86.4 mL. If the
C) sugar density of Cu is 8.9 g/cm 3, what is its mass?
D) coffee A) 178 g B) 132 g C) 871 g D) 128 g

49. Which term correctly describes a liquid that is 57. Two clear, colorless liquids have the same
one-phase, colorless, has a boiling temperature of melting point; but we are not sure if they are the same
96.5 0C to 97 0C? substance. What further test/tests can we try to
A) a solution determine whether they are the same or different
B) pure substance A) boiling point only

38
B) density only A) it is made up of light atoms arranged far away from
C) both a and b each other
B) it is made up of heavy atoms arranged far away
58. In which property must two solids differ if they are from each other
to be separated merely by dissolving at room C) it is made up of heavy atoms closely packed
temperature and filtering? together
A) solubility B) density C) boiling point D) it is made up of light atoms closely packed
together
59. Mass is also a measure of an objects
A) inertia B) weight C) density

60. A graduated, cylinder is filled with water to a level


of 50.0 mL. When a piece of copper is lowered into
cylinder, the water level rises to 79.0 ml. If the density
of copper is 8.9 g/cm 3, what is its mass?
A) 25.81 g B) 258.1 g C) 2581 g

61. Which of the following substance is a solution?


A) copper B) air
C) salt D) gold

62. The state of matter that exist only at a very high


temperature and consist of electrically charged
particles is
A) plasma B) solid
C) gas D) ultraviolet rays

63. Layers of pollutants in the air caused by exhaust


gases from automobiles which reacts in sunlight is
called
A) smog
B) carbon dioxide
C) carbon monoxide
D) chlorofluorocarbon

64. The atmosphere consists of gases, but four of


them are the most important. They are:
A) oxygen, carbon dioxide, nitrogen and argon
B) carbon dioxide, hydrogen, oxygen and carbon
C) oxygen, hydrogen, helium and nitrogen
D) oxygen, nitrogen, argon and water vapor

65. The most abundant gas in the atmosphere which


makes up 78 % of it is
A) oxygen B) nitrogen
C) argon D) helium

66. Why does copper sink in water while oak wood


floats?
A) An object with higher density than water sinks
while an object with lower density than water floats.
B) Copper is heavier than oak wood.
C) Oakwood is lighter than copper.
D) An object with a lower density than water sinks
while an object with higher density than water floats.

67. Copper has a high density because

39
SCIENCE AND CHEMISTRY D) Pound E) Kelvin
1. I. Searching around in order to get information.
II. Examining the relations of different substances
with each other. 7. Which one of the following is equal to 1 meter?
Ill. Collecting information related to an event regularly. A) 10-2 centimeter
Which one (s) of the above statements is (are) B) 10-6 micrometer
considered within the definition of science? C) 10-12 Pico meter
A) I only B) II and Ill C) I and Ill D) 103 kilometer
D) II only E) I, II and Ill E) 10-9gigameter

2. I. Physics examines the effects of the force on 8. Martin wants to test the effect of the absence of
matter. sunlight on the growth of pea plants. The number of
II. Chemistry examines the reactions of matter. pea plants to be observed, the amount and kind of
Ill. Biology examines the behaviors of living things. soil, and the amount of water are known as
Which one (s) of the above statements is (are) true A) Experimental variable
for the given branches of science? B) Constant variable
A) I only B) II C) All of these
only C) I D) None of these
and Ill
D) I and
II E) I, II
and Ill

3. I. Observation
II. Experiment
III. Hypothesis
IV. Sharing the results with other people
V. Searching the reasons of orders
Which of the following is the correct order of steps
given above for a scientific study.
A) I, V, III, IV, II
B) I, II, V, III, IV
C) I, III, IV, II, V
D) I, III, II, V, IV
E) I, V, II, III, IV

4. I. To be patient
Il. To be careful and sensitive
Ill. To be ready for surprises
Which one (s) of the above statements is (are)
characteristic(s) of a good observer?
A) I only B) I and II C) II and III
D) I and Ill E) I, II and Ill

5. A student observes naphthalene and writes down


his observations.
Which one of the following is not an "observation"
written by the student?
A) Naphthalene is white color
B) Naphthalene has crystalline structure
C) It smells
D) It shines
E) When it is burnt, CO2 and H2O are formed.

6. Which of following is not the base unit of SI unit


system?
A) Meter B) kilogram C) Second

196
ATOMIC THEORIES D) Dalton's law
E) Planck's law
Which statement about the
Rutherford’s atomic model is true?
I. All of the positive charge of an atom is concentrated in 7. If 2.4 g Mg is used in order to prepare 9.5 g
very small region, MgX2compound, what is the atomic mass of X?
II. The electrons occupy most of the total volume of (Mg: 24 g/mol)
the atom,
A) 71 B) 35.5 C) 32 D) 17 E) 16
III. Electrons can only exist in specific orbits around
the nucleus.

A) I only B) II only C) III only 8. If 8.4 g XCO3 solid is heated, 2.24 L of CO2
D) I and II E) I and III produced at STP while a quantity of metal oxide is
formed. Find the atomic mass of X. (C: 12 g/mol, O:
16 g/mol)
1. For a neutral atom;
A) 12 B) 16 C) 24 D) 32 E) 48
I. There are protons and neutrons in the nucleus.
II. The densest part of the atom is nucleus.
III. The number of protons is equal to the number of
electrons. 9. A compound of XCIO3 is 31.83% X by mass. Find
Which one(s) is (are) true? the atomic mass of X.

A) I only B) II only C) I and III A) 17 B) 34 C) 35 D) 39 E) 51


D) II and III E) I, II and III
10. Who had first proposed the law of definite
2. I. All electrons are attracted equally by the proportions?
nucleus.
II. All electrons are at equal distances from nucleus. A) Proust B) Lavoisier C) Dalton
III. The attractive force of nucleus is only due to the D) Thomson E) Aristotle
protons.
Which one(s) is (are) wrong for a neutral atom? 11. In the compound, Al2O3, the mass ratio between
Al and O is, 9/8. How many moles of Al required
A) I only B) II only C) III only preparing 51 g of Al2O3?
D) I and II E) I, II and III
A) 5 B) 4 C) 3 D) 2 E) 1

4. Who was the famous scientist, saying, "we can


divide a matter into infinitely many subdivisions as 12. In a compound of X and Y, the mass ratio
many as we want. And we could not obtain an between X and Y is 7/20. If 35 g of X is reacted with
indivisible piece of matter eventually"? 80 g of Y, which element and how many grams of it
will be remain without reacting?
A) Democritus B) Aristotle C) Khayyan
D) Mendeleev E) Lavoisier A) 209 Y B) 7 g X C) 8 g Y D) 10 g Y
E) 209 X

5. Who put the atomic concept into a scientific base?


13. In a compound of X3Y4 , the mass ratio between
A) Aristotle B) Dalton C) Bohr the constituents is X/Y = 21/8. If the molecular mass
D) Mendeleev E) Lovoisier of this compound is 232 g. What are the atomic
masses of
6. Which law of chemistry says, "The amount of X and Y?
substances before and after a chemical reaction does
not change"? X Y
A) 24 32
A) Lavoisier's law B) 32 56
B) Law of definite proportion C) 56 32
C) Law of multiple proportions D) 56 16

41
E) 16 32
A) XY3 B) XY4 C) X2Y3 D) X3Y2 E) XY

14. The graph given below, show the relationship


between the masses of Y and the masses of the 19. The elements C and H form more than one
compound X2Y. If the atomic mass of Y is 16, what is compound. The graph below gives the relationship
the molecular formula of this compound? between the masses of the constituents C, H and the
masses of the both compounds formed. If the
Mass of X2Y (g) molecular formula of the first compound is CH4, what
is the molecular formula of the second compound?
11 m(C) g II: ?

5.5 18 I: CH4
Mass of Y (g) 12
9
2 4 6
m(H)g
A) N2O B) NO C) NO2 D) N2O3 E)
N2O5 2 3 4
A) CH3 B) C2H3 C) C3H4 D) CH2 E) CH4

15. "If two elements, A and B. from more than one


compound, the amounts of A that are combined in 20. Which of the following statements best explains
these compounds with a fixed amount of B stand in a the Dalton's atomic model?
small whole - number ratio" What is the name of this
law? A) Atom is a sphere; at the center of this sphere there
is a nucleus.
A) Law of definite proportion B) Atoms are filled spheres.
B) Lavoisier's law C) Atom is composed of a nucleus, around which the
C) Law of multiple proportions electrons are turning.
D) Law of conservation of mass D) The protons are distributed randomly among the
E) MW of conservation of energy. surface of the atoms
E) Electrons are moving around the nucleus randomly

16. In one of the compound between X and Y, if 11.2


g of X combines with 4.8 g of Y, the formula of the 21. Which one of the following statements is not the
corresponding compound becomes X2Y3. What will mistake of Dalton’s atomic model?
be the formula of the second compound in which 16.8
g of X combined with 6.4 g of Y? A) Atoms are emptied structures
B) The atoms of the same type elements are totally
A) X3Y2 B) X2Y4 C) X3Y4 D) XY4 E) XY2 identical
C) It is not true that atoms are indivisible
D) Atoms form compound by combining with a
17. The elements X and Y form more than one definite number ratio.
compound. In the first of their compound, if 1.2 g of X E) Atoms show the characteristics of that element.
combines with 1.6 g of Y, the formula of the
compound becomes XY. What will be the formula of
the second compound in which 3 g of X combined 22. Which of the following is (are) electromagnetic
with 8 g of Y? waves?
I. Radio waves
A) X2Y B) XY3 C) X3Y D) XY2 E) XY II. Infrared waves
Ill. Visible light
18. In a compound of XY2 type formed between the IV. X - rays
elements of X and Y, the mass ratio between X and Y
is mX/mY = 1 If the second compound formed A) I only B) I and II C) II and III
between X and Y is 40% X by mass, what is the D) IV only E) I, II, III and IV
formula of the second compound?

42
28. What is the wavelength of light with the frequency
23. I. All electromagnetic waves have the same of 5x1014 Hz?
speed with light at vacuum.
II. The speed of waves in vacuum depends on their A) 6000 Ao B) 7500 Ao C) 6500 Ao
wavelengths D) 8000Ao E) 8000 Ao
III. The speeds of all of the electromagnetic waves
are always the same in every medium (air, water,
vacuum etc). 29. What is the frequency of a TV signal with the
Which of the above inferences is (are) correct? wavelength of 10 m?

A) I only B) II only C) Ill only A) 300 KHz B) 300 MHz C) 3x1010s-1


D) I and II E) II and Ill D) 3xlO9 Hz E) 4x108 Hz

24. Human eye can see the radiations 30. The frequency of a radio broadcasting is 105
between……….nm and……..nm. Which of the MHz. What is the wavelength of this signal radiated?
following values should be replaced into the blanks
given above? A) 285.7 m B) 285 cm C) 2.85 m
D) 285 nm E) 2857 m
A) 500; 600 B) 600; 700 C) 400; 700
D) 400; 600 E) 650; 700
31. What is the wavelength of a light in nm with the
frequency of 6x1014 Hz?
25. When an ordinary white light is passed through a
prism, ………..spectrum is obtained. However, when A) 450 B) 500 C) 550 D) 600 E) 650
a light coming from the vapor of a heated element in
an electric arc is passed through a
prism…………….spectrum is formed. Which of the 32. What is the energy of a red light with the
following statements should be replaced into the frequency of 4x1014Hz?
blanks given above respectively?
A) 2.84x10-19 J
A) Line; continuous B) 3x10-19 J
C) Atomic; electromagnetic C) 2.42x10-18 J
B) Continuous; line D) 2.4x10-19 J
D) X - rays;  - rays E) 1.21x10-18 J
E) N.M.R; hydrogen

33. What is the energy of a red light with the


26. What is the frequency of a violet light with the frequency of 7.5x1014s-1?
wavelength of 400 nm?
A) 4.97x10-19 J
15 -1
A) 7.5x10 s B) 4.97x10-18 J
B) 7.5x1014 Hz C) 2.84x10-20J
C) 6.0x1014 s-1 D) 2.84x10-15 J
D) 6.0x1015 Hz E) 4.9x10-20 J
E) 6.5x1015 s-1

34. What is the energy of a yellow light with the


27. What is the frequency of a red light with the wavelength of 580 nm?
wavelength of 700 nm?
A) 3.43xl0-20 J
14 -1
A) 4.29x10 s B) 3.43x10-19 J
B) 5x1014 s-1 C) 3.85x10-20 J
C) 4.29x1015s-1 D) 3.85x10-19J
D) 5x1015s-1 E) 4.5x10-20 J
E) 6x1015s-1

43
35. What is the energy of a TV signal with the 41. What is the energy of a radiation obtained by an
wavelength of 2 m in joule? electron transition from n= 4 to n = 3 level in the
hydrogen atom?
A) 9.94x10-26
B) 9.94x10-25 A) 1.05x10-20 J
C) 9.5x10-26 B) 1.52x10-20 J
D) 9.5x1025 C) 1.59x10-20 J
E) 9.5x1024 D) 1.882x10-20 J
E) 1.60x10-19 J

36. What is the energy of green light with the


wavelength of 5500 Ao in kcal/mol? 42. How many photons with the wavelength of 2m
should radiate to increase the temperature of the 10 g
A) 520 B) 5.2 C) 52.08 D) 5.20 E) of water in meat of chicken inside a microwave by 90
o
5.0 C?

A) 9.94x1026
37. What is the energy of a radiation required to B) 9.94x1025
excite an electron of hydrogen atom from n=2 to n= C) 3.78x1024
4? D) 3.78x1028
E) 9.5x1024
A) 4.08x1020 J
B) 4.08x10-19 J
C) 4.08x10-21 J 43. Which one of the following electron structures
D) 4.5x10-19 J belongs to 12Mg+2 ion?
E) 4.5x1020 J
A) [Ne]s1 B) [Ne]3s2 C) [Ne]
D) 1s2 2s2 2p4 E) 1s2 2s2 2p5
38. A red light is produced in the visible region when
an electron of hydrogen atoms falls from a higher
energy level to n = 2. If the energy of this red light is 44. Which one of the following electron distributions
3.03x10-19 J, from which higher level did this electron does not suit to Hund's rule?
fall? A) 1s2 2s2 2p2x 2p2y 2p2z
B) 1s2 2s2 2p1x2p1y
A) n=6 B) n=5 C) n=4 D) n=3 E) n=2 C) 1s2 2s2 2p1x 2p1y 2p1z
D) 1s2 2s2 2p2x 2p1y2p0z
E) 1s2 2s2 2p2x 2p2y 2p1z
39. What is the frequency of the spectral line that
corresponds to an electron transition from n = 3
to n = 2 level in the hydrogen atom? 45. What is the atomic number of an element having
5 electrons totally in n = 4?
A) 4.56x1014 s-1
B) 5.5x1014 s-1 A) 24 B) 28 C) 30 D) 32 E) 35
C) 5.5x1015 s-1
D) 4.56x1015 s-1
E) 4.5x1015 s-1 46. In a compound X2O3, the ratio of the mass of X to
the mass of compound is 7/19. If the atom of element
X has 7 neutrons in its nucleus totally, what is the
40. What is the wavelength of the spectral line that electron configuration of X?
corresponds to an electron transition from n= 3
to n = 2 level in the hydrogen atom? A) 9X:1s2 2s2 2p5
B) 8X:1s2 2s22p4
A) 650.3 nm B) 656.3 nm C) 660.3nm C) 7X:1s2 2s22p3
D) 663.3nm E) 650 nm D) 6X:1s2 2s2 2p2
E) 5X:1s2 2s2 2p1

44
47. Which one of the following electron distributions C) X: 1s2 2s2 2p6 2s2 3p6 4s2 4p1
does not fit into Aufbau principle? D) X: 1s2 2s2 2p6 2s2 3p6 4s2
E) X: 1s2 2s2 2p6 2s2 3p6 4s2 4p4
A) 9F:1s2 2s2 2p5
B) 8O:1s2 2s2 2p4
C) 7N:1s2 2s2 2p3 54. Who first proposed that the universe was
D) 6C:1s2 2s1 3s2 3p1 composed of atoms, which are indivisible,
E) 5B:1s2 2s2 2p1 unchangeable, discomfortable and eternal?

A) Djabr Bin Khayyan B)


48. What is the maximum number of electrons in Dalton C) Democritus
n = 3 having the ms value of + 1/2? D)
Thomson E) Rutherford
A) 9 B) 8 C) 7 D) 7 E) 6

55. Dalton proposed about the atom that;


49. How many electrons can be present in n=2 with I. All substances are composed of atoms,
ml value of 0? II. Atoms are filled spheres,
III. Atoms cannot be further divided.
A) 2 B) 4 C) 6 D) 8 E) 10 Which one(s) of these purposes is (are) correct?

A) I
50. How many electrons can be present in n=4 with I only
value of 2? B) II
only C) III
A) 10 B) 8 C) 6 D) 4 E) 2 only
D) I and
II E) II
51. For the electron configuration; X:1s2 2s2 2p6 3s2 and III
3p3, find the set of four quantum numbers n, l, ml and
ms for the valence electrons of the X atom
respectively? 56. Who first discovered the electron?

A) 3, 2, +1, +1/2 A)
B) 3, 2, +1, +1/2 Thomson B)
C) 2, 1, +1, -1/2 Rutherford C) Dalton
D) 3, 1, -I, +1/2 D) Bohr E)
E) 4, 1, +1, +1/2 Einstein

52. For the electron configuration; X: [Ne]3s2 3p1, find 57. Which one(s) of the following ideas is wrong
the set of four quantum numbers n, I, ml and ms about the Thomson’s atomic model?
for the valence electrons of the X atom respectively. I. The mass of electron is so small
II. Atom has the shape of sphere
A) 3, 1+1, +1/2 III. Protons and electrons are charged particles
B) 3, 2, +1, +1/2
C) 3, 0, +1, -1/2 A) I only B) II
D) 3, 1, -1, +1/2 only C) I and II
E) 3, 1, 0, +1/2 D) II and III E) I
and III

53. Which one of the following configurations is the


electron configuration of an element, whose valence 58. According to the today’s knowledge which one(s)
electron has the set of four quantum numbers of; of the purposes of Rutherford is (are) incorrect?
n=4, l=1, ml=-1 and ms=+1/2? I. Electrons move at a certain distance from nucleus.
II. Electrons move in circular orbital.
A) X: 1s2 2s2 2p6 3s2 3p6 4s1 III. Energy levels are named either by letters or by
B) X: 1s2 2s2 2p6 3s2 3p64s2 4p3 positive integer numbers.

45
A) I only B) II
only C) I and III
D) I and II E) I, II
and III

46
BORON

1. Which of the following is the first member of group 3A in the periodic table?
A) Carbon B) Nitrogen
C) Aluminum D) Boron

2. Which of the following is not the property of boron?


A) It is a metalloid.
B) It is quite hard and brittle.
C) It is good conductor of electricity and heat.
D) It is found in nature as crystalline structures.

3. Which of the following is not the mineral of boron?


A) borax B) kernite
C) mirabilis D) colemanite

4. The compounds of boron and hydrogen are called…………


A) boranes B) borates
C) boric acid D) borax

5. Which of the following is diborane?


A) B2H6 B) B3H7
C) B5H11 D) B4H8

6. Which of the following formula represents boric acid?


A) B(OH)3 B) BH
C) H2BO4 D) H3BO4

7. Which of the following is one of the uses of boron?


A) In cement industry
B) In paint industry
C) In plastics
D) In glass industry

8. Which of the elements in group 3A in the periodic table is a non metal?


A) In B) Ga C) B D) Al

9. Which of the following elements is harder?


A) Al B) B C) Mg D) Cu

10. …………….is used in analytical chemistry to prepare a standard HCl solutions.


A) Borane B) Borax
C) Boric acid D) Boron

11. Which of the following is wrong about uses of boron in industry?


A) It is an additive substance in lightweight alloys.
B) It is used in control rods in nuclear power stations.
C) It is used in laboratory glassware.
D) It is the source of hydrogen energy in the future.

47
12. Which of the following elements is used as a control rod in a nuclear power stations?
A) Tl B) In C) Al D) B

48
CARBON

13. Which of the following is correct about the elements in group 4A in the periodic table?
A) Si and Ge are good conductors of electricity.
B) C and Si make mostly ionic bonds in their compounds.
C) C is the first member in group 4A.
D) Ge has two allotropes called diamond and graphite.

14. C-14 is the radioactive isotope of carbon.


A) True B) False

15. ……….. is the basic element in living organisms.


A) Hydrogen B) Oxygen
C) Carbon D) Silicon

16. Which of the following is wrong about the sources of carbon in the nature?
A) It is found as petroleum and natural gas.
B) It is in the atmosphere as CO2 and CO.
C) It is in earth’s crust as coal.
D) It is found in oceans as compound of oxygen.

17. Which of the following best explains the hardness of diamond?


A) Carbon atoms are so close to each other.
B) Carbon atoms make bonds with 109.28o angle.
C) Carbon atoms make strong covalent bonds with 4 neighbor carbon atoms in the shape of tetrahedron.
D) Carbon atoms make strong ionic bonds.

18. Fullerenes are …………..


A) compounds of silicon.
B) isotopes of germanium.
C) allotropes of boron.
D) allotropic form of carbon.

19. Which of the following is poisonous oxide of carbon?


A) CO B) CO2 C) HCOOH D) H2CO3

20. CO is an acidic oxide.


A) True B) False

21. Carbon dioxide is a colorless, odorless, nonpoisonous acidic oxide of carbon.


A) True B) False

22. Which of the following is (are) correct common names of compounds?


I. CaCO3: limestone
II. CaO: quicklime
III. Ca(OH)2: limewater
A) I only B) II only
C) I and II D) I, II and III

23. Which of the following is wrong about the uses of carbon dioxide?
A) It is used in fire extinguishers.
B) It is used in carbonated drinks.
C) It is used in manufacturing of yeast, baking powder.
D) It is used in alcoholic beverages.

24. What is dry ice?


A) It is an isotope of carbon.
B) It is solid form of water.
C) It is CO2 at -78oC.
49
D) It is an allotrope of carbon.

25. Which of the following is washing soda?


A) Na2CO3.10H2O B) NaHCO3
C) NaCN D) CCl4

26. Ionic compounds of carbon with metals are called carbides.


A) True B) False

27. Which of the following carbon compounds is quite poisonous?


A) CH4 B) HCN
C) CO2 D) C6H12O6

28. ……….is radioactive isotope of carbon and used in archeology.


A) C-13 B) C-12
C) C-14 D) C-15

29. Which of the following is (are) correct about the uses of active carbon in industry?
I. In gas masks to absorb pollutants.
II. In water distillation.
III. In photo copy machines and laser printers.
A) I only B) II and III
C) I and III D) I, II and III

30. Which of the following gases has a great role in green house effect?
A) CH4 B) CO2
C) O2 D) SO2

31. In the periodic table group 4A is called “carbon family”.


A) True B) False

32. Following properties are given for an element;


I. It is soft.
II. It is good conductor.
III. It is dark black.
Which one might be this element?
A) Diamond B) Graphite
C) Boron D) Silicon

33. In the preparation which substance is the CO2 is used?


A) Bread B) Baking powder
D) Tooth paste D) Oil

34. Which one(s) is (are) given correctly about the usage areas of carbon black?
I. As filler in rubber tires
II. As a pigment in printing inks
III. In plastics and fibers
IV. As a transfer material in carbon papers
A) I and II B) II and III C) I and III
D) II and IV E) I, II and IV

35. Which one(s) of the following oxides is (are) the chief oxides of carbon?
I. CO II. CO2 III. C3O2
50
A) I and II B) II only C) III only
D) I and III E) I, II and III

36. Which of the following ions makes the water harder?

A) OH- B) CN- C) Cl- D) CO3-2 E) Br-

37. Which one(s) of the following oxides of carbon is (are) stable at STP?
I. CO II. C3O2 III. CO2

A) I only B) II only C) I and III


D) II and III E) I, II and III

38. Which one(s) of the following is (are) the usage areas of CO ?


I. Synthesis of methanol
II. As a fuel
III. In metallurgy

A) I only B) II only C) III only


D) I and III E) I, II and III

39. The industrial use of diamond depends on two properties of it. These are ………..and ……….

A) electrical / high thermal conductivity


B) extreme hardness / high thermal conductivity
C) electrical conductivity / optical property
D) high oxidizing property / high reactivity
E) extreme hardness / electrical conductivity

40. Which one of the following is wrong about the usage of carbon containing compounds?

A) In refrigerators
B) In fire extinguishers
C) In metallurgy
D) In plastic sand fibers
E) In pesticides

41. ..… is a bio molecule made up of carbon, hydrogen and oxygen. It is the main source of energy in the body.

A) carbohydrates
B) protein
C) fats
D) nucleic acid

42. Diamond is made up of carbon. It is the hardest substance known. It is considered to be a

A) metal
B) non-metal
C) metalloid
D) halogen

43. Which one(s) of the following gases are responsible of the air pollution?
I. N2 II. CO2 III. CO IV. He V. SO2

A) I, II and III B) I and II C) II and III


D) II, III and V E) III and IV

51
44. Which of the following is (are) correct about the uses of active carbon in industry?
I. In gas masks to absorb pollutants.
II. In water distillation.
III. In photo copy machines and laser printers.
A) I only B) II and III
C) I and III D) I, II and III

45. In the preparation which substance is the CO2 is used?


A) Bread B) Baking powder
D) Tooth paste D) Oil

52
Halogens

1. Which of the following is not one of the oxidation states of chlorine?

A) –1 B) +1 C) +4 D) +5 E) +7

2. Which one(s) of the following is (are) wrong about the usage of chlorine?
I. In the production of chlorinated organic compounds, including PVC
II. In paper and textile industry, pool and municipal water.
III. In automobile batteries.

A) I only B) II only C) III only


D) I and III E) II and III

3. Which of the following nonmetal is the most active?


A) Cl B) F C) H D) N

4. Which of the following is not a property of halogens?


A) They are nonmetals.
B) They are found in diatomic molecules.
C) They form acids with hydrogen.
D) They are good conductors of heat.

5. Which is wrong for the uses of chlorine?


A) It is used in PVCs.
B) It is used in the production of insecticides.
C) It is used in pools to kill the germs.
D) It is used in fertilizers.

6. Acids are usually stored in glass bottles except………


A) H2SO4 B) H2CO3
C) HF D) HCl

7. Which element is not a halogen?


A) Cl2 B) H2 C) Br2 D) I2

8. Which halogen is a liquid at room temperature?


A) Cl2 B) F2 C) Br2 D) I2

9. Which of the following is correct about halogens?


I. They are the most active nonmetals in each period in the periodic table.
II. Only Iodine is metal in the group.
III. They form salts with hydrogen.
A) I only B) II only C) III only
D) I and II D) I and III

10. Which of the following is correct about the color of related elements?
A) F2 has pale yellow color
53
B) Cl2 has yellowish green color.
C) Br2 has pale blue color.
D) I2 has purple color.

11. Which one is wrong about uses of halogens?


A) Fluorine is used in Teflon.
B) Chlorine is used in PVCs.
C) Bromine is used in photographic films.
D) Iodine is in medicine as an antiseptic.
E) Bromine and chlorine is used in swimming pools.

54
HYDROGEN

46. Which of the following is one of the properties of hydrogen?


A) It is green-colored gas.
B) It is heavier molecule than other gas molecules.
C) Its diffusion rate is much greater than other molecules.
D) It has five isotopes.

47. Which of the following is not the isotope of hydrogen?


1 2 3 4
A) 1 H B) 1 D C) 1T D) 2 He

48. Which of the following is the most important compound of hydrogen?


A) CaH2 B) NaOH C) H2O D) H2SO4

49. Which preparation method can be used in laboratories to obtain hydrogen gas?
A) 2Na + 2H2O → 2NaOH + H2
B) C + H2O → CO + H2
C) C3H8 + 2H2O → 3CO2 + 10H2
D) CH4 + heat → C + 2H2

50. Which of the following is not the use of hydrogen in industry?


A) In the production of ammonia
B) In paints as oxidizing agent.
C) In the production of metals from their oxides.
D) In space spacecrafts as fuel.

51. Which of the following is wrong about hydrogen?


A) It is the first element in the periodic table.
B) It is the most abundant element in the atmosphere.
C) It has three isotopes.
D) It forms ionic hydrides with active metals.

52. Which of the following is water gas?


A) H2O B) CO
C) CO + H2 D) CO2 + H2O

53. Which one is wrong for hydrogen?


A) Its symbol is H.
B) Its atomic number is 1.
C) It has no neutron.
D) It is a liquid element at room temperature.

54. In which of the given compounds does hydrogen have different oxidation states?
A) HCl B) HNO3
C) CH3COOH D) KH

1 2 3
55. 1 H , 1 D and 1T are the isotopes of hydrogen. What makes them different?
A) Number of protons
B) Number of electrons
C) Number of neutron
D) Number of orbitals.

56. Although hydrogen is placed in the periodic table at the head of group 1A, in fact it does not show alkali metal properties. It is placed in
group 1A, because …………….
A) it is the first element known.
B) of its valance electron.
C) of number of proton.
D) it does not have neutrons.
55
57. Which of the following is heavy water?
A) H2O B) T2O C) HDO D) D2O

58. In the electrolysis of water …………..


A) it is separated into hydrogen and oxygen gases.
B) it is purified and it becomes drinkable.
C) it is heated to its boiling temperature.
D) it reacts with active metals.

59. Which of the following is not the usage area of hydrogen?


A) It is used in the production of ammonia.
B) It is used to saturate oils.
C) It is used as rocket fuel.
D) It is used in the production PVCs.

60. Which of the following is wrong about hydrogen?


A) Tritium is a radioactive isotope of hydrogen.
B) Ordinary hydrogen is called protium.
C) Hydrogenation is a process of hydrogen removal.
D) Hydrogen resembles halogens.

56
NITROGEN

1. Which of the following elements in not a nitrogen family element?


A) Antimony B) Phosphorus
C) Oxygen D) Arsenic

2. Which of the following elements is only the one metal in the group 5A in the periodic table?
A) Bi B) N C) As D) P

3. Which of the elements in group 5A in the periodic table is gaseous at normal conditions?
A) As B) N C) P D) Bi

4. Which of the elements in group 5A in the periodic table is found in the atmosphere in the greatest ratio by volume and by mass?
A) O2 B) N2 C) H2 D) C

5. The main source of nitrogen is the air and it is obtained by fractional distillation of liquefied air in industry.
A) True B) False

6. Which of the following compounds is obtained by Haber process developed by Fritz Haber, Nobel Prize winner?
A) NH4Cl B) HNO3 C) N2H4 D) NH3

7. Which of the oxides of nitrogen is well known as “laughing gas”?


A) NO2 B) N2O4 C) N2O D) N2O3

8. …………….is a reddish-brown gas that is produced when the nitrates of heavy metals are heated.
A) N2O3 B) NO2 C) HNO3 D) N2

9. ………….is third most important industrial acid, after H2SO4 and H3PO4, is used to prepare in fertilizers, explosives, nylon and
plastics.
A) NH3 B) H2CO3 C) HNO3 D) HCl

10. What is “aqua regia”?


A) It is the solution of NH3 and HCl.
B) It is the solution of H2SO4 and H2O.
C) It is the solution of HCl and HNO3.
D) It is a kind of explosive made from HNO3.

11. What is inert atmosphere in which foods are kept without spoiling?
A) Atmosphere made of nitrogen gas.
B) Atmosphere made of oxygen gas.
C) Atmosphere made of chlorine gas.
D) Atmosphere made of bromine gas.

12. ………..is an essential element in proteins.


A) Silicon B) Fluorine
C) Helium D) Nitrogen

13. Which of the nitrogen oxide is used in dentistry as a mild anesthetic, and is known as laughing gas?
A) NO B) N2O C) N2O5 D) NO2

14. By which method is nitric acid commonly produced in industry?


A) Contact method B) Bayer’s method
C) Ostwald’s method D) Haber process

15. Which of the following metals cannot react with HNO3?


A) Cu B) Fe C) Ag D) Au

16. Which of the following is not one of the uses of nitrogen?


57
A) It is used in explosives.
B) It is used as rocket fuel.
C) It is used in plastics.
D) It is used in fertilizer production.

17. Which of the following oxides of nitrogen is the most important in living organisms?
A) N2O B) NO2 C) NO
D) N2O4 E) N2O3

18. Which one(s) of the following is (are) source of nitrogen containing compounds?
I. KNO3 and NaNO3
II. Atmosphere
III. Plants and Animals
IV. Proteins and Coal
A) I, II and III B) I and III C) III and IV
D) II, III and IV E) I, III and IV

19. Who manufactured NH3 in 1903 firstly and efficiently?


A) Fritz Haber B) Good Year C) N.Bronsted
D) T.M.Lowery E) Mendeliev

20. The essential problem is in the synthesis of ammonia that, under most conditions the reaction does not go to the completion. It is
reversible. High yield of ammonia requires,
I. A high temperature, 400oC
II. A catalyst
III. Cooling down to -40oC
IV. A high pressure, about 200 atm
A) I and II B) I, II and IV C) II and IV
D) II and III E) I only

21. What is the name of the technique, invented in 1895, that is used to convert nitrogen to ammonia?
A) Liquefaction of air
B) Titration
C) Polymerization
D) Haber method
E) Synthesis of ammonia

22. Which of the following is wrong about the usage of nitrogen containing compounds?
A) Fertilizers
B) Household cleaning products
C) Manufacture of dry cell batteries
D) Explosive matter
E) As a blench in paper and textile industry

23. In which of the following compounds is the oxidation state of nitrogen given wrongly?
A) NO2: +4
B) N2O: +1
C) N2O3: +3
D) NH3: +3
E) N2O4: +4

24. Which of the following oxides of nitrogen is responsible for the air pollution?
A) N2O B) NO2 C) N2O4
D) NO3 E) N2O3

25. Which of the following gases causes acid rain?


58
A) N2 B) Cl2 C) NO2 D) H2

26. The main source of nitrogen is in earth’s crust is KNO3 and NaNO3.
A) True B) False

27.The chemical symbol Sb stands for


A) Antimony
B) Tungsten
C) Mercury
D) Lead

59
Noble Gases

61. Which of the following noble gases is used in ordinary light bulbs?
A) He B) Ar C) Xe D) Kr

62. What is the group number of noble gases?


A) 2A B) 6A C) 8A D) 3B

63. Which of the following is wrong?


A) Argon is the most abundant noble gas.
B) All noble gases have 8e- in their valance shells.
C) Radon is radioactive.
D) He is used in airship balloons.

64. Who was the first scientist synthesized the compounds of noble gases?
A) Neils Bohr B) Neil Barlett
C) William Ramsay D) Thomas Edison

65. Which of the following noble gases is the lightest?


A) He B) Ne C) Ar D) Xe

60
Oxygen

12. The most abundant element on the earth's crust is


A) oxygen
B) hydrogen
C) silicon
D) aluminum

13. Which of the following statements is wrong for oxygen?


A) Oxygen forms ozone.
B) Oxygen is used in metallurgy.
C) Oxygen is used in diving.
D) Oxygen is a flammable gas.

14. Which one is wrong for combustion reactions?


A) Oxygen is needed.
B) Heat is needed.
C) Water is used.
D) They are exothermic.

15. Which of the following is wrong about oxygen?


A) Oxygen is second most electronegative element.
B) It forms covalent compounds with metals
C) Its common oxidation state is -2.
D) It is colorless and odorless gas.
E) It has six valance electrons.

16. Which one(s) are correct given below?


I. O2 in atmosphere is second most abundant gas.
II. Oxygen has two allotropes, O2 and O3.
III. In industry, O2 gas is prepared from sea water.
A) I only B) II only C) I and II
D) II and III E) I, II and III

17. Oxygen has +2 charge in ……..


A) H2O B) OF2 C) CaO
D) CHO2H D) CO2

18. What is the ratio of gases by volume collected as a result of electrolysis of water?
A) 2/1 B) 3/1 C) 4/3
D) 5/2 E) 1/3

19. Which of the following is wrong about the uses of oxygen?


A) It is used by divers, patients and astronauts.
B) It is used in acetylene welding.
C) It is required in plastic industry.
D) Liquid oxygen is a powerful rocket fuel.
E) It is used in the production of H2SO4 and HNO3.

61
PHOSPHORUS

1.Which of the following element is known as light spreader in Greek, because of its ability to glow in the dark?
A) Oxygen B) Argon
C) Phosphorus D) Carbon

2.Which of the following is not one of the allotropes of phosphorus?


A) White B) Red C) Green D) Black

3.Which one is known as white phosphorus?


A) P B) P4 C) P2O3 D) P4S3

4.Which allotrope of phosphorus is poisonous and because of this effect it is prohibited in many countries in match
manufacturing?
A) White B) Green C) Red D) Black

5.Which of the following is the most stable allotrope of phosphorus?


A) White B) Red C) Green D) Black

6.Which of the following is correct for phosphorus?


A) It can have +7 oxidation state.
B) White phosphorus is the most stable allotrope.
C) It must be stored under water.
D) None of the above.

7.Which of the following is one of the acids of phosphorus?


A) P6H10 B) H3PO4 C) PH3 D) PCl3

8.Natural or chemical substances put on the land to make crops grow better are called ………..
A) ammonia B) insecticides
C) fertilizers D) phosphine

9.In the fertilizers which are the three basic elements used to enrich soil?
A) Nitrogen, phosphorus and calcium
B) Chlorine, phosphorus and calcium
C) Nitrogen, magnesium and calcium
D) Nitrogen, phosphorus and potassium

10. Which one(s) of the following is (are) the properties of white phosphorus?
I. It is condensed, collected, and stored under water.
II. It is conductor of electricity
III. It is soluble in water.
IV. It can be cut with a knife.

A) I and IV B) I and III C) II and IV


D) III and IV E) II and III

11.Today, heading phosphorus-containing fertilizers in use are the ammonium phosphates, MAP and DAP. They are made by the
controlled neutralization of ........ with ..........

A) H2SO4 / NH3
B) HNO3 / H2SO4
C) H3PO4 / H2SO4
62
D) CaSO4 / NH3
E) H3PO4 / NH3

12.Which one of the following is the formula of red phosphorus?


A) P B) P4O6 C) P4O10D) P4 E) P2O3

13.Which one of the following phosphorus containing compounds is used to prepare match heads?
A) P4O6 B) P4O10C) P4S3
D) H3PO4 E) P2O5

14.Which one(s) of the following is (are) true about the usage of phosphorus?
I. In the living organisms in ADP and ATP synthesis
II. As fertilizers
III. In food industry, cheese making, in curing hams, baking powder

A) I only B) II only C) III only


D) II and III E) I, II and III

63
SILICON
11. What is ordinary window glass called?
1. Which of the following is the most abundant element in the
earth’s crust by mass? A) soda glass B) pyrex glass
A) Carbon B) Silicon B) Aluminum D) Oxygen C) water glass D) silica glass

2. Which of the following elements is most similar to silicon? 12. Which nation was first used glass?
A) Al B) Ge C) Ge D) C A) Romans B) Greeks
3. Acids are usually stored in glass bottles except……… C) Egyptians D) Mongolians
A) H2SO4 B) H2CO3
C) HF D) HCl 13. The main raw material for glass is ………..
A) air B) sand
4. SiO2 C) clay D) wood
I. is the only one stable oxide of silicon,
II. has lattice crystalline structure, 14. The elemental silicon can be prepared from
III. is similar to diamond, very hard. ............ or ............ by the reaction with coke in an electric
Which of the given above is (are) correct? furnace.
A) I only B) III only
A) Quartz / Sand
C) II and III D) I, II and III B) Diamond / Graphite
C) SiO / SiO2
D) Na2SiO4 / Na2SiO3
E) Na2SiO4 / SiO2
5. Silicates are compounds of silicon with oxygen.
A) True B) False 15. One of the following is a practical use of silicon.

6. Which of the following is a substance made of A) for making porcelain and glazes for pottery
silicon? B) as insulator for electric irons
A) Plastics B) Rubber C) as conductor of heat
C) Paper D) Glass D) for making computer chips

7. ……….. is made by adding water to a mixture of 16. Which of the following is a substance made of
cement, sand, and stone gravel. silicon?
A) Clay B) Concrete A) Plastics B) Rubber
C) Glass D) Silicone C) Paper D) Glass

8. Cements are used in the production of ceramics,


rubber, paint, and porcelain.
A) True B) False

9. Which of the following names of compounds is


given wrongly?
A) H4Al2SiO3 : Clay
B) Na2O.CaO.6SiO : Ordinary glass
C) CaSO4.2H2O : Feldspar
D) H2SiO3 :Silicic acid

10. Because silicon is ………….., it is used to prepare


electronic devices.
A) a metal B) a nonmetal
C) a good conductor D) a semi-conductor
64
Sulfur II. Pesticides
In which one(s) of the following ions and III. Sulfuric acid
compounds of sulfur, is (are) the oxidation states IV. Natural rubber
of sulfur atom given correctly?
-2
I. SO4 : +6 A) I and III B) II and III C) I, II and III
II. SO2: +4 D) II and IV E) I, II, III and IV
III. H2S: +2
IV S2O3-2: -2 and +6
24. The main use of ……… is in the synthesis of SO 3
A) I and II B) I and III C) I, II and III to make H2SO4. And the process to make H2SO4 is
D) II, III and IV E) I, II, III and IV called ………

A) Sulfur / Vulcanization
20. Which one(s) of the following is (are) the properties B) Sulfur / Contact process
of sulfur? C) SO / Fermentation
I. S8(s) is a poor oxidizing agent. D) S8 / Vulcanization
II. Its possible oxidation states are between -2 and +6. E) SO2 / Contact process
III. It forms ionic sulphides with most active metals.

A) I only B) II only C) I and II 25. In the world approximately, 70% of H2SO4


D) I and III E) I, II and III production is used in the manufacture of…………

A) Pesticides
21. In 1839, ……………..accidentally discovered that B) Fertilizes
by heating a sulfur-rubber mixture, a product could be C) Dye industry
obtained that was stronger , more elastic, more D) Cleaning agents
resistant to cold and heat than natural rubber. Such E) Metallurgy
process is called ……….

A) H.W. Nernst / Titration 26. Which of the following sulfur compounds is


B) C. Goodyear / Hydrolysis responsible from acid rains?
C) Le Chatelier / Fermentation
D) C. Goodyear / Vulcanisation A) SO2 B) SO3 C) H2S2O3
E) H.W Nernst / Contact process D) H2SO4 E) H2S

22. Which one of the following is not a source of 27. In which of the following is the oxidation state(s) of
sulfur? sulfur in S2O3-2 ion given correctly?

A) Natural gas A) –2 B) +6 C) +2 D) -2, +6 E) –3


B) Mineral sulphides and sulphates
C) Oil
D) Coal 28. Which one(s) of the following is (are) true for the
E) Living organisms usage of sulfur containing compounds?
I. Thiosulfate solutions are used in photographic
23. Which one(s) of the following is (are) the products processing.
of sulfur? II. Sulphites are used in pulp and paper industry.
I. Plastics III. CuSO4 is used in electroplating processes.
65
A) I only B) II only C) I and III
D) II and III E) I, II and III

29. Which of the following is wrong for sulfur?


A) It reacts with hydrogen.
B) It reacts with aluminum.
C) It reacts with oxygen.
D) It reacts with gold.

30. Which of the acid is used in industry mostly?


A) HNO3 B) H2SO4 C) HF D) H2CO3

31. Which of the following methods is used to produce


H2SO4 ?
A) Haber Process B) Down Method
C) Contact Process D) Ostwald’s Process

32. Which of the following sulfur compounds is toxic


and smells like a rotten egg?
A) H2S B) SO2 C) H2SO4 D) H2SO3

66
GASES
Patm=76 cmHg
If 4.0 g of a gas at 25oC occupies 4 L at a pressure of 1.00 atm, what will 10.
be its volume in L at a pressure of 2.5 atm at the same temperature?
A) 0.16 B) 0.32 C) 0.1 D) 1.6 E) 3.2

1. A gas has a volume of 50 L at 77oC. What would be the volume in L X(g) Y(g)
at -77oC assuming constant pressure?
A) 2.86 B) 28 C) 0.286 44cm
32cm
D) 286 E) 1.43

2. A gas has a volume of 152 mL at 15oC and under a pressure of 72


cm Hg. What will be the volume in mL at STP? In the system given in figure above, what is the pressure of X in
A) 1.3 B) 136.5 C) 13.65 D) 0.1 E) 0.01 cmHg?

3. What would be the pressure in atm exerted by 16 g He in 16.4 L A) 152 B) 88 C) 76 D) 44 E) 32


container at -73oC? (R: 0.082)
A) 1.84 B) 18.4 C) 0.1 D) 4 E) 0.184
11. Which one of the following temperature values at which the
4. What is the molecular weight of the gas whose density is 2.2 g/L at
1.64 atm and 127oC? motions of gas molecules stop?
A) 11 B) 22 C) 32 D) 44 E) 84
A) 0oC B) 273oC C) 0 K D) 273 K E) -4 K
5. A gas mixture contains 1.5 mol of H2, 0.5 mol of O2 and 0.5 mol of
Ne gases. If the total pressure is 2 atm, what will be the partial pressure in
atm of Ne? 12. If the mean free path of a gas molecule at constant
A) 0.4 B) 0.8 C) 1.2 D) 0.2 E) 0.16 temperature is doubled, how many times the volume of the gas
will increase?
6. Which one of the following does not affect the pressure of a gas?
A) Number of molecules A) 2 B) 4 C) 6
B) Temperature
C) Speed of the gas molecules
D) 8 E) remains constant
D) Number of atoms in the gas molecule
E) Volume of the container
13. An ideal gas known at STP. For this gas
I. Temperature
7. Which one of the following statements is false for the gases? II. Pressure
III. Volume
A) They are compressible. Which of the quantities given above can be calculated?
B) They can mix in every portion.
C) Their densities are changeable by depending on conditions. A) I only B) III only C) I and III
D) Their inter molecular forces of attraction are so high. D) I and II E) I, II and III
E) They occupy the volume of their containers.

14. Which one of the following quantities will not affect the
8. Which one of the following units is not a pressure unit? pressure of 1 g of gas?

A) mmHg B) g/cm2 C) atm A) Volume


D) torr E) kgm B) Temperature
C) The number of moles
D) Molecular mass
9. The normal freezing and boiling points of water are 0oC and E) The number of atoms in a molecule
100oC respectively.
Which one of the following pairs states these two temperature
values in Kelvin (K)? 16. Calculate the pressure of CH4 gas in cmHg in the manometer
given in the figure?
A) 0, 273 B) 273, 373 C) -273, 0 (dHg=13.6 g/cm3, dH2O=1 g/cm3)
D) -273, 273 E) -273, 373

67
Patm=76 cmHg A) 8 B) 4 C) 2 D) ¼ E) ½

21. When the valves of M opened simultaneously, the gases of


He and X meet at the point of A. What is the molecular mass of
CH4
Water X? (H: 4)
h=136 cmHg
20 cm 10 cm
4 cm M A M
Mercury He X

A) 114 B) 90 C) 82 D) 76 E) 41
A) 64 B) 32 C) 24 D) 16 E) 8

17.
22. For the gases of H2 and Ne;
X2Y I. H2 molecule is faster than Ne molecules at the same
3 temperature.
XY 102oC II. The velocities of H2 molecules at 25oC and Ne molecules at
50oC are the same.
27oC 60 cm III. Ne molecules at 0oC are faster than H2 molecules at 273oC
oil
Which of the given statements is (are) exactly true?
Mercury (H: 1, Ne: 20)

The gases at the bulbs of manometer, having equal volume A) I only B) III only C) I and II
contain the same number of atoms. What is the pressure of the XY D) II and III E) I and III
gas in cmHg? (dHg= 15.doil)

A) 48 B) 56 C) 64 D) 72 E) 112 23. For the O2 and CH4 gases having equal volume, temperature
and the number of molecules;
I. Their pressures are the same.
18. The masses of the H2, He and Ne gases are equal in a II. The average velocities of molecules are the same.
container. Which of the given quantities of these gases is (are) the III. The average kinetic energies of molecules are the same.
same? Which of the given statements is (are) correct?
I. The numbers of atoms of the gases. (O: 16, H: 1, C: 12)
II. The average kinetic energies of the gases.
III. The average velocities of the gases. A) I only B) III only C) I and III
(H: 1, He: 4, Ne: 20) D) II and III E) I, II and III

A) II only B) III only C) II and III


D) I and III E) I, II and III 24. A flexible balloon contains O2 gas. When the balloon is
heated, how will the volume (V), pressure (P) and the kinetic
energy (Ek) of O2 change?
19. The temperature of a gas in a closed container is increased
from 177oC to 627oC. V P Ek
I. The average kinetic energy. A) Increases No change No change
III. The average velocity. B) Increases No change Increases
III. The absolute temperature. C) No change Increases Decreases
Which of the given quantities above is (are) doubled? D) No change Increases No change
E) Decreases Increases Increases
A) I only B) III only C) I and III
D) II and III E) I, II and III
25. A closed container vessel contains a given amount of N2O4
gas is heated, the reaction given below occurs;
20. The molecular mass of an O2 molecule is 16 times greater N2O4(g)  2NO2(g)
than that of a H2 molecule. What is the ratio between the velocity in the vessel;
of O2 molecule and the velocity of H2 molecule? I. The total number the molecules increases.
II. The total pressure increases.
68
III. The average velocity of molecules increases.
Which of the given statements above is (are) true?

A) I only B) II only C) I and III


D) II and III E) I, II and III 30. The volume of the balloon at room temperature is 5 L. What
will be the final volume in dm3 if the temperature of the balloon is
doubled?
26. There are two flexible balloons of which contains O2 and
other contains SO2 gases at the same temperature. A) 0.54 B) 5.4 C) 54 D) 540 E) 5400
If the masses of O2 and SO2 are equal;
I. The volume of the balloons containing O2 is twice greater than
that of containing SO2. 31.
II. The pressure of SO2 and O2 are the same.
Gas 30 cm
III. O2 molecules are 2 times faster than SO2 molecules.
Mercury
Which of the given statements above is (are) true? 20 cm
(S: 32, O: 16) Mercury 20 cm
Gas 15cm

A) I only B) II only C) I and III Figure I Figure II


D) II and III E) I, II and III
A capillary tube shown figure I is inverted and figure II is
obtained. Calculate the atmospheric pressure by assuming
temperature remains constant.
27. The volume of a gas of which pressure is 1368 torr is
increased from 1.5 L to 2.7 L.
A) 60 B) 65 C) 70 D) 75 E) 76
What will be the pressure of the gas in atm?

A) 1.4 B)
1.2 C) 32. Under a set of conditions of temperature and pressure, 40 g of
1.1 D) 1 E) 0.8 SO3 gas occupies 10 L and 18.4 g of Y2O4 occupies 4 L. What is
the atomic mass of Y? (SO3: 80)

A) 92 B) 46 C) 28 D) 16 E) 14
28. 40 L of a gas at 127oC is cooled to the 27oC and its volume
becomes X L. What is X?

A) 77 B) 70 C) 45 D) 35 E) 30 33. Temperature(oC) Volume(L) Pressure(atm)


I. 0 22.4 1
II. 273 11.2 2
III. 546 5.6 3
29. Which one of the following graphs false for the gases?
In the table given above, the values of pressures, volumes and
temperatures for He gas are shown.
Compare the numbers of the moles of He (n 1, n2 and n3). (R:
A) B) 0.082)
P(atm) V (L)
atm A) n1>n2=n3 B) n1>n2>n3 C) n2>n3>n1
(n, T constant) (P, n constant)
D) n3>n2>n1 E) n1=n2=n3

V (L) T (K)
34. If the temperature of a given amount of a gas is increased
from 0oC to 1oC, what will be the ratio between its initial volume
C) D) and final volume? Assume that pressure remains constant.
P(atm) n (mol)
(V,T constant) (P, V constant) A) 0.1 B) 273 /274 C) 0
D) 1 E) 342 /478

n (mol) T (K)
E) 35. In order to increase the number of collisions of gas molecules
V (L)
atm on unit area in a closed container;
(P, n constant) I. The volume of the container should be decreased.
II. Some more gas should be added into container.
T (oC) 69
-273
III. The temperature of the gas should be increased. PCl5 (g)  PCl3 (g) + Cl2 (g)
Which of the statement(s) given above will be true? Patm=76 cm

A) I only B) II only C) I and II


D) II and III E) I, II and III PCl5

76 cm

36. P1
V (L) P2
P3

What will be the final pressure in atm?


o
t ( C)
-273.15 A) 8 B) 6 C) 4 D) 3 E) 2

The graph shows the relationship between volume and


temperature of a given amount of ideal gas in different pressures. 40. A container with an ideally movable piston has 1 mol of CO
Find the relationship between pressures P1, P2 and P3. gas. When 28 g of N2 gas is added into the same container at the
same temperature;
A) P1>P2>P3 B) P3=P2>P1 C) P1=P2=P3 I. Density remains constant.
D) P3>P2>P1 E) P1>P3>P2 II. Total mass increases.
II Volume increases.
Which of the statements given above is (are) true?
37. If the temperature of helium gas in a closed containers (C: 12, N: 14, O: 16)
increased from 0oC to 1oC;
I. The pressure of He gas increases by 1/273 A) I only B) III only C) I and II
II. The average kinetic energy of the He atoms increases. D) II and III E) I, II and III
III. The average velocity of the He atoms increase.
Which of the statement(s) given above will be true?
41. 2 mol of He gas occupies 22.4 L under a pressure of 2 atm. If
A) I only B) II only C) I and II 3 mol of He gas added into the container and the temperature is
D) II and III E) I, II and III increased by 273oC, what will be the final pressure in atm?

A) 16 B) 14 C) 12 D) 10 E) 8
38. The graph shows the relationship between absolute
temperature and pressure of X, Y, and Z gases of which numbers
of the moles are equal. Compare the volumes of the X, Y, and Z
gases.
P 42. A given amount of XY3 gas has a pressure of 4 atm at 27oC in
Z
a closed container. When some amount of XY3 gas is decomposed
Y
to produce Y2 gases, the pressure is increased to 6 atm and the
temperature is increased to 127oC. What is the percentage of
X decomposing XY3 gas in mole?

A) 75 B) 50 C) 37.5 D) 10 E) 12.5
T

43. COCl2 gas in a container with ideally movable piston


A) Vx=Vy=Vz occupies V L at t oC. When the temperature is increased to 367oC,
B) Vx>Vy>Vz the reaction given below occurs and COCl2(g) is consumed
C) Vx<Vy<Vz exactly;
D) Vx=Vy>Vz COCl2(g)  CO(g) + Cl2(g)
E) Vx>Vy=Vz If the final volume is 4V, what is the value of t?

A) 47 B) 57 C) 147 D) 226 E) 320


39. When the absolute temperature of PCl5 is doubled, the
reaction given below occurs and PCl5 (g) is consumed exactly,
70
44. The amount of the mixture of CS2 - O2 is 5 mol. When the
reaction given below occurs; A) 12.6 B) 9.6 C) 8.4 D) 4.2 E) 2.2
CS2(g) + 3O2(g)  CO2(g) + 2SO2(g)
1 mol of CS2 remains unreacted. What is the percentage of O 2 in
mole, in the mixture before the reaction? 50. What is the density of argon in g/L gas under a pressure of 4
atm and at a temperature of 2730C? (Ar: 40)
A) 80 B) 60 C) 45 D) 40 E) 35
A) 4.00 B) 3.57 B) 3.27 D) 2.57 E) 1.78

45. Gas Volume Pressure Temperature


X V P 300 K 51. The graph shows the relationship between the value of P.V
Y 3V 2P 27oC and absolute temperature of the gas X.
In the table some quantities of X and Y gases are given. If the If the mass of the gas X is 32 g, what will be the molecular mass
number of the mol of X is n, what is the number of the mol of Y? of X?
P.V (atmL)
A) 2/3 n B) 3/2 n C) 2n D) 3n E) 6n
22.4
46. When R constant is equal to 22.4/273, pressure is in atm, 11.2
volume is in L, amount is in moles and temperature is in K. If the
pressure was in cm Hg, and the volume was in cm 3, what would
be numerical value of R constant?

A) 22.4/273 273 546 T (K)


B) 22400/273
C) 760x22.4/273
D) 76x22400/273
E) 273/22400x76 A) 128 B) 96 C) 64 D) 32 E) 16

47. A child leaves 4 L of his balloon under the atmospheric 52. 22.5 g C2Hx occupies 22.4 L at 273oC.What is the numerical
pressure of 750 mmHg and at a temperature of 27 C. o value of X? (C: 12, H: 1)
If the balloon escaped at the peak of a hill where the atmospheric Patm=76 cm
pressure is 520 mmHg and the temperature is -13oC, what would
be the volume of the balloon in L?
C2Hx
A) 10 B) 9 C) 8 D) 7 E) 5
38 cm

48. The graph shows the relationship between number of moles


and pressure of a gas, which occupies 11.2 L. What is the
temperature in oC? (R: 0.082)
P (atm)
A) 2 B) 3 C) 4 D) 5 E) 6
9

6 53. The density of CH4 gas under a pressure of 4.1 atm and at a
temperature of t oC is 2.5 g/L. What is the value of t? (CH4: 16)
3
A) 147 B) 127 C) 107 D) 67 E) 47
1 2 3 n (mol)
A) 136.5 B) 156.5C) 226 D) 273 E) 409.5 54. There is a gas X in a container with an ideally movable piston
at constant temperature. The density of X is 0.04 g/cm3. If 12 g
C2H6 is added into the container at the same temperature, the
49. What is the volume of 33 g CO2 in L under a pressure of 4 volume becomes 500 cm3 and the density of the mixture becomes
atm and at a temperature of 273oC? (R: 0.082) 0.032 g/cm3. What is the molecular mass of the gas X? (C: 12, H:
1)
71
A) 16 B) 24 C) 32 D) 40 E) 64 60. A sample of SO2 has a volume of 120 mL at 273oC, what
would be its volume at standard temperature?(assume constant
pressure)
55. In a container, 3 mol of N2 is reacted with 3 mol of H2
according to the equation below; A) 15 B) 30 C) 60 D) 120 E) 240
N2(g) + 3H2(g)  2NH3(g)
In order to consume both gases, how many grams and which gas
should be put into the container? 61. A sample of CO gas occupies 5 L at 127oC. What must be the
(N: 14, H: 1) temperature in oC at which the gas occupies 0.5 L at constant
pressure?
A) 12 g H2 B) 10 g H2 C) 8 g H2
D) 28 g N2 E) 6 g H2 A) 33 B) 123 C) –123 D) –153 E) –233

56. An 80 g sample of which 65% is zinc is reacted with excess 62. What is the temperature in oC of 0.2 mol of a gas that exerts a
HCl. What is the volume of H2 gas produced in L under a pressure pressure of 2280 mmHg in a 400 mL closed container?
of 4.1 atm and at a temperature of 27oC? (Zn: 65)
A) –100 B) –200 C) 100 D) 200 E) 155
A) 11.2 B) 6.72 C) 5.6 D) 4.8 E) 2.24

63. A 3.12 g of the gas, HX, occupies a volume of 500 mL at -


57. Which one of the following is correct about the comparison 23oC and 760 torr. What would be the atomic weight of the X?
of the pressures Po, P1 and P2?
Po A) 27 B) 87 C) 129 D) 254 E) 25.1

P2
P2 P1 64. What is the partial pressure of H2 in a gas mixture containing
1 g H2, 13.2 g of CO2, and 8 g Ar confined to a volume of 1 L at
27oC? (H: 1, C: 12, O: 16, Ar: 40)
h
A) 0.123 B) 1.23 C) 12.3 D) 6.6 E) 3.3

65. A gas has a density of 3.17 g per litre at STP. What would be
the density of the same gas at 4 atm pressures and same
A) Po>P1>P2 B) Po>P2>P1 C) P1>P2>Po temperature?
D) Po=P1>P2 E) P1=P2=Po
A) 0.123 B) 10.5 C) 12.68D) 13.65 E) 17.6

58.
He SO2 66. When the valve M is opened, the final pressure becomes 50
mmHg. What is the value of X?
I II III IV M
Where will these gases meet in the tube? (He: 4, SO2: 64) 2L XL
150 torr empty
A) I B) II C) III
D) IV E) Between III and IV
A) 2 B) 2.5 C) 3 D) 4 E) 6

59. A container, which has an airtight and movable piston,


contains 3 g of H2 (g). If 42 g of the gas X2 (g) is added to the 67. Which one of the following does not affect the pressure of a
same container at the same temperature the volume of the gas gas?
doubles. What is the atomic weight of X? (H: 1)
A) Number of molecules
A) 14 B) 28 C) 56 D) 112 E) 224 B) Temperature

72
C) Speed of the gas molecules And the graph, which shows the relationship between pressure
D) Number of atoms in the gas molecule and time is drawn. What is the percentage in mole of
E) Volume of the container decomposition of COCl2 gas?
Patm

68. Which one of the following is the same for gases in a mixture
3x
at a definite temperature?
2x
A) Average kinetic energies
B) Average speeds
C) Partial pressures Time (min)
D) Number of molecules
E) Number of moles A) 25 B) 40 C) 50 D) 60 E) 75

69. In order to produce 8.2 L of H2 gas under a pressure of 1.5 73. 16 g of NH4NO3(s) in 14 L of container is decomposed
atm and at a temperature of 27oC, according to the reaction given exactly at 546 K according to the equation given below;
below; NH4NO3(s)  N2O(g) + 2H2O (g)
2Al(s) + 6HCl(aq)  2AlCl3,(aq) + 3H2(g) What is the total pressure of the gases in atm?
How many grams of Al(s) should be used? (Al: 27) (NH4NO3: 80)

A) 81 B) 54 C) 27 D) 13.5 E) 9 A) 1.92 B) 1.8 C) 1.76 D) 1.66 E) 1.22

70. For the gases of Ar and C3H4 of which masses are equal
under the same conditions, which one of the following statements 74. Some quantities of helium are given below in the table below.
is wrong? Pressure Temperature Density
(Ar: 40, C: 12, H: 1) P 3T d1
2P 2T d2
A) They occupy equal volume 3P T d3
B) They contain the same number of molecules. What is the relationship between d1, d2 and d3?
C) Their densities are the same.
D) The average kinetic energy of their molecules is the same. A) d1=d2=d3 B) d1>d2>d3 C) d3>d2>d1
E) The average velocity of their molecules is the same. D) d3=d1>d2 E) d2>d1=d3

71. Gas Temperature Pressure Density 75. The graph shows the relationship between the pressure and
X T P d the value P.V of Ne gas. What is the relationship
Y 2T P/4 d/4 between T1, T2 and T3 ?
Z 2T P d
Some quantities of the gases X, Y and Z with equal volumes are P.V (atm.L)
given in the table above.
If the molecular mass of X is equal m, what are the molecular T3
masses of Y and Z?
T2

Y Z T1
A) 2m 3m
B) m/2 2m P (atm)
C) 4m 2m
D) 2m 2m
E) 4m m/2 A) T1=T2=T3 B) T1>T2>T3 C) T3>T2>T1
D) T3>T1>T2 E) T2>T1>T3

72. A closed vessel contains COCl2 gas. Some of the gas is


decomposed according to the equation given below. 76. A container has CS2 and O2 gases with equal numbers of
COCl2(g)  CO(g) + Cl2(g) moles at 127oC. Gases react with each other, according to
equation given below;
CS2(g) + 3O2(g)  CO2(g) + 2SO2(g)
73
After the reaction the temperature is increased by 200oC and the III. The molecular masses and the numbers of atoms of X, Y
pressure is increased by 1 atm. gases.
What is the initial pressure of the gases before the reaction in Which of the statements given above should be equal?
atm?
A) I only B) II only C) III only
A) 2 B) 3 C) 4 D) 5 E) 6 D) I and II E) I and III

77. Two different gases occupy the same volume at STP; 83. A gas mixture of which pressure is 1 atm, made from N2, O2
I. The number of moles of these gases and CO2 gases is 65% N2, 15% O2 and 20% CO2 by mass. Which
II. The masses of these gases one of the following comparisons for the partial pressures of N2,
III. The partial pressures of these gases CO2, and O2 gases is true?
Which of the quantities given above is (are) the same? (N: 14, C: 12, O: 16)

A) I only B) III only C) I and III A) PN2>PCO2>PO2


D) II and III E) I, II and III B) PO2>PCO2>PN2
C) PO2>PN2>PCO2
D) PN2>PO2>PCO2
78. The relationship between the molecular masses of equal E) PCO2>PN2>PO2
masses of X, Y and Z is X>Y>Z;
I. The partial pressure of Y is the highest.
Patm: 76 cmHg
II. The density of X is the smallest.
III. The number of moles of Z is the biggest. 84.
Which of the statements given above is (are) true?
76 M 76
A) I only B) II only C) III only cm CO O2 cm
D) I and II E) I and III

79. He gas occupies 1 L under 2.5 atm in container A. Ne gas V V


occupies 1.5 under 2 atm in container B. If 20% of He gas is
transferred into container B at the same temperature, what will be When the valve M, is opened the reaction given below occurs;
the pressure of gases in container B in atm? CO(g) + 1/2O2(g)  CO2(g)
What is the difference between the levels of the mercury in cm in
A) 1.75 B) 1.82 C) 1.95 D) 2.33 E) 3.25 left manometer?
(Assume that temperature remains constant)

80. Ne and SO3 gases, of which partial pressures are equal, are A) 0 B) 19 C) 38 D) 57 E) 76
mixed. What will be the ratio between masses of the gases (mNe
/mSO3)? (O: 16, Ne: 20, S: 32)

A) 0.25 B) 0.5 C) 1 D) 2 E) 4
85.
81. A closed flask contains equal masses of X and Y gases. In
order to calculate the total pressure;
I. The molecular masses of X and Y 30 M 45
II. Temperature cm N2 H2 cm
III. The partial pressure of Y
Which of the quantities given above should be known? V V

A) I only B) III only C) I and III


D) II and III E) I, II and III
When the valve M, is opened, the reaction given below occurs.
N2(g) + 3H2(g)  2NH3(g)
82. In order to make the pressures, of equal masses X and Y What will be the pressure of NH3 in mmHg? (Assume that
gases, equal at the same temperature; temperature remains constant.)
I. The molecular masses and volumes of X, Y gases.
II. The volumes and the numbers of atoms of X and Y gases. A) 5 B) 10 C) 15 D) 20 E) 30

74
90. Some N2 gas is collected over water at 18.5oC and a total
86. Container II
pressure of 756 mmHg. Calculate the partial pressure of N2 in
Container I mmHg. (The vapour pressure of water is equal to 16 mmHg at
18.5oC)
M
He SO2 A) 772 B) 620 C) 752 D) 750 E) 740
V
2V
91. A student collects some amount of H2 gas over water. The
The containers, which are connected valve M, contain He and SO2 mixture contains 3x10-3 mol of H2 (g) and 1x10-4 mol of H2O (g)
gases of which numbers of atoms are equal. under a pressure of 760 mmHg.
When the valve, M, is opened the final pressure becomes 8 atm. Calculate the vapour pressure of water at a given temperature in
What is the pressure of He gas in atm before the valve, M, is mmHg.
opened?
(Assume that temperature remains constant.) A) 738 B) 620 C) 27 D) 24.5 E) 20

A) 3 B) 6 C) 9 D) 12 E) 18
92. Pure water boils at 92oC at 2800 m, and boils at 100oC at sea
level.
According to this information given above at 2800 m and at sea
level;
I. The vapour pressure of pure water at the same temperature.
87. M II. The vapour pressure of water at the boiling point.
III. The atmospheric pressure.
CH4 He Which of the given quantities above is(are) certainly different?
V
4V A) I only B) II only C) III only
D) II and III E) I, II and III
The balloons contain CH4 and He gases with equal masses at the
same temperature. If the pressure of CH4 gas is equal 1 atm, what
will be the final pressure in atm after the valve, M, is opened? 93. The vapour pressure of water is 760 mmHg of sea level and is
567 mmHg at the peak of Uludag;
A) 2 B) 4 C) 6 D) 8 E) 10 I. Boiling points,
II. The vapour pressures at boiling point,
III. The average kinetic energies of molecules at boiling point,
88. The total pressure of equal moles of H2, O2 and N2 gases, is Which of the quantities given above is (are) certainly different for
750 mmHg. water at sea level and at the peak of Uludag?
According to reaction given below the total pressure of the system
becomes 402 mmHg and the initial conditions are obtained; A) I only B) II only D) I and II
H2(g) + 1/2O2(g)  H2O(l) D) II and III E) I, II and III
What will be the vapour pressure of water?
(Neglect the volume of H2O(l) )
Vapour pressure (mmHg)
94.
A) 250 B) 125 C) 67.5 D) 37 E) 27
760
X Y Z

89. The total pressure of the mixture of the excess H2 and 3.2 g
O2 gases is 1.1 atm. Gases react with each other according to the
equation given below; and initial conditions are obtained.
H2(g) + 1/2O2(g)  H2O(l)
Temperature (oC)
If the pressure is equal to 407 mmHg, what is the amount of H2 0
gas in grams, before the reaction occurs?
(The vapour pressure of water is equal to 27 mmHg at 27oC, O:
16, H: 1) The graph shows the relationship between the vapour pressures
and the temperatures of X, Y and Z liquids. According to graph;
A) 0.9 B) 1.2 C) 1.4 D) 1.6 E) 1.8 I. The boiling point of Z is the highest.
II. The vapour pressure of X is the highest.

75
III. The volatility of Y is more than that of X and less than that of Patm=76 cmHg
Z.
Which of the statements given for X, Y and Z liquids at the same
conditions is (are) true?
h= 4 cmHg
A) I only B) II only C) I and II He
D) I, II and III E) II and III

Mercury
97. What happens when air is heated?

A) It expands. A) 800 B) 80 C) 8 D) 8 E) 0.8


B) It contracts.
C) It condenses. 103. The volume of CO2 gas under constant pressure and at
D) It decomposes. 25oC is 40 ml. If the gas is heated to 50oC, what will be its final
volume in ml?
98. Hydrogen and Helium remain in the atmosphere of a planet
because A) 40 B) 43.35C) 433.5D) 500 E) 80

A) They have small mass 104. What is the pressure of 1.6 g of He gas in a 7-L container
B) They are almost the same at OoC? (He: 4, R: 0.082)
C) They have high temperature
D) All other gases have escaped A) 2.56 B) 1.28 C) 0.64 D) 0.32 E) 0.16

105. The density of NO gas at 127oC is 1.83 g/L. What is the


99. The instrument used to measure atmospheric pressure is pressure of NO gas in atm?
(NO: 30, R: 0.082)
A) Barometer A) 1 B) 2 C) 4 D) 5 E) 8
B) Anemometer
C) Wind vane
D) None of these.

100. Which one of the following is not a characteristic of a gas


under pressure?

A) Random-moving molecules
B) Liquefaction of gas
C) Increased kinetic energy
D) Collision between molecules

101. The mixing of molecules of two or more substances as a


result of their motion is called

A) Diffusion B) bonding C) fission

102. What is the pressure of He in the bulb of the manometer in


mmHg?

76
ACID-BASE
1. Which pair of the solutions of the substances does not 8. When 50 mL of 0.4M CuSO4 solution is mixed with 100 mL
conduct electricity? of 0.5M NaOH, Cu(OH)2 precipitated. What must be the mass of
Cu(OH)2 in solution? (Cu(OH)2: 98)
A) H2CO3, C6H13OH
B) H2S, NH4OH A) 2.22 B) 1.96 C) 1.82
C) C2H5OH, C12H22O11 D) 1.72 E) 2.72
D) H2SO4, Fe(OH)3
E) HCI, HNO3
9. How many mL of 0.1M H2SO4 acid is needed to neutralize 4
2. Assume that H2SO4 dissociates into ions completely. When g of NaOH? (NaOH: 40)
2.45 g of H2SO4 is dissolved in 0.5 L water what would be the pH
of the solution? (H2SO4: 98) A) 250 B) 300 C) 400
D) 500 E) 600
A) 1 B) 8 C) 7 D) 14 E) 0
10. How many grams of Ba are needed to prepare a 1 L and
3. Which pair of the following metals does not react with 4.93% of Ba(OH)2 solution having a density of 1.2g/cm3? (Ba:
“King’s Vodka”? 137, O: 16, H: 1)

A) Au, Ag A) 59.4 B) 47.4 C) 42.4


B) Mn, Fe D) 48.9 E) 52
C) Os, Ir
D) Na, K
E) Al, Cu 11. When the solution of NaCl was electrolyzed, 500 kg of 20%
concentrated NaOH solution were obtained by mass. What would
be the volume of H2 gas at STP in m3 produced at the same time?
4. An alloy of Cu and Ag is known to contain 64% Cu and 36% (NaOH: 40)
Ag by mass. How many grams of this alloy are required to react
with 350 mL, 4 M concentrated nitric acid? (Cu: 64, Ag: 108) A) 30 B) 28 C) 32 D) 26 E) 25.8

A) 54.32 B) 50.25 C) 70 D) 30 E) 80
12. How many grams of SO3 gas are necessary to prepare 24.5%
H2SO4 solution in 480 g water?
5. How many grams of KOH are needed to prepare 200 mL of (S: 32, O: 16, H: 1)
16% by mass KOH solution?
(d =1.25g/mL, KOH: 56) A) 30 B) 50 C) 82 D) 120 E) 127.2

A) 35.2 B) 36 C) 34.4
D) 40 E) 36.25 13. How many liters of NH3 at STP are necessary to prepare a
17.5 % NH4OH solution by mass in 366 g of water? (N: 14, H: 1,
O: 16)
6. A known amount of Cu was reacted with excess amount of A) 5.6 B) 22.4 C) 67.2
concentrated H2SO4 to produce SO2 gas. Then, SO2 gas is reacted D) 11.2 E) 49.68
with 300 g 6.3%of NaOH solution. What must be the amount of
Cu. (excess amount of H2SO4 is used, Cu: 64, NaOH: 40)
14. How many grams of 40 % and 10% solutions of KOH by
A) 9 B) 8.4 C) 7.8 mass are necessary to prepare a 600 g of 20% KOH solution by
D) 9.6 E) 15.12 mass?

10% 40%
7. A 4.29 g of the mixture of KOH and Ca(OH)2 is reacted with
50 mL of 2N an acid solution. What is the percentage of Ca(OH)2 A) 200 400
in mixture? B) 350 250
(KOH: 56, Ca(OH)2: 74) C) 400 200
D) 150 450
A) 59 B) 72 C) 18 D) 29.2 E) 32.4 E) 300 300

77
15. What must be the concentration of OH- ions in a solution 23. 100 g sample of Ba(OH)2 is dissolved in 1 L solution . If the
having a 6.5x10-8M H+ ions concentration? pH of the solution is measured to be 13, what will be the
percentage purity of Ba(OH)2 sample by mass? (Ba: 137, O: 16,
A) 2x10 –8 B) 1.54x10-7 C) 1.55x10-6 H: 1)
D) 1.88x10-6 E) 1.8x10-6
A) 20 B) 15.7 C) 8.55
D) 13 E) 17.1
16. What is the pH value of 0.01 N CH3COOH solution having a
dissociation constant of 0.042?
24. Dissolving of 36.5 g HX made a 10 L solution. If the pH of
A) 3.38 B) 4.48 C) 6.72 the solution was measured to be 1, what must be the atomic
D) 67.2 E) 32.4 weight of X?

A) 38.5 B) 40 C) 35.5 D) 33.6 E) 31


17. What is the pH value of the solution that is obtained by the
dissolving of 4 g of NaOH in 1 L water? (NaOH: 40) 25. What are the coefficients of the reactants in the following
reaction?
A) 1 B) 7 C) 8 D) 11 E) 13 Fe + HNO3  Fe(NO3)2 + NH4NO3 + 3H2O

A) 4, 9 B) 3, 9 C) 4, 10
18. A 1 L of HI solution with a density of 1.1 gr /mL was D) 5, 10 E) 3, 8
neutralized by 0.25 L of 0.3 M RbOH solution. What must be the
mass percentage of HI in the acid solution? (HI: 128) 26. What are the coefficients of the products in the following
reaction?
A) 0.7 B) 0.72 C) 0.75 H2O+ Cu2S + HNO3Cu(NO3)2+CuSO4 + NO2 +H2
D) 73.5 E) 0.87
A) 1, 2, 10, 4
B) 1, 5, 10, 4
19. When a 0.15 M Ca(OH)2 solution is dissociated by 50% into C) 1, 2, 10, 3
ions, what will be the dissociation constant of Ca(OH)2? D) 1, 1, 10, 6
E) 3, 3, 10, 10
A) 2.8x10-3 B) 2.4x10-3 C) 12x10-5
D) 2.25x10-2 E) 22.4x10-4
27. What are the molarity and normality of 49% H2SO4 with a
density of 1.385 g/cm3 solution? (H2SO4: 98)
20. Which one of the following is correct about the strength of
acids in order of decreasing? A) 6.9 M, 13.8 N
B) 7.6 M, 12 N
A) HClO4 >HClO3> HClO2 >HClO C) 6.9 M, 17 N
B) HClO3 > HClO2 > HClO > HClO4 D) 5.4 M, 12 N
C) HClO> HClO2 > HClO3 >HClO4 E) 8 M, 14 N
D) HClO2 > HClO4> HClO3 >HClO
E) HClO4 > HClO > HClO3 >HClO2
28. What is the percentage of the HCl acid solution, which is
produced by dissolving 33.6 L HCl acid at STP in 245.25 g
21. Which of the following bases is the strongest one? water? (HCl: 36.5)

A) Al(OH)3 B) Ca(OH)2 C) Ba(OH)2 A) 20 B) 23 C) 22.5


D) Fe(OH)2 E) Sr(OH)2 D) 18.25 E) 19.5

22. 475.6 mL of HCl gas at 17oC and 5 atm was dissolved in 29. What is the molarity of the solution produced by mixing 0.4
enough water to make a 100 L solution. What would be the pOH L of 9 M and 0.6 L of 2 M H2SO4 solutions?
of the solution?
A) 4.8 B) 5.3 C) 3.2
A) 5 B) 13.5 C) 2 D) 3 E) 11 D) 5.12 E) 4.31

78
E) [H+] can be 10-2 M in solution
30. What is the percentage of 8 M HCl solution, which has a
density of 1.23 g/cm3? (HCl: 36.5)
37. Which one of the following is correct for the [H+] in 0.1 M
A) 23.74 B) 20.05C) 17.65 NaOH solution?
D) 24 E) 24.56
A) 1x10-14 B) 1x10-13 C) 0.1
D) 0.01 E) none
31. What are the molarity and normality of 47.7% H3PO4
solution by mass, which has a density of 1.315 g/cm 3? (H3PO4)
38. 100 mL of 0.01 M HCl solution is diluted to 1 L with pure
A) 5.1M, 22N water. What is the pH of the resulting solution?
B) 6.4M, 19.2N
C) 3.1M, 22N A) 1 B) 2 C) 3 D) 7 E) 12
D) 7.3M, 18N
E) 4.81M, 13.2N
39. In the reaction;
HNO2(aq) + NH3(aq)  NO2- (aq) + NH4+(aq)
32. In the neutralization of an 80 g solution of 50% a Which one of the following pairs is the conjugate acid-base pair
monohydroxy base by mass, a 5 L of 0.2 M HCl solution was of each other?
used. What must be the molecular weight of the base?
A) HNO2 and NH3
A) 34 B) 43.6 C) 40 B) HNO2 and NO2-
D) 48.7 E) 51 C) HNO2 and NH4+
D) NH3 and NO2-
E) NO2- and NH4+
33. A diprotic acid has the following compositions by mass:
0.917% H, 47.7% Cr and 51.38% O. If 0.05 L of 4 M NaOH may
neutralize 21.8 g of this acid, what is the molecular formula of 40. How many mL of 0.04 M NaOH solution is needed to
this acid? neutralize 300 mL of HCl solution having a pH of 2?
(Cr: 52, O: 16, H: 1)
A) 75 B) 150 C) 300
A) HCrO4 B) H2Cr2O3 C) H2Cr2O7 D) 600 E) 1200
D) H2SO4 E) H2Cr2O5

41. '' Substances whose solutions conduct electricity are called


34. To neutralize a 1.56 g sample of a trihydroxide base 0.5 L of electrolytes."
HClO4 solution is necessary. If 0.3 L of 0.4 M NaOH solution can According to this, which ones of the following are called
neutralize 1 L of the same HClO4 solution, what must be the electrolytes?
molecular weight of the base? I. NaCl
II. NH3
A) 61 B) 73 C) 78 III. C2H5OH
D) 82.5 E) 74.8 IV. Sugar
V.HNO3

35. What is the [H+] in 0.25 M solution of benzoic acid, A) I, II and III
C6H5COOH? (Benzoic acid, Ka = 6.4x 10-5) B) III, IV and V
C) I, II and V
A) 6.4x10-5 B) 0.25 C) 4x10-3 D) III and IV
D) 8x10-5 E) 8x10-3 E) I and V

36. Which of the following is not a property of acid solutions? 42. " A Swedish chemist, making important contributions to the
study of electrolyte solutions and rates of reactions. Because of
A) turns litmus paper from red to blue his great studies, he was awarded the Nobel Prize in chemistry in
B) have sour taste 1903. And, he defined an acid as a substance that produces H+
C) good conductor of electricity ions, and a base as a substance that produces OH- ions in aqueous
D) react with Zn to produce H2(g) solutions."
79
According to information above, who would be this scientist? 48. Which of the following substances is not a strong electrolyte?

A) Bronsted B) Lewis C) Curie A) NaOH B) HCl C) CH3COOH


D) Arrhenius E) Lowery D) H2SO4 E) NaCl

43. Which one(s) of the following is (are) the common properties 49. Which one(s) of the following ions do(es) not hydrolyze?
of bases? I. Li+ II. NO3- III. K+ IV. NH4+ V. CN-
I. They have bitter taste.
II. They can react with Cu, Ag, Hg, Au and Pt. A) I and II B) II only C) III and IV
III. They react with metal carbonates. D) IV and V E) I, II and III
IV. They form conducting solutions in water.

A) I and IV B) I, II and III C) II, III and IV 50. How many gram Ca(OH)2 must be dissolved in a 500 mL
D) II only E) I only solution whose pH is 12?
(Ca: 40, O: 16, H: 1)

44. A sample of lemon juice has a hydrogen ion concentration of A) 0.74 B) 1.48 C) 0.37
1x10-3 M. In which of the following is [OH-], pH and pOH of the D) 1.1 E) 0.185
lemon juice given correctly?

[OH-] pH pOH 51. According to the reaction below, how many cal heats would
A) 1.3x10-11 3.3 11.7 be liberated from the reaction of 200 mL of HCl whose pH is 1
B) 1x10-12 12 2 with enough NaOH?
C) 1x10-3 11 3 H+ + OH-  H2O H= -14 kcal
D) 4x10-11 3.3 10.7
E) 1x10-11 3 11 A) 840 B) 500 C) 420
D) 320 E) 280

45. Which of the following acids and bases are strong?


I. HCIO4 II. KOH III. NH3 52. Which of the following is false for the properties of bases?
IV. HBr V. CH3COOH
A) They form conducting solutions.
A) I, II and V B) They affect the colors of many dye materials.
B) I, II and IV C) Only amphoter metals react with bases and produce water.
C) II and V D) They react with acids to form salt and water.
D) III and V E) They have bitter taste.
E) I, III and V

53. Which of the following reactions cannot take place?


46. What is the pH value of the resulting solution after 25 mL of
0.1 M Ba(OH)2 has been added to titrate 50 mL of 0.1 M HNO3 A) CaCO3(s) + HCl 
solution? B) Ag (s) + H2SO4 (aq) 
C) Zn + NaOH 
A) 5 B) 11 C) 3 D) 1 E) 7 D) HClO4 + Fe(OH)2 
E) Cu + HCl 
47. A test tube contains pieces of an alloy of silver and
aluminum, and chips of marble, CaCO3. If concentrated HNO3 is 54. What is the ratio of the molarity of H+ ions to the molarity of
added to this test tube, which gas (es) will be produced? OH- ions in a solution whose pH is 9?
A) H2, SO2, CO2 A) 10-7 B) 10-5 C) 10-4
B) H2 NO, CO D) 10 +4
E) 10 +7
C) Cl2, NO2, CO2
D) H2, NO2, CO2
E) H2, N2O, CO2 55. Which of the following is the strongest acid?
A) HCl B) HClO C) HClO2
D) HClO3 E) HClO4
80
64. When a 100 mL of 10 M HCl solution is diluted to 100 L
with pure water, what would be the pH of the resulting solution?
56. Which of the following is the conjugate base of HS- ion?
A) H+ B) S-2 C) H2S D) H2O E) H3O+ A) 2 B) 12 C) 1x10-12 D) 0 E) 1

57. Which of the following acids and bases are weak? 65. 820 mL of HCl gas at 27oC and 3 atm is dissolved in enough
I. NH3 II. HBr III. KOH IV. HNO3 water to make 100 L of solution. What is the pH of the solution?
(R=0.082)
A) I and II B) III and IV C) I and IV
D) II and III E) I, II and III A) 4 B) 11 C) 3 D) 14 E) 7

58. Vinegar is the 5 % of ....................... 66. How many mL of 0.1 M H3PO4 are required to neutralize 150
mL of 0.1 M Ba(OH)2 solution?
A) Sulfuric Acid
B) Citric Acid A) 50 B) 100 C) 150 D) 5 E) 200
C) Acetic Acid
D) Ammonia
E) Sodium Hydroxide 67. A technique in which a known volume and concentration of
one solution in a burette is gradually added to known volume of
another solution until a neutral solution is obtained is called
59. 5.6 g of KOH are dissolved in water to make 1 L of solution. ..............
What is the pH of the solution? (KOH: 56)
A) Neutralization B) Hydrolysis C) Titration
A) 1 B) 2 C) 7 D) 13 E) 14 D) Electrolysis E) Fermentation

60. What is the pH of a 0.00001 M HCl solution? 68. The relative acidity or basicity of a substance is expressed in
A) 10-5 B) –9 C) –5 D) 9 E) 5 terms of pH. pH ranges from 1 to 14. pH 7 is neutral. Acids have
pH values smaller than 7. What is the possible pH value of bases?

61. Which of the following is the conjugate acid of HS- ion? A) exactly 7
B) less than 7
A) H+ B) S-2 C) H2S D) H2O E) H3O+ C) greater than 7
D) same as an acid

62. Which of the following reactions cannot occur?


69. A substance that gives up hydrogen ions to water to form,
A) NH3 + H2S  hydronuim ion is called
B) Al2O3 + NaOH  A) acid B) base C) salt
C) Na2O + HNO3 
D) HCN + HF 
E) Fe + HCl  70. Which of the following changes red litmus paper to blue?

A) acid B) base C) salt


63. In which of the following reactions CO2 gas will be
produced?
71. The pH of a solution tells how acidic or basic the solution is.
A) Zn + HCl  Pure water is neutral and has a pH of
B) NaOH + HCl 
A) 0 B) 7 C) 13
C) Na2CO3 + H2O 
D) MgCO3 + H2SO4 
E) Fe + HNO3  72. A substance with a pH of 13 is a

A) strong acid B) strong base C) salt

81
CHEMICAL EQUILIBRIUM
A) 100 B) 200 C) 10 D) 30 E) 40
1.
What is the equilibrium constant
6. Given the reaction
expression for the following reaction; X2 (g) + 3Y2(g)  2XY3(g) + 22 kcal
3Fe(s) + 4H2O(g)  Fe3O2(s) + 4H2(g)? Which one is the most suitable way to increase the equilibrium
concentration of XY3?
[ H2 O]4 [ Fe]3 [ H 2 ]4
A) Kc= B) Kc= A) Adding a catalyst to the system.
[ Fe3O4 ][ H2 ]4 [ H2 O]4 B) While decreasing the temperature, increasing the volume.
[ H2 O]4 [ H2 ]4 [ Fe3O4 ] C) While increasing the temperature decreasing the pressure.
C) Kc= D) Kc= D) While increasing the temperature, increasing the volume.
[ H 2 ]4 [ Fe]3 [ H2 O]4 E) While decreasing the temperature, decreasing the volume.
[ H 2 ]2
E) Kc=
[ H2 O]4 7. The rate constant of forward and reverse reactions are 3x104
and 6x108, respectively for the reaction,
X2 + Y2  2XY
1. Given the reactions; at a certain temperature. What’s the equilibrium constant of this
A + 2B  2C K1 = 81 reaction?
A + B D K2 = 27
What is the numerical value of K for the following reaction at A) 3 x 104 B) 6 x 108 C) 9 x 108
D) 2 E) 0.5
same temperature; B + D  2C ?

A) 3 B) 9 C) 9x27 D) 27x81 E) 27/81


8. Consider the reaction,
CO + Br2  COBr2 Kc = 4
2. For the reaction; When the reaction reaches to equilibrium there re 0.4
mol of CO, 0.8 mol of Br2 and 0.8 mol of COBr2 at
2SO2(g) + O2(g)  2SO3(g) K = 2.56x10-6
What is the K value for? equilibrium in a container at constant temperature.
What is the volume of the container?
SO3(g)  SO2(g) + 1/2O2(g)
A) 0.5 B) 1 C) 1.6 D) 3.1 E) 2.5
A) 1/25 B) 1/625C) 312.5D) 625 E) 1/1250

9. There are 4 mol of N2O4 in a 1 L container. When 50 % of


3. Which one of the following is the relation of Kp and Kc of
N2O4 dissociates to NO2 gas according to the reaction,
the reaction?
N2O4(g)  2NO2(g)
H2(g) + I2(g)  2HI(g)
the system will be at equilibrium at constant temperature. What
would be the equilibrium constant of the reaction?
A) Kp=Kc B) Kp=1/Kc C) Kp=2Kc
D) Kp= Kc E) Kp=Kc/2 A) 1 B)2 C) 6 D) 8 E) 15

4. What is n for the following reaction? 10. 2SO2(g) + O2(g)  2SO3(g) H  0


CaCO3(s)  CaO(s) + CO2(g) According to the reaction, given above, when the volume of the
system is decreased by half,
A) 2 B) 0 C) 1 D) 4 E) 3 I. Rate of formation SO3 increases
II. Kc decreases
III. Number of O2 molecules decreases
5. Consider the reaction, Which one(s) is (are) true?
N2(g)+ 3H2 (g)  2NH3(g)
0.3 mol of N2 and 0.4 mol of H2 are put into a 1 L vessel at a A) I only B) I and II C) I and III
certain temperature. When the equilibrium is established, there is D) II and III E) I, II and III
0.5 mol of gases totally in the vessel what would be the
equilibrium constant of the reaction?
11. The equilibrium constant, Kc, for the reaction
82
2HI(g)  H2(g) + I2(g) A) I only B) III only C) I and II
is 0.36 at constant temperature. Which will be the mole number of D) I and III E) I, II and III
H2 at equilibrium, if 2 mol of HI is placed into a container at same
temperature?
17. Given:
A. 0.50 B) 1.00 C) 0.54 D) 1.25 E) 0.42 2NO + O2  2NO2 Kc = 5
2NO2  N2O4 Kc = 4
What’s the equilibrium constant of the reaction?
12. N2 (g) + 3H2 (g) 2NH3 (g) H = 22.1 kcal /mol 2NO(g) + O2 (g)  N2O4(g)
Which one(s) do(es) not change the concentration of N2 at
equilibrium? A) 9 B) 1 C) 20 D) 4/5 E) 9
I. Decreasing the volume of container
II. Addition of H2
III. Addition of H2SO4 18. Given the reactions,
IV. Addition of He Y2(g) + 3Y2(g)  2XY3(g)
2Y2(g) + Z2(g)  2Y2Z(g)
A) I only B) III only C) III and IV The reactions are at equilibrium at constant temperature When
D) IV only E) I, II and III some X2 is added to the container;
I. XY3 concentration increases
II. Y2Z concentration decreases
13. Consider the reaction, III. Z2 concentration decreases
2NH3 (g)  N2 (g) + 3H2 (g) Which one(s) would be correct?
How many times the equilibrium constant decreases when the
volume is doubled at constant temperature? A) I only B) I and II C) I and III
D) II only E) I, II and III
A) 10 B) 1 C) 10 D) 1 E) No change

19. When 0.6 mol of H2 and I2 gases are put into a 1L container
14. Following reaction at constant temperature, the following reaction occurred
X + 2Y  Z + T H2 (g) + I2 (g)  2HI(g)
has a equilibrium constant of 104 at 250C. When the temperature After a while the reaction reached to and has a Kc value of 4.
is raised to 1000C it becomes 104. By knowing this, What would be the mole number of HI at equilibrium?
I. The reaction is endothermic
II. The mole number of T at 1000C is greater than that at 250C A) 1 B) 0.6 C) 0.3 D) 2 E) 0.2
III. The reaction is exothermic
Which one(s) is (are) true?
20. Given the reaction,
A) I only B) III only C) I and II H2(g) + S(s)  H2S(g)
D) II and III E) II only I. Increasing the pressure
II. Addition of H2
15. The following reaction was allowed to come equilibrium III. Removal of H2S
XY+Z IV. Decreasing of the pressure
At equilibrium the 11.2 L container has 2.28 mol of X, 4.48 mol Which one(s) of them will shift the direction of the reaction to
of Y and 3.36 mol of Z gases at 00C. What’s the equilibrium forward?
constant depending on to partial pressures of gases at same
temperature? A) I and II B) I and III C) II and
IV
A) 13.44 B) 4.48 C) 8.96 D) 16.8 E) 22.8 D) II and III E) II and IV

16. Consider the reaction and its enthalpy 21. Given the following reactions, which ones of them will shift
X2 + 3Y2  2XY3 H  0 to forward if the volume and temperature are decreased?
To increase the concentration of XY3 which one(s) must be I. N2 (g) + O2 (g)  2NO  Q
applied to the reaction? II. CO2 (g) + C(g)  2CO(g)  Q
I. Temperature must be increased III. 2CO(g) + O2 (g)  2CO + Q
II. X2 must be drawn IV. CO(g) + H2O(g)  CO2 (g) + H2 (g) + Q
III. XY3 must be drawn V. 2SO2 (g) + O2 (g)  2SO3 + Q

83
A) I, II and III 26. Consider the reaction,
B) II, III and IV 2SO2(g) + O2(g)  2SO3(g) H  0
C) II, III and IV When the volume of the container is decreased by half, which
D) III, IV and V one(s) of the following will occur?
E) I, II and IV I. The rate of formation of SO3 increases
II. Kc value decreases
III. Number of O2 molecules decreases
22. Given the following reactions,
I. N2 + O2  2NO  Q A) I only B) I and II C) I and III
II. 2CO + O2  2CO2 + Q D) II and III E) I, II and III
III. CO2 + C  2CO  Q
IV. 2SO2 + O2  2SO3 + Q
V. N2 + 3H2  2NH3 + Q 27. There are 1.12 mole of X, Y and Z gases at equilibrium in a
If the pressure is increased and the temperature is decreased for 2.24 L flask at 0oC for the following reaction.
all reactions which ones of them will shift to forward? X Y+Z
What would be the Kp value of the reaction at same temperature?
A) I, III and IV
B) II, IV and V A) 5.6 B) 11.2 C) 22.4 D) 33.6 E) 44.8
C) I, II and IV
D) III, IV and V
E) II, III and IV 28. Consider the reaction,
PCl3 + Cl2  PCl5 Kc = 2.5
When the reaction is at equilibrium at constant temperature there
23. Given the equilibrium reaction, are 0.6 mol of PCl3, 0.4 mol Cl2 and 1.2 mol PCl5 gases at
H2 (g) + Cl2 (g)  2HCl(g) equilibrium. What’s the volume of the container?
If the pressure of the system is increased how the reaction will A) 0.5 B) 1 C) 2.5 D) 4 E) 5
shift?
I. Reverse
II. Forward 29. Given the reaction,
III. No change N2O4(g)  2NO2 (g)
At initial 4 mol of N2O4 is present in a 1L container. When 50 %
A) I only of N2O4 dissociate the system reaches to equilibrium. What would
B) II only be the Kc value of the reaction?
C) III only
D) All possible A) 1 B) 4 C) 6 D) 8 E) 15
E) Can’t be commended

30. 50 % of 2 mol of COCl2 gas in a 1L flask dissociates at


24. Given the equilibrium reaction, constant temperature according to the reaction
CO + NO2  CO2 + NO COCl2 (g)  CO(g) + Cl2(g)
There are 0.4 mol of CO and CO2, 0.5 mol of NO2 and 0.6 mol of When the system is at equilibrium what would be the equilibrium
NO gases at equilibrium in 2 L container at constant temperature. constant of the system?
Which one is the Kc value of the reaction at same temperature?
A) 0.5 B) 1 C) 2 D) 4 E) 8
A) 0.4 B) 0.6 C) 1 D) 1.2 E) 1.6

31. Given the equilibrium reaction,


25. 0.8 mol of A and 0.6 mol of B are put into a 1 L container at N2(g) + 3H2(g)  2NH3(g)
constant temperature. After a while following reaction takes I. Addition of N2
place II. Addition of He
2A + B  C + 2D III. Addition of H2SO4
When the reaction reaches to equilibrium there are 0.2 mol of C Which one(s) given above affect(s) the direction of the reaction to
in the container. What’s the Kc value of the reaction at same forward?
temperature?
A) I only B) I and III C) I and II
A) 0.2 B) 0.4 C) 0.5 D) 1 E) 1.25 D) III only E) I, II and III

84
A) The quantity of CO at equilibrium increases.
32. Consider the equilibrium reaction given below, B) The quantity of O2 at equilibrium increases.
N2(g) + 2O2(g)  2NO2(g) H  0 C) The quantity of CO2 at equilibrium decreases.
I. Addition of reactants D) The total pressure increases.
II. Increasing the pressure E) The value of equilibrium constant increases.
III. Increasing the temperature
Which one(s) of the above will change the equilibrium to forward
side? 38. In an equilibrium reaction, which of the following will
change the equilibrium constant itself?
A) I only B) III only C) I only II
D) II only III E) I, II and III A) A change in temperature
B) A change in pressure
C) A change in the catalyst used
33. Consider the reaction, D) A change in the molar concentration of reactants
N2 (g) + 3H2 (g)  2NH3 (g) Kc = 25 E) A change in the molar concentration of products
What would be the Kc value of the reaction given below?
NH3(g)  1/2 N2 (g) + 3/2 H2(g) Kc =?
39. In which one of the following reactions, the equilibrium will
A) 0.2 B) 0.04 C) 5 D) 2.5 E) 1/ 625 not shift in the direction of products by the increasing of
temperature?

34. 2H2 + O2  2H2O K1 = a A) H2O(l)  H2O(g)


H2 + CO2  CO + H2O K2 = b B) H2O(s)  H2O(l)
What is the Kc value of the reaction ? C) CO2(g)  C(s) + O2(g) H= +94 kcal
2CO + O2  2CO2 D) SO2(g) + 1/2O2(g)  SO3(g) + 23 kcal
E) CH3OH(g) + heat  CO(g) + 2H2(g)
A) a B) a xb C) a
a b b2
D) a2 E) a x b2 40. For the equilibrium A(g) + B(g)  2C(g) the rate
2b constant for the forward and reverse reactions are 3.16x10-2 and
0.79 respectively. What is the numerical value of the equilibrium
constant?
35. Consider the reaction,
3A(g) + 2B(g)  2C(g) + D(g) + Q A) 1.58x10-2 B) 2.5x10-2 C) 0.02
I. Increasing the temperature D) 0.04 E) 1
II. Removal of B
III. Removal of C
Which one(s) of the above will increase 41. Which one of the following will increase the equilibrium
the concentration of D? concentration of Cl2 for the reaction?
PCl5(g)  PCl3(g) + Cl2(g) H= +22 kcal
A) I only B) II only C) III only
D) I and III E) I, II and III A) Decreasing the temperature
B) Increasing the concentration of PCl3
C) Increasing the pressure (temperature constant)
36. Given the reaction, D) Increasing the concentration of PCl5
2XO3 (g)  2XO2 (g) + O2(g) E) Adding a catalyst
A 2 mol of XO(g) placed in a 2 L flask at 4000C decomposes 50 % to
establish equilibrium. What would be the Kc value of the reaction at
same temperature? 42. If K is small it indicates that equilibrium occurs

A) 0.25 B) 0.50 C) 0.75 D) 1.25 E) 2.50 A) at a low product concentration.


B) at a high product concentration.
C) after a considerable time.
37. Which one of the following doesn’t occur with the increasing D) with a help of a catalyst.
of temperature, after equilibrium is achieved? E) with no forward reaction .
CO(g) + 1/2O2(g)  CO2(g) H= -67.6 kcal

43. For the reaction,


85
2A(g) + B(g)  2C(g) H= -Q kcal 49. Which one is the equilibrium constant expression for the
Which of the following pairs will shift the equilibrium in the following reaction?
direction of the production of more C? MgCO3(s)  MgO(s) + CO2(g)

A) Increasing the temperature and pressure A) Kc = [MgO] [CO2] B) Kc = [CO2] C) Kc = [CO2]


B) Decreasing the pressure and adding a catalyst [MgCO3]
C) Decreasing the temperature and decreasing the pressure D) Kc = [CO2] [MgO] E) Kc = [MgCO3]
D) Decreasing the temperature and increasing the pressure [MgCO3]
E) Increasing the volume of the container

50. At a certain temperature a mixture of X2(g) and Y2(g) was


44. At a particular temperature, Kp for the reaction, prepared by placing 2 mol of X2 and 4 mol of Y2 into a 2 L
CO2(g) + C(s)  2CO(g) container. After a while the equilibrium
is 2.5 and partial pressure of CO2 at equilibrium is 0.4 atm. What X2(g) + Y2(g)  2XY(g)
is the partial pressure of CO at equilibrium? was established. It was determined that the mole of XY was 2 at
equilibrium. What is the Kc value of the reaction at this temperature?
A) 1 atm B) 1.25 atm C) 2.5 atm
D) 0.8 atm E) 0.2atm A) 2.66 B) 2 C) 4 D) 1.33 E) 0.5

45. At a particular temperature, K is 625 for the reaction, 51. What is the relationship between Kp and Kc for the reaction
2SO2(g) + O2(g)  2SO3(g). given below?
What is K for the reaction? NH4HS(s)  NH3(g) + H2S(g)
SO3(g)  SO2(g) + 1/2O2(g) ?
A) Kp = Kc
A) 1/25 B) 1/625C) 312.5D) 625 E) 1/1250 B) Kp = Kc(RT)2
C) Kp = Kc(RT)-2
D) Kp = KcRT
46. Which one of the following reactions cannot be equilibrium E) Kp = Kc
reaction? RT

A) C2H5OH(l)  C2H5OH(aq) + heat


B) 2NH3(g)  N2(g) + 3H2(g) H = +44 kcal 52. Given the reaction,
C) I2(s)  I2(g) A(g) + 3B(g)  2C(g) + D(g)
D) C(s) + H2O(g)  H2(g) + CO(g) H = +31.4 kcal 1 mole of each A and B are placed in a 1 L container. After the
E) CaCO3(s) + heat  CaO(s) + CO2(g) equilibrium has been established, 0.4 mol of C is present in the
container. What is the Kc value for the reaction?

47. What is the unit of Kc for the reaction given below? A) 0.25 B) 0.625C) 1.56 D) 1.1 E) 2.785
2SO3(g)  2SO2(g) + O2(g)

A) M B) M2 C) M-2 D) M3 E) M-1 53. For the reaction,


X2(g) + Y2(g)  2XY(g)
Kp = 0.02 and Kc = 0.001 at the same temperature. What is the
48. Suppose that some Pb(NO3)2(s) was put into a 4.0 L tank and equilibrium temperature in K?
heated to 227oC. At this temperature Pb(NO3)2 partially
decomposed according to the reaction. A) 273 B) 244 C) 96 D) 20 E) 12.2
2Pb(NO3)2(s)  2PbO(s) + 4NO2(g) +O2(g)
At equilibrium the total pressure of the gases in the tank was
measured to be 10 atm. What is the equilibrium constant, Kc, for 54. Given the reactions with their equilibrium constants
the reaction at 227oC? A(g) + B(g)  C(g) + D(g)K1 = 4
(R = 0.082 L.atm/mol.K) E(g) + B(g)  2C(g) K2 = 0.5
What is the equilibrium constant of the reaction?
A) 4x10-5 B) 2x10-5 C) 1x10-5 A(g) + C(g)  E(g) + D(g) K=?
-5
D) 0.4x10 E) 8x10-5
A) 2 B) 0.5 C) 4 D) 8 E) 0.125

86
55. Consider the reaction and its Kc value, A) 0.18 B) 0.09 C) 0.9 D) 0.45 E) 1.8
N2(g) + O2(g)  2NO(g) Kc = 2
When 1 mole of each N2 and O2, and 2 mole of NO gases are
mixed in a 1 L box, what would be the direction of the reaction 61. 4 moles of NH4Cl are placed in a container at a particular
and Qc value at same temperature? temperature. The following reaction occurs;
NH4Cl(s)  NH3(g) + HCI (g)
A) reverse, 4 B) forward, 2 C) reverse, 8 At equilibrium the total pressure of the mixture is 1.2 atm. What
D) forward, 4 E) forward, 8 is the numerical value of Kp at the same temperature?

A) 0.06 B) 0.36 C) 0.6 D) 1.2 E) 1.44


56. For the reaction
H2 (g) + CO2 (g)  H2O (g) + CO (g) K = 1.6
When equilibrium is established, [CO2] = 0.5 M, [H2]=1.0 M and 62. 2 moles of HI are placed in a 5-L container. The following
[CO] = 2.0 M. What is [H2O] at equilibrium? reaction occurs;
2HI (g)  I2(g) + H2(g)
A) 0.1 M B) 0.4 M C) 0.5M D) 1.0 M E) When equilibrium is established, the weight of unreacted HI is found
2.0 M to be 200 grams. Calculate the value of K.
(HI: 128)

57. If Kp for the reaction A2 (g) + B2 (g)  2AB (g) was equal A) 50.0 B) 3.12 C) 0.01 D) 0.02 E) 1.28
to 1.5 at 250C, what would be the numerical value of K in terms
of molar concentrations?

A) 1/1.5 63. In which one of the following reactions, tendency toward


B) l.5/(0.082x298) minimum energy favors the formation of reactants?
C) l.5x 0.082
D) 1.5 A) H2(g) + Cl2(g)  2HCI(g) H= -44kcal
E) l.5x25 B) CO(g) + 1/2O2(g)  CO2(g) + heat
C) H2O(g)  H2O(l)
D) 5SO2(g) + 1/2O2(g)  5SO3(g) + 23 kcal
58. At a definite temperature, when 2 moles of PCI5 are plated in E) l2(g) + 1.6 kcal  l2 (in alcohol)
a 2-L container, 60% PCl5 dissociates into PCl3(g) and Cl2 (g) and
comes equilibrium:
PCI5 (g)  PCI3 (g) + Cl2 (g) 64. In the reaction
What is the numerical value of K? PbSO4(s) + H+(aq)  Pb+2(aq) + HSO4- (aq)
Which of the following will not occur by the addition of Pb+2 to
A) 0.9 B) 1.1 C) 1.8 D) 0.55 E) 1.0 the system at equilibrium?

A) The amount of PbSO4 increases


B) [H+] increases
59. What occurs when a reaction is at equilibrium and more C) [HSO4-] increases
reactant is added to the container? D) [Pb +2] increases
E) Equilibrium shifts to left.
A) The equilibrium remains unchanged.
B) The forward reaction rate increases.
C) The reverse reaction rate increases. 65. Which of the following statements concerning catalysts is not
D) The forward reaction rate decreases. correct?
E) The reverse reaction rate decreases.
A) Catalysts do not affect the position of equilibrium
B) Catalysts affect the speed at which equilibrium is reached
60. Consider the reaction; C) Catalysts increase the rate of forward reaction
COCI2 (g)  CO (g) + Cl2 (g) D) Catalysts increase the rate of reverse reaction
1 mole COCI2 is placed in a 2-L container at a definite E) Le Chatelier's principle is useful in predicting the effect of
temperature. When the system reaches equilibrium, 60 percent of catalysts upon a system at equilibrium.
the original COCl2 is used up. What is the equilibrium constant of
the reaction?
66. The equilibrium constants of the following reactions are
given.
87
NO(g) + 1/2O2(g)  NO2(g) K1 = 1.1 x l0-6
2NO2(g)  N2O4 K2 = 7.4x l0-I2
What is the value of K at the same temperature for? 72. The equilibrium constant for the reaction
2NO(g) + O2(g)  N2O4(g)? 2CO(g) + O2(g)  2CO2(g)
has a value of 2.2 x 1022 at 1000 K and 9.7 x l07 at 2000 K. Which
A) 8.95x10-24 B) 8.6x10-14 C) 9.8x10-12 of the following statements concerning this reaction is correct?
D) 6.2x 10-12 E) 6.17
A) At constant volume, the pressure at 1000 K is lower than the
pressure at 2000 K
67. In which one of the following reactions the tendency toward B) The activation energy for the forward reaction is higher than
maximum randomness favors reactants? that of the reverse reaction.
C) At high temperatures, the CO2 molecules are more stable than
A) 4Fe(s) + 3O2(g)  2Fe2O3(s) + 400 kcal CO molecules
B) l2(s) + 1.6 kcal  l2 (alcohol) D) The temperature must be increased in order to increase the rate
C) CaCO3(s) + heat  CaO(s) + CO2(g) of forward reaction
D) H2O(l)  H2O(g) E) The production of CO2 is an endothermic reaction.
E) H2O(s)  H2O(l)

68. A mixture of 1 mole of H2 and 1 mole of I2 are heated


together in a 1-L vessel at 448oC. These substances react as
follows;
H2(g) + I2(g)  2HI(g)
K for this equilibrium is 50. What is the partial pressure of H2 in
atm at equilibrium?

A) 2.1 B) 1.3 C) 2.6 D) 1.0 E) 3.9

69. A mixture of 0.8 mole of A and 0.8 mole of B are placed in a


1-L vessel. The following equilibrium is established;
2A + B  2C + D
At equilibrium, n c = 0.6 mole. Calculate K.

A) 1.8 B) 0.9 C) 9.0 D) 5.4 E) 4.5

70. In the reaction


N2(g) + 3H2(g)  2NH3(g) + 22 kcal
Which of the following will not increase the amount of NH3 at
equilibrium?

A) Increasing the concentration of N2


B) Increasing the concentration of H2
C) Removal of NH3 as rapidly as it is formed
D) Increasing the pressure (temperature constant)
E) Increasing the temperature.

71. A quantity of NH3 was placed in a 1-L vessel at a particular


temperature. When the following equilibrium is reached at the
same temperature;
2NH3(g)  N2(g) + 3H2(g)
the vessel contains 0.6 mole H2. If K = 0.12 at this temperature,
what is the number of mole of NH3 before reaction?

A) 0.2 B) 0.4 C) 0.6 D) 0.8 E) 1.0

88
ELECTROCHEMISTRY A) Potential of the cell is 0.2 V
1. The activity order of the X, Y, and Z metals is Z>X>Y. B) Sn electrode is cathode
According to this; C) Cu electrode is anode
D) Concentration of Cu+2 ion decreases
X Y Z E) Electrons in the external circuit flow toward the Sn electrode

Y+2 Cl- Z+2 Cl- X+2 Cl-


6.
I II III V

In which of the beakers given above do(es) reaction take place?


Salt Bridge
A) I only B) I only C) III only X(s) Y(s)
D) I and II E) I and III

2. How many grams of Zn(s) is required to convert all Ag+ ions


to Ag in 100 mL of 0.2 M AgNO3 solution? (Zn: 65)
In the electrochemical cell given above, electrons in the external
A) 0.65 B) 1.3 C) 2.6 D) 6.5 E) 13 circuit flow from Y electrode to X electrode. According to this;
I. X electrode is cathode.
II. Y electrode is anode.
3. Aluminum (Al) reacts with MnCl2, ZnCl2 and FeCl2 III. After a while the mass of X decreases.
solutions. Zinc (Zn) does not react with AlCl3 and MnCl2 but it Which one(s) of the following given above is (are) true?
reacts with FeCl2. Which of the following is the activity the Al,
Zn, Fe and Mn in decreasing order? A) I only B) II only C) III only
D) I and II E) I, II and III
A) Al, Zn, Mn, Fe
B) Al, Mn, Zn, Fe
C) Al, Fe, Zn, Mn 7. Which statement is true for the following balanced equation?
D) Fe, Al, Zn, Mn Al + 3NaOH  Na3AlO3 + 3/2H2
E) Fe, Zn, Al, Mn
A) Al is reduced
B) Al is oxidizing agent
C) Na is reducing agent
4. Zn(s) + 2Ag+  Zn+2 + 2Ag Eo =+1.56 V
D) H+ is reduced
Zn+2 + 2e-  Zn(s) Eo = +0.76 V
E) Al is a better oxidizing agent than H2
According to the reactions given above, what would be the
oxidation potential of Ag?

A) 0.8 B) 2.32 C) -0.8 8. Which of the following reactions will be spontaneous at


standard conditions?
D) -2.32 E) 0.78
A) Zn + Mg+2  Zn+2 + Mg
5. B) Cu + Co+2  Cu+2 + Co
C) Fe + Zn+2  Fe+2 + Zn
V
D) Cd + Co+2  Cd+2 + Co
Salt Bridge E) 2Ag + Ni+2  2Ag+ + Ni

Sn Cu
9. Which one of the following reactions will be non
spontaneous at standard conditions?

A) Zn+2 + Mg  Zn + Mg+2
1M Sn(NO3)2 1M Cu(NO3)2 B) Cu+2 + Co  Cu + Co+2
Sn + 2e  Sn(s) E = -0.14 V
+2 - o
C) Fe+2 + Zn  Fe + Zn+2
According
Cu+2 + to figure
2e-the Cu(s)andEocell potentials
= +0.34 V which one of the D) Cd+2 + Co  Cd + Co+2
following is true? E) 2Ag+ + Ni  2Ag + Ni+2

89
E) 100 ml water must be added to the II. container

10. Which one is wrong for the following reaction?


Ca(s) + 1/2O2(g)  CaO(s) 15. “ When the metallic copper is immersed in a solution
containing Ag+ ions, the metallic copper dissolves in the solution
A) Ca loses electrons producing Cu+2 ions, and the metallic silver is deposited on the
B) Ca is oxidized surface of the copper.”
C) O is reduced substance Which one(s) of the following is (are) true about this experiment?
D) Ca is reductant I. Cu is oxidized
E) O is more active than Ca II. The related reaction must be
Cu(s) + 2Ag+(aq)  2Ag(s) + Cu+2(aq)
III. Ag is reductant
11. Which one is wrong for the following reaction? IV. Cu is better reducing than Ag
Fe+2 + Cr2O7-2 + H+  Fe+3 + Cr+3 + H2O
A) I only B) I and II C) I, II and IV
A) Fe+2 is oxidized D) I and IV E) II, III and IV
B) Cr is reduced
C) Fe+2 is a better reducing agent than Cr2O7-2
D) Cr2O7-2 is a oxidizing agent 16. In which one(s) of the following redox reactions is the Mn
E) Fe+2 is oxidant atom both oxidized and reduced?
I. MnO2 + NO2- + Cl-  NO3- + NO3- + Mn+2 + MnCl2
II. MnO4-2  MnO2 + MnO4-
12. Which one(s) is(are) reducing agent in the following reaction III. Cl- + MnO4- + H+  Cl2 + Mn+2 + H2O
given below?
FeS + NO3- + H+  Fe+3 + SO4-2 + NO + H2O A) I only B) II only C) I and II
D) II and III E) III only
A) Fe only B) S only C) N only
D) N and H+ E) Fe and S
17. Which one(s) is correct about the following redox reactions?
5Fe+2 + Mn+7  5Fe+3 + Mn+2
13. Which one(s) is (are) oxidizing agent in the following +2
I. Fe is oxidized
reaction given below? II. Mn+7 is reduced
Bi2S3 + NO3-  Bi+2 + SO4-2 + NO III. Mn+7 is more active than Fe+2
IV. Fe+2 is a better reductant than Mn +7
A) Bi only B) N only C) S only
D) Bi and N E) N and S A) I and III B) I and IV C) I, II and IV
D) II and III E) I, II and III

14. V
18. Which one(s) is (are) true about the following redox reaction?
6Fe+2 + Cr2O7-2 + 14H+  6Fe+3 + 2Cr+3 + 7H2O
Salt Bridge I. The reaction is electrically balanced
Zn Zn II. Cr2O7-2 is oxidized
III. Fe+2 is a better reducing agent than Cr 2O7-2
IV. In the reaction, six electrons are transferred

A) I and III B) I and IV C) II, III and IV


D) I, II and III E) I, III and IV
100 mL 0.1M Zn +2 100 mL 0.2 M Zn +2
I II
In order to make the cell voltage zero, which of the following 19. When the following reaction is balanced in acidic media,
must be done? what would be the coefficient of water?
Cl2 + SO2  SO4-2 + Cl-
A) 50 mL water must be added to the I. container
B) 50 ml water must be added to the II. Container A) 4 B) 3 C) 2 D) 1 E) 6
C) To the I. container 100 mL of 0.2 M Zn +2 solution must be
added
D) 50 ml water must be vaporized from II. container
90
20. When the following reaction is balanced in basic media, what
would be the coefficient of water? 26. For the cell,
Cr2O7-2 + I-  Cr+2 + I2 Cd(s) + 2Ag+(aq)  Cd+2(aq) + 2Ag(s) Eocell=1.2V ?
What is the cell potential if the concentrations of ions are as
A) 7 B) 5 C) 4 D) 3 E) 2 follows? [Cd+2]=0.1M, [Ag+]=1M

A) 1.14 B) -1.14 C) 1.23


21. A strip of magnesium immersed in 1M-magnesium nitrate, D) -1.23 E) 1.20
and a strip of zinc immersed in 1M zinc nitrate solutions. A wire,
a voltmeter, and a salt bridge complete the circuit. During the cell
operates it is observed that the weight of magnesium electrode 27. For the following cell,
decreases. According to this information, which one(s) of the Mg(s) + Zn +2(aq)  Mg+2(aq) + Zn(s) Eocell = 1.61 V
following is (are) true? What is the cell potential if the concentrations of ions are as
I. Anode electrode is Zn follows? [Mg+2]=0.01M, [Zn+2]=1M
II. Cathode electrode is Mg
III. The concentration of Mg+2 will increase A) +1.66 B) +1.54 C) +1.60
D) +1.23 E) +0.66
A) I only B) II only C) I and II
D) III only E) I, II and III
28. Which one(s) of the following may be applied to protect
metals from corrosion?
22. Which one is not correct for the Mg-Zn standard cell? I. Metals may be painted
(Eo(Mg+2/Mg)= -2.36V, Eo(Zn+2/Zn)= -0.76V) II. Metals may be galvanized
III. Metals may be connected to a less active metal
A) Eocell = +1.6V
B) Mg is oxidized A) I only B) II only C) I and II
C) Zn is oxidizing agent D) II and III E) I, II and III
D) Zn electrode is anode
E) Mg is a better reducing agent than Zn
29. Which one(s) of the following cannot be applied to protect
metals from corrosion?
23. Which one is correct for the Zn-Cu standard cell? I. They may be plated with a thin layer of less oxidized substance
(Eo(Zn+2/Zn)= -0.76V, Eo(Cu+2/Cu)= 0.34V) such as gold, platinum or thin
II. They may be painted
A) Zn is reduced III. They may be connected to a less active metal
B) Cu is oxidant
C) Eocell = +1.1V A) I only B) II only C) III only
D) Zn is a worse reducing agent than Cu D) I and II E) I and III
E) Cu electrode is anode
30. I. Mg + 1/2O2  MgO
II. CO + 1/2O2  CO2
24. Given the following balanced reduction half reaction; III. Fe2O3 + 3CO  2Fe + 3CO2
MnO4- + 8H+ + 5e-  Mn+2 + 4H2O Eo= 1.51V In the reaction, given above which one(s) of the substances
What would be the standard potential of the following reaction? underlined is (are) oxidized?
2Mn+2 + 8H2O  2MnO4- + 16H+ + 10e- Eo= ?
A) I only B) II only C) III only
A) -3.02 B) +3.02 C) -0.755 D) I and II E) I, II and III
D) +0.755 E) -1.51

31. In which of the following reactions is the substance


25. Given the following balanced reduction half reaction; underlined reduced?
O2(g) + 2H+ + 2e-  H2O2 Eo= -0.69V
What would be the standard potential of the following reaction? A) Zn + 2HCl  ZnCl2 + H2
4H2O2  4O2(g) + 8H+ + 8e- Eo= ? B) 2Na + 1/2O2  Na2O
C) H2 + CuO  Cu + H2O
A) -2.76 B) +2.76 C) +0.69 D) 3CO + Fe2O3  2Fe + 3CO2
D) +1.38 E) -1.38 E) N2 + 3H2  2NH3

91
A) 96 B) 64 C) 56 D) 40 E) 24

32. Following reactions are given;


Au(s)  Au+3 + 3e- Eo = +1.4 V
Cd+2 + 2e-  Cd(s) Eo = -0.4 V
According to these, what is the potential of this cell?

A) 2.1 B) 1.8 C) 1.5 D) 1.1 E) 0.9

33. According to the reactions below;


Sn+2 + I2(s)  Sn+4 + 2I- Eocell=0.39V
Sn + 2e  Sn
+4 - +2
Eo=0.15V
What would be the standard reduction potential of iodine?

A) -0.54 B) -0.24 C) 0.24


D) 0.54 E) 0.78

34. If the following oxidation-half reaction were balanced, what


would be the coefficient of the electron?
HSO3- + H2O  SO4-2 + H+ + e-

A) 1 B) 2 C) 3 D) 4 E) 5

35. If the following reduction-half reaction were balanced, what


would be the coefficient of the electron?
ClO3- + H+ + e-  Cl2 + H2O

A) 10 B) 8 C) 6 D) 5 E) 3

36. When molten AlCl3 is electrolyzed at one of the electrodes,


2.24 L of Cl2 gas at 0oC and 3 atm is collected. At the other
electrode how many grams of Al will be deposited? (Al: 27)

A) 5.4 B) 8.1 C) 16.2 D) 24.3 E) 32.4

37. 200 mL of 0.5 M CuSO4 solution is electrolyzed with a 2 A


current for 4825 s. What must be the concentration of Cu+2 ions at
the end of the electrolysis?

A) 0.5 B) 0.25 C) 0.125D) 0.1 E) 0.02

38. When XCln is electrolyzed, 8 grams X and 4.48 L Cl2 gas at


STP are collected. According to this, what must be the value of n?
(X: 40)

A) 1 B) 2 C) 3 D) 4 E) 5

39. When molten XCl2 salt is electrolyzed with a 2 A current for


4825 s, 3.2 grams X metal will be deposited. What must be the
atomic weight of the X metal?

92
ELEMENTS
38. Which of the following oxides of nitrogen is the most
33. Which one(s) is (are) given correctly important in living organisms?
about the usage areas of carbon black?
I. As filler in rubber tires A) N2O B) NO2 C) NO
II. As a pigment in printing inks D) N2O4 E) N2O3
III. In plastics and fibers
IV. As a transfer material in carbon papers
39. Which one(s) is (are) given correctly about the usage
A) I and II B) II and III C) I and III areas of carbon black?
D) II and IV E) I, II and IV I. As filler in rubber tires
II. As a pigment in printing inks
III. In plastics and fibers
33. Which one(s) of the following is (are) the properties of IV. As a transfer material in carbon papers
white phosphorus?
I. It is condensed, collected, and stored under water. A) I and II B) II and III C) I and III
II. It is conductor of electricity D) II and IV E) I, II and IV
III. It is soluble in water.
IV. It can be cut with a knife.
40. Which of the following ions makes the water harder?
A) I and IV B) I and III C) II and IV
D) III and IV E) II and III A) OH- B) CN- C) Cl- D) CO3-2 E) Br-

34. The elemental silicon can be prepared from


............ or ............ by the reaction with coke in an electric 41. Today, heading phosphorus-containing fertilizers in use are
furnace. the ammonium phosphates, MAP and DAP. They are made by the
controlled neutralization of ........ with ..........
A) Quartz / Sand
B) Diamond / Graphite A) H2SO4 / NH3
C) SiO / SiO2 B) HNO3 / H2SO4
D) Na2SiO4 / Na2SiO3 C) H3PO4 / H2SO4
E) Na2SiO4 / SiO2 D) CaSO4 / NH3
E) H3PO4 / NH3

35. Which of the following naming of compounds is given


wrongly? 42. Which one(s) of the following oxides of carbon is (are) stable
at STP?
A) CaCO3 Calcite I. CO II. C3O2 III. CO2
B) CaCO3 . MgCO3 Dolamite
C) SiO2 Silica A) I only B) II only C) I and III
D) 3Ca3(PO4)2 . CaF2 Phosphate rock D) II and III E) I, II and III
E) CO(NH2)2 TNT(trinitrotoluene)

43. Which one(s) of the following is (are) the usage areas of CO ?


36. Which one(s) of the following oxides is (are) the chief I. Synthesis of methanol
oxides of carbon? II. As a fuel
I. CO II. CO2 III. C3O2 III. In metallurgy

A) I and II B) II only C) III only A) I only B) II only C) III only


D) I and III E) I, II and III D) I and III E) I, II and III

44. Which one(s) of the following is (are) source of nitrogen


37. Which of the following is not one of the oxidation states containing compounds?
of chlorine? I. KNO3 and NaNO3
II. Atmosphere
A) –1 B) +1 C) +4 D) +5 E) +7 III. Plants and Animals
IV. Proteins and Coal
93
stronger , more elastic, more resistant to cold and heat than natural
A) I, II and III B) I and III C) III and IV rubber. Such process is called ……….
D) II, III and IV E) I, III and IV
A) H.W. Nernst / Titration
B) C. Goodyear / Hydrolysis
45. Who manufactured NH3 in 1903 firstly and efficiently? C) Le Chatelier / Fermentation
D) C. Goodyear / Vulcanisation
A) Fritz Haber B) Good Year C) N.Bronsted E) H.W Nernst / Contact process
D) T.M.Lowery E) Mendeliev

52. Which one of the following is not a source of sulfur?


46. Which one(s) of the following gases are responsible of the air
pollution? A) Natural gas
I. N2 II. CO2 III. CO IV. He V. SO2 B) Mineral sulphides and sulphates
C) Oil
A) I, II and III B) I and II C) II and III D) Coal
D) II, III and V E) III and IV E) Living organisms

53. Which one(s) of the following is (are) the products of sulfur?


47. The essential problem is in the synthesis of ammonia that, I. Plastics
under most conditions the reaction does not go to the completion. II. Pesticides
It is reversible. High yield of ammonia requires, III. Sulfuric acid
I. A high temperature, 400oC IV. Natural rubber
II. A catalyst
III. Cooling down to -40oC A) I and III B) II and III C) I, II and III
IV. A high pressure, about 200 atm D) II and IV E) I, II, III and IV

A) I and II B) I, II and IV C) II and IV


D) II and III E) I only 54. The main use of ……… is in the synthesis of SO3 to make
H2SO4. And the process to make H2SO4 is called ………
48. Which one of the following is the formula of red phosphorus?
A) Sulfur / Vulcanization
A) P B) P4O6 C) P4O10 D) P4 E) P2O3 B) Sulfur / Contact process
C) SO / Fermentation
D) S8 / Vulcanization
49. In which one(s) of the following ions and compounds of E) SO2 / Contact process
sulfur, is (are) the oxidation states of sulfur atom given correctly?
I. SO4-2: +6
II. SO2: +4 55. In the world approximately, 70% of H2SO4 production is used
III. H2S: +2 in the manufacture of…………
IV S2O3-2: -2 and +6
A) Pesticides
A) I and II B) I and III C) I, II and III B) Fertilizes
D) II, III and IV E) I, II, III and IV C) Dye industry
D) Cleaning agents
E) Metallurgy
50. Which one(s) of the following is (are) the properties of sulfur?
I. S8(s) is a poor oxidizing agent.
II. Its possible oxidation states are between -2 and +6. 56. Which of the following sulfur compounds is responsible from
III. It forms ionic sulphides with most active metals. acid rains?

A) I only B) II only C) I and II A) SO2 B) SO3 C) H2S2O3


D) I and III E) I, II and III D) H2SO4 E) H2S

51. In 1839, ……………..accidentally discovered that by heating 57. In which of the following is the oxidation state(s) of sulfur in
a sulfur-rubber mixture, a product could be obtained that was S2O3-2 ion given correctly?

94
A) –2 B) +6 C) +2 D) -2, +6 E) –3

64. Which one of the following phosphorus containing


58. Which one(s) of the following is (are) true for the usage of compounds is used to prepare match heads?
sulfur containing compounds?
I. Thiosulfate solutions are used in photographic processing. A) P4O6 B) P4O10C) P4S3
II. Sulphites are used in pulp and paper industry. D) H3PO4 E) P2O5
III. CuSO4 is used in electroplating processes.
A) I only B) II only C) I and III
D) II and III E) I, II and III 65. Which one(s) of the following is (are) true about the usage of
phosphorus?
I. In the living organisms in ADP and ATP synthesis
59. What is the name of the technique, invented in 1895, that is II. As fertilizers
used to convert nitrogen to ammonia? III. In food industry, cheese making, in curing hams, baking
powder
A) Liquefaction of air
B) Titration A) I only B) II only C) III only
C) Polymerization D) II and III E) I, II and III
D) Haber method
E) Synthesis of ammonia
66. The industrial use of diamond depends on two properties of it.
These are ………..and ……….
60. Which of the following is wrong about the usage of nitrogen
containing compounds? A) electrical / high thermal conductivity
B) extreme hardness / high thermal conductivity
A) Fertilizers C) electrical conductivity / optical property
B) Household cleaning products D) high oxidizing property / high reactivity
C) Manufacture of dry cell batteries E) extreme hardness / electrical conductivity
D) Explosive matter
E) As a blench in paper and textile industry
67. Which one of the following is wrong about the usage of
carbon containing compounds?
61. In which of the following compounds is the oxidation state of
nitrogen given wrongly? A) In refrigerators
B) In fire extinguishers
A) NO2: +4 C) In metallurgy
B) N2O: +1 D) In plastic sand fibers
C) N2O3: +3 E) In pesticides
D) NH3: +3
E) N2O4: +4
68. I. Chalcocite
II. Chalcopyrite
62. Which one(s) of the following is (are) wrong about the usage III. Siderite
of chlorine? Which one(s) of the following compounds given above is (are) the
I. In the production of chlorinated organic compounds, including minerals of copper?
PVC
II. In paper and textile industry, pool and municipal water. A) I only B) II only C) III only
III. In automobile batteries. D) I and II E) I and III

A) I only B) II only C) III only


D) I and III E) II and III 69. Which one(s) of the following compounds is (are) not the
minerals of copper?
I. Hematite
63. Which of the following oxides of nitrogen is responsible for II. Cuprite
the air pollution? III. Malachite
IV. Copper (II) sulphide
A) N2O B) NO2 C) N2O4
D) NO3 E) N2O3 A) I and II B) I and IV C) I, II and III
95
D) II and IV E) II and III II. Mercury vapor is used in street lamps and fluorescent tubes
III. HgO is used in pigments

70. Which one(s) of the following is (are) the properties of A) I only B) II only C) I and II
metallic copper? D) II and III E) I, II and III
I. Malleable and ductile
II. Very good conductor of electricity and heat
III. Soluble in water 77. Which one(s) of the following is (are) wrong about the
IV. Reacts with HCI properties of mercury and its compounds?
I. Most of the mercury compounds are not hydrated
A) I only B) I and II C) I and IV II. Compounds of mercury are much more toxic than pure element
D) III and IV E) II, III and III III. Mercury alloys are called anthracite

A) I only B) II only C) II and III


71. Which one(s) of the following is (are) not the properties of D) I and III E) III only
metallic copper?
I. It can react with HCI and HNO3
II. Malleable and ductile 78. Which one(s) of the following is (are) wrong about the
III. It is insoluble in water common oxidation states of related metals?
IV. It is good conductor of electricity and heat I. Cu: +2 and +3
II. Hg: +1 and +2
A) I only B) II only C) II and IV III. Ag: +1 and +2
D) I and III E) II, III and IV
A) I only B) II only C) I and III
D) II and III E) III only
72. Brass is a kind of alloy and it is formed by the mixing of
……….. and ………..
79. Which one(s) of the following is (are) true about the common
A) Cu / Cd B) Cu / Zn C) Sn / Ag oxidation states of related metals?
D) Hg / Fe E) Cu / Pb I. Au: +1 and +3
II. Pb: +1 and +4
III. Cd: +2 and +3
73. Bronze is a kind of alloy and it is formed by the mixing of
………. and ………. A) I only B) II only C) II and III
D) I and III E) III only
A) Hg / Fe B) Cu / Pb C) Cu / Cd
D) Cu / Zn E) Sn / Cu
80. Which one has the highest electrical conductivity among the
others?
74. Which of the following metals exists as liquid at room
temperature? A) Cu B) Au C) Pt D) Ni E) Ag

A) Pb B) Au C) Ag D) Hg E) Zn
81. Which one(s) of the following is (are) wrong about the
properties of Ag and its compounds?
75. Which one(s) of the following is (are) true about the I. Ag is resistant to air oxidation
properties of mercury and its compounds? II. It does not react with H2SO4 and HNO3
I. HgO is thermally unstable III. AgI is used to seed clouds
II. HgF2 is covalent compound
III. Because of its high density it is used in barometers A) I only B) II only C) III only
D) I and II E) II and III
A) I only B) II only C) I and III
D) II and III E) I, II and III
82. Which one(s) of the following is (are) true about the uses of
Ag and its compounds?
76. Which one(s) of the following is(are) true about the uses of I. Ag compounds are used in electroplating in batteries and
mercury and its compounds? medicine
I. Hg is used in barometers and thermometers II. Ag is used as catalyst
96
III. Its most important use is in photographic processes
A) Cu B) Hg C) Pb D) Cd E) Zn
A) I only B) II only C) III only
D) II and III E) I, II and III
90. Which one of the following metals is used in the manufacture
of standard voltaic cells known as Weston cells?
83. Which one of the following is the most important use of gold?
A) Ni B) Pb C) Pt D) Fe E) Cd
A) In electrical industry
B) In monetary reserves
C) In photography 91. Which one is not correct about the uses of cadmium or its
D) In medicinal chemistry compounds?
E) In the manufacture of special glasses and ceramics
A) In the protection of iron against corrosion
B) In electroplating and batteries
84. Which of the following compounds completes the following C) As a catalyst mainly in organic reactions
reaction? D) In cement and glass
Au + HCI HNO3  …………+ H2O + NO E) In solar cells and pigments

A) HAuCI2 B) AuCl4 C) HAuCl4


D) AuCl E) AuCl3 92. Which one(s) of the formulas of the oxides of iron is (are)
written wrongly?
I. Fe2O3 II. Fe2O4III. Fe3O4 IV. FeO
85. The lead (II) sulphide is first converted to lead (II) oxide by
strongly heating it air, such process is called………. A) I only B) II only C) III only
D) IV only E) II and III
A) Flotation B) Hydrogenation C) Roasting
D) Electrolysis E) Blast furnace

93. Which one is wrong about the uses of iron or its compounds?
86. Which one(s) of the following oxides is (are) the best known
oxides of lead? A) Major commercial use of iron is in steel making
I. PbO2 B) Various iron oxides are used in electronic devices and
II. PbO recording tapes
III. Pb3O4 C) Iron complexes are used in inks and dyes
D) Iron is widely used in construction industry
A) I only B) II only C) III only E) Iron is also used in batteries as electrode
D) I and II E) I, II and III

94. The minerals of nickel are mainly ………, ……… and


87. Which one is not correct about the uses of lead or its ………
compounds?
A) Sulphides / oxides / arsenides
A) In batteries B) Sulphides / oxides / carbonates
B) In glass and cement C) Oxides / sulphates / arsenides
C) In metal protecting processes and ceramics D) Sulphates / arsenides / carbonates
D) In paints and matches E) Sulphites / chlorites / arsenides
E) In medicinal applications

95. Which one is wrong about the nickel or its compounds?


88. Which one of the following metals is used in the manufacture
of semiconductors? A) The most stable oxidation state of it is +2
B) It is used in Ni-Cd batteries
A) Pb B) Cd C) Fe D) Cu E) Hg C) Large amount of Ni is deposited in Canada
D) Accumulation of Ni in body causes serious damages in brain
and kidney
89. Which one of the following metals causes a disease known as E) It is also used as catalyst in organic reactions
“itai-itai kyo”?
97
96. In chemical reactions metals tend to 105. Name of the copper is coming from .........

A) gain electrons A) Greek B) Cyprus C) Turkey


B) lose electron D) Italy E) China
C) be reduced
D) gain protons
E) gain neutrons 106. What is (are) the oxidation state(s) of Ag metal?

A) only +1 B) only +2 C) +1 and +2


97. What is the mass percentage of Cu metal on the earth crust? D) only +3 E) +1 and +3

A) 0.3 % B) 0.2% C) 0.1%


D) 0.01% E) 0.02% 107. Most common oxidation state of Silver is

A) +1 B) +2 C) +3 D) +4 E) +5
98. In the production of Cu metal in industry, which compound of
copper is used?
108. What is the electron configuration of the Silver metal at
A) CuO B) Cu2O C) Cu2S neutral form? (29Ag)
D) CuSO4 E) Cu(OH)CO3
A) Ar |4s2 3d9 B) Ar |4s1 3d10 C) Ar |4s2 3d3 4p6
D) Ar |4s2 4p6 4d3 E) Ar |4s2 4d9
99. Which one is the second most conductive element after the Ag
metal?

A) Al B) Cu C) Pt D) Au E) Ni 109. The most conductive metal is .........

A) Ag B) Al C) Cu D) Au E) Pt
100. What is (are) the oxidation states of the copper metal?

A) +1 only B) +2 only C) +1 and+2 110. Which compound of silver is used to identify aldehydes from
D) +3 only E) +2 and+3 other organic compound?

A) AgCl B) AgBr C) AgOH


101. What is the electron configuration of the copper metal at D) Ag2O E) AgNO3
neutral form? (Atomic number of Cu is 29)

A) Ar |4s2 3d9 B) Ar |4s1 3d10 C) Ar |4s2 3d3 4p6 111. How many gram of O2 gas is produced by AgNO3 when
D) Ar |4s2 4p6 4d3 E) Ar |4s2 4d9 21.25 gram of silver is decomposed under the temperature.

A) 1.4 B) 2.8 C) 5.6 D) 11.2 E) 2.24


102. Most oxidation state of the Cu metal is......

A) +1 B) +2 C) +3 D) +1 and +2 E) +3 112. Which of the following compounds of silver gives the


reaction with acetylene?

103. Which gas would be produced when concentrated H2SO4 A) AgCl B) AgBr C) AgOH
reacted with Cu metal? D) AgI E) AgNO3

A) SO B) SO2 C) SO3 D) O2 E) H2
113. Which of the following gases is (are) produced, when
concentrated H2SO4 is reacted with silver metal?
104. What is the formula of the basic carbonate?
A) SO B) SO2 C) H2 D) O2 E) H2O (g)
A) CuCO3 B) CuFeS2 C) Cu(OH)2
D) Cu(OH)CO3 E) Cu(OH)2CO3
114. In ancient time Silver is known as ........
98
A) Saturn B) Moon C) Mars 124. What is the proportion between the HCl and HNO3
D) Neptune E) Uranus (mixture), which gives reaction with Au?

A) 5 B) 4 C) 3 D) 2 E) 1
115. What is the color of Ag+ ion when reacts with I- ion

A) yellow B) red C) green 125. What is the coefficient of the water in reaction of gold with
D) orange E) green 3HCl+HNO3?

A) 1 B) 2 C) 3 D) 4 E) 5
116. Which of the following properties would be expected
generally for the transition metals? 126. What is (are) the principle oxidation state(s) of the lead?

A) Low melting point A) +2 B) +4 C) +3 D) +2 and +4 E) +3 and +4


B) High ionization energy
C) Variable oxidation number 127. What is the electron configuration of the lead (82Pb)?
D) Positive standard electrode potential
E) Having low electrical conductance A) Xe |4f14 5d10 6s2 6p4
B) Xe |5f14 5d10 6s2 6p2
C) Xe |5f 14 6d10 6s2 6p2
117. Which of the following ions is diamagnetic? D) Xe |5d10 6s2 6p2
E) Non of them
A) Cr2+ B) Fe3+ C) Cu2+ D) Sc3+ E) Ag+

128. What is the formula of GELENA?


118. Which of the following elements has to display an oxidation
state of the +6 in any of its compound? A) PbO B) PbO2 C) PbS
D) PbI2 E) PbCO3
A) Ti B) Cr C) Cu D) Mn E) Fe

129. Which of the following is the formula of litharge and red


119. The best oxidizing agent among the following is ....... lead?

A) Na+ B) Al3+ C) Ag2+ D) Cu2+ E) Li+ A) PbO, PbO2


B) PbO2, Pb3O4
C) PbO, PbS
120. Which of the following elements has an oxidation state +3 in D) PbS, PbI2
its most compounds? E) None of them

A) Au B) Ni C) Cu D) Ag E) Cr
130. What is (are) the oxidation number(s) of the lead in Pb3O4
compound?
121. What is (are) the oxidation state(s) of the Platinum metal
A) +2 B) +4 C) +8/3
A) +4 and +6 B) +3 and +4 C) +3 only D) +2, +4 E) +2 , +8/3
D) +2 and +4 E) +4 only

131. What is the coefficient of the water in the reaction of Pb3O4


122. What is (are) the oxidation state(s) of gold? with diluted HNO3 solution?
Pb3O4(s) +HNO3 (aq) 
A) +3 B) +2 C) +1 D) +1 and +3 E) +1 and +2
A) 1 B) 2 C) 3 D) 4 E) 5

123. The most common oxidation state of gold is


132. What is the formula of the white lead?
A) +1 B) +2 C) +3 D) +4 E) +5
A) 2PbCO3.Pb(OH)2
99
B) PbCrO4
C) Pb(NO3)2
D) Pb3O4 141. What are the oxidation states of iron atoms in
E) PbO2 Fe4 [Fe(CN)6]3 ?

A) both +2 B) both +3 C) +2, +3


133. When 20.7 gram of Pb is reacted with enough amount of D) both +6 E) +3, +6
concentrated HNO3, how many grams of which gas is produced?
(Pb=207 g/ mol)
142. What are the formula of magnetite, hematite and ilmetite?
A) N2O, 4.48L
B) NO, 4.48L A) Fe3O4, Fe2O3, FeTiO3
C) NO2, 4.48L B) Fe3O4, FeCO3, FeS
D) NO, 2.24L C) FeS, Fe3O4, Fe2O3
E) NO2, 2.24L D) HFeO3, Fe3O4, FeTiO2
E) FeO, FeCO3, Fe2O3

134. What is the oxidation state of the iron metal?


143. What is (are) the oxidation state(s) of nickel?
A) +2 B) +3 C) +2, +3
D) +3, +4 E) +2, +3, +6 A) +2 B) +3 C) +3 and +6
D) +4 E) +2 and +3

135. How many gram of iron is produced, when a 2.8-gram of


carbon monoxide is reacted with iron (III) oxide? 144. In the neutral media electrolysis of NiSO4 is produced ........
(C: 12, H: 1, Fe: 56)
A) Ni, H2 B) Ni, O2 C) Ni, SO2
A) 56 B) 5.6 C) 11.2 D) 112 E) 2.8 D) H2, O2 E) only O2

136. What is the formula of iron carbide? 145. In the acidic media electrolysis of NiSO4 is produced ........

A) Fe4C2 B) Fe4C3 C) Fe3C A) Ni, H2 B) Ni, O2 C) Ni, SO2 D) H2, O2 E) only O2


D) Fe2C E) Fe4C6

146. What is the electron configuration of Ni3+ ion? (28Ni)


137. Which of the following has amphoteric properties?
A) Ar |4s2 3d5 B) Ar |4s13d6 C) Ar |4s0 3d7
2+ 6+ 3+ 2 5
A) Fe B) Fe C) Fe , Fe2O3 D) Ar |4s 3d E) Ar |4s24p5
D) Fe2+, Fe(OH)3 E) Fe2O3 ,Fe(HO)3

147. What is the mass percentage of carbon element in the steel?


138. Which of the following ions is used to determine Fe+2 and
Fe+3? A) 1.7 B) 2.9 C) 3.5 D) 4.6 E) 5.8

A) OH- B) Cl- C) I- D) NO3- E) CO3-


148. Which of the following elements will dissolve in both acidic
and basic solutions?
139. Iron is the .............most abundance element in Earth crash.
A) Na B) Zn C) Al D) Hg E) K
A) first B) second C) third
D) fourth E) fifth
149. Which of the following ions has the greatest hydration
energy?
140. What is the electron configuration of Fe2+ ion? (26Fe)
A) Na+ B) Al3+ C) Ba2+ D) K+ E) Cs+
2 3 0 5 2 3
A) Ar |4s 3d B) Ar |4s 3d C) Ar |4s 4d D)
Ar |4s04d5 E) Ar |4s24p3
100
150. In the production of a metal by reduction with carbon at A) Al2O3 B) AlCl3 C) Al(OH)3
moderate temperature is not feasible with..... D) Al2(SO4)3 E) Al2(CO3)3

A) CdO B) Al2O3 C) PbO D) HgO E) Cu2O


160. In the industry, which compound of aluminum is used in the
production of aluminum metal?
151. Which of the following sulfides produces a free metal
directly on roasting? A) Al2O3 B) AlCl3 C) Al(OH)3
D) Al2(SO4)3 E) Al2(CO3)3
A) HgS B) PbS C) Na2S D) SnS2 E) FeS

161. Which of the following formula is wrong?


152. Which of the following oxides is not soluble in NaOH
solution? A) AlF B) AlCl3 C) AlBr3 D) Al(OH)3 E) Al2(SO4)4

A) ZnO B) Al2O3 C) Fe2O3 D) SnO2 E) None of them


162. Al metal has +1 charge when it forms a compound with
..........
153. Which of the following ions is difficult to obtain freely metal
by reduction in an aqueous solution? A) F B) Cl C) Br D) I E) C

A) Zn2+ B) Cd2+ C) Pb2+ D) Hg2+ E) Cu2+


163. How many liters of H2 will be produced at STP when a 13-
gram of Zn metal reacted with enough amount of NaOH solution?
154. Which of the following is the acidic solution?
A) 2.24 B) 22.4 C) 4.48 D) 3.36 E) 8.96
A) ZnSO4(aq) B) NaAl(OH)2(aq) C) NaHCO3(aq)
D) KNO3(aq) E) KHCO3(aq)
164. What is the molecular weight of the compound formed by
the reaction of Zn with NaOH solution?
155. Which of the following oxides is the best oxidizing agent?
A) 120 B) 143 C) 158 D) 166 E) 208
A) SnO2 B) PbO2 C) HgO D) MgO E) Al2O3

165. What is the oxidation state of zinc?


156. Which of the following metals has a appreciable use at low
melting points to form alloys? A) +1 B) +2 C) +3 D) +4 E) +5

A) Sn B) Pb C) Cd D) Mg E) Ni
166. Which one(s) of the following compound of zinc is (are)
amphoteric?
157. What is the electron configuration of Cd2+ ion? (30Cd) I .Zn(OH)2
II.ZnSO4
A) Ar |4s03d10 B) Ar |4s23d8 C) Ar |4s23d24p6 III. ZnS
2 8
D) Ar |4s 4d E) Ar |4s24p65s2 IV. ZnO

A) IV only B) I and II C) I and III


158. Which one(s) of the following has an amphoteric character? D) I, II and III E) I and IV
I. Al2O3
II. Al(OH)3
III. AlCl3 167. Which one(s) of the following can be produced by the
electrolysis of zinc solution?
A) I only B) II only C) III only I. Zn metal
D) I and II E) I and III II. O2 gas
III. H2 gas
IV. SO2 gas
159. What is the formula of CORUNDUM?
A) I only B) II only C) I and II
101
D) III only E) I, II and IV A) I and II B) III and IV C) I and IV
D) II and III E) II and IV

168. Which of the following gas is produced when Al metal is


reacted with cold HNO3 solution? 176. Which one(s) of the following is (are) basic compound of
chromium metal?
A) NO2 B) N2O C) NO I. Cr2O3
D) N2O4 E) No production of any gas II. CrO3
III. CrO
IV. Cr(OH)2
169. Who discovered firstly the element of Chromium?
A) I and II B) III and IV C) I and III
A) Vokelen B) Oregot C) Klaprot D) II and III E) II and IV
D) Arkel E) Berzelius

177. Which one(s) of the following is (are) acidic compound of


170. What is the maximum bond capacity of zinc? chromium metal?
I. Cr2O3
A) 1 B) 2 C) 3 D) 4 E) 5 II. CrO3
III. H2Cr2O7
171. Which one(s) of the following is (are) not soluble in water? IV. Cr(OH)2
I. Mg(OH)2 V. H2CrO4
II. Cu(OH)2
III. Fe(OH)2 A) I and II B) II, III and IV C) II, III and V
D) III, IV and V E) III and IV
A) I only B) II only C) III only
D) I and II E) II and III
178. Which one(s) of the following has the lowest melting point?

172. Which one(s) of the following metals is (are) obtained as A) Al B) Hg C) Fe D) Cr E) Zn


solid form when solution of them is electrolyzed?
I. Ni
II. Sn 179. What is the oxidation state of Pb metal?
III. Cr
A) +2 B) +2 and +3 C) +3 and +4
A) I, II and III B) II and III C) I and II D) +2 and +4 E) +4 only
D) II only E) I only

180. What is the oxidation number of Pb element in Pb3O4


173. What are the common oxidation states of chromium metal? compound?

A) +1 and +2 B) +2 and +3 C) +1, +4 and +6 A) +8/3 B) +2 C) +4 D) +2 and +3 E) +2 and +4


D) +2, +5 and +6 E) +3 and +6

181. According to the reaction, which compound(s) of Pb is (are)


174. Chromium metal is produced from ....... produced?
Pb3O4 (s) + HNO3 (aq) 
A) Cr2O3 B) Fe(CrO2)2 C) Cr(OH)2 I. Pb(NO3)2
D) KCrO4 E) NaCrO4 II. Pb(NO3)4
III. PbO2
IV. PbO
175. Which one(s) of the following chromium compounds has
(have) amphoteric properties? A) I and II B) III and IV C) I and IV
I. Cr2O3 D) I and III E) II and IV
II. CrO3
III. CrO
IV. Cr(OH)2 182. What is the electron configuration of chromium? (24Cr)

102
A) Ar |4s2 3d4 B) Ar |4s2 4p4 C) Ar |4s0 3d6
D) Ar |4s2 4d4 E) Ar |4s1 3d5

183. One of the following is a practical use of silicon.

A) for making porcelain and glazes for pottery


B) as insulator for electric irons
C) as conductor of heat
D) for making computer chips

184. The most abundant element on the earth's crust is

A) oxygen
B) hydrogen
C) silicon
D) aluminum

185. Diamond is made up of carbon. It is the hardest substance


known. It is considered to be a

A) metal
B) non-metal
C) metalloid
D) halogen

186. ..… is a bio molecule made up of carbon, hydrogen and


oxygen. It is the main source of energy in the body.

A) carbohydrates
B) protein
C) fats
D) nucleic acid

187. The chemical symbol Sb stands for

A) Antimony
B) Tungsten
C) Mercury
D) Lead

188. The only liquid

A) mercury B) tin C) bronze

189. All of the following are important ores except

A) silver B) diamond C) gold D) iron

103
HEAT 8. The heat, which is released by the combustion of 1 g ethyl
1. Which one of the reactions has a negative H? alcohol, C2H5OH, raises the temperature of 2.5 kg water by 25oC.
If the specific heat of water is 1 cal/goC, which of the statements
A) N2(g)  2N(g) gives the standard heats of combustion of ethyl alcohol?
B) Mg(g)  Mg+ (g) + e-
A)1x25x2.5x46 B)1x25x2.5x1 C)1x1x46x25
C) H2O(s)  H2 (g) + 1/2 O2(g)
46 2.5
D) Br2(s)  Br2 (g)
E) Cl(g) + e-  Cl- (g) D)1x1x46x2.5 E)1x 1 x 1 x46
25 25 2.5
2. H2(g)  2H+(g) +2e- what is the name of enthalpy
change in this reaction ? 9. The standard heats of combustion of butane, C4H10, is 690
kcal/mol. What final temperature would you expect to see, when a
A) Heat of formation 11.6 g of butane were burnt to rise the temperature of 2 kg of
B) Heat of combustion water at 15oC? (C: 12, H: 1)
C) Heat of neutralization
D) Heat of ionization A) 22 B) 42 C) 69 D) 84 E) 100
E) Heat of dissolving

10. The heat that is liberated by the combustion of 12.4 g of


3. The heat of formation of NH3 is -11 kcal/mol. According to phosphorus to form P4O10 is used to raise the temperature of 4 kg
this, what is the H of the reaction given below? of water from 10oC to 28oC. According to this, what is the
2NH3(g)  N2(g) + 3H2(g) standard heat of formation of P4O10 in kcal/mol?
(P: 31,O: 16) (Specific heat of water is 1 cal/g oC)
A) -22 B) -11 C) +5.5 D) +11 E) +22
A) 720 B) 144 C) -144 D) –360 E) -720

4. In the formation of CaO(s), 30.4 kcal heat is liberated by the


action of 8 g of Ca(s) on 8 g O2(g). According to this, what is the 11. According to reactions and their H values given below,
heat of formation of CaO(s) in kcal/mol? (Ca: 40, O: 16) 2C(s) + 2H2(g)  C2H4(g) H = 12.5 kcal
2C(s) + 3H2(g)  C2H6(g) H = -20.2 kcal.
A) -152 B) -121.6 C) -106.4 D) -76 E) -60.8 Which one is the enthalpy change for the formation of C2H6(g)
from C2H4(g) in kcal ?
5. The heat of neutralization of NaOH with HCl is -14 kcal/mol. A) -7.7 B) -20.2 C) -32.7 D) 7.7 E) 32.7
When a 100 mL of 0.5M NaOH solutions is neutralized with a
200 mL of 0.4M HCl solutions, how much kcal heat is liberated?
12. Which one(s) of the followings is(are) necessary to calculate
A) 2.24 B) 1.82 C) 1.12 D) 0.7 E) 0.85 the enthalpy change of the following reaction,
CH4(g) + 2O2(g)  CO2(g) + 2H2O(g)
I. The standard heats of formation of CH4(g)
6. The standard heat of formation of X2O3 is -400 kcal/mol.
II. The standard heats of formations of CO2(g) and H2O(g)
The formation of 20.4 g of X2O3 releases 80 kcal heats. What is
III. Bond energy of O2(g)
the atomic weight of X? (O: 16)
A) I only B) II only C) III only
A) 11 B) 27 C) 56 D) 70 E) 120
D) I and II E) I, II and III

7. Mg + 2HCI  MgCl2 + H2 H = -100 kcal


13. K(s)+H2O(l) KOH(aq) + 1/2H2(g)
When a 200 mL of HCl solution is reacted with excess Mg, 5 kcal
heats is liberated. What is the molar concentration of HCl H1 = -48 kcal
solution? H2(g) + 1/2O2(g)  H2O(l) H2= -68 kcal
KOH(s)  KOH(aq) H3= -13 kcal
A) 0.25 B) 0.5 C) 0.75 D) 1 E) 2 According to these, what is the standard heat of formation of
KOH(s) in kcal/mol?

A) +129 B) -129 C) -64.5 D) +103 E) -103

104
20. When a 5.6 g of CO(g) is burnt, the temperature of the
14. I. K(g)  K+(g) + e- H1 colorimeter whose heat capacity is 1 cal/oC rises by 13.6 oC. What
II. F2(g)  2F(g) H2 is the molar heats of combustion of CO(g) in kcal/mol ? (C: 12,O:
III. F(g) + e-  F-(g) H3 16)
Which one(s) of the enthalpy change of the reactions above is
(are) positive? A) 102 B) 68 C) 51 D) 34 E) 28

A) I only B) II only C) III only


D) I and II E) II and III 21. Given the reactions below,
I. C(s) + O2(g)  CO2(g) H1= -94 kcal
II. CO(g) + 1/2O2(g)  CO2(g) H2= -68 kcal
15. The heats of formation of H2O(l) is -68 kcal/mol. According III. H2(g) + 1/2O2(g)  H2O(g) H3= -58 kcal
to this, what is the H of the following reaction in kcal; what is the H of the following reaction,
2H2O(l)  2H2(g) + O2(g) ? C(s) + H2O(g)  CO(g) + H2(g)

A) -136 B) +136 C) -68 D) +68 E) +34 A) -87 B) +87 C) +32 D) +34 E) +104

16. The heats of formation of CO2(g) is -94 kcal/mol. How many 22. The standard heats of formation of CO and HCOOH are -94
kcal heats are released in the formation of 5.6L of CO2(g) and -98 kcal/mol, respectively. According to this information,
measured at 0oC and 2 atm pressure ? what is the H of the following reaction in kcal/mol ?
H2 + CO2  HCOOH H =?
A) 11.8 B) 23.5 C) 23.2 D) 37.5 E) 47
A) +192 B) -192 C) -96 D) +4 E) -4

17. C + O2  CO2 H1


H2 + 1/2O2  H2O H2 23. 647 cal heats are released in the formation of 1 g of NH3
C2H5OH + 3O2  2CO2 + 3H2O H3 measured at 25oC and 1 atm pressure. What is the molar heat of
According to these reactions, what is the H of the following formation of NH3 in kcal/mol? (N: 14, H: 1)
reaction? A) -0.647x17 B) -0.647 C) +0.647x17
2C + 3H2 +1/2O2  C2H5OH H = ? 17
D) +0.647 E) - 17
A) (2H1 + 3H2) - H3 17 0.647
B) H3 - (2H1 + H3 )
C) H1 + H2 + H3
24. I. The standard heats of combustion of an organic compound
D) H1 /2 + H2 /2 + H3
which is formed by C and H is -335 kcal/mol,
E) H3 -( H1 /2 + H2 /3)
II. The heat is liberated in the combustion of 5.6 g of this
compound is used to rise the temperature of 1 kg of water from
20oC to 87oC. (The specific heat of water is 1 cal/g oC)
18. 1/2N2(g) + O2(g)  NO2(g) H = 8 kcal According to this information, what is the molecular weight of
2NO2(g)  N2O4(g) H = -13.7 kcal this organic compound?
According to these reactions, which one is the heats of formation
of N2O4(g) in kcal/mol? A) 88 B) 56 C) 32 D) 28 E) 16
A) -2.3 B) 57 C) +2.3 D) +5.7 E) 14.8
25. Given the reaction,
Zn + Cu+2  Zn+2 + Cu H = -52 kcal
19. Which one(s) of the following reactions given below is (are) how many kcal heat is released when 400 cm3 of 0.25M CuSO4
exothermic reaction? reacted with excess Zn ?
I. 2H2(g) + O2(g)  2H2O(g)
II. H2(g)  2H(g) A) 2.6 B) 5.2 C) 7.8 D) 10.4 E) 15.6
III.Ca(g)  Ca+2(g) + 2e-(g)

A) I only B) II only C) III only 26. Which one of the following reactions is endothermic?
D) I and II E) II and III
A) C(s) + O2(g)  CO2(g) + 94 kcal

105
B) H2(g) + 1/2O2(g)  H2O(l) H=-68.3 kcal CH4(g) + 2O2(g)  CO2(g) +2H2O(g) H= -191.9 kcal
C) 2C(s) + 3H2(g) + 20.1 kcal  C2H6(g)
D) H2(g) + Cl2(g)  2HCl(g) + 22 kcal A) +115.8 B) -115.8 C) -57.9
E) CH3OH(l) + 3/2O2(g)  CO2(g) + 2H2O(g) H= -174 D) +57.9 E) +231.6
kcal

33. Given,
27. S(s) + 3/2O2(g)  SO3(g) H = -94.5 kcal H-H: 104 kcal/mol O=O: 118.2 kcal/mol
To obtain 18.9 kcal, how many grams of sulfur must be burned? H-O: 110.8 kcal/mol
(S: 32) What is the molar heat of formation of H2O in kcal?

A) 3.2 B) 6.4 C) 9.6 D) 12.8 E) 94.5 A) +58.5 B) -58.5 C) +10.6


D) -10.6 E) +52.3

28. Which one of the following statements is wrong for the


following reaction? 34. The molar heat of combustion of C2H5OH(l) to form CO2(g)
Ca(OH)2(aq) + CO2(g)  CaCO3(s) + H2O(l) and H2O(l) is 295 kcal. How much heat is liberated when 9.2 g of
H = -20 kcal C2H5OH is burned?

A) It is an exothermic reaction A) 78.5 kcal B) 73.5 kcal C) 58 kcal


B) 1 kcal is given off when 0.05 mol of Ca(OH)2 is used. D) 85 kcal E) 67.5 kcal
C) 5 kcal is given off when 5.6 L of CO2 is used in the reaction.
D) 33g of CO2 must be used to produce 15 kcal.
E) The heat content of products is 20 kcal more than that of 35. According to the following reaction,
reactants. N2(g) + 2O2(g)  2NO2(g) H= -16.2 kcal
Calculate the heat of formation, Hfo, of NO2 in kcal/mol?

29. The standard heats of formation of Ag2S(g), H2O(l) and A) +8.1 B) +81 C) -58 D) -8.1 E) 181
H2S(g) are -8 kcal/mol, -68 kcal/mol and -5kcal/mol respectively.
Which one of the following is the H in kcal of the reaction given
below? 36. When 128 g of sulfur in its standard state S8 is burned to form
2Ag2S(g) + 2H2O(l)  4Ag(s) + 2H2S(g) + O2(g) SO2(g), 283.84 kcal heat is liberated. Calculate the amount of
heat by the burning of 1 mol of S8?
A) -65 B) -81 C) +81 D) +142 E) +256
A) 567.68 B) 500.68 C) 675.85
D) 765.68 E) none
30. Given the heats of reactions,
2NO(g) + O2(g)  2NO2(g) H= -27 kcal
37. According to the following reaction;
2NO2(g)  N2(g) + O2(g) H= -43.2 kcal
Xe(g) + 2F2(g)  XeF4(s) H= 62.5 kcal
What is the H of the reaction?
What is the heat of formation of ,Hfo, XeF4(s) in kcal?
2NO2(g)  N2(g) + 2O2(g)
A) +62.5 B) +31.25 C) -31.25 D) -62.5 E) +125
A) -16.2 B) +16.2 C) -70.2 D) -32.4 E) -32.4

38. When 0.4 mol of C2H5OH(l) is burned 118 kcal are liberated.
31. 2Fe(s) + 3/2O2(g)  Fe2O3(s) H= -197 kcal
C2H5OH(l) + 3O2(g)  2CO2(g) + 3H2O(g)
2Al(s) + 3/2O2(g)  Al2O3(s) H= -399 kcal
H of CO2(g)= -94kcal
The mixture of 0.2 mol of Fe2O3 and 0.04 mol of Al(s) is treated
to form Al2O3(s) and Fe(s). How much heat is liberated in kcal? H of H2O(l) = -57.8 kcal
What is the heat of formation of C2H5OH(l) in kcal ?
A) 202 B) 101 C) 40.4 D) 8.08 E) 4.04
A) +66.4 B) +132.8 C) -132.8
D) +33.2 E) -66.4
32. What is the heat of decomposition of one mole of water vapor
into hydrogen and oxygen gases?
39. Which one of the following reactions is endothermic?
C(s) + 2H2(g)  CH4(g) H= -17.9 kcal
C(s) + O2(g)  CO2(g) H= -94.0 kcal
106
A) C(s) + O2(g)  CO2(g) + 94 kcal Hf(CH4) = 17.9 kcal
B) H2(g) + 1/2O2(g)  H2O(l) H=-68.3 kcal Hf(CaO) = -37.10 kcal
C) 2C(s) + 3H2(g) + 20.1 kcal  C2H6(g) Hf(CO2) = -94 kcal
D) H2(g) + Cl2(g)  2HCl(g) + 22 kcal Hf(H2O) = -68.3 kcal
E) CH3OH(l) + 3/2O2(g)  CO2(g) + 2H2O(g)
H = -174 kcal A) 4.905 B) -5.39 C) -3.16
D) 5.34 E) -12.46
40. Calculate Hfo of C6H12O6(s) from the following information
given below?
H combustion of C6H12O6(s) = -2816 kcal/mol 46. 2H2 (g) + O2(g)  2H2O(g) H = -115.6 kcal
Hfo of CO2(g) = -393.5 kcal/ mol How much heat in kcal will be liberated when 8 grams of
Hfo of H2O(l) = -285.9 kcal/ mol hydrogen reacts with oxygen as in the equation above? (H: 1)

A) +1.260 B) +260 C) –260 A) 231. 2 B) 115.6 C) 462.4


D) 1060 E) -1260 D) 68. 5 E) 230.6

41. Given the heats of reaction; 47. The Ho of SbCl3 (g) = -75 kcal/mol and the Ho of SbCl5 (l)
1/2N2(g) + 1/2O2(g)  NO(g) H=+21.6 kcal = -105.2 kcal/mol. What is the Hof in kcal the reaction given
1/2N2(g) + O2(g)  NO2(g) H=+8.1 kcal below?
Determine the heat of the reaction for the following reaction? SbCl3 (g) + Cl2 (g)  SbCl5 (l)
NO(g) + 1/2O2(g)  NO3(g)
A) +30.2 B) -30.2 C) -15.1
A) 27 B) -13.5 C) -27 D) +13.5 E) 81 D) +15.1 E) +60.4

42. S(s) + 3/2O2(g)  SO3(g) H = -94.5 kcal 48. Given the reactions,
To obtain 18.9 kcal, how many grams of sulfur must be burned? Fe(s) + 1/2O2(g)  FeO(g) H = -64.04 kcal
(S: 32) 2Fe(s) + 3/2O2(g)  Fe2O3(s) H = -196.5 kcal
What is the H in kcal of the reaction given below?
A) 3.2 B) 6.4 C) 9.6 D) 12.8 E) 94.5 2FeO(s) + 1/2O2(g)  Fe2O3(s)

A) -324.58 B) +324.58 C) -68.42


43. According to the reactions given below; D) -457.04 E) +68.42
2NO(g) + O2(g)  2NO2(g) H= -27 kcal
2NO(g)  N2(g) + O2(g) H = -16.67 kcal
What is the H of the reaction? 49. Given the reaction;
2NO2(g)  N2(g) + 2O2(g) 4NH3(g) + 5O2(g)  4NO(g) + 6H2O + 216 kcal
How many kcal heats are released when 1 mol of NH3 is burned?
A) -16.2 B) +16.2 C) -70.2
D) +70.2 E) -32.4 A) 54 B) 5.4 C) 10.8 D) 27 E) 216

44. P4(s)  4P(g) H = +300.8 kcal 50. Which of the following reactions would you expect to be
exothermic?
H2(g)  2H(g) H = +104 kcal
PH3(g)  P(g) + 3H(g) H = + 223 kcal
What is the heat of formation of PH3(g) in kcal ? A) C12H22O11(s) + 12O2(g)  12CO2(g) + 11H2O(l) + 1351
kcal
A) +624 B) +16.4 C) +32.8 B) 2H2O(l)  2H2(g) + O2(g)
D) +8.2 E) +313.5 C) CO2(g) + 94 kcal  C(s) + O2(g)
D) H2O(s)  H2O(l)
E) CO2(g) + 2H2O(g)  CH4(g) + 2O2(g)
45. According to the reaction and the standard heat of formation H=191.8 kcal/mol
values of the compounds,
CH4 (g) + 4CuO (s)  CO2 (g) + 2H2O(l) + 4Cu(s)
how much kcal heat will be released if 12.7 g of Cu is obtained ? 51. Given the reactions;
(Cu: 63.5) C(s) + O2(g)  CO2(g) H= -94.05 kcal
107
C(s) + 1/2O2(g)  CO(g) H= -26.41 kcal
What is the H in kcal of the following reaction?
CO(g) + 1/2O2(g)  CO2(g)

A) +135.28 B) +67.64 C) -67.64


D) -52.48 E) +47.02

52. Which of the thermo chemical equations represents the


combustion of 1 mol of C(s) to CO2(g) to release 94 kcal ?

A) CO2(g) + 94 kcal  C(s) + O2(g)


B) CO2(g) + 94 kcal  C(s) + 1/2O2(g)
C) CO2(g)  C(s) + O2(g) + 94 kcal
D) CO2(g)  C(s) + O2(g) H= +94 kcal
E) C(s) + O2(g)  CO2(g) + 94 kcal

53. The standard heats of formation of Ag2S(g), H2O(l) and


H2S(g) are -8 kcal/mol, -68 kcal/mol and -5 kcal/mol respectively.
Which is the Ho in kcal for the reaction?
2Ag2S(s) + 2H2O(l)  4Ag(s) + 2H2S(g) + O2(g)

A) -65 B) -81 C) +81 D) +142 E) +256

108
QUESTIONS c. 2KMnO4 + K2SO3 + KOH  2K2MnO4 + K2SO4 + H2O
1) Which one of the following reactions are oxidation-reduction
reactions.
a. KOH + CuCl2  KCl + Cu(OH)2
b. KBr + Cl2  KCl + Br2
c. Fe(OH)3  Fe2O3 + H2O
d. Pb(NO3)2  PbO + NO2 + O2
2) Complete the following processes.Find the oxidation and reduction
processes.
a. Al + -3e-  d- Fe+3 + 1e- 

b. S+6 + 8e-  e- Br2 + 2e- 


c. N-3 - 5e-  f- Mn+2 –5e- 
3) For each of the following oxidation-reduction reactions identify the
substance oxidized, the substance reduced, the oxidizing agent and
reducing agent:
a. Zn + Cl2  ZnCl2
b. 2ReCl5 + SbCl3 2ReCl4 + SbCl5
c. Mg + CuCl2  MgCl2 + Cu
d. 2NO + O2  2NO2
e. WO3 + 3H2  W + 3H2O
f. 2NaBr + Cl2  2NaCl + Br2
g. Zn + HCl  ZnCl2 + H2
h. Fe2O3 + 2Al  Al2O3 + 2Fe
i. OF2 + H2O  O2 + 2HF
j. 2HgO  2Hg + O2

4) Balance the following oxidation-reduction reactions by using


electron balance method.And find the oxidizing and reducing agents
for each of the reaction.
a. H2S + K2Cr2O7 + H2SO4  S + Cr2(SO4)3 + K2SO4 + H2O
b. C + HNO3  CO2 + NO + H2O
c. MnO2 + NaBiO3 + HNO3  HMnO4 + BiONO3 + NaNO3
+ H2O
d. NaI + NaIO3 + H2SO4  I2 + Na2SO4 + H2O
e. Mg + HNO3  Mg(NO3)2 + N2O + H2O
f. PbO2 + MnSO4 + HNO3  PbSO4 + Pb(NO3)2 + HMnO4 +
H2O
g. KMnO4 + NH3  KNO3 + MnO2 + KOH + H2O
h. K2Cr2O7 + HClO4 + HI  Cr(ClO4)3 + KClO4 + I2 + H2O
i. H2C2O4 + KMnO4  CO2 + K2CO3 + MnO2 + H2O
j. C6H12O6 + KMnO4 + H2SO4  CO2 + MnSO4 + K2SO4 +
H2O
5) For each of the following oxidation-reduction reactions:
I) H2O2 + K2Cr2O7 + HCl  CrCl3 + KCl + O2 + H2O
II) H2O2  H2O + O2
III) H2O2 + KNO2  KNO3 + H2O
a. Find the oxidized, reduced substances and oxidizing,
reducing agents.
b. What is the role of hydrogen peroxide in these reactions.
c. Balance the reactions by electron balancing method.

6) For each of the following reaction equations:( page 159) complete


the question
a. 2KMnSO4 + 5K2SO3 + 3H2SO4  2MnSO4 + 6K2SO4 +
3H2O
b. 2KMnSO4 + 5K2SO3 + 3H2O  2MnO2 + 3K2SO4 +
2KOH
109
REACTION RATE Which one of the following is the rate expression for the reaction?
Given the reaction; A) rate=k[X][Z]2
H2O2  H2O + 1/2O2 + 26 kcal B) rate=k[X][Y][Z]
If the Ear for this reaction is 44 kcal which one of the following is C) rate=k[X][Y]2[Z]
its PE - RC diagram? D) rate=k[X]2[Y][Z]2
E) rate=k[X][Y][Z]2
A) B)
PE PE
44 44 4. The mechanism of a reaction is given as fallows:
26 18 AB + C  DE + F
0 0 DE + G  K + L
L + F  M + C
RC RC Which one of the species is a catalyst?
C) D)
PE A) DE B) F C) G D) L E) C
PE 18
26

0 5. The rate expression for the reaction is


0 3A2 + 2B2  2A3B2, rate = k [A2][B2]1/2
-44
18 Which one of the following reactions is the rate-determining step
RC in the mechanism of this reaction?
RC
E) PE
18 A)A2B + 1/2B2  A2B2
B) A2 + 1/2B2  A2B
0 C) A2 + B2  A2B2
D) A2 + B2  2AB
-26
E ) A2 + 2B2  2AB2
RC

1. A + 2B  AB2 rate = k[A][B]2 6. The activation energy for the reaction


Doubling the concentration of B increases the rate of reaction by a 2XY(g)  X2(g) + Y2(g)
factor of is 60 kcal and for the reaction
X2(g) + Y2(g)  2XY(g) is 40 kcal. Calculate the
A) 2 B) 3 C) 4 D) 6 E) 8 numerical value of the H for the reaction
2XY(g)  X2(g) + Y2(g)

2. The following data were obtained for the reaction A) -20 B) -60 C) +20 D) +100 E) -100
X + Y  Z
Exp. No: [X] [Y] Rate
1 0.2 0.2 0.06 7. For the reaction A + B  C, H = -20 kcal
2 0.2 0.4 0.24 and the activation energy is 16 kcal.
What is the activation energy for the reaction?
3 0.4 0.4 0.24
CA +B
The rate of reaction is proportional to
A) 20 B) 16 C) 4 D) -4 E) 36
A) [X][Y] B) [Y] C) [X]2[Y]
D) [Y]2 E) [X][Y]2
8. Which one of the following reactions may be fastest at room
temperature?
3. The following data were obtained for the reaction
X + 2Y + Z  K + 2L A) H2(g) + F2(g)  2HF (g)
Exp. No: [X] [Y] [Z] Rate B) CO(g) + 1/2O2(g)  CO2(g)
1 0.01 0.2 0.1 1.8x10-4 C) C2H6(g) + 7/2O2(g)  2CO2(g) + 3H2O(g)
2 0.02 0.2 0.1 3.6x10-4 D) Ag+(aq) + Cl+2(aq)  AgCl(s)
3 0.01 0.4 0.1 1.8x10-4 E) Fe+2(aq) + Cu+2(aq)  Cu+(aq) + Fe+3(aq)
4 0.01 0.2 0.2 7.8x10-4 9. The mechanism of the overall reaction
110
3X(g) + 2Y(g)  X3Y2(g) A) 0.135 B) 0.90 C) 0.09
is given as, D) 9.0 E) 1.35x10-3
X + X  X2 (slow)
X2 + Y  X2Y (fast)
X2Y + Y  X2Y2 (fast) 15. The following mechanism has been proposed for the reaction;
X2Y2 + X  X3Y2 (fast) H2O2(aq) + 2I- (aq) + 2H3O+ (aq)  I2(aq) + 4H2O(l)
By what factor does the rate of the reaction would be increased if H2O2 + I-  H2O + IO- (slow)
the volume of the container were reduced to one half of its IO- + 2H3O+  HIO + H2O (fast)
original value? HIO + H3O+ + I-  2H2O + I2 (fast)
According to these data which one of the following doesn’t affect
A) 2 B) 4 C) 8 D) 16 E) 32 the rate of the overall reaction?

10. Which one of the following phenomenons is (are) A) Doubling the concentrations of H2O2 and I-, and halving the
endothermic? concentration of H3O+
I. Evaporation of H2O B) Doubling the concentrations of H2O2 and halving the
II. Electrolysis of H2O concentration of H3O+ .
III. Freezing of H2O C) Doubling the concentrations of H3O+ and halving the
concentration of H2O2 and I-.
A) I only B) III only C) I and III D) Adding water and doubling the volume.
D) I, II and III E) I and II E) Doubling the concentrations of H3O+ and I- , and halving the
concentration of H2O.

11. Which one of the following is WRONG for a catalyst?


16. Which one of the following substances acts as a catalyst in a
A) It remains unchanged at the end of the reaction reaction that has the following mechanism?
B) It decreases the activation energy HCOOH + H+  HCOOH2+
C) It increases the rate of the reaction HCOOH2+  HCO+ + H2O
D) A catalyzed reaction is always endothermic HCO+  H+ + CO
E) It doesn’t change the amount of products
A) HCOOH B) H+ C) HCOOH2+
+
D) HCO E) H2O
12. Which one of the following type of reactions may be fastest at
room temperature?
17. The following data were collected for the reaction;
A) Uncatalyzed reaction 2X(g) + 3Y(g) + Z (g)  2K(g) + L(g)
B) Reaction with low activation energy
C) Reaction with high activation energy Exp. No: [X] [Y] [Z] Rate
D) Exothermic reaction
E) Endothermic reaction 1 0.2 0.1 0.1 4x10-3
2 0.2 0.2 0.1 1.6x10-3
3 0.2 0.2 0.2 1.6x10-3
13. Consider the hypothetical reaction, 4 0.1 0.2 0.1 8x10-2
2X + 3Y + Z  2K + L What can be the slowest step in the mechanism of the overall
Exp. No: [X] [Y] [Z] Rate reaction?
1 0.2 0.2 0.2 2x10-3
2 0.4 0.2 0.2 2x10-3 A) X + Y  products
B) X + Z  products
3 0.4 0.4 0.2 8x10-3
C) 2X +Y  products
4 0.4 0.4 0.4 1.6x10-2
D) X + 2Y  products
What is the numerical value of the rate constant?
E) 2Y + Z  products
A) 0.25 B) 0.05 C) 0.01 D) 0.50 E) 0.20
18. A(s) + B(g)  C(g) + heat
I. Volume is decreased
14. The reaction 3X + Y  X3Y is found to have the II. Temperature is increased
following rate law Rate= k[X]2 . When [X]=[Y]=0.1 M, rate = III. Some A is added
0.06 M/sec. What is the rate in M/sec, when [X]=[Y]= 0.15 M? Which one(s) of the following affects above increase(s) the rate?

111
A) II only B) I and II C) I and III
D) II and III E) I, II and III 24. When a 0.04 mol of sample of X reacts with 0.02 mol of Y in
a 2 L container the following reaction will occur
2X + Y  Z + T
19. In a closed container and at constant temperature the at constant temperature. If the rate is equal to 4 x 10-6 m/s at the
following net reaction occurs in two steps. same temperature what will be the rate constant?
2X + 3Y  Z + 2T
I. If the concentration of X is doubled the rate of the reaction also A) 0.5 B) 1 C) 4x 10-2 D) 5 x 10-3 E) 200
doubled
II. If the concentration of Y is decreased one half the rate of the
reaction decreases one fourth. 25. A + B  C H=15 kcal/mol Eaf = 45 kcal
According to these, which one of the following is the slowest step If appropriate catalyst is added to the reaction media;
of the reaction? I. Eaf will become less than 45 kcal
II. H will become more than 15 kcal
A) X + 2Y  products III. Activation energy of the reverse reaction will become equal to
B) X + Y  products 30 kcal.
C) 2X + 2Y  products Which one(s) is (are) true?
D) X + 3Y  products
E) 2X + Y  products A) I only B) III only C) I and II
D) II and III E) I, II and III

20. If the temperature is increased at any reaction,


I. Reaction rate increases. 26. A + 2B  C + D
II. Average kinetic energies of the molecules increase. When the concentrations of A and B are 2x102 M and 102 M
III. Activation energy increases. respectively, the rate of the reaction is 10 6 M/s. What will be the
Which one(s) is (are) true? rate of the reaction when the concentrations of A and B are 4x10 3
and 103 respectively?
A) I only B) II only C) I and II
D) II and III E) I, II and III A) 2x109 B) 106 C) 8x109
9
D) 4x10 E) 4x106
21. Which one is wrong for catalysts?
27. CaO(g) + H2O(l)  Ca(OH)2 + 15.6 kcal
A) It doesn’t affect the H. Which one of the following is wrong for the reaction above?
B) It changes the value of the rate constant. (Ca: 40, O: 16,H: 1)
C) It changes the reaction rate.
D) It doesn’t affect the activation energy. A) Reaction is exothermic
E) It changes the mechanism of the reaction. B) 15.6 kcal/mol heat is released when 1 mol of CaO reacts with
H2O (l)
C) H = -15.6 kcal/mol
22. Which one increases the rate of this reaction; D) 1.56 kcal/mol heat is released if 7.4 g of Ca(OH)2 is obtained
A(g) + B(s)  C(g) E) Standard heat of formation of Ca(OH)2 is -15.6 kcal/mol
A) Increasing the volume of the container
B) Addition of B 28. 2A2(g) + B2(g) + heat  CD(g) the mechanism of this
C) Addition of inert X reaction is shown as below ;
D) Decreasing the temperature A2(g) + A2(g)  E(g) (slow)
E) Increasing the amount of A
E(g) + B2(g)  CD(g) (fast)
According to this, which one does not increase the rate of the
reaction?
23. The following reaction
N2 + 3H2  2NH3 A) Using a appropriate catalyst
occurs in a 1L container. If the mole number of N2 is doubled and B) Addition of B2
the volume of the container is halved how many times does the C) Addition of A2
rate of the reaction increase? D) Increasing the partial pressure of A2
E) Increasing the temperature
A) 4 B) 8 C) 16 D) 32 E) 64
112
II. The rates of all reactions increase when the temperature is
increased.
29. Given the reaction and its rate law; III. Catalysts increase the rate of reactions.
2H2 +O2  2H2O Rate = k[H2]2 [O2] IV. As the interacting area between reacting molecules increases
if the concentration of H2 is triplet, how will the rate of the the rate of the reaction will decrease. Which one(s) is (are)
reaction increase? correct?

A) 9 B) 6 C) 2 D) 4 E) 12 A) I and II B) III and IV C) I, II and III


D) I, II and IV E) I, II, III and IV

30. Given the reactions:


I. Ag+(aq) + Cl-(aq)  AgCl(s) 35. At high temperatures, N2O3(g) decomposes to NO2(g) and
II. C8H18(l) + 25/2O2(g)  8CO2(g) + 9H2O(g) NO(g). If the rate of appearance of NO2 is 1.4x10-3 atm/s, what
III. 2NO(g) + O2(g)  2NO2(g) must be the rate of disappearances of N2O3 in atm/s?
IV. 2MnO4-(aq) + 5Sn+2(aq) + 16H+(aq) 
2Mn+2(aq) + 5Sn+4(aq) + 8H2O(l) A) 0.14x10-3 B) 1.4x10-3 C) 0.7x10-3
-2
In which of the following are the rates of the reactions given in D) 0.7x10 E) 0.28x10-3
decreasing order?

A) I>II>III>IV B) II>I>III>IV C) IV>II>I>III 36. Given the reaction and its PE-RC diagram:
D) III>IV>II>I E) I>IV>III>II C(s) + O2(g)  CO2(g) + Energy
PE (kcal)
a
31. Given the reactions:
I. Ba+2(aq) + SO4+2(aq)  BaSO4(s)
II. H2(g) + Cl2(g)  2HCl(g)
0
III. CH4(g) + 2O2(g)  CO2(g) + 2H2O(g)
In which of the following are the rates of the reactions given in b
RC
increasing order?
I. a is the activation energy of forward reaction.
A) II>III>I B) III>II>I C) II>I>III
II. b is the enthalpy of the reaction
D) I>II>III E) II=III>I
III. a-b is the activation energy of the reverse reaction.
Which one(s) is (are) correct given above?
32. Given the reaction,
A) I only B) II and III C) I and II
2A(g) + B(g)  2C(g)
D) I and III E) I, II and III
Which one of the following cannot be the rate expression of the
reaction?
37. The rate of a reaction slows down with time at constant
A) Rate = k[A]2[B]
temperature because
B) Rate = k[A]2
C) Rate = k[A] [B]
A) the concentration of reactants decreases
D) Rate = k[A]2[B] [C]
B) the concentration of products decreases
E) Rate = k[B]
C) the temperature decreases
D) the catalyst is used up
E) the energy of activation increases
33. Given the reaction and its rate law;
SO2(g) + 1/2O2(g)  SO3(g) Rate = k[SO2][O2]2
What is the order of the reaction with respect to SO2 and overall? 38. The most important effect of increasing temperature of a
reaction is to
A) 1,2 B) 2,1 C) 1,3 D) 2,3 E) 3,1
A) change the reaction mechanism
B) increase the number of collusion
34. The factors given below affect the rate of reactions. C) increase the number of effective collusion
I. When concentrations of reactants are increased the rate of the D) lower the energy of activation
reaction will increase. E) increase the activation energy for reverse reaction

113
(colorless) (violet) (colorless)
I. Color II. Pressure III. Concentration

A) I only B) II only C) I and III


D) I and II E) I, II and III
39. Which one of the following graph is wrong for the reaction?
X2(g) + Y2(g)  2XY(g) + heat
43. Which one(s) of the following reactions may be faster than
A) [X2] B) [Y2] others?
I. Ba+2(aq) + SO4-2(aq)  BaSO4(s)
II. CH4(g) + 2O2(g)  CO2(g) + 2H2O(g)
III. Ag+(aq) + Cl-(a)  AgCl(s)
time time A) I only B) II only C) I and III
D) II only E) I and III
C) [XY D) to C
]
44. The relationships among the rate expressions for the rate of a
reaction in terms of the changes in concentrations in a time
interval are given as follows.
time time Rate = -[N2O5] = [NO2] = [O2]
2t 4t t
Which one of the following is the equation of the reaction?
E) P(atm
)(at constant) A) 2N2O5  4NO2 + O2
B) 4N2O5 + O2  2N2O5
C) N2O5  2NO2 + 1/2O2
D) 4N2O5  8NO2 + 2O2
time E) 2NO2 + 1/2O2  N2O5

40. The following information is given for the function of a


catalyst. 45. In which one(s) of the following reactions pressure change
I. It changes the reaction mechanism. cannot be used to determine the rate?
II. It lowers the activation energy. I. Ba+2(aq) + 2Cl-(aq)  BaCl2(s)
III. It changes the nature of reactants and products. II. I2(g) + H2(g)  2HI(g)
IV. It does not change the yield of the reaction. III. NO2(g)  NO(g) + 1/2O2(g)
Which one(s) is (are) wrong?
A) I only B) I and II C) II and III
A) I and II B) III only C) I, II and III D) I and III E) I, II and III
D) III and IV E) IV only

46. Substances that change the speed of a chemical reaction but


41. Which of the following cannot be the rate expression of the are not themselves changed by the reaction.
reaction below?
2NO(g) + 5H2(g)  2NH3(g) + 2H2O(g) A) Electrolyte B) solute C) catalyst

A) Rate = -[NO] B) Rate = -[H2]


t t
C) Rate = PH2O D) Rate = -[NH3]
t t
E) Rate = [NO]
t

42. Which of the following changes can not be used to determine


the rate of the following reaction below?
H2(g) + I2(g)  2HI(g)
114
SOLUBILITY EQUILIBRIA
1. At 25oC, the solubility product of MgCO3 is 2.5x10-5. What is
the molarity of Mg2+ ion in the saturated MgCO3 solution? 8. Which one of the following is WRONG in case of adding
solid AgCl to a saturated solution of AgCl?
A) 5x10-3 B) 0.25 C) 2.5x10-5
D) 5x10-5 E) 2.5x10-4 A) The value of Ksp doesn’t change.
B) [Ag+] increases in the solution.
C) The density of solution doesn’t change.
2. At 25oC, the solubility product of BaSO4 is 1.5x10-9. Equal D) The addition of solid AgCl doesn’t affect equilibrium.
volumes of 0.1 M BaCl2 and 0.2 M Na2SO4 solutions are mixed. E) The concentration of the solution doesn’t change.
What is the molarity of Ba2+ions after precipitation?

A) 0.05 B) 0.1 C) 3x10-8 9. The solubility of BaSO4 is 3.9x10-5 mol/L at room


D) 3.9x10-5 E) 7.5x10-9 temperature. What is the Ksp for BaSO4 at this temperature?

A) 3.9x10-5 B) 1.7x10-5 C) 1.95x10-9


3. 0.68 g of the compound XSO4 having a Ksp of 2.5x10 can -5 D) 1x10-5 E) 1.5x10-9
be dissolved in 1-liter water at room conditions. What is the
atomic mass of X?
10. Which one of the following is CORRECT in case of adding
A) 20 B) 24 C) 40 D) 137 E) 207 solid AgBr to a saturated solution of AgBr?

A) The value of Ksp increases


4. At a particular temperature, the solubility product of CaCO3 B) The value of Ksp decreases
is 1x10-10. How many grams of CaCO3 can be dissolved in 100 C) [Ag+] increases in the solution
liters of water at this temperature? D) The density of solution increases
(CaCO3: 100) E) [Br -] doesn’t change in the solution

A) 0.01 B) 0.1 C) 1 D) 1 E) 10-5


11. Which one of the following substances has the lowest
solubility in water?
5. The solubility of AB2 at a particular temperature is 0.02
mol/L. A) CuCl : Ksp = 3.2x10-7
What is the solubility product of the following equilibrium? B) BaSO4 : Ksp = 1.5x10-9
AB2(s)  A+2 (aq) + 2B- (aq) C) AgCl : Ksp = 1.7x10-10
D) PbCrO4 : Ksp = 2x10-16
A) 3.2x10-5 B) 4x10-4 C) 2x10-2 E) AgBr : Ksp = 5x10-13
-6
D) 8x10 E) 6x10-6

6. x moles of AB2 can be dissolved in 200 mL of water by the 12. The solubility of XCl2 in water is 4.4 g/L and its solubility
following reaction. product is 1.6x10-5. What is the atomic mass of X in the
AB2(s)  A+2 (aq) + 2B-(aq) compound XCl2? (Cl: 35.5)
What is the Ksp of AB2?
A) 25 B) 88 C) 108 D) 40 E) 207
3 2 2 3 3
A) x B) 5x C) 50x D) 100x E) 500x

7. Which one of the following represents the relationship


between Ag+ and CrO42- ion concentrations in a saturated
Ag2CrO4 solution?

A) [Ag+]=[CrO42-]
B) 2x[Ag+]=[CrO42-]
C) [Ag+]=2x[CrO42-]
2
D)[Ag+]= [CrO4 ]

E) [CrO42-] = [ Ag ]

115
ALKALINE EARTH METALS B) Salts of Sr give red light in signal rockets.
C) Ra is used in radiotherapy.
1. Which one is correct about Alkaline Earth Metals? D) Ca is used in metallurgy to purify metals.
I. They lose 2 electrons in order to get +2 charges.
II. They are found in nature in elemental forms. 12. Which color is observed when Mg is burnt?
III. Their oxides have strong acidic characters. A) Yellow B) Blue C) White D) Gray
A) I only B) I and II
13. Which gas is produced when an alkaline earth metal reacts
C) I and III D) II and III with water?
A) Oxygen B) Hydrogen
2. Which one is not a member of alkaline earth metals C) Carbon dioxide D) Nitrogen
group?
A) B B) Ca C) Mg D)Ba 14. Which of the following naming of compounds is given
wrongly?
3. Which of the following ions makes water hard? A) CaCO3 Calcite
A) Ca+2 B) Ma+2 B) CaCO3 . MgCO3 Dolamite
+2 +2
C) Ca and Ba D) Ca+2 and Mg+2 C) SiO2 Silica
D) 3Ca3(PO4)2 . CaF2 Phosphate rock
4. Which of the following is a radioactive member of Alkaline E) CO(NH2)2 TNT(trinitrotoluene)
Earth Metals Group?
A) Rn B) Ra C) Fr D) Be

5. In which one is the formula of marble given correctly?


A) CaSO4.H2O B) Ca(OH)2
C) CaCO3 D) CaF2

6. Which of the following common names of calcium


compounds is given correctly?
I. Ca(OH)2 : Slaked Lime
II. CaO : Lime
III. CaCO3 : Lime Stone
IV. CaC2: Lime Water
A) I only B) II and III
C) I, II and III D) II, III and IV

7. Which of the following is gypsum salt?


A) CaSO4.2H2O B) MgSO4
C) CaSO4.1/2H2O D) MgSO4.7H2O

8. Which of the following alloy of metals is used to manufacture


airplanes?
A) Be B) Mg C) Ba D) Ra

9. Which of the compounds is used to get X-ray of intestines?


A) CaSO4 B) Be(OH)2
C) BaSO4 D) CaC2

10. What is milk of magnesia?


A) It is pure magnesium.
B) It is a compound used in acetylene welding.
C) It is a pain reliever.
D) It is a compound of magnesium used as stomach antacid

11. Which one is wrong about the uses of alkaline earth metals?
A) Mg is used in rocket fuels and signal rockets.
116
ALKALINE METALS B) lose electron
C) be reduced
1. Which of the following elements is not in the alkaline metals D) gain protons
group? E) gain neutrons
A) Na B) K C) Cs D) Al
10.Which of the following ions has the greatest hydration
2. Some alkaline metals and their electron configurations are energy?
given below. Which one is wrong? A) Na+ B) Al3+ C) Ba2+ D) K+ E) Cs+
A) K: 1s2 2s2 2p6 3s2 3p6 4s1
B) Na: 1s2 2s2 2p6 3s2 11. Which one of the following atoms has the smallest volume?
C) Li: 1s2 2s1 A) Li B) Na C) K D) Rb
D) H: 1s1

12. What are the oxidation states of the nonmetals for the given
3. Which of the following is a radioactive element in compounds, respectively: NaH, K2O, Li3N?
alkaline metals group? A) -1, +2, +3 B) -1, +1, +2
A) H B) Cs C) Fr D) Rb C) +1, -2, -3 D) -1, -2, -3

4. Which one is accepted as the most active metal?


A) Fr B) Rb B) Ca D) Li

5. Which one is not the characteristic of alkaline


metals?
A) They are active metals.
B) They give H2 gas when reacted with water.
C) They have +1 oxidation state in their compounds.
D) They form acidic compounds when reacted with
water.

6. Which of the following methods is used to obtain


pure Na metal?
A) Haber Process B) Ostwald Method
C) Down Method D) Contact Process

7. Which of the following is wrong about NaCl salt?


A) It is known as table salt as well.
B) It is a covalent compound.
C) It is colorless cubic structured crystals.
D) It is readily soluble in water.

8. Which one is not correct about the uses of alkaline


metals?
A) Some Li compounds are used in organic reactions.
B) Na is used in metallurgy to produce pure metals
from their oxides.
C) K is used commonly in fertilizers.
D) Cs is used in nuclear reactors to transfer excess
heat.

9. In chemical reactions metals tend to


A) gain electrons

117
Aluminium 199. Which one of the following is not amphoteric
190. Which one(s) of the following has an amphoteric metal?
character? A) AI B) Zn C) Cr D) Br
I. Al2O3
II. Al(OH)3 200. What is the name of the compound, Na3AIO3?
III. AlCl3 A) Aluminum sodium oxide
A) I only B) II only C) III only B) Sodium aluminate
D) I and II E) I and III C) Sodium alumminum oxide
D) Aluminum (III) Sodium oxide

191. What is the formula of CORUNDUM? 201. Which of the following elements is the most
A) Al2O3 B) AlCl3 C) Al(OH)3 abundant metal in earth’s crust?
D) Al2(SO4)3 E) Al2(CO3)3 A) Fe B)
O C)
Al
192. In the industry, which compound of aluminum is D) Si E) C
used in the production of aluminum metal?
A) Al2O3 B) AlCl3 C) Al(OH)3 202. What is the name of process of which is used to
D) Al2(SO4)3 E) Al2(CO3)3 get pure Al from bauxite mineral?
A)
Mond
193. Which of the following formula is wrong? B) Vulcanisation C) Contact
A) AlF B) AlCl3 C) AlBr3 D) Al(OH)3 E) D)
Al2(SO4)4 Baeyer
E) Haber

194. Al metal has +1 charge when it forms a compound 203. Which of the following is wrong about the uses
with .......... of aluminium?
A) F B) Cl C) Br D) I E) C A) It is used in many alloys.
B) It is used in electrical wires.
195. Which of the following gas is produced when Al C) It is used for packaging foods.
metal is reacted with cold HNO3 solution? D) It is used in construction industry.
A) NO2 B) N2O C) NO E) It is used in camera flashes.
D) N2O4 E) No production of any gas

196. In the production of a metal by reduction with


carbon at moderate temperature is not feasible with.....
A) CdO B) Al2O3 C) PbO D) HgO E)
Cu2O

197. Which one of he following metals is more


active than aluminum?
A) Cr B) Cu C) Ag D) Ca

198. What is the place of the Al among the metals


exist in the earth’s crust by abundance?
A) 1 B) 2 C) 3 D) 4

118
Inert Metals

204. I. Chalcocite 209. Bronze is a kind of alloy and it is formed by the


II. Chalcopyrite mixing of ………. and ……….
III. Siderite
Which one(s) of the following compounds given above A) Hg / Fe B) Cu / Pb C) Cu / Cd
is (are) the minerals of copper? D) Cu / Zn E) Sn / Cu

A) I only B) II only C) III only


D) I and II E) I and III 210. Which of the following metals exists as liquid at
room temperature?

205. Which one(s) of the following compounds is (are) A) Pb B) Au C) Ag D) Hg E) Zn


not the minerals of copper?
I. Hematite
II. Cuprite 211. Which one(s) of the following is (are) true about
III. Malachite the properties of mercury and its compounds?
IV. Copper (II) sulphide I. HgO is thermally unstable
II. HgF2 is covalent compound
A) I and II B) I and IV C) I, II and III III. Because of its high density it is used in barometers
D) II and IV E) II and III
A) I only B) II only C) I and III
D) II and III E) I, II and III
206. Which one(s) of the following is (are) the
properties of metallic copper?
I. Malleable and ductile 212. Which one(s) of the following is(are) true about
II. Very good conductor of electricity and heat the uses of mercury and its compounds?
III. Soluble in water I. Hg is used in barometers and thermometers
IV. Reacts with HCI II. Mercury vapor is used in street lamps and
fluorescent tubes
A) I only B) I and II C) I and IV III. HgO is used in pigments
D) III and IV E) II, III and III
207. Which one(s) of the following is (are) not the A) I only B) II only C) I and II
properties of metallic copper? D) II and III E) I, II and III
I. It can react with HCI and HNO3
II. Malleable and ductile
III. It is insoluble in water 213. Which one(s) of the following is (are) wrong about
IV. It is good conductor of electricity and heat the properties of mercury and its compounds?
I. Most of the mercury compounds are not hydrated
A) I only B) II only C) II and IV II. Compounds of mercury are much more toxic than
D) I and III E) II, III and IV pure element
III. Mercury alloys are called anthracite

208. Brass is a kind of alloy and it is formed by the A) I only B) II only C) II and III
mixing of ……….. and ……….. D) I and III E) III only

A) Cu / Cd B) Cu / Zn C) Sn / Ag
D) Hg / Fe E) Cu / Pb
119
214. Which one(s) of the following is (are) wrong about 219. Which one of the following is the most important
the common oxidation states of related metals? use of gold?
I. Cu: +2 and +3
II. Hg: +1 and +2 A) In electrical industry
III. Ag: +1 and +2 B) In monetary reserves
C) In photography
A) I only B) II only C) I and III D) In medicinal chemistry
D) II and III E) III only E) In the manufacture of special glasses and ceramics

215. Which one(s) of the following is (are) true about 220. Which of the following compounds completes the
the common oxidation states of related metals? following reaction?
I. Au: +1 and +3 Au + HCI HNO3  …………+ H2O + NO
II. Pb: +1 and +4
III. Cd: +2 and +3 A) HAuCI2 B) AuCl4 C) HAuCl4
D) AuCl E) AuCl3
A) I only B) II only C) II and III
D) I and III E) III only 221. What is the mass percentage of Cu metal on the
earth crust?

216. Which one has the highest electrical conductivity A) 0.3 % B) 0.2% C) 0.1%
among the others? D) 0.01% E) 0.02%

A) Cu B) Au C) Pt D) Ni E) Ag
222. In the production of Cu metal in industry, which
compound of copper is used?
217. Which one(s) of the following is (are) wrong about
the properties of Ag and its compounds? A) CuO B) Cu2O C) Cu2S
I. Ag is resistant to air oxidation D) CuSO4 E) Cu(OH)CO3
II. It does not react with H2SO4 and HNO3
III. AgI is used to seed clouds
223. Which one is the second most conductive element
A) I only B) II only C) III only after the Ag metal?
D) I and II E) II and III
A) Al B) Cu C) Pt D) Au E) Ni

218. Which one(s) of the following is (are) true about


the uses of Ag and its compounds? 224. What is (are) the oxidation states of the copper
I. Ag compounds are used in electroplating in batteries metal?
and medicine
II. Ag is used as catalyst A) +1 only B) +2 only C) +1 and+2
III. Its most important use is in photographic processes D) +3 only E) +2 and+3

A) I only B) II only C) III only


D) II and III E) I, II and III 225. What is the electron configuration of the copper
metal at neutral form? (Atomic number of Cu is 29)

A) Ar |4s2 3d9 B) Ar |4s1 3d10C) Ar |4s2 3d3 4p6


120
D) Ar |4s2 4p6 4d3 E) Ar |4s2 4d9

234. Which compound of silver is used to identify


226. Most oxidation state of the Cu metal is...... aldehyde from other organic compound?

A) +1 B) +2 C) +3 D) +1 and +2 E) +3 A) AgCl B) AgBr C) AgOH


D) Ag2O E) AgNO3

227. Which gas would be produced when concentrated


H2SO4 reacted with Cu metal? 235. How many gram of O2 gas is produced by AgNO3
when 21.25 gram of silver is decomposed under the
A) SO B) SO2 C) SO3 D) O2 E) H2 temperature.

A) 1.4 B) 2.8 C) 5.6 D) 11.2 E) 2.24


228. What is the formula of the basic carbonate?

A) CuCO3 B) CuFeS2 C) Cu(OH)2 236. Which of the following compounds of silver gives
D) Cu(OH)CO3 E) Cu(OH)2CO3 the reaction with acetylene?

A) AgCl B) AgBr C) AgOH


229. Name of the copper is coming from ......... D) AgI E) AgNO3

A) Greek B) Cyprus C) Turkey


D) Italy E) China 237. Which of the following gases is (are) produced,
when concentrated H2SO4 is reacted with silver metal?

230. What is (are) the oxidation state(s) of Ag metal? A) SO B) SO2 C) H2 D) O2 E) H2O (g)

A) only +1 B) only +2 C) +1 and +2


D) only +3 E) +1 and +3 238. In ancient time Silver is known as ........

A) Saturn B) Moon C) Mars


231. Most common oxidation state of Silver is D) Neptune E) Uranus

A) +1 B) +2 C) +3 D) +4 E) +5
239. What is the color of Ag+ ion when reacts with I-
ion
232. What is the electron configuration of the Silver
metal at neutral form? (29Ag) A) yellow B) red C) green
D) orange E) green
A) Ar |4s2 3d9 B) Ar |4s1 3d10C) Ar |4s2 3d3 4p6
D) Ar |4s2 4p6 4d3 E) Ar |4s2 4d9 240. What is (are) the oxidation state(s) of the Platinum
metal

A) +4 and +6 B) +3 and +4 C) +3 only


233. The most conductive metal is ......... D) +2 and +4 E) +4 only

A) Ag B) Al C) Cu D) Au E) Pt
121
241. What is (are) the oxidation state(s) of gold? A) Ag2O B) AgNO3 C) Agl D) AgCI

A) +3 B) +2 C) +1 D) +1 and +3 E) +1 and +2 251. Which one of the following is not an inert


metal?
A) Cu B) Ag C) Au D) Mn
242. The most common oxidation state of gold is
252. Which one of the following metal is in liquid
A) +1 B) +2 C) +3 D) +4 E) +5 state at room temperature?
A) H2 B) He C) Ag D) Hg

243. What is the proportion between the HCl and HNO3 253. Which one of the following was discovered in
(mixture), which gives reaction with Au? Cyprus?
A) Silver B) Copper C) Gold D)
A) 5 B) 4 C) 3 D) 2 E) 1 Platinium
254. What is karat of pure gold?
A) 12 B) 14 C) 18 D) 24
244. What is the coefficient of the water in reaction of
gold with 3HCl+HNO3? 255. Which one of the following is formula of king
water?
A) 1 B) 2 C) 3 D) 4 E) 5 A) 3HNP3 + HCI B) HNO3 + 3HCI
C) HNO3 + HCl D) H2SO4 + 3HCI
245. The best oxidizing agent among the following is
.......

A) Na+ B) Al3+ C) Ag2+ D) Cu2+ E)


Li+

246. Which one(s) of the following has the lowest


melting point?

A) Al B) Hg C) Fe D) Cr E) Zn

247. The only liquid

A) mercury B) tin C) bronze

248. Which of the following elements has an oxidation


state +3 in its most compounds?

A) Au B) Ni C) Cu D) Ag E) Cr

249. Which of the following elements will dissolve in


both acidic and basic solutions?

A) Na B) Zn C) Al D) Hg E) K

250. Which one of the following compound is used


in cloud seeding?
122
Lead
A) +8/3 B) +2 C) +4 D) +2 and +3 E) +2
256. Which one(s) of the following oxides is (are) the and +4
best known oxides of lead?
I. PbO2
II. PbO 263. According to the reaction, which compound(s) of
III. Pb3O4 Pb is (are) produced?
Pb3O4 (s) + HNO3 (aq) 
A) I only B) II only C) III only I. Pb(NO3)2
D) I and II E) I, II and III II. Pb(NO3)4
III. PbO2
IV. PbO
257. Which one is wrong about the uses of lead or its
compounds? A) I and II B) III and IV C) I and IV
A) In batteries
B) In glass and cement D) I and III E) II and IV
C) In metal protecting processes and ceramics
D) In paints and matches
E) In medicinal applications
264. What is (are) the oxidation number(s) of the lead
258. Which one of the following metals is used in the in Pb3O4 compound?
manufacture of standard voltaic cells known as Weston
cells? A) +2 B) +4 C) +8/3
D) +2, +4 E) +2 , +8/3
A) Ni B) Pb C) Pt D) Fe E) Cd

259. What is the formula of GELENA? 265. What is the coefficient of the water in the reaction
of Pb3O4 with diluted HNO3 solution?
A) PbO B) PbO2 C) PbS Pb3O4(s) +HNO3 (aq) 
D) PbI2 E) PbCO3
A) 1 B) 2 C) 3 D) 4 E) 5

260. Which of the following is the formula of litharge


and red lead? 266. What is the formula of the white lead?

A) PbO, PbO2 A) 2PbCO3.Pb(OH)2


B) PbO2, Pb3O4 B) PbCrO4
C) PbO, PbS C) Pb(NO3)2
D) PbS, PbI2 D) Pb3O4
E) None of them E) PbO2

261. What is the oxidation state of Pb metal?


A) +2 B) +2 and +3 C) +3 and +4 267. When 20.7 gram of Pb is reacted with enough
D) +2 and +4 E) +4 only amount of concentrated HNO3, how many grams of
which gas is produced? (Pb=207 g/ mol)

262. What is the oxidation number of Pb element in A) N2O, 4.48L


Pb3O4 compound? B) NO, 4.48L
123
C) NO2, 4.48L
D) NO, 2.24L
E) NO2, 2.24L

268. The lead (II) sulphide is first converted to lead (II)


oxide by strongly heating it air, such process is
called……….

A) Flotation B) Hydrogenation C) Roasting


D) Electrolysis E) Blast furnace

269. What is (are) the principle oxidation state(s) of the


lead?

A) +2 B) +4 C) +3 D) +2 and +4 E) +3 and +4

270. What is the electron configuration of the lead


(82Pb)?

A) Xe |4f14 5d10 6s2 6p4


B) Xe |5f14 5d10 6s2 6p2
C) Xe |5f 14 6d10 6s2 6p2
D) Xe |5d10 6s2 6p2
E) None of them

271. I.It is a heavy metal.


II. It is a transition metal.
III. All its compounds are poisonous.
Which of the above is (are) incorrect for lead?
A) I only B) III only
C) I and II D) II and III

272. Which of the following oxidation numbers of


lead are given wrong?
I. PbO + 2
II. PbO2 + 4
III. Pb3O4 + 4/3
A) I only B) III only
C) I and II D) II and III

124
Some Transition Metals
273. Which one of the following metals causes a A) The most stable oxidation state of it is +2
disease known as “itai-itai kyo”? B) It is used in Ni-Cd batteries
C) It is a magnetic element.
A) Cu B) Hg C) Pb D) Cd E) Zn D) It is used in industry in different alloys.
E) It is also used as catalyst in organic reactions
274. Which one is not correct about the uses of
cadmium or its compounds? 279. Which of the following properties would be
expected generally for the transition metals?
A) In the protection of iron against corrosion
B) In electroplating and batteries A) Low melting point
C) As a catalyst mainly in organic reactions B) High ionization energy
D) In cement and glass C) Variable oxidation number
E) In solar cells and pigments D) Positive standard electrode potential
E) Having low electrical conductance
275. Which one(s) of the formulas of the oxides of
iron is (are) written wrongly?
I. Fe2O3 II. Fe2O4 III. Fe3O4 280. What is the oxidation state of the iron metal?
IV. FeO
A) +2 B) +3 C) +2, +3
A) I only B) II only C) III only D) +3, +4 E) +2, +3, +6
D) IV only E) II and III

281. How many gram of iron is produced, when a


2.8-gram of carbon monoxide is reacted with iron (III)
276. Which one is wrong about the uses of iron or oxide? (C: 12, H: 1, Fe: 56)
its compounds?
A) 56 B) 5.6 C) 11.2 D) 112 E) 2.8
A) Major commercial use of iron is in steel making
B) Various iron oxides are used in electronic devices
and recording tapes 282. What is the formula of iron carbide?
C) Iron complexes are used in inks and dyes
D) Iron is widely used in construction industry A) Fe4C2 B) Fe4C3 C) Fe3C
E) Iron is also used in batteries as electrode D) Fe2C E) Fe4C6

The minerals of nickel are mainly ………,


277. 283. Which of the following has amphoteric
……… and ……… properties?
A) Fe2+ B) Fe6+ C) Fe3+,
A) Sulphides / oxides / arsenides Fe2O3
B) Sulphides / oxides / carbonates D) Fe2+, Fe(OH)3 E) Fe2O3
C) Oxides / sulphates / arsenides ,Fe(HO)3
D) Sulphates / arsenides / carbonates
E) Sulphites / chlorites / arsenides
284. Which of the following ions is used to
determine Fe+2 and Fe+3 ?
278. Which one is wrong about the nickel or its A) OH- B) Cl- C) I- D) NO3- E)
-
compounds? CO3
125
A) Ni, H2 B) Ni, O2 C) Ni, SO2 D) H2, O2 E) only
O2
285. Iron is the .............most abundance element in
earth’s crust.
A) first B) second C) third 292. What is the electron configuration of Ni3+ ion?
D) fourth E) fifth (28Ni)

A) Ar |4s2 3d5 B) Ar |4s13d6 C) Ar |4s0 3d7


2+
286. What is the electron configuration of Fe ion? D) Ar |4s2 3d5 E) Ar |4s24p5
(26Fe)

A) Ar |4s23d3 B) Ar |4s03d5 C) Ar |4s24d3 293. What is the mass percentage of carbon element
D) Ar |4s04d5 E) Ar |4s24p3 in the steel?

A) 1.7 B) 2.9 C) 3.5 D) 4.6 E) 5.8


287. What are the oxidation states of iron atoms in
Fe4 [Fe(CN)6]3 ? 294. What is the electron configuration of Cd2+ ion?
(30Cd)
A) both +2 B) both +3 C) +2, +3
D) both +6 E) +3, +6 A) Ar |4s03d10 B) Ar |4s23d8 C) Ar |4s23d24p6
D) Ar |4s24d8 E) Ar |4s24p65s2

288. What are the formula of magnetite, hematite 295. How many liters of H2 will be produced at STP
and ilmetite? when a 13-gram of Zn metal reacted with enough
amount of NaOH solution?
A) Fe3O4, Fe2O3, FeTiO3
B) Fe3O4, FeCO3, FeS A) 2.24 B) 22.4 C) 4.48 D) 3.36
C) FeS, Fe3O4, Fe2O3 E) 8.96
D) HFeO3, Fe3O4, FeTiO2
E) FeO, FeCO3, Fe2O3
296. What is the molecular weight of the compound
formed by the reaction of Zn with NaOH solution?
289. What is (are) the oxidation state(s) of nickel?
A) 120 B) 143 C) 158 D) 166 E) 208
A) +2 B) +3 C) +3 and +6
D) +4 E) +2 and +3
297. What is the oxidation state of zinc?

290. In the neutral media electrolysis of NiSO4 is A) +1 B) +2 C) +3 D) +4 E) +5


produced ........

A) Ni, H2 B) Ni, O2 C) Ni, SO2 298. Which one(s) of the following compound of
D) H2, O2 E) only O2 zinc is (are) amphoteric?
I .Zn(OH)2
II.ZnSO4
291. In the acidic media electrolysis of NiSO4 is III. ZnS
produced ........ IV. ZnO

126
A) IV only B) I and II C) I and III
D) I, II and III E) I and IV
305. Which one(s) of the following is (are) basic
compound of chromium metal?
299. Which one(s) of the following can be produced I. Cr2O3
by the electrolysis of zinc solution? II. CrO3
I. Zn metal III. CrO
II. O2 gas IV. Cr(OH)2
III. H2 gas
IV. SO2 gas A) I and II B) III and IV C) I and III
D) II and III E) II and IV
A) I only B) II only C) I and II
D) III only E) I, II and IV
306. Which one(s) of the following is (are) acidic
300. Who discovered firstly the element of compound of chromium metal?
Chromium? I. Cr2O3
II. CrO3
A) Vokelen B) Oregot C) Klaprot III. H2Cr2O7
D) Arkel E) Berzelius IV. Cr(OH)2
V. H2CrO4

301. What is the maximum bond capacity of zinc? A) I and II B) II, III and IV C) II, III and
V
A) 1 B) 2 C) 3 D) 4 E) 5 D) III, IV and V E) III and
IV
302. What are the common oxidation states of
chromium metal? 307. What is the electron configuration of
chromium? (24Cr)
A) +1 and +2 B) +2 and +3 C) +1, +4 and +6
D) +2, +5 and +6 E) +3 and A) Ar |4s2 3d4 B) Ar |4s2 4p4 C) Ar |4s0 3d6
2 4
+6 D) Ar |4s 4d E) Ar |4s1 3d5

308. Which one of the following metals is used in


303. Chromium metal is produced from ....... the manufacture of semiconductors?

A) Cr2O3 B) Fe(CrO2)2 C) Cr(OH)2 A) Pb B) Cd C) Fe D) Cu E) Hg


D) KCrO4 E) NaCrO4
309. Which of the following is wrong about the
general characteristics of transition metals?
304. Which one(s) of the following chromium A) They are good conductors of heat and electricity.
compounds has (have) amphoteric properties? B) They have high melting and boiling points.
I. Cr2O3 C) They are d block elements in the periodic table.
II. CrO3 D) They are usually active metals than 2A group
III. CrO metals.
IV. Cr(OH)2
310. If compared with other metals according to
A) I and II B) III and IV C) I and IV abundances in earth’s crust what would be the place of
D) II and III E) II and IV iron?
127
A) 1 B) 2 C) 3 D) 4 322. Which one is a property of zinc?
A) Zinc is an active metal.
311. Which one is fool’s gold? B) Generally, zinc is found in pure metallic form in
A) FeSO4 B) FeCO3 C) FeS2 D) Fe3O4 nature.
C) Charge of zinc is always 2+ in its compounds
312. Which of the following elements shows D) NO2 is used to produce pure Zn metal from ZnO
magnetic property?
A) Al B) Na C) Ni D) C 323. Which of the following compound is known as
Chinese white?
313. Which of the following elements is the raw A)
material used in the preparation of steel? FeO
A) Zn B) Fe C) Ba D) Ag B)
NiO
314. Which of the following is not one of the main C) ZnO D) TiO2
categories of steel?
I. Carbon steel 324. ……………poisoning is a painful bone disease,
II. Stainless steel was observed first in Japan in rice fields.
III. Nickel steel A)
A) I only B) III only Zinc
C) I and II D) II and III B) Iron C)
Cadmium
315. Which of the following metals is used in
organic reactions to absorb hydrogen gas? D)
A) Ni B) Fe C) Li D) Al Nickel
E) Mercury
316. Which of the following metals is obtained as
pure by using Mond method?
A) Fe B) Co C) Cr D) Ni

317. Which of the following is one of the oxidation


numbers of Fe?
A) +1 B) +2 C) -2 D) +5

318. Which element is similar to Cd according to its


chemical properties?
A) Zn B) Na C) Ca D) Ba

319. Which of the following is the greatest oxidation


number that Cr may take?
A) 7 B) 5 C) 4 D) 3

320. Which of the following elements is very


important for the aircraft industry?
A) Cs B) Ca C) Ti D) Au

321. Lack of which element is the reason for anemia?


A) Fe B) AI C) Ca D) Sn

128
HYDROCARBONS
A) phenol
1. Which one is the general formula of B) hydroxy toluene
cycloalkadienes? C) propanoic acid
D) acetic acid
A) CnH2n B) CnH2n-2 C) CnH2n-4 E) benzoic acid
D) CnH2n-6 E) CnH2n-8

9. If 37 carbon atoms are present in an alkane


2. Which one of the numbers given cannot be the total molecule, total number of atoms in the molecule is
number of atoms in an alkane molecule?
A) 74 B) 76 C) 111 D) 113 E) 114
A) 83 B) 64 C) 50 D) 32 E) 17

10. Which one cannot be a cyclohydrocarbon?


3. Which one of the following alkanes has the highest
boiling point? A) C6H11Br B) C15H27Cl3 C) C9H12
D) C4H8 Br2 E) C3H6
A) 2-methyl pentane
B) 3-methyl pentane
C) 2,3-dimethyl butane 11. Which is an isomer of 2,3-dimethyl 3-ethyl
D) n-hexane hexane?
E) 2,3,4-trimethyl pentane
A) 2,6-dimethyl heptane
4. Which substance is called “ chloroform’’? B) 3-ethyl octane
C) 2-phenyl propane
A) CH3Cl B) CH2Cl2 C) CHCl3 D) 2,5-dimethyl nonane
D) CHCl2 F E) CCl2F2 E) 3-methyl decane

5. How many grams of benzene can be produced 12. Which ones are not saturated?
using 36 grams of acetylene? I. acetylene II. methane III. cyclobutane IV. styrene

A) 18 B) 24 C) 36 D) 48 E) 72 A) I and II B) II and IV C) I and IV


D) I and III E) II, III

6. Which one is the molecular formula of


naphthalene? 13. When 2 moles of a 4-carbon hydrocarbon are
burned, 10 moles of oxygen gas is used up. The
A) C12H12 B) C12H10 C) C10H10 molecular formula of hydrocarbon is
D) C10H8 E) C8H8
A) C4H8 B) C4H6 C) C4H10 D) C4H4 E) C4H3

7. Which one(s) is (are) the hybridization type(s) of


carbons in toluene? 14. Which of the following substances will turn
KMnO4 solution rapidly from purple to colorless?
2 3 2 3 3
A) sp B) sp C) sp D) sp , sp E) sp d
A) methane B) ethane C) ethene
D) acetylene E) toluene
8. Which one is the strongest acid among the others?
129
15. Which of the following is not an alkane?
22. How many atoms are there in one molecule of
A) C2H6 B) C3H8 C) C4H6 D) C5H12 E) C6H14 TNT?

A) 17 B) 18 C) 19 D) 20 E) 21
16. Which is the property of alkanes?

A) the general formula is CXH2X-2 23. Which one of the following alkanes has the highest
B) soluble in water boiling point?
C) can be hydrogenated
D) does not burn A) 2-methyl pentane
E) may react with substitution B) 3-methyl pentane
C) 2,3-dimethyl butane
D) n-hexane
17. Which one of the following substances will turn E) 2,3,4-trimethyl pentane
bromine water rapidly from orange-red to colorless?

A) CH4 B) C2H6 C) C4H10 D) C2H4 E) C3H8 24. How many sigma() and pi() bonds are there in
2-butyne?

18. Which is not a property of alkenes? A) 2 , 9 B) 3 , 8 C) 2 , 8 D) 3 , 9 E) 2 , 10

A) the general formula is CXH2X


B) soluble in water 25. In Wurtz reaction, bromides of ethyl and propyl are
C) it can be hydrated to give an alcohol used. So, the product(s) would be . . . . . . . . . .
D) it may be burned I. C4H10 II. C5H12 III. C6H14
E) it may react by addition reaction .
A) I only B) II only C) III only
D) all of them E) none of them
19. There are . . . . . . . . .hydrogens in a toluene
molecule.
26. Which of the following elements must be found in
A) 7 B) 8 C) 9 D) 10 E) 11 all organic compounds?

A) Oxygen B) Carbon C) Nitrogen


20. The organic family that has geometrical isomerism D) Sulfide E) Phosphorus
is…

A) alkanes 27. Which one of the following is the name of the


B) alkenes compound whose structural formula is given below?
C) alkynes
CH3 C2H5
D) aromatics | |
E) all CH3 - CH - C - H
|
CH3
21. When 0.3 moles of acetylene is burned, total mole
number of the products will be . . . . . . . . .
A) 2,3-dimethyl-3-ethyl propane
A) 0.3 B) 0.45 C) 0.60 D) 0.75 E) 0.90 B) 2-methyl-3-ethyl butane
130
C) 2-ethyl-3-methyl butane A) III and IV B) I and IV C) IV only
D) 2,3-dimethyl pentane D) II and III E) II and IV
E) 3,4-dimethyl pentane

33. Two most important natural sources of alkanes are


28. Which one(s) of the following is (are) the .......... and ................
characteristic properties of the organic compounds?
I. They have covalent bonds between their atoms. A) petroleum / forests.
II. They are soluble in water but insoluble in organic B) natural gas / coal.
solvents. C) petroleum / natural gas.
III. Organic reactions are slow and complex. D) gasoline / coal.
E) marshes / swamps.
A) I and III B) II and III C) I and II
D) I only E) I, II and III
34. Which one(s) of the following alkanes is(are)
named wrongly?
29. The rules for naming organic compounds have
I. CH3
been arranged by the ....................... |
CH3-C-CH3 neo-pentane
A) ICC B) ICUC C) IUPC D) IUPAC E) ICHO |
CH3

II. CH2 - CH-Cl


30. Which of the following is the general formula for | | 1-chloro-2-methyl butane
normal alkanes? CH2 - CH-CH3

A) CnH2n B) CnH2n-2 C) CnH2n+4


D) CnH2nO E) CnH2n+2 III. CH3-CH-CH-CH3
| | 3-methyl-4-chloropentane
Cl C2H5

31. Which one(s) of the following is (are) correct for


the alkenes?
I. The bonds between C and H are all sigma.
II. They have carbon - carbon single bonds. A) I and II B) II and III C) II only
III. The simplest alkane is C2H6, ethane. D) III only E) I, II and III

A) I only B) I and II C) II and III


D) I and III E) I, II and III 35. Which one of the following alkanes is at liquid
state at 25oC?

32. Which one(s) of the alkyl groups is (are) named A) C2H6 B) C5H12 C) CH4
wrongly? D) C7H16 E) C19H40
I. C2H5- ethyl

II. CH3-CH2-CH2-CH2- n-butyl 36. Which one(s) is (are) saturated?


I. ethyl alcohol
III. CH3-CH-CH3 n-propyl II. methane
| III. cyclobutane
IV. CH3-CH2-CH2- iso propyl IV. 2-methyl butane

A) I only B) II and IV C) I and IV


131
D) I and III E) I, II, III and IV E) 200 275 25

42. To which of the following hydrocarbons 2 mol of


H2 gas can be added?
37. Which of the following structures is the isomer of
butene? A) CH4 B) C2H4 C) C4H8 D) C3H4 E)
A) H2C-CH2
| B) CH3-CH2-CH2-CH3 C3H6
H2C-CH2
D) CH3-CH2-C=CH2
43. Which of the following substances affects the color
C) CHC-CH2-CH3 |
CH3 of Br2 solution?

E) CH3 A) Alkane B) Alkene C) Alcohol D) Aldehyde E)


| Ether
CH3-CH-CH3
|
44. When one mole of water is drawn from one mole
of propyl alcohol which of the following substances
can be obtained?
38. To burn completely 8 L of equal moles of CH4 and
C2H6 mixture how many liters of O2 is necessary? A) Propane B) Propene C) Propyne
D) Acetone E) Methyl ether
A) 8 B) 12 C) 10 D) 22 E) 24

45. Which of the following hydrocarbons produces a


39. How many liters air is needed to burn 4 grams of white precipitation when reacted with AgNO3 in NH3
C3H4 at STP? (C: 12, H: 1) solution?

A) 89.6 B) 8.96 C) 22.4 D) 44.8 E) 448 A) C2H4 B) C2H6 C) C2H6 D) C2H2 E) CH4

40. At STP a 25 L mixture of CH4 and C2H4 gases is 46. When a-100 grams of sample of CaC2 at STP are
burnt with 65 L of O2 gas. What is the mass ratio of hydrolyzed 33.6 L of C2H2 is obtained. What is the
gases in the mixture? (C: 12, H: 1) impurity percentage of CaC2 in the sample?

A) 15/10 B) 8/21 C) 7/4 D) 5/4 E) 8/5 A) 4 B) 8 C) 36 D) 64 E) 96

41. In the mixture of CH4, C2H6 and H2, each gas has 47. Which of the following information is (are) correct
equal moles. To convert all gases into CO2 and H2O for 2-butene?
960 grams of O2 have been consumed. At initial, what I. It is an unsaturated hydrocarbon.
were the masses of each gas in the mixture? (C: 12, O: II. It has a cyclic structure.
16, H: 1) III. It has 4-carbon atom.
IV. Its molecular weight is 56 g.
CH4 C2H6 H2
A) 64 120 8 A) II and IV B) I and III C) I, II and IV
B) 80 150 10 D) II and III E) I, II and III
C) 120 225 15
D) 160 300 20
132
48. 2R-Br + 2Na  55. In the saturation of ethylene that was obtained form
What is the name of the product obtained in the 20 g of 23% ethyl, alcohol how many liter of H2 will
reaction above? be needed at STP? (O: 16, C: 12)

A) Alcohol B) Alkane C) Ether D) Alkyne E) A) 2.24 B) 4.48 C) 11.2 D) 22.4 E) 44.8


Alkane

56. Which of the following reactions cannot be


49. Which of the following completes the reaction occurred?
given below? Catalyst
C4H9-COONa + NaOH  ………. + Na2CO3 A) C4H10 + H2  C4H12
o
15 C
A) C2H8 B) C4H10 C) C4H6 D) C3H8 E) C3H6
B) S + H2  H2S
o
400 C
50. I. It is an unsaturated hydrocarbon.
II. It has a cyclic structure. C) FeO + H2  Fe + H2S
III. It has five carbon atoms. o
800 C
IV. Its molecular weight is 70 grams.
Which ones of the following properties above are Catalyst
correct for 3-pentene? (C: 12, H: 1) D) CO + 2H2  CH3OH
o
200 C
A) II and IV B) I and III C) I, III and IV
D) II and III E) I, II and III E) PbO + H2  PbO + H2O
o
100 C
51. As a result of burning of one mol of an alkyne 7
mols O2 is necessary. What is the formula of the
alkyne? 57. To burn completely acetylene obtained by the
reaction of 6.4 g of CaC2 with water how many liters
A) C2H2 B) C2H4 C) C3H4 D) C4H6 E) C5H8 of oxygen gas will be needed? (Ca: 40, C: 12)
52. Which of the following hydrocarbons does not
react with H2 gas? A) 2.8 B) 4.48 C) 5.6 D) 11.2 E) 22.4

A) C2H2 B) C2H4 C) C3H4 D) C3H8 E) C4H6


58. Which of the following elements must be found in
all organic compounds?
53. Which of the following hydrocarbons can be
obtained by using Wurtz synthesis? A) Oxygen B) Carbon C) Nitrogen
D) Sulfide E) Phosphorus
A) C2H4 B) CH4 C) C3H4 D) C2H2 E) C2H6

59. Which ones of the following structural formulas of


54. When a 10 g mixture of C2H2 and C2H6 is acted the hydrocarbons can be shown as a general formula of
with the solution of AgNO3 in NH3, 24 g of precipitate II. H H H
CnH2nI.? HH
is obtained. What is the percentage of C2H2 in the I | |
| | H3C-C-C=C-CH3
mixture? H3C-C=C-CH3 |
H
A) 26 B) 74 C) 13 D) 83 E) 79
III. H2C=CH-CH2-CH=CH2 IV. H2C=CH-CH=CH2

133
A) I and II B) I and IV C) II and III 63. What is the name of the following hydrocarbon?
D) II and IV E) III and IV C2H5
|
H3C-CH = C –C=CH-CH2-CH3
|
60. What is the name of the following structure? Cl

Cl
I
H3C-C-CH=CH-CH3 A) 4-chloro-5-ethyl-3-heptene
| B) 4-chloro-5-ethyl-3,4-heptadiene
CH3 C) 4-chloro-2-ethyl-3,4-heptadiene
D) 2-ethyl- 4-chloro-3,4-heptadiene
E) 4-chloro-2-ethyl-2,4-heptadiene
A) 4-chloro-4-methyl-3-pentene
B) 4-methyl-4-chloro-2-pentane
C) 4-methyl-4-chloro-2-pentene 64. Which one is the general formula of alkynes?
D) 2-methyl-2-chloro-3-pentene
E) 4-chloro-4-methyl-2-pentene A) CnH2n-2 B) CnH2n C) CnH2n+2
D) CnH2n-4 E) CnHn

61. What is the name of the following structure?


65. Which one of the numbers given cannot be the total
H3C-CH- CH2 -CH3
number of atoms in an alkyne molecule?
| |
CH3 CH3
A) 4 B) 7 C) 13 D) 19 E) 27

A) 2,3-methyl-butane
B) 2,3-trimethyl-butane 66. Which one of the following names is correct for the
C) 2,3-dimethyl-butene structure given below? CH3-CH2-C=CH
D) 2,3-dimethyl-butane I
E) 2-methyl-pentane CH3

A) 2-methyl-1-butane
62. Which one of the following structures named as B) 2-methyl-1-butene
trans-2-butene? C) 2-methyl-2-butene
H H H D) 3-methyl-3-butene
| | |
A) H3C-C = C -CH3 B) H3C-C = C -CH3 E) 3-methyl-1-butyne
|
H
H
|
C) H3C-C - C -CH3 H H 67. Which one(s) of the following hydrocarbons is
| | | (are) named correctly?
H D) H3C-C - C -CH3 I. Cl
I II. C2H2
H
CH3-C=C-CH3 Acetylene
|
E) H3C-C = C -CH2-CH3 I
| Cl
H Trans-2, 3-dichloro-2-butene

III. NH2 134


| Toluene
A) NO2 B) CH3 C) NH2
| | |
NO2

D) NH2
E) CH3
|
NO2 NO2 |
NO2 NO2

|
NO2 |
A) I only B) I and III C) I and II NO2
D) II and III E) III only

68. If an alkene molecule have 15 atoms totally, what


would be the its molecular formula? 73. Which one is in gaseous state at room temperature?

A) C4H11 B) C6H9 C) C7H8 D) C5H10 E) C10H5 A) CH4 B) C6H14 C) H2O D) CaCl2 E) NaCl

69. What would be the IUPAC name of the following


cyclohydrocarbon?
74. Two most important natural sources of
Br
alkanes are .......... and ................
A) petroleum / forests.
B) natural gas / coal.
A) Cyclobutane bromide C) petroleum / natural gas.
B) Bromide cyclobutane D) gasoline / coal.
C) 1-bromo cyclobutane E) marshes / swamps.
D) Bromo cyclobutane
E) Bromyl butane 75. Which one of the following hydrocarbons reacts
with chlorine to undergo “substitution reaction”?

70. Which one of the following substances is not an A) C2H6 B) C2H4 C) C2H6
alkyne? D) C3H4 E) C2H2

A) C2H2 B) C3H4 C) C4H8 D) C5H8 E) C7H12 76. Which one of the following compounds adds
hydrogen gas in the presence of Ni catalyst?

71. Which one(s) of the following properties do(es) not A) C2H6 B) CH4 C) C2H4I2
belong to alkynes? D) C2H4 E) C4H5Br
I. They have triple bonds
II. They are named as acetylenes
III. The simplest alkyne is C3H4 77. The complete combustion of a 50 liter mixture of
CH4 and C2H4 requires 130 liters of oxygen gas. What
A) I only B) III only C) II and III is the volume of CH4 in the mixture?
D) I and II E) I, II and III
A) 10 B) 20 C) 25
D) 30 E) 15
72. Which one is the structural formula of TNT?
135
78. When a 0.1 mol sample of an alkane is burned, 7.2 84. Which one of the following produces 2- butanol
g of water vapor is produced. What is the formula of the when water adds?
alkane? (C:12, H:1, O:16)
A) butane B) l-butene C) 2-propene
A) C3H8 B) C2H6 C) CH4 D) butadiene E) l-pentene
D) C4H10 E) C5H12
85. Which one of the following is the formula of p-nitro
toluene?
79. How many grams of precipitate is formed when
A) B) C) D) E)
carbon dioxide gas, which is produced by the CH3 CH 3
NO2
combustion of methane gas that is obtained from 8.2 g NO2 CH3
CH3
of sodium acetate, is passed into a solution of
NO2
limewater? (C:12, H:1, O:16, Na:23, Ca:40)

A) 2.5 B) 5 C) 7.5 NO2 NO2 NO2 NO2

D) 10 E) 12.5
86. When a 3 liter mixture containing equal moles of
80. Which one of the following is not isomer of hexane? ethane, ethylene, and acetylene is burned, how many
liters of carbon dioxide are produced?
A) 2,2-dimethylbutane
B) 2,3-dimethylbutane A) 2 B) 3 C) 4
C) 3-methylpentane D) 6 E) 9
D) 2-methylpentane
E) 1,2-dimethylpropane 87. How many liters o oxygen gas at STP are required
to burn a sample o ethylene gas which is obtained from
81. If a 2.8 g sample of an alkane adds 0.1 g of 4.6 g of ethyl alcohol? (C:12, H:1, O:16)
hydrogen, what is the formula of the alkane? (C:12,
H=1) A) 67.2 B) 44.8 C) 22.4
D) 6.72 E) 4.48
A) C2H4 B) C3H2 C) C3H6
D) C4H8 E) C5H10 88. When a sample of gaseous saturated hydrocarbon is
burned, the sum of the volumes of the products (CO2
82. When a 15 liter mixture of ethylene and acetylene is and H2O) is measured to be 7 times greater than the
saturated in the presence of catalyst, 21 liters of volume of the hydrocarbon at the same conditions.
hydrogen gas are consumed at the same conditions. Which one of the following is the molecular formula of
What is the volume of ethylene in the mixture? the hydrocarbon?

A) 5 B) 6 C) 7 A) C4H10 B) C3H8 C) C2H6


D) 8 E) 9 D) CH4 E) C5H12

83. When a 0.2 mol of an unsaturated hydrocarbon is 89. When a 5 liter sample of hydrocarbon is burned
burned, 0.6 mol of carbon dioxide gas produced. In completely, 35 liters of oxygen are consumed and 25
another experiment, it is measured that a 0.2 mol of this liters of carbon dioxide are produced. Which one of the
hydrocarbon reacts completely with 0.4 mol of bromine. following is the molecular formula of the
What is the formula of this compound? hydrocarbon?
A) C3H8 B) C3H6 C) C3H4 A) C5H12 B) C5H10 C) C5H8
D) C6H12 E) C6H6 D) C4H8 E) C4H10

136
90. Which one of the following properties is the same
as the two isomers of a compound?

A) chemical properties
B) physical properties
C) structural formulas
D) molecular weights
E) structure of molecules

91. What is the number of possible isomers for the


compound C3H6Br2?
A) l B) 2 C) 3
D) 4 E) 5

92. If 1.5 mol of O2 is required to burn 0.25 mol of a


compound having a formula of CnH2n, what is the
value of “n”?

A) 2 B) 3 C) 4
D) 5 E) 6

93. When 2 mol of a four carbon hydrocarbon is


burned, 11 mol of oxygen gas is used up. What is the
formula of hydrocarbon?

A) C4H8 B) C4H6 C) C4H10


D) C4H4 E) C4H9

94. In the production of hexane by the Wurtz reaction,


butane and octane are produced as by-products. Which
alkyl halides do you expect to be used?

A) C2H5-X B) C3H7-X C) CH2-X


C4H9-X C3H7-X C5H11-X

D) C2H5-X E) C4H9-X
C2H5-X C8H17-X

137
CHEMICAL BONDS

1. Which one of the following is not an 6. Which of the following compounds would you
intermolecular bond? expect to have the higher melting point?

A) Hydrogen bond A) MgCl2 B) NaF C) BaF2


B) Van der Waals forces D) AlCl3 E) LiF
C) Metallic bond
D) Network structure
E) Polar covalent bond 7. Given the elements 5A and 9B. What would be
the formula and the type of hybridization of the
molecule formed between A and B?
2. Which one of the following electron
configurations is written wrongly? A) AB2 sp
B) AB sp2
A) 11Na: 1s2 2s2 2p6 3s1 C) A2B sp3
B) 3Li+1: 1s2 2s1 D) AB3 sp2
C) 13Al+3: 1s2 2s2 2p6 3s0 3p0 E) AB sp
D) 6C: 1s2 2s2 2p2
E) 2He: 1s2
8. Which of the following bond is more polar than
others?
3. In which of the following are the valance
electrons of atoms written correctly? A) Mg-Cl B) K-Cl C) Li-Cl
D) Ba-Cl E) Na-Br
Atoms No. of valance electrons
A) 11Na 2
B) 20Ca 2 9. Given the elements;
C) 3Li 3 19K, 20Ca, 11Na, 12Mg, 13Al
D) 13Al 4 Which element do you expect to have the higher
E) 16S 7 melting point and hardness in solid state?

A) K B) Ca C) Na D) Mg E) Al
4. Which of the following atom has only one lone
pair electron in its structure?
10. Which of the following atoms gains electrons to
A) 6C B) 8O C) 10Ne complete its outer shell consists of eight electrons?
D) 4Be+2 E) 13Al
A) 20Ca B) 19K C) 9F
D) 10Ne E) 56Ba
5. The information given below is related to the
intramolecular bonds.
I. The ionic bonds are formed between the nonmetals. 11. Which one of the following molecules has an
II. The covalent bonds are formed by sharing of ionic bond?
electrons.
III. In the ionic bonds electron transfer must be done. A) HCl B) BrCl C) PCl3
Which one(s) is (are) true? D) MgF2 E) CF4

A) I only B) II only C) III only


D) II and III E) I, II and III
138
12. Which of the following compounds has both A) 9 and 1 B) 10 and 1 C) 11 and 2
ionic and covalent bonds in its structure? D) 8 and 2 E) 3 and 10

A) CaCO3 B) NaCl C) OF2


D) CO2 E) K2O 18. Following information is given for the metals;
I. They have ability to conduct electricity.
II. They cannot be drawn into wires and sheets.
13. Which of the following compounds has only III. They have empty valance orbitals.
covalent bond? Which one(s) is (are) correct?

A) PCl3 B) Fe(OH)2 C) CaC2 A) I only B) II and III C) I and III


D) MgI2 E) BaF2 D) III only E) I, II and III

14. Which one(s) of the following information is 19. In which of the following is the geometry of the
(are) correct for ionic compounds? compounds given correctly?
I. They do not conduct electricity at solid state.
II. They have low melting points. Compound Geometry
III. They are formed between nonmetals. A) NaF Angular
B) CaCl2 Linear
A) I only B) II only C) III only C) H2O Linear
D) I and II E) I, II and III D) CF4 Triangular
E) CH4 Pyramidal

15. The electro negativities of elements in the


periodic table, 20. Which one of the following bond is the most
I. increase from left to right across a period. ionic?
II. decrease from top to bottom within a group.
III. do not change from left to right across a period. A) Li-F B) Na-F C) Cs-F
Which one(s) is (are) correct? D) Fr-F E) K-F

A) I only B) II only C) I and II


D) III only E) II and III 21. A compound of the element Y with chlorine
has the formula of YCl3. The probable formula of the
oxide is.....
16. Which of the following has a non-polar
structure as a whole molecule? A) YO3 B) Y2O3 C) Y3O2
D) Y3O E) Y2O
A) H2O B) NH3 C) NCl3
D) PCl3 E) CO2
22. Ionic bonding occurs when

17. Which one of the following is correct for the A) the combining atoms both lose electrons to have
number of  and  bonds in, propionic acid? noble
H H O gas electron configuration.
| | || B) the combining atoms both gain electrons to have
H-C-C-C-O-H
| |
noble gas electronic configuration.
H H C) the combining atoms share electrons to have noble
gas electron configuration.

139
D) one of the combining atoms loses an electron and
the other gains an electron, both atoms have noble gas
electron configuration. 28. The noble gases are unreactive because they
E) two non-metals react with each other. have

A) the same number of electrons.


23. Which one of the following elements forms a B) no allotropes.
chloride compound that is ionic in the pure state? C) low boiling points.
D) no isotopes.
A) hydrogen B) carbon C) sulfur E) stable electron configurations.
D) neon E) potassium

29. The atom X forms a molecule with 5 atoms


24. A compound that is ionic probably with nitrogen. The formula of its oxide would be

A) has a high melting point A) XO B) XO2 C) X2O


B) has a high boiling point D) X2O3 E) X3O2
C) is an electrolyte in aqueous solution
D) dissolves in water
E) all could be true 30. The element X forms XCl6 with chlorine. Its
electron configuration could be ending with . . . . . . .

25. Which one of the following particles has only A) s2 B) p2 C) p4 D) p6 E) d2


covalent bonds?

A) argon 31. CH4 has a . . . . . . . . . . . shape, and the


B) solid potassium fluoride geometrical shape of CH3+ is . . . . . . . . . . . . . .
C) dilute sodium hydroxide
D) gaseous propane A) tetrahedral/pyramidal
E) potassium bromide solution B) tetrahedral/planar triangular
C) tetrahedral/tetrahedral
D) pyramidal/tetrahedral
26. Ionic bond is stronger when E) pyramidal/planar triangular
A) there are sp3 hybrid orbitals.
B) bonding metal is an alkali metal.
C) the electronegativity difference of the bounding 32. Na atom becomes Na+ ion when
elements is high.
D) a bonding nonmetal is a halogen. A) gains an electron
E) two metals or two nonmetals combine with each B) loses an electron
other C) gains a proton
D) loses a proton
E) gains a neutron
27. Which of the following pairs of elements is
least likely to form a covalent compound?
33. . . . . . . .. .. have a higher tendency to lose their
A) fluorine and oxygen valence electrons, on the other hand , . . . . . . .. . . .
B) hydrogen and fluorine have a higher tendency to gain electrons than other
C) carbon and oxygen groups.
D) sodium and fluorine
E) carbon and fluorine A) noble gases/halogens
140
B) alkali metals/noble gases
C) earth alkali metals/halogens A) sp, sp3 B) sp2, sp3 C) sp, sp2
D) d-block elements/p- block elements D) sp, sp , sp E) sp2 only
2 3

E) alkali metals/halogens

39. Which ones of the following molecules are


34. Carbon has only two half-filled orbitals, but it polar?
generally forms four chemical bonds. This can be I. BCl3 II. PH3 III. PCl5
explained best with IV. CS2 V. AlF3

A) expanded octet. A) II, III B) III, IV, II C) I, IV, V


B) coordinate covalent bond. D) I, V E) none of them
C) excited electrons.
D) electron promotion.
E) hybridization. 40. When 33X combines with other elements, which
geometrical shapes could be possible for the
molecules?
35. , (pi) bonds are formed by which one of the I. tetrahedral II. pyramidal III. angular
following orbitals? IV. bipyramidal V. octahedral

A) unhybridized p orbitals A) I, III B) II, IV C) I, IV


B) sp hybrid orbitals D) III, IV E) II, III
C) unhybridized s orbitals
D) p2 hybrid orbitals
E) sp3 hybrid orbitals 41. 16X and 20Y form a(n) . . . . . . . . . . . . . .
molecule with a(n) . . . . . . . . . . . . . . . . shape.

36. Which one is the geometrical shape of A) ionic / triangular


hydronium ion , H3O+ ? B) covalent / angular
C) ionic / linear
A) planar triangular D) covalent / pyramidal
B) pyramidal E) ionic / angular
C) tetrahedral
D) planar rectangular
E) angular 42. Ionic bonding is between . . . . . . . . . . . . . . . . .
., and covalent bonding is between . . . . . . .. . . .. . .

37. Which one(s) of the following types of bonds A) one metal and one nonmetal/two nonmetals
present in ammonium chloride, NH4Cl? B) two metals/two nonmetals
I. ionic bond C) two metals/one metal and one nonmetal
II. coordinate covalent bond D) two nonmetals/two metals
III. polar covalent E) one metal and one nonmetal/two metals or two
IV. nonpolar covalent bond nonmetals

A) I, II, III B) I, IV C) II, III , IV


D) I, III E) II, III 43. When 12A and 38B react, the shape and the
formula of the molecule will be............

38. What could be hybrid type(s) of carbons in A) AB, linear B) A2B, angular C) AB3,
C3H4? pyramidal
141
D) A2B, linear E) they never react
A) CO2 B) HF C) SO2
D) SiO2 E) OF2
44. How many ionic bonds are there in a molecule
of K2CO3 ?
51. Which pair of molecules has similar
A) 8 B) 6 C) 4 D) 2 E) 0 geometrical shapes?

A) CO2, SiO2 B) H2O, Li2O C) PH3, BH3


45. When 13X and 15Y react, one molecule of the D) BeCl2, P4 E) SF4, CCl4
compound will have . . . . . . . . . . . . .electrons.

A) 28 B) 41 C) 54 D) 69 E) 71 52. Which of the following elements has the


highest bonding capacity? (Ability to form more
chemical bonds)
46. Which one of the following molecules has more
covalent bonds than the others? A) Chlorine B) Silicone C) Phosphorus
D) Sulfur E) Aluminum
A) H2SO4 B) HNO3 C) Al(CN)3
D) K2CO3 E) FeSO4
53. Which of the substances given below has the
highest boiling point?
47. A compound that is covalent probably.........
A) H2O B) NH3 C) CH4
A) has a high melting point D) HF E) H2S
B) has a high boiling point
C) is an electrolyte when molten
D) dissolves in water 54. Which one of the following substances given
E) is formed from two or more non-metals has the lowest melting point?

A) MgBr2 B) MgF2 C) BeF2


48. The hybrid type of X in XY3 is sp2. So......... D) BeCl2 E) MgI2
I. X must be group 5-A
II. XY3 molecule must be nonpolar
III. Shape of XY3 is pyramidal 55. What is the hybrid type of carbon atom in
Which one(s) of the above statements is (are) correct? carbon anhydride?

A) I only B) II only C) I and III A) sp B) sp2 C) sp3 D) s E) p3


D) I and II E) I and III

56. What is the hybrid type of P in P4O10?


49. Element X forms a four atom consisting
covalent molecule with fluorine .The atomic number of A) sp2 B) sp3 C) sp3d D) sp3d2 E)
X could be......... sp

A) 31 B) 32 C) 33 D) 34 E) 35
57. In which of the following molecules there are
more double bonds?
50. Which one of the given molecules has
resonance structures? A) P2O5 B) SF6 C) H2SO4
142
D) C2H4 E) H3PO4 63. The boiling temperature of a pure substance is
mainly proportional to

58. A substance that melts at 8000C and is an A) polarity of its molecules


electrolyte which is soluble in water is B) the size of the molecules
C) the strength of the bonds inside of its molecules
A) a simple covalent solid. D) the forces between its molecules
B) an ionic solid. E) they are all related
C) a network covalent solid.
D) a metal.
E) a non-metal. 64. Which ones of the following statements are
correct for network solids?
I. having high boiling points
59. Which of the following types of bonds accounts II. very hard substances
for the strange properties of water? III. having ability to dissolve in water
IV. good conductors of heat
A) Van der Waals
B) Ionic A) I and III B) II and IV C) II and III
C) Polar covalent D) I and II E) I and IV
D) Hydrogen
E) Non-polar covalent
65. Calcium, 20Ca, has a higher melting point than
potassium, 19K, and strontium, 38Sr. This is because of
60. A crystal of sodium chloride consists of a I. atomic size
lattice of II. ionization energy
III. number of valence electrons
A) sodium chloride molecules.
B) sodium atoms and chlorine molecules. A) I only B) II only C) III only
C) positive sodium ions and negative chloride ions. D) I and II E) II and III
D) negative sodium ions and positive chloride ions.
E) sodium atoms and chlorine atoms.
66. What type of intermolecular forces exists
between HF molecules?
61. Which one of the given substances forms I. Hydrogen bonds
covalent network solids? II. Dipole - dipole forces
I. CO2 II. SO2 III. SiO2 IV. C V. SeCl2 III. Van der Waals forces
IV. Covalent bonds
A) I and II B) III and IV C) V and I
D) II and III E) III and V A) I only B) I and II C) I, II and III
D) I, II and IV E) they all exist

62. In which ones of the following substances there


are hydrogen bonds between molecules? 67. Which one of the following substances would
I. H2O II. CH3OCH3 III. NO2 IV. have the highest boiling point among others?
CH3NH2
A) Chlorine
A) I and II B) III and IV C) II and IV B) Carbon dioxide
D) II and III E) I and IV C) Nitrogen
D) Oxygen
E) Fluorine
143
73. Which one of the following statements is the
best one for network solids?
68. Chlorine compounds do not form strong
hydrogen bonds. This is mainly because of A) They have high melting points.
B) They are nonconductors of electricity.
A) low electronegativity of chlorine. C) They are hard substances.
B) relatively higher atomic weight of chlorine. D) They are transparent.
C) vacant d orbitals in chlorine atoms. E) All above is true.
D) high ionization energy of chlorine atoms.
E) relatively bigger size of chlorine atoms.
74. At room conditions, chlorine is a gas, whereas
bromine is a liquid. This difference between these two
69. Although CO2 is a gas at normal conditions, halogens can be explained best by the difference in
SiO2 is a solid with a very high melting point
temperature, because SiO2 A) ionization energies.
B) electro negativities.
A) is heavier than CO2 . C) valence electrons.
B) forms hydrogen bonds. D) atomic weights.
C) has metallic bonds. E) hydrogen bonding.
D) is a network solid.
E) has a higher molecular size.
75. Which one(s) of the factors have an affect on
the strength of metallic bonds?
70. The ability to conduct electricity and heat I. number of the vacant orbitals
can be best explained by......... II. ionization energy
III. period number of the metal
A) van der Waals forces.
B) freely moving electrons. A) I and II B) I and III C) II and III
C) covalent network solids. D) I only E) I, II and III
D) size of the molecules.
E) low ionization energies.
76. One water molecule is bonded to . . . . . . . . .
71. The main difference between diamond neighboring water molecules making a . . . . shape.
and graphite is that graphite....... ..
A) 2, triangular
A) is harder than diamond. B) 3, pyramidal
B) is denser than diamond. C) 4, tetrahedral
C) conducts electricity, but diamond does not. D) 4, pyramidal
D) has a higher percentage of carbon. E) 3, planar
E) has a higher melting point.

77. Which one(s) of the following statements is


72. Which one of the following elements forms the (are) correct for the intermolecular bonds?
strongest hydrogen bonds? I. Metallic bond is the electrostatic attraction between
the freely moving electrons and the atomic nuclei.
A) Fluorine B) Oxygen C) Nitrogen II. The attractions of the positive and negative poles of
D) Sulfur E) Chlorine the molecules are called dipole-dipole forces.
III. The strength of the van der Waals forces increases
with the number of atoms.
IV. Hydrogen bond decreases the boiling point.
144
83. Consider the electron configurations of X
A) I, II and IV B) I, II and III C) I and IV and Y:
2 2 6 2
D) II, III and IV E) II and III 12X: 1s 2s 2p 3s
2 2 6 2 5
17Y: 1s 2s 2p 3s 3p
Which one of the following statements is correct for
78. Which one(s) of the following statements is the formation of a compound between X and Y?
(are) wrong?
I. Metals form covalent bonds with metals. A) X gains electrons
II. Metals form ionic bonds with nonmetals. B) Y gives electrons
III. Nonmetals form covalent bonds with nonmetals. C) Between X and Y there are sharing of electrons
IV. Metals form ionic bonds with metals. D) X gives electrons
E) Formula of the compound formed between X and Y
A) I only B) II only C) I and II is X2Y
D) I and III E) I and IV

84. In which of the following compounds are there


79. Which of the following molecules have sp2 2 and 2 bonds?
hybrid orbitals?
A) S2Cl2 B) C2H6 C) C2H2
A) C2H2 B) C2H6 C) CH4 D) CO2 E) NH3
D) HF E) C2H4

85. Which of the following compound is polar?


80. I. O2 and CO2
II. CO2 and C2H2 A)
III. C2H4 and C3H6 NH3
Which of the following pairs of the molecules have B)
the same number of pi () bonds? C3H6 C) C2H4
D) CH4 E) C2H2
A) I only B) II only C) III only
D) I and II E) II and III
86. Which of the following molecules have the
greatest number of  bonds?

81. Which of the following molecules do not A) N2 B)


have pi () bond? (6C, 7N, 8O, 9F) CS2 D)
C4H6
A) O2 B) N2 C) F2 D) CO2 E) C2H2 D)
C5H8 E) C6H6

82. What is the type of the bond formed


between 17Y and 11X in which electron 87. What is the name of the geometry of the
configuration is 1s2 2s2 2p6 3s1? compound formed between 17Cl and 16S atoms?

A) Ionic B) Metallic C) Covalent A) triangular B) linear C) tetrahedral

D) Hydrogen E) Van Der Waals D) pyramidal E) angular

145
4. In which one of the following molecules is the
nitrogen to nitrogen bond distance expected to be the
88. Which of the following elements compounds shortest?
with their atomic numbers have a pyramidal geometry
when forming a compound with fluorine? A) N2O B) N2O4 C) N2H4
D) N2 E) N2O3
A) 15P B) 14Si C) 12Mg D) 12Al
E) 11Na
5. Which one of the following molecules has an
ionic bond?
89. Which of the following substances does not
conduct electricity? A) NCl B) BrCl C)
PCl3
A) liquid silver B) Solid silver C) Aqueous NaCl D) MgF2 E) CF4
D) liquid NaCl E) Solid NaCl
6. Which one of the following molecules has a
shape of planar triangle?
90. Which of the following electron dot
representations for the molecules of NH3, O2 and N2 is A) BF3 B) PCl3 C)
(are) correct? SO3
I. H:N: H II. O::O III. N:::N D) SF2 E) SiH4
H
7. Given the elements 5A, 8B, 12C, 14D, 17E.
A) I only B) I and II C) I and III Which one of them has the highest bonding capacity?
D) II and III E) I, II and III
A) A B) B C) C
ORAN Chemistry-3 D) D E) E

1. Which one of the following species contains a 8. A molecule in which sp2 hybrid orbital are
triple covalent bond? used for bond formation by the central atom is
A) NO3- B) HCN C) CO2 A) CO B) SO2 C) SCl2
D) C2H4 E) PCl3
D) H2CO E) NH4+
2. Which one of the following molecules is not 9. Which one of the following violates the octet
linear? rule?
A) HCN B) F2 C) CO2 A) C2H2 B) O2 C) NO
D) SO2 E) C2H2 D) H2O E) OF2

3. Of the following molecules, all are polar except


one. That one is….. 10. Which one of the following bonds is the most
polar?
A) BCl3 B) CH3C1 C) NO
D) PCl3 E) OF2 A) P-Cl B) P-I C) P-Br
D) P-F E) F-F

146
11. Which one of the following molecules has a A) AB4 tetrahedral
tetrahedral shape? B) A3B planar triangular
C) AB3 pyramidal
A) BeCl2 B) BF3 C)
D) AB2 angular
SiH4
E) AB2 linear
D) PH3 E) H2S

17. Given the elements 14X and 17Y. Which one


12. Which one of the following contains a pi bond?
of the following shows the correct Lewis structure
A) C3H4Br2 B) C3H8 C) C3H7Br (electron dot formula) for the molecule formed
between X and Y?
D) C3H6Br2 E) C3H7OH

:
: Y : X : Y :
13. Given the equations
2Xg   X 2 g   103.4 kcal/mol

:
2Yg   Y2 g   37.7 kcal/mol A)
: Y :
2Zg   Z2 g   226.0 kcal/mol

:
A) Z2>Y2>X2 B) Y2>X2>Z2 : Y :

:
C) Z2>X2>Y2 D) Y2>Z2>X2
: Y : X : Y :
E) X2>Z2>Y2
:

:
14. Which one of the following bonds is the most : Y :
ionic? B) :

A) K-Cl B) Na-Cl C) K-Br


:

:
D) Li-Br E) Na-I
: Y : X : Y :
15. Given the bond energies
:

:
Bond Bond energy (kcal/mol) : Y :
H-H 104
:

C-C 83 C)
C=C 146
:

C-H 99
: X : Y : X :
What is the enthalpy change for the following reaction
:

in kcal/mol? : X :
C2 H 4 g   H 2 g   C2 H6 g 
:

D)
A) - 177 B) -135 C) : X :: Y : : X :
+52
:
:

D) +31 E) -31 E)

16. Given the elements 15A and 9 B. What would 18. The following information is given for the
be the formula and the shape of the molecule formed properties of the molecule formed between X and
between A and the shape of the molecule formed 17Cl.
between A and B?

147
I. The chemical formula of the molecules is C) NaCl>Na=Li>Br2>H2
XCl2. D) NaCl>Li>Na>Br2>H2
II. The molecules as a whole is non polar. E) NaCl>Br2>H2>Na>Li

According to this information, what must be the atomic


4. Which one of the following liquids has the
number of element X?
highest melting point?
A) 5 B) 8 C) 12
A) HF B) HCl C) H2S
D) 14 E) 15
C) CCl4 E) HBr
19. Given the electronegatives for the elements.
H=2.2, F=4.0, Cl=3.2, Br=3.0, Na=0.9, K=0.8, Ca=1.0 5. One of the following substances is a liquid at
Which one of the following bonds is the most ionic? room temperature, whereas all the others are gaseous.
Which one is the liquid?
A) F-H B) Na-F C) K-Cl D)
Na-Br E) Ca-F A) C3H8 B) CH3OH C) O2
D) N2 E) CO
20. Which one of the following is correct for the
number of and bonds in C2H2Cl2 ?
6. Chlorine has an atomic number of 17. Which
one of the following elements forms an ionic
A) 3  ,2  B) 4  ,1  C) 2  ,3  compound with chlorine?
D) 5  ,1  E) 3  ,3 
A) 6A B) 8B C) 10C
D) 11D E) 16E
SELF TEST (sayfa 43)

1. Which one of the following substances has 7. Which one of the following is wrong for the
dipole-dipole forces between its molecules in its solid element 17X?
form?
A) It forms an ionic compound with 11Na.
A) Xe B) LiF C) HCl B) The molecules between 1H and 17X have only
D) F2 E) CH4
dipole-dipole forces.
C) The X2 molecules have only van der Waals
2. Which one of the following substances has only
forces in the solid state.
van der Waals forces between its molecules in the solid
D) The intramolecular bonds in X2 molecules are
state?
covalent.
A) ClBr B) CCl4 C) E) The X2 molecules are nonpolar.
SiO2 D) NaCl E) HF
8. Given the compounds:
I- HF
3. Given the solid state of substances: NaCl, Br 2,
II- H2S
Na, H2, Li. Which one of the following would you
III- NH3
expect to be the correct order of increasing melting
IV- CF4
point?

A) Br2>Na>NaCl>Li>H2 Which substance or substances have hydrogen bonds


between their molecules in the liquid state?
B) Li>Na>NaCl>Br2>H2

148
A) Only I B) I and II C) II and
IV 14. Which elements have the highest melting point
D) III and IV E) I and III in the solid state?

9. Which one of the following elements forms a A) A B) B C) C


network solid involving covalently bonded atoms? D) D E) E

A) 11A B) 13B C)
CH3OH 15. Which element forms the strongest metallic
bond in its solid form?
D) CCl4 E) CO2
A) A B) B C) C
10. Which one of the following substances doesn’t D) D E) E
conduct electricity in its solid forms but a good
conductor when dissolved in water? 16. Which two elements form molecules which are
planar triangular in shape in the gaseous state?
A) BaCl2 B) PH3 C)
CH3OH A) A and C B) B and E C) C and E
D) CCl4 E) CO2 D) D and E E) E and F

17. What type of attractive forces exists between


11. Which one of the following molecular
the molecules of H2S in its liquid state?
substances has the highest solubility in water?

A) PH3 B) HBr C) A) Hydrogen bond; dipole - dipole forces


B) Van der Waals forces and hydrogen bond
CO2
C) Ionic bond and van der Waals forces
D) NH3 E) N2 D) Dipole - dipole forces and van der Waals forces
E) Covalent bonds and hydrogen bond
The following data were given for questions 12, 13,
14, 15 and 16. 18. Which one of the following substances has the
highest boiling point in the liquid state?
A 1s22s22p2
A) CH4 B) CF4 C)
B 1s22s22p3
Br2
C 1s22s22p63s1
D) CBr4 E) Cl2
D 1s22s22p63s23p1
E 1s22s22p63s23p5
19. Which one of the following compounds doesn’t
F 1s22s22p63s23p6 have hydrogen bonds between their molecules in the
liquid state?
12. Which two elements form the most ionic
compound? A) CH3OH B) HF C) NH3
D) PH3 E) H2O
A) B and C B) C and E C) D and E
D) C and D E) D and F
20. What types of intermolecular and
13. Which element doesn’t form a compound? intramolecular forces exist in H2O molecules?

A) A B) B C) D Intramolecular Intermolecular
D) E E) F forces forces
149
A) Ionic bond Van der Waals;
dipole - dipole
forces
B) Covalent bond Hydrogen bond;
dipole - dipole
forces;
van der Waals
forces
C) Ionic forces Hydrogen bond; ionic
bond
D) Covalent bond Van der Waals forces;
ionic bonds and
covalent bonds
E) Covalent bond Van der Waals forces and
covalent bonds

150
Alcohols & Ethers C)
H3C
1.Which of the following represents H2
alcohols? H3C C C OH
A) H-OH B) R-OH
C) R-O-R` D) R-COH
OH
D)
2.Compounds in which the hydroxyl group is bonded to an
alkyl group are called……………
H H2
H3C C C OH
A) aldehydes. B) carboxylic
acids. OH
C) alcohols. D) ketones.
7.In which of the following is (are) the names of structures
3.What is the class of the following alcohol? given correctly?
H3C C OH I.
H2 H2 H2
C C OH
A) Primary B) Secondary
C) Tertiary D) None of them C C
H2 H2
4-phenyl-1-butanol
4.Which of the following alcohols is a tertiary alcohol? II.
A) B)
Cl
C2H5 H3C CH OH H2
C CH OH
H3C C OH Cl CH C
H H2C H2
C) D) Cl
C2H5 2,3,4-trichloro-1-propanol
III.
OH
H3C C OH
H2C C OH
H2 HO CH OH
Br H2C C C
H2 H2
Glycerol
5.What is the IUPAC name of following structure?
H2C C OH A) I only B) II and III
H2 C) I and III D) I and II
Br
A) 2-bromo-ethyl alcohol 8.Which of the following is the simplest alcohol named as
B) 2-bromo-1-ethanol “wood alcohol” which is toxic and can cause permanent
C) 1-bromo-2-ethanol blindness or death if taken internally?
D) Bromo ethyl alcohol A) Isopropyl alcohol B) Methanol
C) Ethyl alcohol D) Isobutyl alcohol
6.Which of the following structures is 1,3-
propanediol? 9.……….. is best known alcohol, used in the chemical
A) industry as a solvent and as an ingredient of fermented
H2 H2 alcoholic beverages, which is produced by the fermentation
HO C C OH of aqueous solutions of simple carbohydrates such as sucrose
C
H2 or glucose.
A) CH3OH B) C6H5OH
B)
C) CH3CH2OH D) CH3CH2CH2OH
H3C
H2 H2 10. Which of the following group(s) of organic substances is
HO C C C OH (are) obtained when primary and secondary alcohols are
H
151
oxidized by potassium dichromate or potassium
permanganate and sulfuric acid? 16. Which one of the following forms a ketone when
A) Tertiary alcohols
B) Ketones oxidized?
C) Aldehydes
D) Carboxylic acids and ketones A) n-butanol B) butanal
C) 2-methyl-2-butanol D) 2-methyl-l-
11. Which of the following is the general representation of butanol
ethers? E) 3-methyl-2-butanol
A) R-O-R` B) R-OH
C) R-COR` D) R-COOH
17. 19 g of a dihydric alcohol produces 5.6 1 of H2
12. Which of the following structure is named as isopropyl gas at STP when reacted with sodium. What is the
methyl ether? formula of the alcohol? (C:12, H:l, O:16, Na:23)
A) B)
CH3
A) C2H4(OH)2 B)
H2 C3H6(OH)2
H3C O C CH3 H3C O CH CH3
C) D) C) C4Hg(OH)2 D)
CH3 C3H7(OH)2
H3C
E) C4H9(OH)2
H2C O CH CH3 H3C O CH3
18. When a monohydric alcohol and an organic acid,
which have equal molecular weights, react together,
13. The following data are given for an alcohol;
an ester with a molecular weight of 102 g is
i) a ketone is formed when it is oxidized in the
obtained. What is the formula of the alcohol? (C:12,
presence of a suitable catalyst.
H:l, O:16)
ii) 0.5 mol of hydrogen is liberated when 1 mol
of the alcohol is reacted with sodium.
A) CH3OH B) C2H5OH
Which one of the following is expected to be
the alcohol? C) C3H7OH D) C4H9OH
E) C5H11OH
A) 1,2-butanediol B) l-butanol
C) 2-butanol D) 1,3-butanediol
E) 2,3-butanediol
19. Which one of the following compounds liberates
14. Which one of the following is the isomer of hydrogen gas with sodium but not zinc?
diethyl ether?
A) CH3 - O - CH3 B) CH3 CHO
A) ethanol B) etanediol
C) CH3CH2CH2OH D) CH3COCH2
C) diethyl ketone D) butanol
E) butanediol E) CH3COOH

15. What is the name of the compound formed from 20. The ratio of molecular weights of one step and
the reaction CH3C2ONa+CH3Br two step oxidation products of a primary alcohol is
11/15. Which one of the following is the alcohol
A) ethyl acetate oxidized?
B) methyl ethyl ether (C:12, H:l, O:16)
C) diethyl ether
D) acetone (dimethyl ketone) A) C5H11OH B) C4H9OH
E) propane C) C3H7OH D) C6H5OH

152
E) CH3OH
A) 80 B) 60 C) 40
21. Which one of the following alcohols forms 37 g D) 20 E) 10
of ester when reacted with 0.5 mol ofacetic acid?
(C:12, H:l, O:16) 27. How many isomers are possible for the 1 alcohol
having a molecular formula of C4H10O?
A) CH3OH B) C2H5OH
C) C3H7OH D) C4H9OH A) 3 B) 4 C) 5
D) 6 E) 7
E) C5H11OH
28. What is the name of the compound:
22. How many grams of formic acid, which is 4.6% H3C
pure, can be produced by the oxidation of 8 g of
H H2
methanol? H2C CH C C CH3
(C:12, H:l, O:16)
HO HO
A) 46 B) 50 C) 92 A) 1,3-pentanediol
D) 184 E) 250 B) 1,3-hexanediol
C) seconder hexanol
23. A 68 g mixture of diethyl ether and ethylene D) 2-methyl-l,3-hexanediol
glycol (1,2-ethanediol) liberates 11.2 of H2 at STP E) 2-methyl-l,3-pentanediol
when reacted with sodium. How many moles of
29. How many degrees can the temperature of 1 kg
oxygen are required to burn the ether in, the
of water be increased by the heat obtained from the
mixture? (C:12, H:l, O:16)
combustion of 9.2 g of ethyl alcohol? (∆HCO2 = -94
kcal/mol, ∆HH2O = -57.8 kcal/mol, (∆HC2H5OH = -
A) 12 B) 9 C) 6
66.4 kcal/mol)
D) 5 E) 3
A) 29.5 B) 39.2
24. If the molecular weight of alkoxide formed from
C) 46.0
the reaction of a trihydric alcohol with sodium is
D) 59.0 E) 74.0
172 g, what is the number of carbon atoms in each
molecule of the alcohol? (Na:23)
30. Given the equation;
CH3OH + MnO4- + H+ → HCOOH + Mn+2
A) 3 B) 4 C) 5
+ H2 O
D) 6 E) 7
What would be the coefficient of water if the
25. If a 5 mol mixture of propanol and propanetriol
equation has been balanced?
is reacted with sodium, 4 mol of hydrogen is
produced. What is the mole of propanetriol in the
A) 5 B) 7 C) 11
mixture?
D) 16 E) 22
A) 1 B) 1.5 C) 2
31. The compound 2-chloro-3-methyl-1- butanol
D) 3 E) 4
has the formula of
26. When a 0.5 mol mixture of methanol and ethanol
is completely burnt, 20.16 L of CO2 gas are A) CH2ClC(CH3)2CH2OH
produced at STP. B) CH3CHOHCH(CH3)CH2Cl
What is the mole percentage of methanol in C) CH3CH(CH3)CHClCH2OH
the mixture? D) CH3CHClCH(CH3)CH2OH
153
E) CH3C(CH3)2CHClCH2OH

A) I only B) II only C) III only


D) I and II E) I and III

32. To prepare methyl ethyl ketone which one


should oxidize?

A) 2-propanol B) l-butanol
C) 2-butanol D) t-butyl alcohol
E) 2-methyl-l-propanol

33. Which one is the name of following alcohol


given below?

A) phenyl methanol
B) metyhl methanol C
C) Triphenyl methane-1-ol |
OH
D) Triphenyl methanol
E) Diphenyl methanol

34. What is the name of the following alcohol?


CH2 - CH – CH2
| | |
OH OH OH

A) Trihydroxyl propanol
B) 1, 2, 3-propane-3-ol
C) 1, 2, 3-propanol
D) Propane-1, 2, 3-ol
E) 1, 2, 3-propanetriol

35. Which one(s) of the following naming of


alcohols is (are) wrong?
I. CH2-CH2 II. C2H5 – OH III. CH3 -OH
| | Methyl Methanol
OH OH alcohol
Glycol

A) I and III B) I and II C) II and III


D) II only E) III only

36. Which one(s) of the following classifications of alcohols


is(are) correct?
III. OH
I. CH3-OH II. CH3 – CH- OH |
Primary | CH3 – C-OH
CH3 | 154
tertiary C2H5
secondary
Aldehydes & Ketones H
O Br O C
O
1. Compounds containing the carbonyl group ( C ) are C2H5 CH C H
called………. 2-bromo propanal benzaldehyde
A) alcohols
B) ethers 6. Which of the following is general representation of ketones?
C) aldehydes & ketones A) R-COH B) R-COR`
D) carboxylic acids C) R-COOH D) R-COOR`

2. What is the common name of the simplest aldehyde? 7. Which of the following is 1,1,3-trichloro-2-butanone?
A) benzanal B) acetone A) B)
C) formaldehyde D) acetaldehyde O
Cl Cl O Cl
H2
3. What is the IUPAC name of the following structure? CH C C C Cl H3C CH C CH
H3C H2
O Cl Cl
C) D)
H3C CH CH Cl Cl O
A) 2-methyl propanal Cl Cl O
B) isopropyl propanal HC CH C CH3
C) 2-methyl ethanal Cl CH C CH C OH
D) 2- methyl acetaldehyde H2 Cl

4. Which of the following structures is 2,3-dihydroxy 8. Which of the following is the simplest ketone?
propanal? A) butanone B) acetylene
A) C) isopentanone D) acetone
HO HO O
9. What is the name of the following structure?
H2C CH C O CH3
O
B) H2
HO HO O H3C C C C
H2
H2C CH C OH A) 1-phenyl-2-butanal
B) 4-phenyl-3-butanone
C)
C) 1-phenyl-2-butanone
HO HO O D) 1-phenyl-2-butanoic acid

H2C CH CH 10. Ketones can be further oxidized to carboxylic acids.


D) A) True B) False
HO O

H3C CH CH

5. Which of the following structures is (are) named wrongly?


A) B)
O O

H C H H3C C H
Methanal Acetaldehyde
SELF TEST (sayfa 156)
C) D)
1. The compound X with the formula C3H8O gives
on oxidation the compound with formula C3H6O. X
is most likely to be
155
C 2 H5 CH 3
A) an aldehyde B) a ketone CH 2 OH C=O C=O
C) a tertiary alcohol D) a secondary alcohol
E) an ether C2 H 5 CH 3
A) B) H C)
CH 3 CH 3
2. Which compound yields a ketone upon mild
oxidation? C=O O
C H
A) CH3CH2CH2CH2OH D) H E) 2 5
B) CH3CH2CHOHCH3 7. If a 10 gram mixture of pentane and
formaldehyde is reduced by hydrogen gas, the
C) CH3CH2OCH2CH3
resulting mass of the mixture becomes 10.2. g. What
D) (CH3)2CHCH2OH is the percentage of pentane by mass in the mixture?
E) (CH3)3COH (C:12, O:16, H:l)

3. Which of the following will reduce silver nitrate, A) 15 B) 30 C) 45


dissolved in aqueous ammonia solution, to silver? D) 60 E) 70

A) CH3COCH3 B) CH3CHO 8. 1 mol of an organic compound


I. gives 2 mol of CO2 when burned,
C) CH3CHOHCH3 D) CCl3CH(OH)2
II. gives an acid when oxidized,
E) CH2ClCHOHCH3 III. doesn’t react with metallic sodium to form
hydrogen. What is the organic compound?
4. What would be the main organic product of the
reaction of chlorine gas with boiling CH3CHO? A) C2H5OH B) HCHO C) CH3CHO D)
CH3OH E) CH3COCH3
A) CH3COOH B) CH3COCl
C) CCl3CHO D) CH3CHCl2 9. Which one of the following gives CH3COC2H5
E) CHCl3 (ethyl methyl ketone) when oxidized?

5. A 2 gram mixture of acetaldehyde-acetone A) 2-butanol B) 2-propanol C) l-butanol


precipitates 5.4 g of silver from silver nitrate D) t-butanol E) isopropyl alcohol
dissolved in aqueous ammonia solution. What is the
percentage by mass of acetone in the mixture? 10. A 10.9 g of ethyl bromide is first converted to an
(Ag: 108, C: 12, O: 16, H: 1) alcohol and then the alcohol is converted on oxidation
to aldehyde. What is the mass of aldehyde obtained?
A) 11 B) 22 C) 45 (Br=80; C=12)
D) 60 E) 90
A) 13.7 B) 8.6 C) 5.8
6. 1 mol of an organic compound D) 4.4 E) 3
I. gives 3 mol of CO2 when burned,
II. gives a primary alcohol when reduced. What 11. Which one of the following compounds reduces
is the formula of the compound? cupric ions (Cu2+) in Fehling’s reagent to cuprous
oxide (Cu2O)?
H C=O O
H C H3 C CH3
A) B)

156
H OH
CH 3 C CH 3 CH 3 C CH 2 CH 2 CH 3
C) CH3CH2OCH3 D) OH E) COOH
H H
C=C 15. The aldehyde which can be obtained from 12 g
E) H H of an alcohol forms 28.6 g of Cu2O from the basic
solution of cupric ion (Cu2+) complexed with the
11. Given the equation: anion of tartaric acid (Fehling’s reagent). What is the
R C = O+ 2Ag + +H 2 O R C = O+ 2Ag +2H + formula of the alcohol? (Cu: 63.5, C: 12, O: 16, H: 1)
H OH
According to the equation, 2.2 g of aldehyde reduces A) CH3OH
10.8 g of silver from ammoniacal silver nitrate B) CH3CH2OH
solution. What is the formula of the aldehyde? C) CH3CH2CH2OH
(Ag: 108, C: 12, O: 16)
H D) CH3CH2CH2CH2OH
H
OH
CH3 C = O
A) H C = O B) CH 3 CH 2 CH CH 3
CH 2 OH E)
C=O H
C2H5 C=O 16. 0.05 mol of an aldehyde weighs 4.3 grams. What
C) H D) is the molecular formula of the aldehyde?
H (C: 12, O: 16, H: l)
C3H7 C=O
E) A) HCHO B) CH3CHO
C) CH3CH2CHO D) CH3CH2CH2CHO
13. Which one of the following is wrong for
ketones? E) CH3CH2CH2CH2CHO
A) They are solvents.
B) They give addition reactions. 17. When a 10 g mixture of acetaldehyde and
C) They are the oxidation products of secondary acetone is oxidized, 6 g of acetic acid is formed. What
alcohols. is the percentage by mass of acetone in the mixture?
D) They are reducing agents. (C: 12, H: l, O: 16)
E) They contain the carbonyl group.
A) 40 B) 44 C) 56
14. Which one of the following gives 2-pentanol D) 60 E) 80
(pentane-2-o1) when reduced?
O 18. How many grams of acetone can be prepared by
heating 9.875 g of 80% calcium acetate?
CH 3 CH 2 CH 2 CH 2 C H
A) (Ca: 40, C: 12, H: l, O: 16)
O
A) 2.9 B) 5.8 C) 7.9
CH 3 CH 2 C CH 2 CH 3
B) D) 11.6 E) 15.8
O
CH 3 C CH 2 CH 2 CH 3 19. How many grams of which alcohol is produced
C) when 0.56 1 of H2 at 0°C and 2 atm pressure adds to
O ethyl methyl ketone? (C: 12, H: l, O: 16)
CH 3 C CH 2 CH 2 COOH
D) A) 1.85 g 2-propanol
157
B) 3. 7 g 2-butanol
C) 3.6 g 2-methyl-2-propanol
D) 7.2 g n-butanol
E) 7.4 g 3-pentanol

20. What is the molecular formula of an aldehyde


which has 40% carbon in its molecule?

A) HCHO B) CH3CHO
C) CH3CH2CHO D) CH3CH2CH2CHO
E) CH3CH2CH2CH2CHO

158
Carboxylic Acids (CH3COOH = 60 g/mol)
1. The common functional group in carboxylic acids is
…………
A) carbonyl group.
A) 12 B) 5 C) 3
B) carboxyl group. D) 6 E) 4.6
C) hydroxyl group.
D) phenyl group. 3. 1 rnol of an organic com pound
I. can add 1 mol Br2
2. Which of the following is a carboxylic acid?
A) B) II. produces 1 g H2 when reacted with zinc,
O O III. produces 2 g H2 when reacted with
H3C C OH H3C C CH3 sodium.
C) D) Which one of the following is this
O OH H compound?
H3C C H H3C CH H HOOCCH CH C=O A)
2 2

3. What is the correct name of the following structure?


H3C O B)
OH OH O
H3C C C OH CH 2 CH=CH CH C OH
CH3 C)
A) 2,2-dimethyl propanoic acid OH OH
B) 2,2-dimethyl-3-propanoic acid
C) 2,2-dimethyl propane aldehyde
CH 2 CH=CH CH 2
D) Isobuthyl acetic acid D)
OH O O
4. Compounds in which the hydroxyl group is bonded to an CH 2 C C OH
alkyl group are called……………
E)
A) aldehydes. B) carboxylic OH O
acids. CH 2 CH=CHCOH
C) alcohols. D) ketones.
4. A 12.8 g sample calcium salt of a dicarboxylic
5. What is the name of the simplest carboxylic acid? acid contains 4 g of Ca. What is the molecular
A) acetic acid B) ethanoic acid
C) formic acid D) benzoic acid
weight of the acid? (Ca;40, C;12, O;16)

Self test A) 48 B) 88 C) 89
1. How many liters of carbon monoxide are D) 90 E) 128
produced at STP if a 23 g sample of formic
acid is decomposed in the pres- ence of
sulfuric acid? (HCOOH = 46 g/mol)

A) 44.8 B) 22.4 C) 11.2


D) 5.6 E) 2.24

2. If a 10 g sample of vinegar is reacted with


marble (mainly CaCO3), 0.112 1 CO2 at STP
is liberated. What is the percentage by mass of
the vinegar?
159
A) HCOOH
B) CH3COOH
C) COOH
D) COOH
5. Which one of the following compounds is E) C3H7COOH COOH CH3OH
optically active?

H C=O
8. 15.68 liters of oxygen at STP are needed to
A) B)
oxidize a 14.8 g sample of a mono carboxylic
COOH H acid. What is the number of carbon atoms in
H C OH the acid ? (C:12, O:16, H:l)
CH 3
A) 1 B) 2 C) 3
D) 4 E) 5
C) D)
COOH COOH
9. Esterification of a mono carboxylic acid and
CH 2 H C NH 2 ethyl alcohol ( C2H5OH) produces 10.2 g
CH 2 H ester and, 1.8 g H3O. What is the molecular
COOH
weight of organic acid ? (C:12, O:16, H:l)

E) A) 46 B) 56 C) 60
H 2 C COOH D) 74 E) 88
HO C COOH Which one of the following compounds
H2 C COOH liberates the same amount of hydrogen when
reacted with either one of zinc and sodium
metals?
6. One mole of an organic compound
I. reacts with sodium hydroxide (NaOH)
A) CH3COH B) CH3CH2OH
II produces 1 mol H2 when reacted with
COOH COOH
metallic sodium
III. Indicates optical activity. Which one of the CH 3 OH
C) D) COOH
following is this compound?
COH
C H 2 OH COOH CH 2
C H 2 OH
CHOH CH 2 CH 3
CHOH E)
C H3 CH 2 OH
A) B) COOH C)
11. A 14.4 g of an aldehyde is converted to 17.6 g
COOH COOH
carboxylic acid when oxidized. What is the
CH 2 CHOH MW of primary alcohol prepared by the
CH 3 reduction of this aldehyde with hydrogen?
D) COOH E) (C;12, H;l, O;16)

7. 0.1 mol of an organic acid is added to 600 ml A) 88 B) 74 C) 60


of 0.5 M potassium hydroxide (KOH) D) 46 E) 32
solution. The resulting basic solution can be
neutralized by 200 ml of 0.5 M HCl solution. 12. Which one of the following is the IUPAC
Which one of the following is the formula of name of
the acid?

160
CH 3 CH CH CH 2 COOH
17. If the pieces of zinc are added to 500 ml of
CH 3 CH 3
oxalic acid, 4.48 1 of hydrogen gas at STP are
A) hexanoic acid produced. What is the molar concentration
B) dimethyl heptanoic acid ofoxalic acid?
C) 2,3-dimethylpentanoic acid
D) 3,4-dimethylpentanoic acid A) 0.2 B) 0.3 C) 0.4
E) 2,3-dimethylbutanoic acid D) 0.5 E) 0.l

18. The name of is


13. An 11.5 g sample of an alcohol reacts with 0,2 A) hydroxypropionic acid
mol of MnO4- by the reaction B) 1,2-dihydroxypropionic acid
5R  CH 2OH  4MnO4  12H   C) ,-dihydroxypropionic acid
5R  COOH  4Mn2  11H 2O D) ,-dihydroxypropionic acid
What is the formula of the acid formed? E) glycolic acid

A) HCOOH B) CH3COOH 19. One mole of


C) C2H5COOH D) C3H7COOH I. is neutralized by 2 rnol of Ca(OH)2
E) C4HgCOOH II. is optically active
III.produces 22.4 1 of H2 at STP when reacted
14. A 39.6 g sample of (CnH2n+1COO)2 Mg with zinc
IV. can be esterified by 2 rnol of a mon- ohydric
contains 4.8 grams of Mg. What is the value of alcohol.
n? (Mg: 24) Which one or ones of these statements are
CORRECT?
A) 1 B) 2 C) 3
D) 4 E) 5 A) I, II B) I, II, III C) I, III
D) I, III, IV E) II, III, IV
15. An organic compound
I. produces hydrogen gas when reacted with 20. What is the pH of 0.0625 M butanoic acid?
sodium
II. forms a precipitate when reacted with Ka=l.6x10-5
Fehling’s reagent. The compound is
A) 2 B) 3 C) 4
D) 5 E) 6
A) formic acid B) acetic acid
C) ethyl alcohol D) acetaldehyde Acetylene + H2O → Carboxylic acid
E) acetone By using 5.2 g. of acetylene how many grams of which acid
16. Given the equations: is produced? (C: 12, H: 1, O: 16)
C2H5–CH=CH2+H2SO4A+H2O A) 12g of CH3 COOH B) 3.2 g of HCOOH C) 6g of
CH3 COOH D) 144 g of HCOOH
E) 14.8 g C2H5 COOH
H2SO4+B+1/2O2C+H2O
R — COOH + Na → R—COONa + 1/2H2
Which one of the following is CORRECT? According to the reaction 14.8 g of carboxylic acid reacts
with Na and produce 2.24 liters of H2 at STP. What is the
A) C is butanoic acid formula of the acid?
B) C gives butanoic acid when oxidized (C: 12, H: 1, O: 16)
A) HCOOH B) CH3COOH C) C2H5 COOH
C) C is butanal D) C3H7COOH E) C4H9COOH
D) B is n-butanol
E) B is 2-butanol Organic substance Z hydrolysis to give X and Y
161
X and Y react with Na metal and produce H2 gas.
X can be oxidized and Y can be reduced.
According to the information above which one is true? A) OH B) OH C) H
X Y Z | | |
A) ester alcohol carboxylic acid CH3 - C - COOH CH3 - C - CN F - C -Br
B) alcohol carboxylic acid ester
| | |
C)carboxylic acid ester alcohol
D) alcohol ester carboxylic acid H H CH3
E)carboxylic acid alcohol ester D) OH E) OH
| |
An 11.2g sample of magnesium salt of dicarboxylic acid HOOC - C - CH2 - COOH OH - C - CH3
contains 2.4g of magnesium. What is the molecular weight of | |
this acid? CH3 C2H5
A) 46 B) 56 C) 88 D) 90 E) 112

O
|| 1 mole of a saturated dicarboxylic acid hydrocarbon is 146 g.
How many hydrogen atoms are there in the saturated
R1 – COOH + R2- OH → CH3 – C – O – CH3+H2O
dicarboxylic acid of hydrocarbon? (C=12, O=16, H=1)
According to the reaction, what are R1 and R2? A) 6 B) 7 C) 8 D) 9 E) 10
R1 R2
A) CH3 CH3 500mL 0.2 M CH3COOH solution is added to 10 g of Ca
B) C2H5 CH3 metal. How many liter of H2 is produced at STP?
C) C2H5 C2H5 A) 1.12 B) 2.24 C) 5.6 D) 11.2 E) 16.8
D) CH3 C2H5
E) CH3 C3H7 An organic compound that contains (-COOH) in a carbon
chain is called carboxylic acid. Which one of the following
Which one(s) is/are true for 1 mol of α- amino acetic acid? statement is wrong for carboxylic acids?
I. It’s optical active A) Carboxylic acids react with bases by neutralization
II. It produces 0.5 mol of H2 gas with Zn metal reaction.
III. It produces 1 mol of H2 gas with Na metal B) Carboxylic acids react with alcohol by esterification.
A) I only B) III only C) I and III D) II C) Carboxylic acids is formed by two times oxidation of
and III E) II only primary alcohol
D) They react with Zn and Mg and produce H2.
E) When carboxylic acid is reduced once to form ketones.

HOOC COOH
CH3
I. It’s aromatic dicarboxylic acid |
II. It polymerizes with glycol to form polyester I. CH3 — C — COOH
III. It reacts with Mg and Na metals |
CH3
Which one(s) is are true for the substance given above?
A) I only B) II and III C) I and III II. CH3 — CH—CH2 — COOH
D) I and II E) I, II and III |
CH3

III. CH3 — CH2 — CH— COOH


|
CH3

Which one of the following compounds is not optically Which one is wrong about the substances given above?
active? A)A) Their OHmolecular formula
B) is C5H10O2C ) H
OH
B) They’re | isomers of each other.
| |
C)CHIf3 I.
- Cis -reduced
COOH twoCH times,
3 - Ctertiary
- CN alcohol F - Cis-Br
formed
D) If II.| Is reduced, 3- methyl butanal | is formed |
E) If III. H is reduced two times, H primary alcohol CH3is formed
D) OH E) OH 162
| |
HOOC - C - CH2 - COOH OH - C - CH3
| |
II. produces 1 g H2 when reacted with zinc,
CH3 III. produces 2 g H2 when reacted with sodium.
I. |
CH3 — C— COOH II. CH3 — CH—CH2 — COOH Which one of the following is this compound?
| |
CH3
H
CH3
HOOCCH 2 CH 2 C=O
A)

III. CH3 — CH2 — CH— COOH OH OH


|
Which of the given names
CH3 is/are wrong? CH 2 CH=CH CH 2
B)
A) I only B) II only C) III only D) I and II OH OH O
E) II and III CH 2 CH=CH CH C OH
C)
0.1 mole of an organic substance; OH O O
I. produces 0.1 ml of H2 gas with Na metal
II. Produces 0.05 ml of H2 gas with Z metal CH 2 C C OH
D)
III. It has asymmetrical C atom
What is the formula of the substance? OH O
CH 2 CH=CHCOH
A) CH3 B) COOH D) CH2-OH E)
| | |
CH3— C—OH CH—OH H— C—CH3
| | | 4. A 12.8 g sample calcium salt of a dicarboxylic
CH2—OH CH3 OH acid contains 4 g of Ca. What is the molecular
weight of the acid? (Ca;40, C;12, O;16)

C) COOH E) COOH A) 48 B) 88 C) 89
| |
OH— C—OH COOH
D) 90 E) 128
|
CH3 5. Which one of the following compounds is
SELF TEST (sayfa 186) optically active?

1. How many liters of carbon monoxide are COOH


produced at STP if a 23 g sample of formic acid
H C OH
is decomposed in the pres- ence of sulfuric
acid? (HCOOH = 46 g/mol) CH 3
A)
H C=O
A) 44.8 B) 22.4 C) 11.2
D) 5.6 E) 2.24 B) H
COOH
2. If a 10 g sample of vinegar is reacted with
CH 2
marble (mainly CaCO3), 0.112 1 CO2 at STP
is liberated. What is the percentage by mass of CH 2
the vinegar?
C) COOH
(CH3COOH = 60 g/mol)
COOH
H C NH 2
A) 12 B) 5 C) 3
D) 6 E) 4.6 D) H

3. 1 rnol of an organic com pound


I. can add 1 mol Br2
163
H 2 C COOH acid. What is the number of carbon atoms in the
HO C COOH acid ? (C:12, O:16, H:l)

H2 C COOH A) 1 B) 2 C) 3
E)
D) 4 E) 5
6. One mole of an organic compound
I. reacts with sodium hydroxide (NaOH) 9. Esterification of a monocarboxylic acid and
II produces 1 mol H2 when reacted with metallic ethyl alcohol ( C2H5OH) produces 10.2 g ester
sodium and, 1.8 g H3O. What is the molecular weight
III. indicates optical activity. Which one of the of organic acid ? (C:12, O:16, H:l)
following is this compound?
A) 46 B) 56 C) 60
C H 2 OH D) 74 E) 88
CHOH
C H3 Which one of the following compounds
A) liberates the same amount of hydrogen when
C H 2 OH reacted with either one of zinc and sodium
CHOH metals?
B) COOH A) CH3COH B) CH3CH2OH
COOH
COH
CH 2
COOH COOH CH 2
CH 2 OH
C) CH 3 OH CH 3
COOH C) D) COOH E)
CH 2
11. A 14.4 g of an aldehyde is converted to 17.6 g
D) COOH carboxylic acid when oxidized. What is the
COOH MW of primary alcohol prepared by the
reduction of this aldehyde with hydrogen? (C;
CHOH
12, H; l, O; 16)
CH 3
E)
A) 88 B) 74 C) 60
D) 46 E) 32
7. 0.1 mol of an organic acid is added to 600 ml of
0.5 M potassium hydroxide (KOH) solution.
12. Which one of the following is the IUPAC name
The resulting basic solution can be neutralized
of
by 200 ml of 0.5 M HCl solution. Which one of
A) hexanoic acid
the following is the formula ofthe acid?
B) dimethyl heptanoic acid
C) 2,3-dimethylpentanoic acid
A) HCOOH
D) 3,4-dimethylpentanoic acid
B) CH3COOH
E) 2,3-dimethylbutanoic acid
C) COOH
D) COOH
E) C3H7COOH COOH CH3OH 13. An 11.5 g sample of an alcohol reacts with 0,2
mol by the reaction
8. 15.68 liters of oxygen at STP are needed to
oxidize a 14.8 g sample of a monocar- boxylic What is the formula of the acid formed?

164
A) HCOOH B) CH3COOH OH
C) C2H5COOH D) C3H7COOH HOOCCHCH COOH
2
19. One mole of
E) C4HgCOOH I. is neutralized by 2 rnol of Ca(OH)2
II. is optically active
14. A 39.6 g sample of (CnH2n+1COO)2 Mg III.produces 22.4 1 of H2 at STP when reacted
contains 4.8 grams of Mg. What is the value of with zinc
n? (Mg: 24) IV. can be esterified by 2 rnol of a monohydric
alcohol.
A) 1 B) 2 C) 3 Which one or ones of these statements are
D) 4 E) 5 correct?
15. An organic compound A) I, II B) I, II, III C) I, III
I. produces hydrogen gas when reacted with D) I, III, IV E) II, III, IV
sodium
II. forms a precipitate when reacted with 20. What is the pH of 0.0625 M butanoic acid?
Fehling’s reagent. The compound is
Ka=l.6x10-5
A) formic acid B) acetic acid
C) ethyl alcohol D) acetaldehyde A) 2 B) 3 C) 4
D) 5 E) 6
E) acetone
16. Given the equations:
C2H5–CH=CH2+H2SO4A+H2O →

H2SO4+B+1/2O2C+H2O

Which one of the following is CORRECT?


A) C is butanoic acid
B) C gives butanoic acid when oxidized
C) C is butanal
D) B is n-butanol
E) B is 2-butanol

17. If the pieces of zinc are added to 500 ml of


oxalic acid, 4.48 1 of hydrogen gas at STP are
produced. What is the molar concentration
ofoxalic acid?

A) 0.2 B) 0.3 C) 0.4


D) 0.5 E) 0.l

18. The name of is


A) hydroxypropionic acid
B) 1,2-dihydroxypropionic acid
C) β, γ-dihydroxypropionic acid
D) α, β -dihydroxypropionic acid
E) glycolic acid

165
SELF TEST D) long-chain fatty acid halides.
E) long-chain fatty acid anhydrides.
1. If an acid with a molecular weight of 60 g reacts
with an alcohol to form an est- er with a molecular 6. Which one of the following acids is found in the
weight of 88 g, what is the molecular weight of the structure of fats which are solids at room temperature?
alcohol?
A) C17H35COOH B) C17H33COOH
A) 18 B) 23 C) 28 C) C17H31COOH D) C15H29COOH
D) 30 E) 46
E) C17H29COOH
2. Which one of the following com pounds is an
ester?

A) CH3OC2H5 B) CH3COONa 7. Which one of the following com pounds is


formed if the isomer of ethyl acetate is reduced by one
C) CH3COOCH3 D) CH3COOH
step?
E) CH3CONO3
A) C2H5CHO B) C3H7CHO
3. Which one of the following pairs may be used to C) C4HgOH D) CH3COC2
O
E) C3H7COOH
CH
3 C
3 7 OC H
obtain the ester
A) CH3OH B) CH3OH C) 8. 35.2 g of ethyl acetate is obtained when 23 g of
C3H7OH ethyl alcohol solution reacts completely with acetic acid.
What is the percentage of alcohol in the solution?
C3H7COOH C3H7CHO (C2H5OH=46 g/mol, CH3COOC2H5=88 g/mol)
HCOOH
D) CH3COOH E) CH7COOH A) 80 B) 70 C) 60
C3H7OH C2H5OH D) 50 E) 40

4. Consider the compounds 9. Which one of the following is not the property
I. CH3OH II. CH3COC2H5 of esters? Esters
III. CH3– O – C2H5 IV. C2H5COOH
A) are hydrolyzed to give their component
Which one of the following statements is compounds.
WRONG? B) have higher boiling points than the acids from
which they are obtained.
A) L reacts with sodium to give hydrogen. C) have characteristic pleasant odors.
B) L and IV react one another to form ester. D) do not form hydrogen bonds.
C) IV is produced by the oxidation of acetaldehyde. E) give alcohols when reduced.
D) III is the structural isomer of pro- panol.
E) II can not be oxidized by Fehling’s reagent. 10. The compound X forms the compound Y when
reduced with hydrogen by two steps. Y reacts with
5. Which one of the following statements defines CH3COOH to form an ester in which alkyl groups are
fats and oils? Fats and oils are
equal. Which one of the following pairs represents the
A) long-chain carboxylic acids. compounds X and Y?
B) sodium salts of long-chain carboxylic acids.
C) esters formed from glycerol and long-chain X Y
carboxylic acids. A) acetic acid ethyl alcohol
B) formic acid ethyl alcohol
166
C) acetic acid methyl alcohol
D) formic acid methyl alcohol According to the reaction, what are R1 and R2?
E) propanoic acid methyl alcohol R1 R2
A) CH3 CH3
B) C2H5 CH3
C) C2H5 C2H5
D) CH3 C2H5
I. Acetic acid II. Formic acid E) CH3 C3H7
III. Ethyl alcohol IV. Methyl alcohol
Which pairs must be used to produce ethyl If 9.2 g of ethyl alcohol reacts with sufficient amount
formate? of propionic acid, how many grams of esters is
A) II and III B) I and IV C) I and III produced?
D) II and IV E) I and II A) 10.2 B) 20.4 C) 316 D) 428 E) 64.2

What is the product of the following reaction?


acid anhydride +alcohol → ester +? Give the name used for this type of reaction:
A) Ester B) Carboxylic acid C) Ketone D) CH3 – CH2 – COOCH3 + H2O →
Aldehyde E) Ethene A) esterification B) saponification
C) addition D) substitution
What is the molar mass of soap formed from the E) hydrolysis
Saponification of a simple triglyceride having a
molar mass of 884 with NaOH? Which one of the following reactions is an
A) 120 B) 212 C) 264 esterification reaction?
D) 292 E) 304 A) C2H5COOH+CH3OH→ C2H5COOCH3+H2O
B) CH3OH + Na → CH3ONa + 1/2H2
What is the product of the following reaction? C) C2H4 + H2 → C2H6
O D) CH4 + 2O2 → CO2 + 2H2O
|| E) CH3CH2OH + 1/2O2 → CH3CHO+H2O
CH3 — C — O Na + C2H5 — Cl → ? +
NaCl
What is the ester which is prepared from 2-methyl
A) Sodium acetate B) Ethylchloride C) Acetic propanoic acid and 2- propanol ?
acid D) Ethyl acetate E) Ethyl
propionate A) CH3 O B) O CH3
| || || |
CH3 — CH— C—O—CH —CH3 CH3 — C—
Which alcohol should react with carboxylic acid to O—CH
obtain this ester? O CH3 | |
CH3 CH3
|| | C) CH3 O
CH3—CH2—C—O—CH | ||
|
CH3 — CH— CH—C—O—CH2 —
CH3 CH3
D) CH3 — CH2— C— O —CH—CH3 E) CH3 — CH— C—O—C2H5
|| | | ||
O CH3 CH3 O

A) ethanol B) 1-propanol C) 2-
propanol D) 2-pentaol E) 2- methyl-3-
pentanol Which one of the following is an ester?
A) O B) O
O || ||
|| CH3 — C— CH2 — CH3 CH3 — C— ONa
R1 – COOH + R2- OH → CH3 – C – O – CH3+H2O 167
C) O D) O E) O CH3
|| || || |
CH3 — CH—C— CH3 CH3 — C— C2H5 — C— O— CH
OH |
What product will form by the reaction of methyl
alcohol with propanoic acid?

A) O B) O C) O
|| || ||
CH3 — C— C2H5 — C— CH3— C—
OC2H5 OCH3 OCH3

D) O E) O
|| ||
C3H7 — C— C3H7 — C— OC2H5
OCH3

168
CHAPTER 1 A) I only B) I and III C) II and III
D) I and II E) I, II and III
1.
C2H5
|
CH3 — C — CH — C2H5
| | 7. I. CH3OH
OH CH3
II. H — C = O
Which one of the following names is correct for the given |
compound? H
A) 2-hydroxy-2-ethyl-3-methylpentane III. CH4
B) 3-hydroxy-3-methyl-4-ethyl
C) 3-hydroxy-3-methyl-4-ethyl Which of the following gives the correct comparison for the above
D) 3-hydroxy-3, 4-dimethylhexane compounds with respect to their boiling points?
E) 4-hydroxy-3-methylpentane
A) I > II > III B) III > II > I C) I = II = III
D) II > I > III E) III > I > II
2. Which one of the following bonds is not found in the structure of
alcohols?
A)C — C B)C — H C)H — O
D) C — O E) H— H 8. I. Isopropyi alcohol and dimethyl ether
II. 2 - butanol and ethyl methyl ether
III. 3 - pentanol and ethyl propyl ether
3. Which one of the following alcohols doesn't have an ether
isomer? Which of the given pairs of compounds are isomers?
A) CH3OH B) C2H5OH C) C3H7OH
D) C4H9OH E) C5H11OH A) I only B) II only C) III only
D) II and III E) I, II and III

4. What is the name of the hydrocarbon in which two methyl, one


ethyl and one hydroxy group are attached to a carbon atom?
A) 2- methyl - 2 - butanol 9. What is the molecular formula of the monoalcohol that contains
B) 2- methyl - 2 - hydroxy- propane 60% carbon by mass?
C) 2,2 - dimethyl - 2 - hydroxy - propane
D) n - pentanol A) CH3OH B) C2H5OH C) C3H7OH
E) 2- pentanol
OH
D) E) OH
5. Which one of the following compounds is not an isomer of the
others?
A) CH3CH2CH2CH2OH
B) CH3 — O — C3H7 10. CH3
C) C2H5 — O — C2H5 |
D) CH3CHCH2CH3 CH3 — C — CH
| |
OH OH

E) CH3 — CH — OH Which of the names below can be used to correctly name the above
| compound?
CH3 I. Isobutanol
II. Tertiary butanol
6. CH3 — O — C2H5 III. Trimethyl carbinol
For the given compound,
I. It is ethyl methyl ether. A) I only B) II and III C) III only
II. Hydrogen bonds exist between the molecules of the compound. D) I and II E) I, II and III
III. It is an isomer of iso-propyl alcohol.

Which of the above statements above is/are true?


11. I. n-pentanol
169
II. Isopentanol
III. Neopentanol A) Ethyl methyl ketone
Which is true for the melting point comparison of the above B) Dimethyl ketone
compounds? C) 2-butanol
D) Butanoic acid
A) I = II = III B) I > II > III C) I > III > II E) Diethyl ketone
D) III > I > II E) III > II > I

18. For the given compounds having the same number of carbon
atoms;
12. At least how many carbon atoms are found in a tertiary alcohol? I. Primary alcohol
A) 1 B) 2 C) 3 D) 4 E) 5 II. Secondary alcohol
III. Tertiary alcohol
Which one is true for their boiling point comparison?

13. Some compounds and their first oxidation products are given A) I > II > III B) I = II = III C) III > II > I
below. D) I > III > II E) III > I > II
Compound Oxidation product
I. Primary alcohol Aldehyde
II. Secondary alcohol Ketone 19. From the reaction of 4.6 grams of a monoalcohol with Na
III. Tertiary alcohol Carboxylic acid metal, 1.12 L of H2 gas is obtained at STP. What is the molar mass
Which is/are correct? of the other product obtained?
(C: 12, H: 1, O: 16, Na: 23)
A) I only B) I and II C) II and III
D) III only E) I, II and III A) 45 B) 46 C) 68 D) 70 E) 72
20. 2.24 L of H2 gas is evolved after the reaction of a 20 gram
mixture of C2H5OH and CH3OCH3 with Na metal.
14. When ethyl alcohol is oxidized 2 times, which one of the What is the mass percentage of dimethyl ether in the mixture?
following products results?
A) 46 B) 54 C) 82 D) 18 E) 72
A) Acetaldehyde B) Acetic acid
C) Oxy-propane D) Dimethyl ether
E) Dimethyl ketone
21. In acidic medium, CH3OH reacts with KMnO4 solution, to give
15. White using an equal mass of ethyl alcohol, the fallowing the following reaction:
substances are produced.
I. Ethylene 5CH3OH + 4KMnO4 + 6H2SO4
II. Ethane 5HCOOH + 4MnS04 + 2K2SO4 + 11H2O
III. Acetic acid
Which one of the given comparison is true, if the mass of products How many grams of CH3OH react with 400 mL, 0.1 M KMnO4 and
are compared? how many grams of HCOOH are produced by the reaction?

A) I>II>III B) III>II>I C) I=II=III CH3OH HCOOH


D) II>HI>I E) III>I>II A) 1.6 2.30
B) 3.2 4.60
C) 1.6 1.15
16. CH2 = CH — CH2 — OH D) 2.4 3.45
E) 4.8 2.30
For the given compound,
I. Its aqueous solution has basic properties.
II. It forms esters with carboxylic acids.
III. It fades the color of an aqueous solution of bromine. Which of 22. How many liters of CO2 gas are produced by the combustion
the above statements is/are true? of 0.5 mole of diethyl ether with a sufficient amount of oxygen gas?

A) I only B) III only C) I and II A) 44.8 B) 22.4 C) 16.8 D) 8.96 E) 5.6


D) II and III E) I, II and III

17. Which one of the following compounds is produced by the 23. CH2 —CH —CH2
primary oxidation of butane-2-ol? | | |
170
OH OH OH

For 1 mole of the above organic compound,


I. It reacts with 3 mol of Na.
II. When 1 mol is reacted with a sufficient amount of Na metal, 3/2
mol of H2 gas is evolved.
III. It reacts with 1 mol of Zn. CHAPTER 2
Which of the above statements is/are true?

1. CH3 – CH2 – CH – CH – C = O
A) I only B) II only C) I and II | | |
D) I and III E) I, II and III CH3 CH3 H

The above compound is correctly named by which of the


24. following?
[O] [O]
C2H5OH X Y 2Z + 2H2O A) 2,4 - dimethylpentanal
- H2O B) 2,3 - dimethylpentanal
C) 2,3 - dimethyl - 1 - pentanol
For given reaction, D) 3 - ethyl - 2 - methyl - 1- butanal
I. The compound X is acetaldehyde. E) n – heptanal
II. The compound Y is acetic acid.
III. The compound Z is carbon dioxide.
Which of the above statements is/are true?
2. Which of the following aldehydes are named incorrectly?
A) I only B) II only C) I and II
D) II and III E) I, II and III I. CH3 – CH – C = O
| |
CH3 H
25. CH3 n – butanal
|
CH3— C— C2H5 CH3
| |
OH II. CH3 – CH2 – C – CH2 – C = O
For the above compound, which one of the following names cannot | |
be used? CH3 H
3,3 dimethylpentanal
A) 2-methyl-2-butanol
B) 2-methyl-2-hydroxybutane
C) Tertiary pentanol III. CH2 – CH2 – C = O
D) Dimethyl ethyl carbinol |
E) 2-pentanol H
3 – phenylpropanal

IV. CH3 – CH – CH – CH – CH – C = O
| | | | |
OH OH OH OH H

2,3,4,5 – tetrahydroxyhexanal

A) I only B) II only C) III only

D) I and III E) II and IV

3. I. CH3OH
II. H — C = O
|
171
H C) C2H5 — C = O D) C3H7 — C = O
| |
III. CH4 H H
Compare the compounds above with respect to their boiling points.
E) C4H9 — C = O H
A) I > II > III B) III > II > I C) I = II = III
D) II > I > III E) III > I > II
8. Which one of the following compounds reduces the Ag+ ion in
silver nitrate and ammonia solution?
4. CH3CHO + 2Ag+ + 3OH- 2Ag + CH3COO - + 2H2O
A) Aldehyde B) Ketone C)Alkene
According to this equation, how many grams of metallic silver D) Carboxylic acid E) Ester
precipitate when 0.22 gram of CH3CHO is reacted?
(C: 12 g/mol, H: 1 g/mol, O: 16 g/mol, Ag: 108 g/mol)
9. Which one of the following compounds produces 2 - methyl
A) 0.54 B) 1.08 C) 1.62 D) 2.16 E) 4.32 - 3 - pentanone when it is partially oxidized?

A) 2 - methyl - 3 - pentanol
5. CH3 — CH — C = O B) 2 - hydroxyhexane
| | C) 2 - methyl - 2 - hydroxypentane
CH3 H D) 3 - methyl - 2 - hydroxypentanol
This compound, E) n – hexanol
I. is named as 2 - methylpropanal.
II. reacts with Na metal.
III. reacts with Tollens' reagent.
Which of the statements above is (are) true? 10. CH2 — CH2 — C — CH3
||
A) I only B) II only C) I and III O
D) I and II E) I, II and III
I. Ethyl methyl ketone
II. 2 - butanone
III. Buten - 2 - one
6. CH3 — CH — CH — C = O
| | | Which of the names above describe the compound correctly?
OH CH3 H
A) I only B) I and II C) II and III
D) I and III E) I, II and III
For the above compound,
I. When it reacts with Na, H2 gas is evolved
II. When it reacts with silver nitrate and ammonia, metallic silver
precipitates. 11. ( R — COO)2Ca R — C — R + CaCO3
III. It undergoes an addition reaction with H2. ||
O
Which of the statements above is (are) true?
In the reaction above, 15.8 gram of (RCOO) 2 Ca produces 10
A) I only B) I and II C) II and III grams of CaCO3.
D) I and III E) I, II and III How many C atoms are there in the structure of the produced
ketone?
(Ca: 40 g/mol, C: 12 g/mol, H: 1 g/mol, O: 16 g/mol)
7. RCHO + 2Ag+ + H2O RCOOH + 2Ag + 2H+
According to the equation above, 5.8 grams of aldehyde RCHO A)3 B)4 C)5 D)6 E) 7
produces 21.6 grams of metallic silver. Which one of the following
is the molecular formula of this
aldehyde ? 12. I. Methanal
(Ag: 108 g/mol, H: 1 g/mol, O: 16 g/mol) II. Ethanal
III. Propanal
A) H—C=O B) CH3—C=O Which of the compounds above do not have a ketone isomer?
| |
H H A) II only B) III only C) I and II
D) II and III E) I, II and III
172
A) They can be reduced.
13. I. H—C=O B) They can be oxidized.
| C) They undergo addition reactions.
H D) The can be polymerized.
II. CH3 — C = O E) Combustion products are CO2 and H2O.
|
CH3
III. CH3 — C = O
| 19. Some compounds have more than one name. Pairs of names are
H given below.
Which of the above compounds can not be oxidized?
I. Dimethyl ketone, acetone
A) I only B) II only C) III only II. Methanol, formaldehyde
D) II and III E) I and III III. Ethanal, acetaldehyde
Which pairings represent the same compound?

14. I. CH3 – C = O A) 1 only B) II only C) I and II


| D) II and III E) I, II and III
H
II. CH3 – C ≡ CH

III. CH3 — CH — C = O 20. 3 methyl and an aldehyde group are attached to a carbon
| | atom.
OH H I. 2,2 - dimethylpropanal
Which of the compounds above precipitate metallic silver when II. Isopentanal
reacted with AgNO3 in ammonia solution? III. n - pentanal
Which of the names above represent this compound correctly?
A) I only B) III only C) I and III
D) II and III E) I, II and III A) I only B) I and II C) III only
D) I and III E) I, II and III
15. I. Undergo reduction to form primary alcohols.
II. React with Fehling's reagent
III. When they burn, CO2 and H2O are formed Which of 21. Which one of the following compounds forms acetone when it
the properties above apply to ketones? is partially oxidized?

A) I only B) III only C) I and III A) CH3CH2 — C = O B) CH3CH2 — COOH


D) II and III E) I, II and III |
OCH3
C) COOH D) CH3
16. Which one of the following is formed when isopropyl alcohol is | |
partially oxidized? CH — OH CH — OH
| |
A) Propanol B) Dimethyl ether C) Acetone COOH CH3
D) Ethyl methyl ketone E) Propanoic acid
E) CH2 — OH
|
17. Aldehydes, CH2
|
I. can be oxidized. CH2 — OH
II. can be reduced.
III. can produce polymers.
Which of the statements above is (are) true? 22. I. 1 - butanol
II. 2 - butanol
A) I only B) II only C) III only III.2 - methylpropanol
D) I and III E) I, II and III Which of the given alcohols form a ketone when oxidized?

A) I only B) II only C) III only


D) I and II E) II and III
18.Which one of the following is not a property of ketones?
173
A) I only B) I and II C) II only
23. The following statements describe formaldehyde, D) I and III E) I, II and III
I. When it polymerizes with phenol, bakelite is
obtained.
II. When it reacts with ammonia, hexamethylene
tetramine (urotropine) is formed.
III. When it is partially oxidized, formic acid is
formed.
Which are true?

A) I only B) II only C) I and II


D) II and III E) I, II and III
24. To reduce 2.2 grams of acetaldehyde, how many liters of H2 are
needed at STP?
(C: 12 g/mol, H: 1 g/mol, O: 16 g/mol)

A) 0.56 B) 1.12 C) 1.68 D) 2.24 E) 4.48

25.CH3CHO + 2Cu+2 + 5OH -


Cu2O + CH3COO - + 3H2O
According to the above equation, how many grams of acetaldehyde
are needed to precipitate 28.8 grams of Cu2O?
(C: 12 g/mol, H: 1 g/mol, O: 16 g/mol, Cu: 64 g/mol)

A) 1.53 B) 9.2 Q8.8 D) 2.35 E) 1.27

26- I. OH II. OH
| |
CH3 — C — H CH3—C —H
| |
H CH3

III. OH
|
CH3 — C — CH3
|
H — C — CH3
|
OH

Which of the above alcohols produce a ketone when oxidized?

A) I only B) II only C) III only


D) I and II E) II and III

27. For

and

I. They are isomers of each other.


II. They are isomers of 3 - methyl - 1- butanol. CHAPTER 3
III. They undergo an addition reaction with NH3.
1. What is the molecular formula of the carboxylic acid that
Which of the above statements is (are) true? contains 40.68% C, 54.24 % O and 5.06 % H by mass?
(C: 12 g/mol, H: 1 g/mol, O: 16 g/mol)
174
A) HCOOH B) CH3COOH O
||
C) COOH D) HOOC (CH2)2COOH II. CH3 — (CH2)2 — C—H
|
COOH E) C6H5OH
III. CH3 — (CH2)3 — OH

2. Which one of the given compounds is a ketoacid? O


||
A) HO — C = O B) HO — C = O IV. CH3 — (CH2)2 — C — OH
| |
OH H What is the correct order of boiling points for the above
compounds?
O O O
|| || || A) I > II > III > IV B) IV > III > II >I
C) CH3 —C —C —OH D) HO —C —O—CH3 C) I > II = III > IV D) IV = III = II >I
E) IV > II = III > I
O
||
E) H2N — CH2 — C — OH
8. CH2 — CH — C = O 1 mol of this compound;
| |
3- CH3 — CH — CH — CH — C = O OH OH
| | | |
CH3 NH2 OH OH I. reacts with 2 mol Na
II. reacts with 1 mol NaOH
What is the correct name of the above compound? III. reacts with 1 mol Zn

A) 1,2-dihydroxy-3-amino-4-methylpentanol which of the above are correct?


B) 2-hydroxy-3-amino-4-methylpentanoic acid
C) 1 -hydroxy-2-amino-3-methylbutanoic acid A) I only B) III only C) I and II
D) l,2-dihydroxy-3-amino-4-methylpentanoic acid D) II and III E) I, II and III
E) 2-hydroxy-3-amino-4-methylpentanol

9. H — C = O reacts with;
4. 2 methyl, 1 carboxyl and 1 hydroxyl group are attached to a |
carbon atom. What is the correct name of this compound? OH
I. ammoniacal AgNO3
A) 2,2-dimethyl-2-hydroxyethanoic acid II. NaOH
B) 2-hydroxy-2-methylpropanoic acid III. hydrogen
C) 1-methyl-1-hydroabutanoic acid which of the above is/are correct?
D) Isopropionic acid
E) 2-hydroxy-3-rnethylpropanoic acid A) I only B) II only C) I and II
5. HCOOHO(I) HCOO-(aq) + H+(aq) Ka = 10-4 What is the D) II and III E) I, II and III
-2
pH of a 10 M HCOOH solution?

A) 2 B)3 C)4 D)5 E) 6 10. What is the molar mass of the aldehyde that is produced by the
one-step reduction of a monocarboxylic acid with a molar mass of
74 grams?
6. Which of the compounds below has the highest boiling point? (C: 12 g/mol, H: 1 g/mol, O: 16 g/mol)

A) CH3CH2CH2CH3 A) 46 B) 58 C) 62 D) 66 E) 72
B) CH3COOH
C) CH3CH2OH
D) HOOC — CH2CH2 — COOH
E) CH3CH2 — O — CH3 11. COOH COONa
I + 2NaOH I + 2H2O
COOH COONa
7. I. CH3 — (CH2)3 — CH3
175
According to above reaction, how many grams of NaOH would
react with 90 grams of a 20% by mass oxalic acid solution? 17. O
(C: 12 g/mol, H: 1 g/mol, O: 16 g/mol, Na: 23 g/mol) ||
I. H —C —OH
A) 4 B)8 C) 16 D)20 E) 40
O
||
II. HO —C —OH
12. Which one of the following compounds does not show acidic
properties? O O
|| ||
A) CH2 — C = O B) OH III. HO —C —C —OH
| | Which of the above compounds show both acid and aldehyde
Cl H properties?

A) I only B) III only C) I and II


C) CH2— C = O D) COOH D) II and III E) I, II and III
| | | 18. Which one of the following is the product of the reaction
OH OH COOH between dilute H2SO4 and formic acid?

A) CH3 — C = O B) CH3 — CH2OH


E) HO — C = O |
| H
OH
C) CO + H2O D) CO2 + H2O

13. Which one of the following compounds is produced by the two E) CO + CO2 + H2O
degree reduction of formic acid?

A) Methanoic acid B) Methanal C) Methanol 19. Which one of the following compounds cannot be oxidized to a
D) Formaldehyde E) Ethyl alcohol carboxylic acid?

A) Methanol B) Ethanol C) Butanol


O O D) n-propanol E) 2-pentanol
|| ||
14. Regarding HO — C — CH2 — C— CH3 ,
I. it is a dicarboxylic acid. 20. Which one of the following carboxylic acids cannot be
II. when it is oxidized, an ether is formed. obtained by the reaction of a Grignard compound with CO2?
III. it is an acid anhydride.
Which of the above statements is/are incorrect? A) HCOOH B) CH3COOH C) C2H5COOH
D) C3H6COOH E) C4H10COOH
A) I only B) II only C) I and III
D) II and III E) I, II and III
21. 134.4 mL CO2 at STP is obtained from the reaction of 6
grams vinegar with an excess of NaHCO3. What is the mass
15. What is the molar mass of the ester that is produced from the percentage of acetic acid in the vinegar?
reaction of 60 grams of a carboxylic acid with 46 grams of an (CH3COOH: 60 g/mol)
alcohol?
A) 4.5 B) 6.0 Q9.6 D) 21 E) 45
A) 106 B) 88 C) 98 D) 124 E) 82

22. Large molecules of saturated monocarboxylic acids are solid at


16. I. C2H5OH + CH3OH C2H5OCH3 + H2O room temperature.
II. CH3OH + CH3COOH CH3COOCH3 + H2O
III. KOH + HCOOH HCOOK + H2O I. C17H35COOH
Which of the above reactions are esterification reactions? II. C15H31COOH
III. C17H33COOH
A) I only B) II only C) I and II Which of the above compounds are solid at room conditions?
D) II and III E) I, II and III
A) I only B) III only C) I and II
176
D) II and 111 E) I, II and III I. resistant to acids and bases
II. optically active is needed.
Which one of compounds below satisfies these two conditions?

23. I. Acetic acid


II. Lactic acid
III. Oxalic acid
Which of the above represent a hydroxy acid?

A) I only B) II only C) III only


D) II and III E) I, II and III

24. Some common acids found in daily life together with their
sources are given below,
I. lemon salt: citric acid
II. aspirin: acetyl salicylic acid
III. vinegar: acetic acid
Which pairings are correct?

A) I only B) I and II C) II and III 28. HO OH


D) I and III E) I, II and III C=C
O=C CH — CH — CH2 — OH
O
25. O = C — CH — CH — CH2 — C = O
| | | | How many asymmetric carbon atoms does this compound
OH OH OH H have?

The compound above, A) 0 B) 1 C) 2 D) 3 E) 4


I. is optically active
II. contains four asymmetric carbon atoms
III. contains aldehyde, alcohol and carboxyl groups 29. H2C — CH — COOH
Which of the above are correct? | |
NH2 OH
A) I only B) II only C) I and III The above compound,
D) II and III E) I, II and III I. has 2 asymmetric carbon atoms.
II. is optically active.
III. has 2 optical isomers.
Which of the above statements is(are) wrong?
26. CH3 — CH — COOH
| A) I only B) III only C) I and II
OH D) II and III E) I, II and III

Regarding above compound,


I. it reacts with KOH
II. it is optically active
III. H2 gas is evolved upon reaction with Mg
30. I. Naturally occurring fatty acids exist in the cis-
Which of the above statements are correct? geometry.
II. The simplest fatty acid is butyric acid,
A) I only B) II only C) I and II C3H7COOH.
D) I and III E) I, II and III III. Double bonds in the structure decrease the
melting points of fatty acids.
Which of the above statements is (are) true?

A) I only B) II only C) I and III


27. In digital clocks, a liquid crystal substance which is D) II and III E) I, II and III
177
A) H- B) CH3- C) C2H5- D) C3H7- E) C4H9-

3. Which one of the following compounds does not contain an acyl


group?

A) Ketones B) Esters C) Carboxylic acids


D) Amides E) Ethers

4. I. Formic acid
II. Acetic acid
III. Propionic acid
Which of the above acids does not have an ester isomer?
A) I only B) II only C) I and II
D) II and III E) I, II and III

5.
O
||
CH2 — CH2 — C — O — CH2 — CH2 — CH — CH3
| |
Cl CH3

What is the correct name for the compound above?

A) Isopentyl ester of  - chloroacetic acid


B) Pentyl ester of - chloroacetic acid
C)  -chloro ethyl ester of isovaleric acid
D) Isopentyl ester of  -chloropropionic acid
E) 2-chloropropionic acid

6. Compounds formed by the reaction of inorganic or organic acids


with alcohols are called esters.
According to this, which one of the given compounds is not an
ester?

A) HCOOCH3 B) C4H9 — C — OCH3

C)CH3 — C = O D) CH2ONO2
| |
O — C2H5 CH2
CHAPTER 4 |
CH2ONO2
1. Esters are;
I. Alkyl salts of carboxylic acids E) CH2 — CH — CH2
II. Formed by the attachment of an alkoxy group | | |
to an acyl group OH OH OCH3
III. Oxidation products of carboxylic acids.
Which of the above statements, is (are) correct?
7. What is the name of CH3COOCH3 ?
A) I only B) I and II C) III only
D) II and III E) I, II and III A) Methyl methanoate B) Ethyl methanoate
C) Ethyl acetate D) Methyl acetate
E) Methyl formate
2. Which one of the following groups cannot be attached to the
carbon of the functional group of an ester of molar mass 102
g/mol?

178
8. Which one of the compounds shown below is isopropyl-
p-methyl benzoate?

Which of the
compounds
shown above can be hydrolyzed with an acid catalyst?

A) I only B) I and II C) I and III


D) III only E) IV only

12. What is the type of reaction occurring when isobutyl acetate


reacts with NaOH?

9. Which A) Oxidation B) Addition C) Polymerization


of the given D) Esterification E) Saponification
pairs of compounds are isomers of
each other? 13. Which of the following pair of compounds are formed by the
reaction of methyl oleate with NaOH?
I. Methyl acetate - methyl propanoate
II. Phenylacetic acid - methyl propanoate A) Methyl alcohol and sodiumoleate
III. Benzoic acid - phenyl formate B) Methyl alcohol and oleic acid
C) Oleic alcohol and methanoic acid
A) I only B) III only C) I and II D) Oleic acid and methanoic acid
D) II and III E) I, II and III E) Oleic alcohol and sodium methylate

10. For esters; 14. Which one of the following species is able to break up the
I. Hydrogen bonds exist between their molecules. structure of the acryl group in an ester?
II. They are more soluble in water than in alcohols.
III. Liquids esters are good organic solvents. A) H2O B) KOH C) H2 D) NH3 E) NaOH
Which of the above statements is (are) correct?

A) I only B) III only C) I and II


D) I and III E) I, II and III

11.

179
| | |
CH3 H2C — OH H2C — OH

Which of the molecules above is (are) carbohydrates?

A) I only B) II only C) III only


D) II and III E) I and II

5. H CH2 — OH
| |
C=O C=O
| |
H — C — OH CH2 — OH
|
CH2 — OH

X Y

CHAPTER 5 Which of the following is(are) correct for the compounds above?
They are;
I. optically active
II. Isomers of each other
1. Which one of the following is the main source of carbohydrates? III. Carbohydrates
A) Stones B) Sedimentary rock
C) Green plants D) Soil A) I only B) II only C) III only
E)Air D) II and III E) I and II

2. Which one of the following is incorrect for 6. Which one of the following is incorrect for carbohydrates?
photosynthesis?
A) Monosaccharides are the simplest sugars.
A) It occurs in the porous leaves of green plants. B) Monosaccharides are straight chained molecules.
B) Sunlight is used as an energy source. C) Disaccharides are formed by the combination of
C) It turns CO2 from the air into carbohydrates. two monosaccharide molecules.
D) Respiration in animals and humans is the same as D) If three or more monosaccharide molecules
photosynthesis. combine they form polysaccharides.
E) It occurs using the catalyst chlorophyll. E) Monosaccharides cannot be hydrolyzed to simpler
molecules.

3. Fill in the blank given below with suitable word.


Carbohydrates are ............... that contain aldehyde or 7. Which one of the following is incorrect for
ketone groups in their structures. monosaccharides?

A) Ethers B) Esters C) Aldehydes A) They cannot be hydrolyzed to simpler molecules.


D) Polyesters E) Polyalcohols B) They can enter the blood stream directly.
C) Monosaccharides are the only class of
carbohydrates that can be fermented.
D) The most important are glucose, fructose, mannose
and galactose.
4. I. H II. H III. H E) They do not show optical activity.
| | |
C=O C=O H — C — OH
| | |
H— C — OH H — C — OH C=O
180
B) Maltose is formed by the combination of two
molecules of glucose.
C) Lactose is formed by linking glucose and mannose
8. Which one of the following is wrong for glucose? molecules together by glucoside bonds.
D) Maltose and cellobiose are geometrical isomers of
A) It is the most common sugar in nature and is each other.
known as blood sugar. E) Cellobiose shows exhibits properties.
B) It can be fermented by yeast.
C) It can be oxidized to a carboxylic acid by Fehling's
reagent. 14. What is the mass percentage of a saccharose solution if 228
D) It can be reduced to a polyalcohol by NaBH4. grams of it produces 72 grams of glucose after hydrolysis in acidic
E) Mutarotation is not a property of aqueous solutions of medium?
glucose. (C: 12 g/mol, H: 1 g/mol, O: 16 g/mol)

A) 60 B) 50 C) 40 D) 30 E) 20
9. How many grams of 20% glucose solution should be used to
produce 28.6 g of Cu2O from Fehling's solution?
(C6H12O6: 180 g/mol, Cu2O: 143 g/mol) 15. Which one of the following is incorrect for
polysaccharides?
A) 180 B) 360 C) 90 D) 45 E) 286
A) They are highly soluble in water.
B) They don't have a stable crystalline structure.
10. The fermentation reaction of glucose is; C) When they are hydrolyzed, they can be broken
down into monosaccharides.
yeast D) They are found in plant and animal tissues.
C6H12O6 2C2H5OH + 2C02 E) The most common polysaccharides are starch, cellulose and
glycogen.
How many grams of ethyl alcohol are produced from 90 grams of a
60% of glucose solution?
16. Which one of the following statements describing
A) 13.8 B) 18.4 Q23 D) 27.6 E) 32.2 polysaccharides is incorrect?

A) They don't have a definite melting point, they


11. Which statement is incorrect for disaccharides? sublime.
B) Starch reacts with Fehling's reagent.
A) Their general formula is Cl2H22O11. C) The hydrolysis of starch by dilute acid produces
B) They are all soluble in water. dextrin.
C) They are the condensation product of at least two D) Cellulose is the most common organic substance
monosaccharides. and forms the walls of plant cells.
D) The most important are saccharose, maltose, E) Their general formula is (C6H10O5)n.
lactose and cellobiose.
E) They are more soluble in alcohol than in water.

12. Which statement is correct for sucrose?

A) It is widely found in grapes and honey.


B) It is formed by combination of two molecules of
glucose. CHAPTER 6
C) It is slightly soluble in water and highly soluble in
alcohol.
D) It doesn't show any optical activity. 1. Some compounds and their types are given. Which pair are
E) It doesn't react with Fehling's and Tollens' reagents. wrong ?

13. Which one of the following statements is incorrect?

A) Lactose is known as milk sugar and is not found in


plants.
181
..
6. I. CH3 — N — CH3
|
CH3
..
2. II. CH3 — NH2
I. CH3 — NH2
II. CH3 — OH O
III. NH3 ||
Which of the compounds above is (are) basic? III. CH3 — C — NH2

A) I only B) III only C) I and III


D) II and III E) I, II and III Which of the above compounds undergoes hydrogen bonding with
itself?

A) I only B) II only C) II and III


D) I and III E) I, II and III
3. For an aqueous solution of the

compound ;

I. The pH > 7.
II. It conducts electricity.
III. It reacts with Zn metal to give H2 gas.
Which of the above statements is (are) correct?

A) I only B) II only C) I and II


D) II and III E) I, II, III

7. Which one of the following is a primary amide?

O O
|| .. || ..
4. What is the name of the alkyl group in the primary amine that A) H — C — NH2 B) H — C — N — CH3
has a molecular weight of 45 grams? |
CH3
A) Methyl B) Ethyl C) Propyl
D) Isopropyl E) Aryl O
.. || ..
C) CH3 — C — NH D) CH3 — C — N — CH3
|| | |
5. Which one of the following is not a secondary amine O CH3 CH3

182
C) CH3 — C — COOH D) CH3 — C — COOH
O | |
|| .. NH2 NH2
E) C2H5 — C — N — CH3
| E) CH3 — C — CH3
H ||
O

8. Which statement is incorrect for - amino acids?


12. Compound Functional Groups
A) They are not optically active.
I. Amine Nitro
B) They contain two kinds of functional groups.
II. Amide Carbonyl, hydroxyl
C) They form Zwitterions in the solid state.
III. Amino acid Amino, carboxyl
D) They are amphoteric.
E) They are slightly soluble in water.
Nitrogen containing organic compounds and their functional groups
are given.
Which description(s) is (are) correct?
9. O
||
A) I only B) III only C) I and II
CH3 — C — NH2
D) II and III E) I, II and III
For the above compound, which statement is correct?

A) It is named aminoethanoic acid.


B) It is optically active.
C) It reacts with HC1.
D) It is neutral.
E) It reacts with NaOH

13. Peptide bonds;


I. are formed between at least two amino acids.
II. are formed between nitrogen and carbon atoms.
III. form Zwitterions (dipolar ions).
Which of the above statements is (are) correct?

A) I only B) II only C) I and II


D) II and III E) I, II and III
10. Which of the statements below is incorrect for 1 mole of
acetamide and 1 mole of N-methylformamide?

A) Their carbon atom numbers are equal.


B) The masses of their nitrogen atoms are equal.
C) The number of their hydrogen atoms are equal.
D) They have different number of pi bonds.
E) They contain the same number of oxygen atoms.
14. Which one of the following compounds is named
incorrectly?

11. An unknown organic compound forms a dipolar ion (a


Zwitterion) and is optically active. Which one of the given
compounds may be the unknown compound?

O O
|| ||
A) CH3 — C — OH B) CH3 — C — NH2

CH3 H
| |
183
D) CH3 — CH — COOH Acidic
|
OH

E) CH3 — CH2 — CH — COOH Basic


|
OH

CHAPTER 7

1. How many sigma bonds are there in one molecule of nitro


toluene?

A) 9 B) 10 C) 12 D) 15 E) 17

2. Which of the following processes is a reduction reaction?

A)

15. Some compounds and the properties of their solutions are given
below. Which pairing is incorrect?

Compound Property

A) C2H5 — C = O Neutral
|
NH2

B) CH3 — CH — COOH Amphoteric


|
NH2

C) CH3 — NH Basic production of aniline from nitrobenzene.


| B) production of phenylchloride from benzene.
CH3 C) production of nitrobenzene from benzene.
184
D) production of benzoic acid from toluene.
E) production of benzoic acid from benzaldehyde.

3. AlCl3
I. C6H6 + C12 8. How many mL of 0.8 M HCl solution is needed to
neutralize the aniline that is produced from 49.2 grams
II. C6H6 + Br2 (aq) of nitrobenzene with 25% efficiency?
(N: 14 g/mol, C: 12 g/mol, H: 1 g/mol, O: 16 g/mol)
H2SO4
III. C6H6 + HNO3 A) 100 B) 125 C) 150 D) 250 E) 500

Which of the above reactions actually occur?

A) I only B) I and II C) II and III 9. Which one of the following changes the color of litmus paper to
D) I and III E) I, II and III blue?

4. How many grams of 63% by mass HNO3 must be used to convert


27.6 grams of toluene to trinitrotoluene in the presence of H2SO4?
(N : 14 g/mol, C : 12 g/mol, H : 1 g/mol, O : 16 g/mol)

A) 60 B) 70 C) 80 D) 90 E) 100

5.

Which one of the following is the 10. 1 mol of which of the following compounds can produce 2 mol
incorrect name for the above compound? of CH3COOH?
A) m-aminotoluene
B) m-toluidine
C) m-methylaniline
D) m-aminomethylbenzene
E) p-methylnitrobenzene
6. Which one of the following groups cannot be directly
attached to the benzene ring?

A) -NO2 B) -NH2 C) -OH D) -Cl E) -Br

7. Which one of the following compounds is an oxidation product


of 2,6-dihydroxyaniline?

11. Compound Preparation Method

185
I. nitrobenzene nitration of benzene C) o-dihydroxybenzene D) Phenol
II. aniline reduction of nitrobenzene E) Phenylmethyl ether
III. phenol addition of base to benzyl chloride

Which of the preparation methods given above is (are) correct? 17. Benzaldehyde can react with;
I. K2Cr2O7/H+
A) I only B) III only C) I and II II. Tollens' reagent
D) I and III E) I, II and III III. Fehling's reagent
IV. LiAlH4
Which of the reagents above is (are) correct?
12. Both phenol (C6H5OH) and ethanol (C2H5OH) can react with;
I. Na metal A) I only B) I and IV C) I, III and IV
II. HBr solution D) I, II and IV E) IV only
III. NaOH solution
Which of the above statements is (are) correct?
18. I. Phenol
A) I only B) II only C) I and II II. Benzole acid
D) I and III E) II and III III. Benzyl alcohol

Which of the above compounds can react with at least two


13. Which of the combinations below shows the given compounds substances from K, KOH and KHCO3?
in order of increasing acidity?
A) I only B) II only C) I and II
I. CH3COOH D) I and III E) II and III
19. How many liters of H2 are produced at STP when 200 grams of
II. CH2COOH 5.4% by mass benzyl alcohol solution reacts with Na metal?
|
Br A) 1.12 B) 2.24 C) 3.36 D) 4.48 E) 5.6

III. C6H5OH
20. Which one of the following is not an isomer of the
A) I > II > III B) III > II > I C) I = II > III others?
D) II > I > III E) III > I > II

14. Which one of the given compounds can react with Na, NaOH
and NaHCO3?

A) C6H5CH2OH B) C6H5OH C) CH3OH


D) HOCH2CH2OH E) C6H5COOH

15. phenylmagnesium benzyl


A B
Bromobenzene bromide alcoho|

What are the reagents A and B?


________A__________ ________B___________
A) phenol benzaldehyde / H2O
B) magnesium bromide acetone/HBr
C) magnesium formaldehyde/HBr
D) magnesium acetaldehyde/H2O
E) magnesium bromide formaldehyde/HBr

16. When 1 mole of C7H8O reacts with Na, 0.5 mole of H2 and an
unknown compound are produced. In addition, when it is oxidized,
salicylic acid is formed. Which one of the compounds below must
be of C7H8O? 21. O
FeCl3
A) m-cresol B) Benzyl alcohol I. X + CH3 — C Y + HCl
186
25. How many pi (n) bonds are there in one molecule of the
H compound that is produced by oxidizing o-xylene?

A) 1 B) 2 C) 3 D) 5 E) 6
II. Y + H2 Ni C6H5 — CH — CH3

Which one of the following statements is correct for the reactions 26. Which one of the statements given
given above? below is wrong for the compound on
the right?
A) X is benzyl chloride.
B) Y is an aromatic aldehyde.
C) The first reaction is an oxidation reaction.
D) The second reaction is an addition reaction.
E) The molar mass of X is greater than that of Y. A) It is a benzoic anhydride.
B) When 1 mole is hydrolyzed, 2 moles
benzoic acid are produced.
22. Which one of the following is benzyl chloride? C) It is produced by the reaction of benzoic acid with
benzaldehyde.
D) The pH value of its solution is less than 7.
E) When 1 mole reacts with 1 mole NH3, 1 mole
benzoic acid and 1 mole benzamide are produced.
27. The compound shown on the right can
be called;

I.
sodium-
2-

23.
AlCl3
+X Y + HCl

hydroxybenzoate
OH II. sodium m-hydroxybenzoate
| III. sodium salicylate
Y+H CH – CH3 Which of these names is(are) correct?

A) I only B) I and II C) III only


What are the substances X and Y in the above reactions? D) II and III E) I, II and III

X Y

A) Acetyl chloride Phenylmethyl ketone 28. For the compound OCH2COOH,


B) Acetic acid Acetophenone
C) Acetyl chloride Phenylmethyl ester I. CO2 is released when it reacts with NaHCO3.
D) Benzoic acid Phenylmethyl ketone II. Phenol and bromoacetic acid are produced by
E) Benzyl chloride Acetophenone its reaction with HBr.
III. When it is hydrolyzed, benzyl alcohol and
acetic acid are produced.
24. Which one of the following is the weakest acid?
Which of the above statements is (are) correct?
A) H2SO4 B) CH3CH2OH C) CH3COOH
A) I only B) I and II C) I and III
D) COOH E) H3 COH D) II and III E) I, II and III
|
COOH

187
29. Which one of the following does not produce benzoic
acid when it is oxidized?
34.

Compound Oxidation Product


I. Aniline nitrobenzene
II. Toluene benzaldehyde
III. p-xylene terephthalic acid
30.
I. CH3COOH Which of the above pairings are correct?
II. CH3CH(OH)CH3
III. C6H5OH A) I only B) I and II C) I and III
D) II and III E) I, II and III
Which of the above compounds can react with Ma to produce H2
gas?

A) I only B) I and II C) I and III


D) II and III E) I, II and III
31. Which one of the following compounds does not show acidic
properties? 35. For the compound

I. It is an alcohol.
II. It is an ester. III. It is a hydroxy acid. Which of the descriptions
above are incorrect?

A) I only B) II only C) III only


D) I and II E) I, II and III

32. Which one of the following compounds cannot be


oxidized to benzoic acid?

A) Toluene B) Benzylalcohol C) Benzaldehyde


D) Acetophenone E) Tert-butylbenzene
36. For the compound;

33. CH3 is heated with alkaline KMnO4,


I. It reacts with NaOH.
the product is then reacted with a strong acidic solution. II. When it is oxidized, salicyclic acid is formed.
Which product will be produced by this process? III. It changes the color of aqueous Br2 solution.
Which of the statements above is(are) correct?

A) I only B) I and II C) I and III


D) II and III E) I, II and III

188
37. Initial Substances Polymer Produced A) I only B) I and III C) IV only
I. Phenol-formaldehyde Bakelite D) I and IV E) I, II and III
II. Terephthalic acid-glycol Dacron
III. Aniline-phenol Acetophenone

Which of the pairings above are correct?

A) I only B) I and II C) I and III


D) II and III E) I, II and III

38.

By which one of the reactions below may the above compound be


formed?

39. I. C6H5COOH
II. C6H5CHO
III.C6H5 – OH
IV.C6H5 – NH2

Which of the above compounds forms a basic salt with NaOH and
has an aqueous solution that is acidic?

189
Chapter 1……………..
7. Which is not correct for CH3—O—CH3
1) All of the following are organic except: and CH3—CH2—OH?
A) Gasoline A) They have the same molecular formula.
B) Paper B) They have the same molecular mass.
C) Plastics C) Their percentage composition are identical.
D) Water D) They have the same empirical formula.
E) Human body E) They are the same compounds.

2. Which one of the elements is present in the smallest amount 8. I. Molar mass
in the human body? II. Types of the atoms
A) Carbon III. Numbers of atoms in a molecule
B) Silicon Which of the above can not be determined by the empirical formula
C) Hydrogen of a compound?
D) Oxygen A) I only
E) Nitrogen B) II only
C) III only
3. I. Air D) I and III
II. Petroleum E) II and III
III. Natural gas
Which of the above is/are a source of organic compounds? 9. Which of the following is
A) I only incorrect for carbon?
B) II only A) It is tetravalent.
C) III only B) It can form single, double and triple bonds.
D) I and II C) It can form stable long chains.
E) II and III D) It is very electronegative.
E) It can form cyclic structures.
4. Which of the following is not valid for organic compounds?
A) The main element is carbon 10. An organic compound whose molar mass is 60 g/mol
B) They are soluble in water contains 40% carbon, 53.3% oxygen and 6.7% hydrogen.
C) They are often liquid or gas at What is the molecular formula of this compound?
STP. A) CH2O
D) Their reactions are usually slow B) C3H8O
E) Their molecules may contain C) C4H12
oxygen and hydrogen. D) C2H4O2 E) H2CO3
Chapter 2……………..
5. I. To detect carbon, CuO is used.
II. To detegc nitrogen, NaOH is used 1. How many sigma bonds are there in a molecule of a
III. Hydrogen in organic compounds forms water when heated saturated aliphatic hydrocarbon with 5 carbon atoms?
with CuO. A) 5
Which of the given statements is/are correct? B) 8
A) I only C) 12
B) II only D) 16
C) I and II E) 17
D) II and III
E) I, II and III 2. 3 methyl groups and 1 ethyl group are attached to one
carbon atom. This compound can be named as;
6. All of the following have the same empirical formula except:
A) CH3—CH=CH—CH3 B) H2C—CH2 I. 2,2 – dimethylbutane
II. Neohexane
C2H—CH2 III. Isopentane
C) CH2 A) I only
B) II only
H2C CH D)CH2=C—CH3 C) I and II
CH3 D) II and III
CH3 E) I, II and III
E) H2C—CH2
HC CH2 3. Which one of the following is not a isomer of n-hexane?
A) CH3—CH—CH2—CH2—CH3
CH

190
CH3 C) I > III > II
B) CH3—CH2—CH—CH2—CH3 D) III > I > II
C) CH3—CH—CH—CH3 E) II > III > I
CH3 CH3
D) CH3—CH2—CH—CH—CH3
CH3 CH3
CH3 9. When 8.8 grams of an alkane is burned, 26.4 grams of
CO2 is produced.
E) CH2—C—CH2—CH3 What is the molecular formula of this alkane?
A) C3H
CH3 B) C2H6
4. I. Chloroethane C) C3H8
II. Propane D) C4H10
III. Methane E) C3H12
Which of the above compounds do not have isomer?
A) I only 10. I. The alkane that produces 5 moles of water when 1
B) I and II moles is burned is C4H10
C) II and III II. The alkane that gives 1 mol of CO2 when 0.5 mole is
D) I and III burned is C4H6
E) I, II and III III. The alkane that gives 22.4 L CO2
(at STP) when 0.25 mole is burned is C4H10.
5. Which one of the following compounds has isomers? Which of the above is/are correct?
A) Methane
B) Methyl chloride A) I only
C) Ethane B) I and II
D) Propane C) II and III
E) Butane D) I and III
E) I, II and III
6. I. Molecular weight
II. Molecular formula 10. Which one of the given compounds cannot be produced by
III. Density a Wurtz reaction?
IV. Boiling point
X and Y are isomers. Which two of the given properties are A) C6H14
exactly the same for X and Y? B) C4H10
A) I only C) C3H8
B) I and II D) C2H6
C) II and III E) CH4
D) I and III
E) I, II and III 12. To produce 2 – methypropane by a Wurtz reaction, which two
alkyl halides should be reacted together?
7. I. They are insoluble in water because they are nonpolar. I. 1 – chloropropane
II. Van der Waals forces exist II. Methyl chloride
between the molecules. III. 2 - chloropropane
III. Boiling point increases with increasing number of IV. Ethyl chloride
carbon atoms.
Which of the given statements is/are correct for alkanes? A) I and III
A) I only B) II and III
B) II only C) I and IV
C) I and II D) II and IV
D) II and III E) I and II
E) I, II and III
13. Which of the following might be produced by the reaction of
8. Compare the boiling points of: C2H5Cl and C3H7Cl with Na?
I. n-pentane
II. isopentane I. CH4
III. neopentane II. C4H10
III. C5H12
A) I > II >III
B) III > II > I A) I only
191
B) II and III B) C3H6
C) I and III C) C4H8
D) I and II D) C5H10
E) I, II and III E) C6H12

14. I. CH2=CH2+H2 C2H6 2. How mane σ bonds are there in an alkene whose molecular
II. 2CH3Br+2Na C2H6+2NaBr
weight is 56 grams?
III. C2H5Cl+KOH C2H6+KCl
A) 10
Which of the given reactions is/are redox reaction?
B) 11
A) I only
C) 12
B) II only
D) 13
C) I and II
E) 14
D) II and III
E) I, II and III
3. A mixture containing equal moles of C2H4 and C3H8 contains
HC2
3.6 grams of hydrogen. What is the mass of carbon in the mixture?
15. CH2 H2C—CH2 H2C CH2
A) 6
B) 9
H2C—CH2 H2C—CH2 H2C—CH2
cyclopropane cyclobutane cyclopentane
C) 12
I. II. III. D) 18
E) 24
Which of the above names is/are correct?
A) I only 4. Compound Name
B) II only C3H CH3
C) I and II I. C=C trans-2-butene
D) II and III
E) I, II and III H H

16. How many grams of Al4C3 must react with water to produce
6.72 L of CH4 gas at STP.
A) 14.4
B) 28.8 H C2H5
C) 43.2 II. C=C cis – 2 – pentene
D) 57.6
E) 100.8 CH3 H
17. Which of the methods can be used to produce methane?
I. From its elements using catalysts H2C H
II. Addition of H2 to alkynes II. C=C 1,3 – butadiene
III. The reaction of CO2 with H2O
A) I only H CH2
B) II only Which of the given compounds is/are named correctly?
C) I and II A) I only
D) II and III E) I, II and III B) III only
18. The molar mass of an alkyl bromide is 123 g/mol. C) I and II
Which of the compounds below can be produced from the reaction D) II and III
of this alkyl bromide with Na? E) I and III
(H : 1, C : 12, Br : 80)
A) Ethane 5. Br—CH—CH=CH
B) Propane CH3 CH3
C) Sodium Propionate What is the correct name of the compound?
D) Heptane A) 1- bromo-1,3-dimethylpropene
E) 2,3 – dimethylbutane B) 1-bromo-1-methyl-3-butene
C) 4-bromo-4-methyl-butene
Chapter 3……………………. D) 4-bromo-2-pentene
E) 2-bromo-4-pentene
1. What is the molecular formula of an alkene of which 0.3 mol 6. all of the followings are isomers of each other except:
weighs 21 grams? A) Cyclohexane
B) 2-hexene
A) C2H2
192
C) 3-methyl-1-pentene Which of the above occur?
D) 1,3-hexadiene A) I only
E) 3,3-dimethyl-1-butene B) II only
C) I and II
7. For CH2=CH—CH=CH2 D) I and III
I. Its name is 1,4-butadiene. E) I, II and III
II. 1 molecule contains 9 σ and 2 π bonds.
III. It is an isomer of 1-butyne. 12. All of the following hydrocarbons undergo addition reaction
Which of the above statements is/are correct? except for;
A) I only A) CH4
B) II only B) C2H2
C) I and II C) C2H4
D) I and III D) C3H4
E) II and III E) C3H6

13. 27 grams of a straight chained hydrocarbon containing two π


bonds in its structure is fully saturated with 2 grams of hydrogen.
How many hydrogen atoms are there in one molecule of this
compound?

8. In which one of the following compounds is cis-trans isomerism


observed? A) 1,2-dibromobutane
A) CH2=CH2 B) 2,3-dibromobutane
B) CH3-CH=CH2 C) 2,2-dibromobutane
C) CH2=CH-CH2-CH3 D) 1,3-dibromobutane
D) CH3-CH=CH-CH3 E) 2,2-dibromobutane
E) CH3-CH2-CH2-CH=CH2
15. Regarding the compound that is formed by the addition of HBr
9. Formula Name to 1-methylcyclopentane;
H F I. It is aliphatic.
II. Its molecular formula is C6H13Br.
I. C=C trans-1,2 difluorotene
III. It is named as 1-bromo-1-methylcyclopentane;
F H Which of the above statements is/are wrong?
A) I only
II. CH3—C=CH—CH3 B) II only
C) I and II
CH3 D) I and III
E) I, II and III
III. CH—CH—CH3
CH CH2 16. H3C H
Which of the above is/are named correctly? C=C
A) I only H CH3
B) II only For the above compound;
C) I and II I. It is named as trans-2-butene.
D) II and III II. It decolorizes ruminated water.
E) I, II and III III. 1 mole is saturated with 1 gram of H2.
Which of the above statements is/are correct? (H:1)
10. Which of the given aliphatic compounds undergoes an addition A) I only
reaction with H2 in the presence of Pt catalyst? B) II only
A) CH4 C) I and II
B) C2H6 D) II and III
C) C2H4Cl2 E) I and III
D) C2H5OH
E) C3H6
H H H
11. In the saturation of 1 mole of acetylene with H2:
I. 2 moles of π bonds are broken. 17. H—C= C—C—H + HBr
II. 4 moles of σ bonds are formed.
CH3
III. Ethane is formed.
193
What is the compound that is formed in the above reaction? According to this data, what is the mass percentage of C3H6 in the
A) 1-methyl-2-bromopropane starting mixture?
B) 1-bromo-3-methylpropane A) 12.5
C) 2-bromo-3-methylpropane B) 25
D) 2-bromopropane C) 40
E) 1-bromopropane D) 50
E) 75
18. Which of the following cyclic compounds undergo addition
reactions? 25. Which one of the following compounds cannot be obtained
A) C2H4 from the reaction between 2-chlorobutane and KOH in alcohol
B) C3H6 solution?
C) C4H8 A) Potassium chloride
D) C5H10 B) 1-butene
E) C6H12 C) 2-butene
D) 1-butyne
20. CH3—CH=CH—CH3 E) Water
For 1 mol of this compound;
I. It undergoes an addition reaction with 1 mol of H2;
II. When it is burned, 4 moles of CO2 is produced.
III. It undergoes an addition reaction with 81 grams of
HBr.
Which of the above statements is/are correct?
A) I only
B) I and II
C) I and III
D) II and III
E) I, II and III

21. How many liters of oxygen at STP are required to burn the
propene gas that is obtained from 1.2 grams of propyl alcohol? Chapter 4 ………………………..
(C: 12, H: 1, O: 16)
A) 1.344
B) 1.680 1. I. Acetylene
C) 1.792 II. Cyclopropane
D) 2.016 III. Cis-2-butene
E) 2.240 In which of the compounds given above do the carbon atoms
undergo only sp hybridization?
22. 1.4 mol of O2 is needed to burn 0.2 mole of a cycloalkene A) I only
completely. What is the number of hydrogen atoms in one molecule B) II only
of this cycloalkene? C) III only
A) 6 D) I and II
B) 8 E) I, II and III
C) 10
D) 12 2. Which of the given acyclic compounds can decolorize bromine
E) 14 water?
I. C6H10
23. What is the molecular formula of 2,3,4- trimethyl-1,3- II. C4H10
pentadiene? III. C5H10
A) I only
A) C5H10 B) II only
B) C5H8 C) I and II
C) C8H16 D) I and III
D) C8H14 E) I, II and III
E) C8H18
3. The names of three compounds are given below.
24. C3H6 gas and an excess amount of H2 are put into a closed I. CH3—C≡C—CH3 2-butyne
container. The gases react with each other until all of the C3H6 is II. CH3—C≡CH 2-propyne
consumed. After the reaction and at the initial temperature, the ratio III. CH≡C—CH—CH3 2-methyl-3-butyne
of the initial pressure to the final pressure is found to 8/7.
CH3
194
Which of them is/are named correctly? III. IF water I added to propyne, dimethyl ketone (2-propanone) is
A) I only produced.
B) II only Which of the above statements is/are correct?
C) I and II
D) I and III A) I only
E) II and III B) I and II
C) II and III
4. I. Acetylene D) I and III
II. 1,3 – pentadiene E) I, II and III
III. 2 – butene
If we have 1 mole of each of the compounds above, to which one(s) 10. CH3—C≡C—Na + CH3—Cl
can 2 moles of H2 be added? Which one of the names below is the correct hydrocarbon produced
in this reaction?
A) I only A) 2-butyne
B) II only B) 1-butyne
C) I and II C) 2-butene
D) II and III D) Butane
E) I, II and III E) Propyne

5. A 30 L of mixture of C2H2 and C2H4 is saturated with 50 L of 11. With ammoniacal AgNO3 solution;
H2. What is the mole percentage of acetylene in the mixture? I. H—C≡C—H
A) 25 % II. CH3—C≡C—H
B) 50 % III. CH3—C≡C—CH3
C) 75 % which of the above undergoes a reaction?
D) 33 % A) I only
E) 67 % B) I and II
C) II and III
6. Which one of the given compounds below will not change the D) III only
color of bromine water? E) I, II and III
A) C2H2
B) C2H4 12. When 11 grams of a mixture of C2H6, C2H4 and C2H2 gases is
C) C3H4 passed through ammoniacal AgNO3 solution, 12 grams of silver
D) CH4 acetylide precipitates. The remaining mixture is passed through
E) C3H6 brominated water and 24 grams of Br 2 is used up.
What is the mass percentage of C2H6 in the mixture? (H: 1, C: 12,
7. CH—C≡C—CH3 Br: 80, Ag: 108)
CH2=CH—CH=CH2 A) 25
2 moles of BR2 is added to one mole of each of above compounds. B) 35
Which property is/are the same for the reaction products? C) 50
I. Molecular formula D) 60
II. Molecular weight E) 75
III. Structural formula 13. For the compound CH3—CH2—C≡CH
A) I only I. It undergoes a reaction with ammoniacal AgNO3 solution.
B) II and III II. It decolorizes bromine water.
C) I and III III. It undergoes an addition reaction with hydrogen.
D) I and II Which of the above statements is/are correct?
E) I, II and III

8. Propyne is saturated with an excess of HBr. What is the name of


the product? A) I only
A) 2,2-dibrromopropane B) II only
B) 2-bromopropane C) III only
C) 2,2-dibromopropene D) I and III
D) 2-bromopropene E) I, II and III
E) propane

9. I. If water is added to ethylene, ethyl alcohol is produced.


II. IS water is added to acetylene, acetaldehyde (ethanal) is
produced.

195
Chapter 5……………………. CH3
7. I. Its molecular formula is C10H8.
II. Hydrogen can be added to it at room temperature.
1. For benzene; III. It is an aliphatic hydrocarbon.
I. It is an aromatic compound. Which of the above statements is/are correct for naphthalene?
II. Its empirical formula is CH. A) I only
III. It contains 3 π and 12 σ bonds. B) II only
Which of the above statements is/are correct? C) III only
A) I only D) I and II
B) II only E) I and III
C) I and III
D) II and III
E) I, II and III

2. Which of the following pairs of compounds are isomers?


I. Cl Cl

II. CH3 C2H5

CH3
III.
OH O—CH3

CH
A) I only
B) I and II
C) I and III
D) II and III
E) I, II and III

3. All of the following react with benzene except:


A) Water
B) Nitric acid
C) Bromine
D) Sulfuric acid
E) Methyl chloride

4. I. II. III.

Which of the above compounds is/are aromatic?


A) I only
B) II only
C) I and II
D) II and III
E) I, II and III

5. Which of the following is correct for benzene molecule?


A) It contains 12 hydrogen atoms.
B) It is unsaturated.
C) The carbon-carbon bonds are single.
D) The carbon-carbon bonds are double.
E) It has an acyclic structure.

6. All of the following are isomers of each other except:


A) C2H5 B) CH3 C) CH3 D) CH3 E) CH3
CH3
CH3 H3 C CH3

196

Вам также может понравиться